1600 John F. Kennedy Blvd. Ste 1800 Philadelphia, PA 19103-2899
Otologic Surgery ISBN: 978-1-4160-4665-3 Copyright © 2010, 2001, 1994 by Saunders, an imprint of Elsevier Inc. All rights reserved. No part of this publication may be reproduced or transmitted in any form or by any means, electronic or mechanical, including photocopying, recording, or any information storage and retrieval system, without permission in writing from the publisher. Permissions may be sought directly from Elsevier’s Health Sciences Rights Department in Philadelphia, PA, USA: phone: (+1) 215 239 3804, fax: (+1) 215 239 3805, e-mail:
[email protected]. You may also complete your request on-line via the Elsevier homepage (http://www.elsevier.com), by selecting “Customer Support” and then “Obtaining Permissions”.
Notice Knowledge and best practice in this field are constantly changing. As new research and experience broaden our knowledge, changes in practice, treatment and drug therapy may become necessary or appropriate. Readers are advised to check the most current information provided (i) on procedures featured or (ii) by the manufacturer of each product to be administered, to verify the recommended dose or formula, the method and duration of administration, and contraindications. It is the responsibility of the practitioner, relying on his or her own experience and knowledge of the patient, to make diagnoses, to determine dosages and the best treatment for each individual patient, and to take all appropriate safety precautions. To the fullest extent of the law, neither the Publisher nor the Editors assume any liability for any injury and/or damage to persons or property arising out of or related to any use of the material contained in this book. The Publisher
Library of Congress Cataloging-in-Publication Data Otologic surgery / [edited by] Derald E. Brackmann, Clough Shelton, Moisés A. Arriaga. — 3rd ed. p. ; cm. Includes bibliographical references and index. ISBN 978-1-4160-4665-3 1. Ear—Surgery. I. Brackmann, Derald E. II. Shelton, Clough. III. Arriaga, Moises A. [DNLM: 1. Ear—surgery. 2. Otologic Surgical Procedures. WV 200 O878 2010] RF126.O87 2010 617.8’059—dc22 ���������������������������������������������������������������������� 2009032119
Acquisitions Editor: Stefanie Jewell-Thomas Developmental Editor: Rachel Yard Project Manager: Jagannathan Varadarajan Design Direction: Ellen Zanolle Publishing Services Manager: Hemamalini Rajendrababu
Printed in the United States of America Last digit is the print number: 9 8 7 6 5 4 3 2 1
Dedication This book is dedicated to our mentors and teachers, Drs. Howard P. House, William F. House, and James L. Sheehy. Each of these outstanding physicians has special talents and characteristics that, when melded together, resulted in an outstanding clinical, research, and educational facility, The House Ear Clinic and Institute. Regrettably, Drs. Howard House and James Sheehy have passed away since the publication of the previous edition of this book. Howard House, the founder of our institution, was among the first to concentrate his activities in the field of otology. He devoted his career to the treatment of otosclerosis. In addition to his surgical genius, Howard was recognized as an outstanding statesman and fundraiser. Without him the House Ear Institute, which has provided so many opportunities for all of us, would not exist. He died in 2003 at the age of 95. At the time of his death, he was still coming to the office regularly and was active in development work for the Institute.
William F. House joined his brother in practice after completing his residency. A creative genius, Bill recognized that the future of otology lay in the diagnosis and treatment of diseases of the inner ear. He introduced the operating microscope and microsurgical techniques to the field of neurosurgery and revolutionized the treatment of acoustic tumors and other neurotologic problems. Bill is also recognized as instrumental in bringing the cochlear implant to the state of a practical clinical device that is now widely applied. Bill is now retired from clinical practice but, at the age of 86, remains extremely creative and is currently pursuing a number of new innovations in otology and audiology.
The final link in the chain that resulted in the success of the House Ear Clinic and Institute was Dr. James L. Sheehy. His special interest was in the field of chronic otitis media. In addition to his outstanding surgical ability, Jim possessed exceptional talent in organizational ability and teaching. Jim was responsible for developing all the patient educational materials as well as serving as the editor for all of the many publications produced by members of the House Ear Clinic. His course development, panel discussions, and slide preparation techniques became standards for our specialty. Jim had been a member of the House Ear Clinic for 48 years and died in 2006. It was our great privilege to be under the personal tutelage of each of these outstanding men. In addition to all the attributes enumerated above, first and foremost each was an outstanding physician. They practiced the art and science of surgery in the finest fashion, making it most appropriate that this book on surgical technique be dedicated to them. Derald E. Brackmann, MD Clough Shelton, MD Moisés A. Arriaga, MD
In Memoriam
On October 19, 1996, the field of otology lost one of its most influential leaders of modern times. Harold Frederick Schuknecht, MD, Professor Emeritus of the Department of Otology and Laryngology at the Harvard Medical School and Chief Emeritus of the Department of Otolarynology at the Massachusetts Eye and Ear Infirmary, was a world-renowned clinical otologist, otopathologist, teacher, and scholar. His contribution to human otopathology is unparalleled. His book, Pathology of the Ear, which he solely authored, is without question the most complete and comprehensive thesis on the subject. His clinical approach and technical innovations were based on scientific principle, and he unabashedly held others to the same standard. His influence as a teacher and role model is evidenced by the unprecedented number of his students who have followed in his footsteps and have risen as leaders in our specialty. Through his life’s work and through the lives of those he has touched, his influence lives on. Harold Frederick Schuknecht
Mendell Robinson, MD, known for his eponymous stapes prosthesis, passed away on September 29, 2007. A sketch on a napkin during an air flight in 1960 led to the development of this popular and successful prosthesis. Dr. Robinson was an internationally renowned otosclerosis surgeon and had a successful otologic practice in Providence, Rhode Island, for almost 50 years. He was so appreciated that the mayor of Providence officially declared “Mendell Robinson Day” on two separate occasions. We have chosen to leave his chapter unchanged from the previous edition.
Mendell Robinson
vii
viii
In Memoriam
As this edition of Otologic Surgery was going to press, we were saddened by the sudden death of our dear colleague Antonio De la Cruz. He succumbed to a malignant lymphoma after a very brief illness. Antonio was a member of the House Ear Clinic and Institute for 34 years and director of the Institute’s Department of Education. He directed hundreds of temporal bone dissection courses at the Institute and was responsible for teaching otologic surgery to thousands of physicians. His colleagues recognized him by election to the presidency of the American Academy of Otolaryngology– Head and Neck Surgery and the American Otologic Society. Antonio participated in more national and international courses than any physician in the history of our specialty. All of us marveled at his tireless energy, which allowed him to travel at least on a monthly basis to courses around the world. In addition to his teaching activities, Antonio maintained an active otologic and neurotologic practice, benefiting many patients with his expertise. He contributed greatly in many areas, particularly in the surgical correction of congenital atresia of the external auditory canal. His techniques are described in the chapter that he contributed to this volume. A former House Fellow wrote the following: “I am saddened to hear of Antonio’s passing. He had a unique ability to encourage others to perceive the skills of the expert to be achievable by them. His humble style, though, belied a high level of skill and savvy. His focused energy, his keen intellect, and his eagerness to teach all made him a great mentor and colleague, roles that touched so many of us over the last 30+ years. I am sure many, many will miss him but will forever cherish the perspective, skills, and tips he gave so freely. His contributions will live on.”
Antonio De la Cruz
Contributors Oliver F. Adunka, MD
Derald E. Brackmann, MD, FACS
Assistant Professor, Department of Otolaryngology−Head and Neck Surgery, University of North Carolina School of Medicine, Chapel Hill, North Carolina
Clinical Professor, Otolaryngology-Head and Neck Surgery and Neurological Surgery, University of Southern California School of Medicine; President, House Ear Clinic, and Board of Directors, House Ear Institute, Los Angeles, ���������� California
Translabyrinthine Vestibular Neurectomy
Moisés A. Arriaga, MD, MBA, FACS Clinical Professor of Otolaryngology and Neurosurgery, and Director of Otology and Neurotology, Department of Otorhinolaryngology−Head and Neck Surgery, Louisiana State University Health Science Center, New Orleans; Medical Director, Hearing and Balance Center, Our Lady of the Lake Regional Medical Center, Baton Rouge, Louisiana
Malignancies of the Temporal Bone—Limited Temporal Bone Resection; Mastoidectomy—Canal W all Down Procedure; Overview of Transtemporal Skull Base Surgery; Surgery for Glomus Tumors and Other Lesions of the Jugular Foramen; Anterior and Subtemporal Approaches to the Infratemporal Fossa
Gregory A. Ator, MD Associate Professor, Department of Otolaryngology−Head and Neck Surgery; Director, Division of Otology/Neurotology, University of Kansas Medical Center, Kansas City, Kansas
Traumatic Facial Paralysis
James E. Benecke, Jr., MD Clinical Professor, Otolaryngology−Head and Neck Surgery, St. Louis University School of Medicine; Section Chief, Otolaryngology, Missouri Baptist Medical Center; Director, Otology Associates, Inc. St. Louis, Missouri
Otologic Instrumentation
Leonard P. Berenholz, MD Trumbull Memorial Hospital, Department of Otolaryngology, Warren, Ohio
Special Problems of Otosclerosis Surgery
K. Paul Boyev, MD Assistant Professor, �������������������������������������� Department of Otolaryngology-Head and Neck Surgery������������������������������������������������ ; Director, Division of Otology/Neurotology, and Director, Hearing and Balance Center, University of South Florida College of Medicine, Tampa, Florida
Drainage Procedures for Petrous Apex Lesions; Surgery for Glomus Tumors and Other Lesions of the Jugular Foramen; Middle Fossa Approach; Auditory Implants for the Central Nervous System; Management of Postoperative Cerebrospinal Fluid Leaks
Craig A. Buchman, MD, FACS Professor and Chief, Division of Otology, Neurotology, and Skull Base Surgery, Department of Otolaryngology-Head and Neck Surgery, University of North Carolina School of Medicine, Chapel Hill, North Carolina
Translabyrinthine Vestibular Neurectomy
John P. Carey, MD Associate Professor, Otolaryngology-Head and Neck Surgery, Johns Hopkins University School of Medicine; Attending, The Johns Hopkins Hospital, Baltimore, �������� Maryland
Superior Semicircular Canal Dehiscence Syndrome
Ricardo L. Carrau, MD Professor, Departments of Neurological Surgery and Otolaryngology, University of Pittsburgh School of Medicine; Minimally Invasive EndoNeurosurgery Center and Center for Skull Base Surgery, University of Pittsburgh Medical Center, Pittsburgh, Pennsylvania
Anterior and Subtemporal Approaches to the Infratemporal Fossa; Endoscopic Endonasal Approaches to the Skull Base and Paranasal Sinuses
Stephen P. Cass, MD Associate Professor, Department of Otolaryngology, University of Colorado, Denver, Colorado
Chemical Treatment of the Labyrinth
Ray C. Chang, MD Department of ������������������������������������� Otolaryngology, University of Miami; Department of ������������������������������������������ Otolaryngology, Jackson Memorial Hospital/ University of Miami, Miami, Florida
Intraoperative Neurophysiologic Monitoring
Cochleosacculotomy ix
Contributors
Douglas A. Chen, MD
Shervin R. Dashti, MD, PhD
Adjunct Associate Professor of Surgery, Allegheny University of the Health Sciences; Director, Division of Neurology, Hearing and Balance Center, Allegheny General Hospital, Pittsburgh, ������������ Pennsylvania
Neurosurgical Institute of Kentucky, Norton Neuroscience Institute, Louisville, Kentucky
Dural Herniation and Cerebrospinal Fluid Leak
Henry H. Chen, MD, MBA Resident Physician, ����������������������������������������� Department of ��������������������������� Otolaryngology, University of Colorado Health Sciences Center, Denver, Colorado
Traumatic Facial Paralysis
Vascular Considerations in Neurotologic Surgery
M. Jennifer Derebery, MD Clinical Professor of Otolaryngology, University of Southern California School of Medicine; Staff, St. Vincent’s Medical Center, and Associate, House Ear Clinic, Los Angeles, California
Surgery of Ventilation and Mucosal Disease
Joseph M. Chen, MD, FRCS(C)
Shaun C. Desai, MD
Associate Professor, Department of Otolaryngology, University of Toronto; Staff Surgeon, Sunnybrook and Women’s College Health Science Center, Toronto, Ontario, Canada
The George Washington University Medical Center, Washington, DC
Middle Cranial Fossa—Vestibular Neurectomy; Transotic Approach
Vivek R. Deshmukh, MD
Sarah S. Connell, MD Neuro-otology Fellow, ������������������������������ Department of ���������������� Otolaryngology, University of Miami Ear Institute, Miami, Florida; Physician, Otolaryngology, Kaiser Permanente, Walnut Creek, California
Intraoperative Neurophysiologic Monitoring
Benjamin T. Crane, MD, PhD Assistant Professor, ����������������������������������������� Department of ��������������������������� Otolaryngology, University of Rochester; Assistant Professor, Otolaryngology, Strong Memorial Hospital, Rochester, New York
Superior Semicircular Canal Dehiscence Syndrome
Antonio De la Cruz, MD† Formerly Clinical Professor, Otolaryngology-Head and Neck Surgery, University of Southern California, Los Angeles, California
Congenital Malformation of the External Auditory Canal and Middle Ear; Transcochlear Approach to Cerebellopontine Angle Lesions
Robert D. Cullen, MD Otologic Center, Inc., and Midwest Ear Institute, Kansas City, �������� Missouri
Surgery for Cochlear Implantation; Translabyrinthine Approach
Calhoun D. Cunningham III, MD Consultant Clinical Professor of Otolaryngology−Head and Neck Surgery, Duke University School of Medicine; VicePresident of Carolina Ear and Hearing Clinic, and Co-Director of Carolina Research Institute, Raleigh, North Carolina
Ossicular Reconstruction †Deceased
Vascular Considerations in Neurotologic Surgery
Director, Cerebrovascular and Endovascular Neurosurgery, Department of Neurosurgery, George Washington University, Washington, DC
Vascular Considerations in Neurotologic Surgery
John L. Dornhoffer, MD, FACS Professor and Director, Division of Neurotology, Department of Otolaryngology−Head and Neck Surgery; Professor, Department of Neurobiology and Developmental Sciences; and ���������������������������������������������������� Medical Director, ENT Clinic and Audiology Services, University of Arkansas for Medical Sciences, Little Rock, Arkansas
Cartilage Tympanoplasty
Adrien A. Eshraghi, MD Associate Professor, Department of Otolaryngology, University of Miami, Miller School of Medicine, Miami, Florida
Intraoperative Neurophysiologic Monitoring
Jose N. Fayad, MD Associate Professor, Clinical Otolaryngology, University of Southern California; Associate, Otology/Neurotology, House Ear Clinic, Los Angeles, California
Otologic Instrumentation; Tympanoplasty—Outer Surface Grafting Technique; Auditory Implants for the Central Nervous System
Ugo Fisch, MD Professor Emeritus of Otolaryngology, University of Zurich; Head of ENT and Skull Base Surgery, University Hospital, Zurich, Switzerland
Middle Cranial Fossa—Vestibular Neurectomy; Transotic Approach
Contributors
xi
David R. Friedland, MD, PhD
Neil A. Giddings, MD
Associate Professor and Chief, Division of Otology and Neuro-otologic Skull Base Surgery, Department of Otolaryngology and Communication Sciences, Medical College of Wisconsin; Attending Physician, Division of Pediatric Otolaryngology, Children’s Hospital of Wisconsin; Attending ��������������������������������������������� Physician,����������������������������������� Department of Otolaryngology-Head and Neck Surgery, Froedtert and The Medical College of Wisconsin Hospital, Milwaukee, Wisconsin
Sacred Heart Medical Center and Deaconess Medical Center, Spokane, ���������� Washington
Stereotactic Radiosurgery of Skull Base Tumors
Rick A. Friedman, MD, PhD Associate Professor, Otolaryngology, University of Southern California; Chief, Division of Skull Base Surgery, Cedars-Sinai Medical Center; Chief, Section on Hereditary Disorders, Cell and Biology Genetics Division, House Ear Clinic/House Ear Institute, Los Angeles, ���������� California
Surgery of V entilation and Mucosal Disease; Translabyrinthine Approach; Extended Middle Cranial Fossa Approach
Takanori Fukushima, MD, MMSc Consulting Professor, Department of Neurosurgery, Duke University Medical Center, Durham, North Carolina; Professor, Department of Neurosurgery, West Virigina University, Morgantown, West Virginia
Extreme Lateral Infrajugular Transcondylar Approach for Resection of Skull Base Tumors
Bruce J. Gantz, MD, FACS Professor and Department Head, Otolaryngology-Head and Neck Surgery, University of Iowa; Professor and Department Head, Otolaryngology-Head and Neck Surgery, University of Iowa Hospitals and Clinics, Iowa City, Iowa
Canal W all Reconstruction Tympanomastoidectomy; Management of Bell’s Palsy and Ramsay Hunt Syndrome
Gale Gardner, MD Clinical Professor, Otolaryngology/Head and Neck Surgery, Louisiana State University Health Science Center-Shreveport; Active Staff, Otolaryngology-Head and Neck Surgery, Louisiana State University ��������������� Medical Center, Shreveport, ��������� Louisiana
Tympanoplasty—Undersurface Graft Technique: Transcanal Approach
Paul A. Gardner, MD Department of Neurological Surgery, University of Pittsburgh School of Medicine, Pittsburgh, Pennsylvania
Tympanoplasty—Undersurface Graft Technique: Transcanal Approach; Endoscopic Endonasal Approaches to the Skull Base and Paranasal Sinuses
Drainage Procedures for Petrous Apex Lesions
Michael E. Glasscock III, MD, FACS Adjunct Professor, Otolaryngology−Otology, Vanderbilt University Medical Center, Nashville, Tennessee
Tympanoplasty—Undersurface Graft Technique: Postauricular Approach
Samuel P. Gubbels, MD Assistant Professor, Surgery, Division of Otolaryngology, University of Wisconsin–Madison, Madison, ��������� Wisconsin
Canal Wall Reconstruction Tympanomastoidectomy; Management of Bell’s Palsy and Ramsay Hunt Syndrome
Ophir Handzel, MD, LLB Department of Otology and Laryngology, Harvard Medical School; Fellow, Otology/Neurotology, Otolaryngology, Massachusetts Eye and Ear Infirmary, Boston, ������������� Massachusetts
Diagnosis and Management of the Patulous Eustachian Tube
Steven A. Harvey, MD Clinical Assistant Professor, Department of Otolaryngology, Medical College of Wisconsin, Milwaukee, Wisconsin
Complications of Surgery for Chronic Otitis Media
Todd A. Hillman, MD Adjunct Clinical Faculty, Otolaryngology, Louisiana State University, New Orleans, Louisiana; Associate, Pittsburgh Ear Associates, and Staff, Division of Neurotology, Allegheny General Hospital, Pittsburgh, Pennsylvania
Petrosal Approach
William E. Hitselberger, MD Neurosurgery, St. Vincent’s Hospital, Los Angeles, California
Auditory Implants for the Central Nervous System
Barbara Stahl Hoskins, RN, BSN Allied Health Practitioner, Doheny Surgery, St. Vincent’s Medical Center, Los Angeles; Allied Health Practitioner, Surgery, Kaiser Sunset, Hollywood; Private Scrub Nurse, Neurosurgical Department, St. Vincent’s Medical Center, Los Angeles, ���������� California
Otologic Instrumentation
xii
Contributors
Howard P. House, MD†
Herman A. Jenkins, MD
Formerly Professor Emeritus, University of Southern California; Founder and Chairman Emeritus, House Ear Institute, St. Vincent’s Medical Center, Los Angeles, California
Professor and Chairman, Department of Otolaryngology, University of Colorado Health Sciences Center; Chief, Otolaryngology Service, University of Colorado Hospital, Denver, Colorado
Total Stapedectomy
Traumatic Facial Paralysis
John W. House, MD
Amin B. Kassam, MD, FRCS(C)
Clinical Professor, Department of Otolaryngology, University of Southern California School of Medicine; President, House Ear Institute, Los Angeles, California
Associate Professor, Department of Neurological Surgery, University of Pittsburgh School of Medicine; Director, Minimally Invasive EndoNeurosurgery Center, and Co-Director, Center for Skull Base Surgery, University of Pittsburgh Medical Center, Pittsburgh, ������������ Pennsylvania
Translabyrinthine Approach
William F. House, MD Formerly at Hoog Hospital, Newport Beach, California
Middle Fossa Approach
Brandon B. Isaacson, MD Assistant Professor, Otolaryngology-Head and Neck Surgery, University of Texas Southwestern Medical Center at Dallas; Attending, Otolaryngology-Head and Neck Surgery, Zale Lipshy University Hospital, Dallas, Texas
Office Management of Tympanic Membrane Perforation and the Draining Ear
Robert K. Jackler, MD Sewall Professor and Chair and Associate Dean (CME), Otolaryngology-Head and Neck Surgery, Stanford University School of Medicine, Stanford, ���������� California
Retrosigmoid Approach to Tumors of the Cerebellopontine Angle
C. Gary ���������������������� Jackson, MD, FACS Clinical Professor, Department of Surgery, University of North Carolina School of Medicine, Chapel Hill, North Carolina
Tympanoplasty—Undersurface Graft Technique: Postauricular Approach
Ivo P. Janecka, MD, FACS Professor, Harvard Medical School; Director, Skull Base International; Staff, Children’s Hospital, Brigham and Women’s Hospital, Boston, Massachusetts
Anterior and Subtemporal Approaches to the Infratemporal Fossa; Endoscopic Endonasal Approaches to the Skull Base and Paranasal Sinuses
David M. Kaylie, MD Associate Professor, Department of Surgery, Division of Otolaryngology, Duke University Medical Center, Durham, North Carolina
Tympanoplasty—Undersurface Graft Technique: Postauricular Approach
Bradley W. Kesser, MD Assistant Professor, Department of Otolaryngology-Head and Neck Surgery, University of Virginia Health System, Charlottesville, Virginia
Surgery of Ventilation and Mucosal Disease
Joe W. Kutz, MD Assistant Professor, Otolaryngology-Head and Neck Surgery, University of Texas Southwestern Medical Center at Dallas; Attending, Otolaryngology-Head and Neck Surgery, Zale Lipshy University Hospital, Dallas, Texas
Office Management of Tympanic Membrane Perforation and the Draining Ear
Jed A. Kwartler, MD, MBA Clinical Associate Professor, Division of Otolaryngology, University of Medicine and Dentistry of New Jersey, Newark, New Jersey
Total Stapedectomy
Malignancies of the Temporal Bone-Radical Temporal Bone Resection
John P. Leonetti, MD
Peter J. Jannetta, MD
Otolaryngology-Head and Neck Surgery, ������������������ Loyola University Medical Center,���������� Maywood, Illinois ���������
Professor of Neurosurgery, Department of Neurosurgery, Drexel University College of Medicine, Philadelphia; Vice Chairman, Department of Neurosurgery, Allegheny General Hospital, Pittsburgh, Pennslyvania
Operations for V ascular Compressive Syndromes †Deceased
Malignancies of the Temporal Bone—Limited Temporal Bone Resection
Contributors
xiii
Robert E. Levine, MD
Lloyd B. Minor, MD
Clinical Professor, Ophthalmology, University of Southern California Keck School of Medicine; Co-Founder and CoDirector, Facial Nerve Clinic, House Ear Clinic, Los Angeles, California,
Andelot Professor and Director, Department of Otolaryngology-Head and Neck Surgery, The Johns Hopkins University School of Medicine, Baltimore, Maryland
Care of the Eye in Facial Paralysis
James Lin, MD Assistant Professor, Department of Otolaryngology, Louisiana State University Health Sciences Center, New Orleans, Louisiana
Translabyrinthine Approach
William H. Lippy, MD, FACS Chief Physician/Surgeon, Otolaryngology, The Lippy Group for Ear, Nose, and Throat; Physician, Otolaryngology, Trumbell Memorial Hospital, Warren, Ohio
Special Problems of Otosclerosis Surgery
Philip D. Littlefield, MD Otology and Neurotology, Department of Otolaryngology, Walter Reed Army Medical Center, Washington, DC
Complications of Surgery for Chronic Otitis Media
Teresa M. Lo, MD Department of Otolaryngology, The Southeast Permanente Medical Group, Atlanta, Georgia
Perilymphatic Fistula
Larry B. Lundy, MD Associate Professor, Otolaryngology-Head and Neck Surgery, Mayo Clinic, Jacksonville, Florida,
Laser Revision Stapedectomy
William M. Luxford, MD Clinical Professor, Otolaryngology, University of Southern California Keck School of Medicine; Associate, House Clinic, Los Angeles, ���������� California
Hypoglossal Facial Anastomosis Surgery for Cochlear Implantation
John T. McElveen, Jr., MD Consultant Clinical Professor of Otolaryngology–Head and Neck Surgery, Duke University School of Medicine; President of Carolina Ear and Hearing Clinic, and Director of Carolina Research Institute, Raleigh, North Carolina
Ossicular Reconstruction
Michael J. McKenna, MD Professor of Otology and Laryngology, Department of Otology and Laryngology, Harvard Medical School; Surgeon in Otolaryngology, Department of Otolaryngology, Massachusetts Eye and Ear Infirmary, Boston, ������������� Massachusetts
Cochleosacculotomy; Transcanal Labyrinthectomy
Superior Semicircular Canal Dehiscence Syndrome
Edwin M. Monsell, MD, PhD Professor, Otolaryngology-Head and Neck Surgery, Wayne State University, Detroit, Michigan
Chemical Treatment of the Labyrinth
Joseph B. Nadol, Jr., MD Walter Augustus Lecompte Professor and Chair, Department of Otology and Laryngology, Harvard Medical School; Chief of Otolaryngology, Department of Otolaryngology, Massachusetts Eye and Ear Infirmary, Boston, ������������� Massachusetts
Transcanal Labyrinthectomy
Julian M. Nedzelski, MD, FRCS Professor Emeritus, Department of Otolaryngology-Head and Neck Surgery, University of Toronto; Consultant, Department of Otolaryngology-Head and Neck Surgery, Sunnybrook Health Sciences Centre, Toronto, Ontario, Canada
Chemical Treatment of the Labyrinth
J. Gail �������������������� Neely, MD, FACS Professor and Director, Otology/Neurotology/Base of Skull Surgery; Director of Research, Department of OtolaryngologyHead and Neck Surgery, Washington University School of Medicine; Attending, Otolaryngology-Head and Neck Surgery, BarnesJewish Hospital and St. Louis Children’s Hospital, St. Louis, Missouri
Surgery of Acute Infections and Their Complications
James L. Netterville, MD The Mark C. Smith Professor, and Director of Head and Neck Surgery, Vanderbilt Department of OtolaryngologyHead and Neck Surgery, Vanderbilt Medical Center, Nashville, Tennessee
Rehabilitation of Lower Cranial Nerve Deficits after Neurotologic Skull Base Surgery
Steven R. Otto, MA Chief Audiologist and Coordinator, ABI Program, Department of Auditory Implants and Perception, House Ear Institute, Los Angeles, California
Auditory Implants for the Central Nervous System
Mark D. Packer, MD Neurotology Fellow, Otolaryngology, The Ohio State University; Lieutenant Colonel, United States Air Force, Columbus, Ohio
Surgery of the Endolymphatic Sac
xiv
Contributors
Lorne S. Parnes, MD, FRCS(C)
Miriam I. Redleaf, MD
Professor, Otolaryngology and Clinical Neurological Sciences, University of Western Ontario; Site Chief, Otolaryngology, University Hospital–London Health Sciences Centre, London, Ontario, Canada
Assistant Professor, Otolaryngology, University of Chicago and University of Chicago Medical Center, Chicago, Illinois
Posterior Semicircular Canal Occlusion for Benign Paroxysmal Positional V ertigo
Joseph B. Roberson, Jr., MD
Rodney Perkins, MD Clinical Professor of Surgery, Stanford University, Palo Alto, California
Canalplasty for Exostoses of the External Auditory Canal and Miscellaneous Auditory Canal Problems
Brian P. Perry, MD, FACS Clinical Associate Professor, The University of Texas, Health Science Center–San Antonio, San Antonio, Texas
Management of Bell’s Palsy and Ramsay Hunt Syndrome
Thomas M. Pilkington, MD Resident Physician, Division of Otolaryngology-Head and Neck Surgery, Department of Surgery, Duke University Medical Center, Durham, North Carolina
Extreme Lateral Infrajugular Transcondylar Approach for Resection of Skull Base Tumors
Dennis S. Poe, MD Associate Professor, Otology and Laryngology, Harvard Medical School; Associate Surgeon, Otolaryngology, Children’s Hospital, Boston, ������������� Massachusetts
Diagnosis and Management of the Patulous Eustachian Tube
Sanjay Prasad, MD Clinical Assistant Professor, Department of OtolaryngologyHead and Neck Surgery, Georgetown University Medical Center, Washington, DC; President and Founder, Metropolitan Ear Group, Besthesda, Mayland
Malignancies of the Temporal Bone—Radical Temporal Bone Resection
Daniel M. Prevedello, MD Department of Neurological Surgery, University of Pittsburgh School of Medicine; Minimally Invasive EndoNeurosurgery Center and Center for Skull Base Surgery, University of Pittsburgh Medical Center, Pittsburgh, Pennsylvania
Endoscopic Endonasal Approaches to the Skull Base and Paranasal Sinuses
Management of Bell’s Palsy and Ramsay Hunt Syndrome
Stanford University Hospital, Palo Alto, and San Ramon Regional Medical Center, San Ramon, California
Canalplasty for Exostoses of the External Auditory Canal and Miscellaneous Auditory Canal Problems; Avoidance and Management of Complications of Otosclerosis Surgery
Michael A. Roberts, MD Comprehensive Ophthalmologist, St. Vincent’s Medical Center, Los Angeles, California
Care of the Eye in Facial Paralysis
Mendell Robinson, MD† Formerly Clinical Associate Professor, Brown University School of Medicine; Senior Surgeon, Miriam Hospital, Rhode Island Hospital, Providence, Rhode Island
Partial Stapedectomy
Grayson K. Rodgers, MD Birmingham Hearing and Balance Center, Birmingham, Alabama
Management of Postoperative Cerebrospinal Fluid Leaks
Peter S. Roland, MD Professor and Chairman, Otolaryngology-Head and Neck Surgery, University of Texas Southwestern Medical Center at Dallas; Chief of Service, Otolaryngology-Head and Neck Surgery, Zale Lipshy University Hospital, Dallas, Texas
Office Management of Tympanic Membrane Perforation and the Draining Ear
Christina L. Runge-Samuelson, PhD Associate Professor and Co-Director, Koss Cochlear Implant Program, Department of Otolaryngology and Communication Sciences, Medical College of Wisconsin; Division of Pediatric Otolaryngology, Children’s Hospital of Wisconsin, and Department of Otolaryngology-Head and Neck Surgery, Froedtert and The Medical College of Wisconsin Hospital, Milwaukee, Wisconsin
Stereotactic Radiosurgery of Skull Base Tumors
Leonard P. Rybak, MD, PhD Professor of Surgery, Southern Illinois University; Memorial Medical Center and St. John’s Hospital, Springfield, Illinois
Chemical Treatment of the Labyrinth
Contributors
xv
Barry M. Schaitkin, MD
David W. Sim, FRCS Ed (URL)
Professor, Department of Otolaryngology, University of Pittsburgh School of Medicine, University of Pittsburgh Medical Center Shadyside, Pittsburgh, Pennsylvania
Clinical Tutor, University of Edinburgh, Edinburgh, Scotland
Facial Reanimation Techniques
Harlod F. Schuknecht, MD† Formerly Professor and Chairman Emeritus, Department of Otology and Laryngology, Harvard Medical School; Emeritus Chief of Otolaryngology, Department of Otolaryngology, Massachusetts General Hospital, Boston, Massachusetts
Cochleosacculotomy
Raymond F. Sekula, Jr., MD Assistant Professor Neurological Surgery, Department of Neurological Surgery, Allegheny General Hospital, Pittsburgh, Pennsylvania
Operations for V ascular Compressive Syndromes
Robert V. Shannon, PhD Adjunct Professor, Biomedical Engineering, University of Southern California; �������������������������� Department Head, Auditory Implants ������������� and Perception, House Ear Institute, Los Angeles, California
Auditory Implants for the Central Nervous System
M. Coyle Shea, Jr., MD Associate Clinical Professor, Department of Otolaryngology-Head and Neck Surgery, University of Tennessee Center for the Health Sciences; Active Staff, Baptist Memorial Hospitals; Courtesy Staff, Methodist Hospitals, Memphis, ��������� Tennessee
Tympanoplasty—Undersurface Graft Technique: Transcanal Approach
James L. Sheehy, MD† Formerly Clinical Professor of Surgery and Otolaryngology, University of Southern California School of Medicine, Los Angeles, California
Tympanoplasty—Outer Surface Grafting Technique; Ossicular Reconstruction; Mastoidectomy—Intact Canal W all Procedure; Tympanoplasty—Staging and Use of Plastic
Retrosigmoid Approach to Tumors of the Cerebellopontine Angle
George T. Singleton, MD Professor Emeritus, Otolaryngology-Head and Neck Surgery, University of Florida College of Medicine; OtolaryngologyHead and Neck Surgery, Shands Teaching Hospital; Otolaryngology-Head and Neck Surgery, Malcolm Randall VA Medical Center, Gainesville, ������� Florida
Perilymphatic Fistula
William H. Slattery III, MD Clinical Professor, University of Southern California; ���������� Director, Clinical Studies, House Ear Institute; Associate, House Ear Clinic, Los Angeles, California
Implantable Hearing Devices; Neurofibromatosis
Carl H. Snyderman, MD Departments of Neurological Surgery and Otolaryngology, University of Pittsburgh School of Medicine; Minimally Invasive EndoNeurosurgery Center and Center for Skull Base Surgery, University of Pittsburgh Medical Center, Pittsburgh, Pennsylvania
Endoscopic Endonasal Approaches to the Skull Base and Paranasal Sinuses
Robert F. Spetzler, MD Professor, Department of Surgery, Section of Neurosurgery, University of Arizona College of Medicine, Tuscon; ��������� Director and J. N. Harber Chairman of Neurological Surgery, Barrow Neurological Institute, Phoenix, Arizona
Vascular Consideration in Neurotologic Surgery
Barry Strasnick, MD, FACS Professor and Chairman, Department of Otolaryngology-Head and Neck Surgery, Eastern Virginia Medical School, Norfolk, Virginia
Tympanoplasty—Undersurface Graft Technique: Postauricular Approach
Clough Shelton, MD, FACS
Christopher A. Sullivan, MD
C. Charles Hetzel, Jr., MD and Alice Barker Hetzel Presidential Endowed Chair in Otolaryngology; Professor, Otology, Neuro-Otology and Skull Base Surgery; Chief, Otolaryngology-Head and Neck Surgery, University of Utah; Medical Director, Otolaryngology-Head and Neck Surgery and Otology/Neuro-Otology Surgeon, University of Utah Hospital and Clinics, Salt Lake City, Utah
Assistant Professor, Otolaryngology-Head and Neck Surgery, Wake Forest University School of Medicine; Staff Surgeon, Otolaryngology-Head and Neck Surgery, North Carolina Baptist Hospital; Assistant Professor, Regenerative Medicine, Wake Forest Institute for Regenerative Medicine, Winston Salem, �������������� North Carolina
Tympanoplasty—Staging and Use of Plastic; Laser Stapedotomy, Facial Nerve Tumors; Middle Fossa Approach
†Deceased
Rehabilitation of Lower Cranial Nerve Deficits after Neurotologic Skull Base Surgery
†Deceased
xvi
Contributors
Mark J. Syms, MD
P. Ashley Wackym, MD, FACS, FAAP
Neurologist, Section of Neurotology, Department of Neurosurgery, Barrow Neurological Institute, Phoenix, Arizona
John C. Koss Professor and Chairman, Department of Otolaryngology and Communication Sciences, Medical College of Wisconsin; Chief, Section of Otology, Division of Pediatric Otolaryngology, Children’s Hospital of Wisconsin; Chief, Department of Otolaryngology-Head and Neck Surgery, Froedtert and The Medical College of Wisconsin Hospital, Milwaukee, Wisconsin; ������������������� Vice President for Research, Legacy Health System, Portland, Oregon
Mastoidectomy—Intact Canal W all Procedure
Charles A. Syms III, MD Clinical Professor, Department of Otolaryngology-Head and Neck Surgery, University of Texas Health Science Center, San Antonio; President, Ear Medical Group, San Antonio, Texas
Mastoidectomy—Intact Canal Wall Procedure
Steven A. Telian, MD John L. Kemink Professor of Otorhinolaryngology; Director, Division of Otology, Neurology, and Skull Base Surgery; and Medical Director, Cochlear Implant Program, Department of Otolaryngology-Head and Neck Surgery, University of Michigan, Ann Arbor, Michigan
Retrolabyrinthine and Retrosigmoid Vestibular Neurectomy
Fred F. Telischi, MEE, MD, FACS Professor, Neurological Surgery, Biomedical Engineering, and Otolaryngology, University of Miami Miller School of Medicine; ������������������������������������������������ Vice Chairman and Director, University of Miami Ear Institute, Miami, Florida
Intraoperative Neurophysiologic Monitoring
Karen B. Teufert, MD House Ear Institute, Los Angeles, California
Congenital Malformation of the External Auditory Canal and Middle Ear; Transcochlear Approach to Cerebellopontine Angle Lesions
Spereotactic Radiosurgery of Skull Base Tumors
P. Daniel ������������������� Ward, MD, MS Resident, Department of Otolaryngology-Head and Neck Surgery, University of Michigan, Ann Arbor, Michigan
Retrolabyrinthine and Retrosigmoid V estibular Neurectomy
Frank M. Warren, MD Assistant Professor, Otolaryngology-Head and Neck Surgery, University of Utah, Salt Lake City, Utah
Facial Nerve Tumors
D. Bradley ������������������������������ Welling, MD, PhD, FACS Professor and Chair, Otolaryngology-Head and Neck Surgery, The Ohio State University, Columbus, Ohio
Surgery of the Endolymphatic Sac
Richard J. Wiet, MD Professor of Clinical Otolaryngology and Neurosurgery, Northwestern University, and Ear Institute of Chicago, Chicago, Illinois
Complications of Surgery for Chronic Otitis Media
Anders M.R. Tjellström, MD, PhD
Eric P. Wilkinson, MD
Department of Otolaryngology, Sahlgrenska University Hospital, Gőteborg, Sweden
Clinical Assistant Professor of Otolaryngology, University of Southern California Keck School of Medicine; Associate, House Clinic, Los Angeles, California
The Bone-Anchored Cochlea Stimulator (Baha)
Debara L. Tucci, MD Associate Professor, Division of Otolaryngology-Head and Neck Surgery, Department of Surgery, Duke University Medical Center, Durham, North Carolina
Extreme Lateral Infrajugular Transcondylar Approach for Resection of Skull Base Tumors
Canal W all Reconstruction Tympanomastoidectomy; Drainage Procedures for Petrous Apex Lesions; Management of Postoperative Cerebrospinal Fluid Leaks; Hypoglossal Facial Anastomosis
Acknowledgements Publication of a book of this scope requires a tremendous effort on the part of many, all of whom I wish to sincerely thank. First, my thanks to my lovely wife, Charlotte, who supports all my efforts and forgave my absence for the time devoted to this project. No less supportive are our four sons, David, Douglas, Mark, and Steven, who provide diversion and pleasure by taking me hunting and fishing. Since the publication of the second edition, four additional grandchildren have blessed Charlotte and me. Lauren, Nicholas, Sammy, Kaylee Daniel, and Megan provide immeasurable pleasure to us. What we have been told about the joy of grandparenting was underemphasized. My associate editors, Dr. Shelton and Dr. Arriaga, worked tirelessly to bring this volume to fruition. Anthony Pazos deserves special recognition. He spent countless hours in the temporal bone laboratory learning at first hand the various operations that he then illustrated. All the authors have appreciated his attention to detail and willingness to work with them until everything was “just right.” The publishers have been extremely supportive throughout the development of this book. From the beginning, they made a major commitment to ensure that this volume would be of the highest quality. I wish to thank particularly Rachel Yard and Scott Scheidt. Finally, I wish to thank our office staff, who work tirelessly on behalf of our patients and us. I would particularly like to thank Rita Koechowski, my surgery counselor, who not only schedules my surgery but also offers tremendous encouragement and support to all my patients. Her assistant, Patricia McGrath, has been a tremendous help to both of us in supporting our patients. Finally, I wish to thank my surgical assistants, Matthew Layner and Nancy Aguilar, and my executive assistant, Cathy Weise. She maintains my focus and orientation on a daily basis, and her efforts are greatly appreciated. Derald E. Brackmann, MD My family’s patience and support in this book and all my academic projects continues to motivate and encourage me. My wife Rosemary and children (Becca, Moi, and Toby) participated in this and my other professional activities through tolerating my time away, enduring family moves, and now even offering occasional editorial suggestions on substance and form. Moisés Agusto and Leticia are a continued inspiration. I particularly want to thank Derald Brackmann for his mentorship and stellar example of skill in otologic surgery and graceful balance of complex competing demands. The physicians and alumni of the House Ear Clinic provide an active network of otologic innovation whose concepts serve as a thread of continuity in this book. My Louisiana State University residents and partners have provided insightful questions and suggestions that have prompted some of the changes in this edition. Additionally, they have shown that even in the terrific upheaval of post-Katrina Louisiana, calm focus on basic principles ensures quality education and superb patient care as long as we remain flexible to use all resources available. Moisés A. Arriaga, MD, MBA, FACS
xvii
1
Otologic Instrumentation Jose N. Fayad, Barbara Stahl Hoskins, and James E. Benecke, Jr.
Sophisticated micro-otosurgical techniques mandate that the otologic surgeon and surgical team have an indepth understanding of the operating room (OR) layout and surgical instrumentation. This chapter describes in detail different surgical procedures. The OR setup and instruments necessary for the various types of otologic procedures are described. Appendix 1 provides a comprehensive list of instruments and equipment.
OPERATING ROOM The OR for otologic surgery requires features that differ from ORs used for nonotologic surgery. The following sections elaborate on the general environment of the OR designed for ear surgery. A word about the sterile field is in order. Respecting the sterile field is vital during routine otologic surgery, and takes on special significance during neurotologic procedures. Maintaining the proper environment means limiting traffic through the OR, and keeping the number of visitors to a minimum. It is preferable for observers to be in a remote room watching the procedures on video. Individuals allowed in the OR should be experienced in sterile technique and should wear jackets over scrubs so that all skin surfaces are covered (Fig. 1-1). Before entering the OR, the patient identifies the operative site. The correct ear is marked using a marking pen. The psychological environment of the OR must be respected because many otologic procedures are performed on awake patients under local anesthesia. Members of the surgical team and visitors must use discretion when making comments during surgery. The first piece of OR equipment to be discussed is the operating table. The surgeon must be comfortable while performing microsurgery. Adequate leg room under the table can be achieved with older OR tables by placing the patient 180 degrees opposite the usual position; in other words, the patient’s head is where the feet would normally be (Fig. 1-2). Newer electric tables easily accommodate the patient and surgeon. Because most otologists spin the OR table 180 degrees after the
induction of anesthesia, the new tables allow for spinning the table without unlocking it. Nonetheless, after the patient is properly positioned, the table must be firmly locked in place. All ORs are equipped with wall suction. Standard suction devices are acceptable for otologic surgery. It is preferable, however, to use a multiple-canister suction setup, minimizing the number of times the bottles must be emptied (Fig. 1-3). Suction systems have several locations where the amount of suction can be varied, but the surgeon should also employ a control clamp on the suction tubing on the sterile field (Fig. 1-4). The tubing that is attached to the suction tips and suction irrigators should be highly flexible. The readily available disposable tubing is not flexible enough for microsurgery, and places awkward torque on the surgeon’s hands. Suction setup problems are common in every OR. The prudent team troubleshoots the system in advance and has access to backup equipment. Electrocautery equipment should be in a ready-touse state on all procedures except perhaps stapes surgery. The patient must be properly grounded. It is advantageous to have unipolar and bipolar cautery on the field for all chronic ear and neurotologic procedures. Polytef (Teflon) tips are available for most cautery devices and are desirable. Surgeons have at their disposal a wide array of safe cautery devices, but they must be thoroughly familiar with these electric instruments before use. The surgical drill is another essential piece of equipment for otologic surgery. The vast array of available drills precludes an in-depth discussion of each system. Generally, otologic drills fall into two categories: air driven and electric. There are advantages and disadvantages to each type, and most surgeons have a distinct preference based on training and experience. For surgeons using air-driven drills, it is preferable to use a central source of nitrogen to power the drill, instead of using room tanks of the gas. Using a central source eliminates the need for changing tanks during long cases. High-speed drills capable of doing most of the bone work in the temporal bone include the Fisch, Midas Rex, and Anspach drill systems. These drills generally are
OTOLOGIC SURGERY
FIGURE 1-3. Multiple-canister suction setup.
FIGURE 1-1. Observer in jacket. FIGURE 1-4. Suction tubing with control clamp.
FIGURE 1-2. Operating table with patient’s head at foot of bed. FIGURE 1-5. Synergy microdrill for footplate work.
unsuitable for work in the middle ear, especially around the stapes footplate. For the latter purposes, a microdrill, such as the Skeeter drill or Synergy, is suitable (Fig. 1-5). Whatever drill is used in the middle ear, it must have a variable speed control and a wide array of drill bits. Most larger otologic drills are equipped with straight and angled handpieces. Most surgeons prefer straight handpieces for early gross removal of the mastoid cortex, switching to angled handpieces for working deeper in the temporal bone. The Anspach drill system has a handpiece that can be converted from straight to angled simply by rotating the connection. A full complement
of cutting and diamond burrs is mandatory. Figure 1-6 shows the Anspach drill system. Most drill systems have attachments that vary in shape, diameter, and length. It is the surgeon’s responsibility to be intimately familiar with the drill system and to have all of the attachments and burrs that might be needed. The otologic drill should be held in the hand like a pencil, with the hand resting comfortably on the sterile field. The side of the burr should be used to provide maximum contact between the bone and the flutes of the burr, affording safer and more efficient drilling (Fig. 1-7). The newer drills are remarkably reliable, but, similar
Chapter 1 • Otologic Instrumentation
FIGURE 1-6. Anspach drill system.
FIGURE 1-7. Proper holding of the drill.
to other tools, may malfunction. Drill systems require proper care and inspection before use. A backup system should be readily available. The introduction of the operating microscope revolutionized otologic surgery. Most otolaryngologists are familiar with the use of the microscope. Several brands of optically superior instruments are available; most are sufficiently similar to share the same general principles. The otologic surgeon must be familiar with the adjustments on the microscope, and must be prepared to troubleshoot the problems that may arise with the scope. The focal length of the objective lens is a matter of personal preference. Most otologists use a 200 mm or 250 mm objective. If a laser is attached to the microscope, one might consider a 300 mm objective. The objective lens should be selected, confirmed, and properly mounted before draping the microscope. Other adjustments, such as the most comfortable interpupillary distance, also should be done before the scope is draped. Par focal vision should be established so that the surgeon can change magnification without having to change focus. This is accomplished by first setting the diopter setting of both eyepieces to zero. The 40× magnification (or highest available setting) is selected. The locked microscope
is focused on a towel using the focus knob only. Without disturbing any of the settings, the magnification is now set at 6× (or the lowest available setting). The eyepieces are individually adjusted to obtain the sharpest possible image. The diopter readings are recorded for future use. The surgeon should have par focal vision when these appropriate eyepieces are used. The microscope should move easily. All connections should be adjusted so that the microscope does not wander by itself, yet permits movement to any position with minimal effort. Wrestling with the microscope during microsurgery is an extreme distraction. Proper posture at the operating table is crucial. To perform microsurgical procedures, rule number one is that the surgeon must be comfortable. The surgeon should be seated comfortably in a proper chair with the back support at the correct height. Both feet should be resting comfortably on the floor. Fatigue is avoided by assuming a restful position in the chair, rather than a rigid upright posture (Fig. 1-8). The overall OR setup for routine otologic surgery is shown in Figure 1-9. For neurotologic surgery, more space must be available for additional equipment. Middle cranial fossa procedures require some modifications to the OR setup (Fig. 1-10). Basically, the surgeon and the microscope trade places such that the surgeon is seated at the head of the table. Cooperation and careful orchestration between the surgeon, nursing personnel, and anesthesiologist are required for otologic surgery. The needs of the otologist are best served by having the anesthesiologist at the foot of the bed and the scrub nurse opposite the surgeon. Space for the facial nerve monitoring equipment and personnel is reserved.
STAPES SURGERY The following description of the instrumentation and operative setup for stapes surgery also provides information useful for other middle ear procedures. Under most circumstances, it is preferable to perform stapes surgery under local anesthesia, and surgeons who do so usually employ some type of preoperative sedation. Numerous regimens are available, and their description is beyond the scope of this text. If sedation is administered by the surgeon or nursing personnel, without the assistance of an anesthetist or anesthesiologist, the agents used should be short-acting and reversible. It is far safer for the patient to be psychologically prepared for the procedure than to be oversedated. At the House Clinic, the associates prefer to perform all local anesthesia cases (including stapes surgery) under monitored anesthesia care. This approach requires the presence of anesthesia personnel in the OR to sedate the patient, as is required for the operation, and to monitor vital functions. The surgeon is relieved from this duty, allowing total concentration on the microsurgery.
OTOLOGIC SURGERY
A
B
FIGURE 1-8. A, Proper posture for the surgeon. B, Wrong posture for the surgeon.
FIGURE 1-9. Usual otologic/neurotologic operating room setup.
About 30 minutes before the operation, the patient is brought to the preoperative holding area. If the surgeon routinely harvests a postauricular graft, this area is now shaved. A plastic aperture drape is applied to the operative site and trimmed so as not to cover the patient’s face (Fig. 1-11). An intravenous line is started, and the patient is now ready to go to the OR. When the patient is on the OR table, the monitors are placed on the patient by the nursing or anesthesia staff. Minimal monitoring includes pulse oximetry, automatic blood pressure cuff, and electrocardiogram electrodes. The ear and plastic drape are scrubbed
with an iodine-containing solution, unless the patient is allergic to iodine. A head drape is applied, and the ear is draped with sterile towels so as not to cover the patient’s face; this can be facilitated by supporting the drapes with a metal bar attached to the OR table, or by fixing the drapes to the scrub nurse’s Mayo stand (Fig. 1-12). The patient’s head is now gently rotated as far away from the ipsilateral shoulder as possible, and the table is placed in slight Trendelenburg position. These maneuvers increase the surgeon’s working room and help to straighten the external auditory canal (EAC). The EAC
Chapter 1 • Otologic Instrumentation
FIGURE 1-10. Operating room layout for middle fossa surgery.
FIGURE 1-11. Plastic drape applied for stapes surgery.
is gently irrigated with saline heated to body temperature. Vigorous cleaning of the canal is avoided until the ear is anesthetized. The local anesthesia is administered with a plastic Luer-Lok syringe that has finger and thumb control holes. A 11⁄2 inch, 27 gauge needle is firmly attached to the syringe. If the ear is injected slowly and strategically, excellent anesthesia and hemostasis can be achieved with a solution of 1% lidocaine with 1:100,000 epinephrine (e.g., 1:40,000). When using stronger concentrations of epinephrine, the patient’s blood pressure and cardiac status must be considered, in addition to the possibility of mixing errors. The canal is injected slowly in four quadrants starting lateral to the bony-cartilaginous junction. The final injection is in the vascular strip. If one routinely harvests fascia or tragal perichondrium, these areas are now injected. Before describing stapes surgical instruments, a few general comments are in order. All microsurgical
FIGURE 1-12. Patient draped in the operating room for stapes surgery.
instruments should be periodically inspected to ensure sharp points and cutting surfaces. The instruments for delicate work should have malleable shanks, enabling the surgeon to bend the instruments to meet the demands of the situation. If the surgeon prefers a total stapedectomy over the small fenestra technique, an oval window seal must be selected. If fascia is used, the tissue is harvested before exposing the middle ear. The tissue is placed on a Teflon block or fascia press to dry. If perichondrium is preferred, this may be harvested immediately before footplate removal. For the small fenestra technique, a small sample of venous blood is obtained when the intravenous line is started. This blood sample is passed to the scrub nurse and placed in a vial on the sterile field. Various ear specula should be available in oval and round configurations. Sizes typically range from 4.5 to 6.5 mm (Fig. 1-13). It is desirable always to work
OTOLOGIC SURGERY
through the largest speculum that the meatus permits, without lacerating canal skin. Some surgeons prefer to use a speculum holder for stapes and other middle ear procedures. The tympanomeatal flap is started with incisions made at the 6 and 12 o’clock positions with the No. 1, or sickle, knife. These incisions are united with the No. 2, or lancet, knife. This instrument actually undermines the vascular strip instead of cutting it. The strip is cut with the Bellucci scissors. The defined flap is elevated to the tympanic annulus with the large round knife, known as the large “weapon.” When properly identified, the annulus is elevated superiorly with the Rosen needle, and inferiorly with the annulus elevator,
FIGURE 1-13. Speculum array.
FIGURE 1-14. Stapes instruments
or gimmick. Figure 1-14 shows a typical set of stapes instruments, including suction tips. Adequate exposure usually requires removal of the bony ledge in the posterosuperior quadrant. This can be initiated with the Skeeter microdrill and completed with a stapes curette (Fig. 1-15). From this point on, the steps differ depending on the technique preferred by the surgeon. The diagnosis of otosclerosis should be confirmed on entering the middle ear, and a measurement should be taken from the long process of the incus to the stapes footplate with a measuring stick. The next step is to make a control hole in the footplate with a sharp pick-needle (Barbara needle) or the laser. The incudostapedial joint is separated with the joint knife, the tendon is cut with scissors or laser, and the superstructure is fractured inferiorly and extracted. For work on the footplate, the surgeon must have a variety of suitable instruments available. A stapedotomy can be created with a microdrill, laser, or needles and hooks. The 0.3 mm obtuse hook is useful for enlarging the fenestra. For total footplate extraction, a right angle hook or excavator (Hough hoe) is used. The harvested graft is guided into place with a footplate chisel. The prosthesis is grasped with a smooth alligator or strut forceps and placed on the incus. It is positioned on the graft, or into the fenestra, with a strut guide. The wire is secured onto the incus with a crimper, or wire-closing forceps. The McGee crimper is useful, especially if followed by a fine
Chapter 1 • Otologic Instrumentation
FIGURE 1-15. Stapes curette.
FIGURE 1-17. Rosen suction tubes with House adapter; Baron tubes.
TYMPANOPLASTY AND TYMPANOPLASTY WITH MASTOIDECTOMY
FIGURE 1-16. Crimpers and footplate hook.
alligator forceps for the last gentle squeeze. A small right angle hook may be necessary to fine-tune the position of the prosthesis (Fig. 1-16). Suction tubes for stapes surgery include Nos. 3 to 7 Fr Baron suctions plus Rosen needle suction tips (18 to 24 gauge) with the House adapter (Fig. 1-17). The Rosen tips are useful when working near the oval window, with the surgeon’s thumb off the thumb port. Ear packing after stapes surgery is accomplished with an antibiotic ointment to hold the flap in place. A piece of cotton suffices as a dressing, unless a postauricular incision has been made, in which case a mastoid dressing is applied. For all middle ear procedures, the surgeon should hold the instruments properly. The instrument should rest, like a pencil, between the index finger and thumb, allowing easy rotation around the shank. The hands should always be resting on the patient and the OR table. The middle and ring fingers should rest on the speculum so that the hand moves as a unit with the patient. Proper hand position and holding of instruments should afford the surgeon an unimpeded view (Fig. 1-18).
The preparation and draping for tympanoplasty with or without mastoidectomy are much the same as for stapes surgery. The major difference is the amount of hair shaved before draping. Usually, enough hair is shaved to expose about 3 to 4 cm of skin behind the postauricular sulcus. The plastic drape is applied to cover the remaining hair (Fig. 1-19). The patient is positioned on the OR table as described earlier. Whether the procedure is performed under local or general anesthesia depends on the extent of the surgery, the surgeon’s preference, and the desire of the patient. After appropriate sedation or induction of anesthesia, the ear and plastic drape are scrubbed with the proper soap or solution. Some surgeons place a cotton ball in the meatus if a perforation exists, preferring not to allow the preparation solution to enter the middle ear. The field is draped as described earlier, the head is rotated toward the contralateral shoulder, and the table is placed in slight Trendelenburg position (Fig. 1-20). The postauricular area, canal, and tragus (if necessary) are injected with 1% lidocaine with 1:100,000 epinephrine for local and general anesthesia cases. Most chronic ear procedures begin in a similar fashion. Through an ear speculum, vascular strip incisions are made with the sickle or Robinson knife and united along the annulus with the lancet knife. The vascular strip incisions are completed with a No. 64 or 67 Beaver blade. This same blade can be used to transect the anterior canal skin just medial to the bony-cartilaginous junction. The postauricular incision is made with a No. 15 Bard-Parker blade behind the sulcus. The level of the temporalis fascia is identified, and a small selfretaining (Weitlaner) retractor is inserted. The fascia is cleared of areolar tissue and incised. A generous area of fascia is undermined and removed with Metzenbaum scissors. The scrub nurse can assist by using a Senn retractor to elevate skin and soft tissues away from the
OTOLOGIC SURGERY
FIGURE 1-18. Proper holding of instruments as shown by Dr. William House.
FIGURE 1-19. Drape (3M 1020) applied for chronic ear surgery. FIGURE 1-20. Chronic ear surgery draping for local anesthesia.
fascia. The fascia is thinned on the Teflon block and dehydrated by placing it under an incandescent bulb, carefully monitoring its progress. The fascia may also be dehydrated by placing it on a large piece of Gelfoam and compressing this complex in a fascia press. Figure 1-21 shows the instruments used in the initial stages of chronic ear surgery. Continued postauricular exposure is obtained by incising along the linea temporalis with a knife or with the electrocautery. A perpendicular incision is made down to the mastoid tip. Soft tissues and periosteum are elevated with a Lempert elevator (Fig. 1-22), the vascular strip is identified, and a large self-retaining retractor is inserted. A very large retractor, such as an Adson cerebellar retractor with sharp prongs, is preferred. Next, under the microscope, the anterior canal skin is removed down to the level of the annulus with the large weapon. The plane between the fibrous layer of the drum remnant and the epithelium is developed with a sickle knife, and the skin is pulled free with a cup forceps. The
anterior canal skin is placed in saline for later use as a free graft. The ear canal is enlarged with the drill and suctionirrigators. An angled handpiece and medium to small cutting burr are used. Irrigation through the suctionirrigators is done with a physiologic solution such as TisU-Wol, lactated Ringer, or saline. Two large (3000 mL) bags of irrigant are hung and connected by way of a threeway stopcock to the delivery system (Fig. 1-23). For mastoidectomy surgery, the surgeon must have a full array of cutting and diamond burrs, and a complete set of suction-irrigators. It is advisable to have bone wax and absorbable knitted fabric (Surgicel) readily available. Cholesteatoma removal can be accomplished with middle ear instruments such as the gimmick, weapon, and fine scissors. Although the setup for closing and packing after chronic ear surgery varies with the specifics of the situation, a few generalities should cover most situations encountered by the otologist. To maintain the middle ear space, silicone elastomer (Silastic) sheeting works
Chapter 1 • Otologic Instrumentation
A
B FIGURE 1-21. A, Instruments for making a canal incision. B, Instruments for handling fascia.
FIGURE 1-22. Periosteal elevators.
well and is still readily available. This sheeting comes in various thicknesses, with and without reinforcement. For middle ear packing, absorbable gelatin sponge (Surgifoam) is the usual choice, soaked in saline or an antibiotic otic preparation. Surgifoam is also used to pack the EAC, although some surgeons prefer an antibiotic ointment, as described in the section on stapes surgery. For meatoplasty packing, 1-inch nonadhesive Curity packing
strip or nasal packing gauze is saturated with an antibiotic ointment and rolled around the tip of a bayonet forceps; this creates a plug that conforms to the new meatus and is easily removed (Fig. 1-24). Wound closure is accomplished in two layers with absorbable sutures. The skin is closed with a running intradermal suture of 1-0 polyglactin 910 (Vicryl) or polyglycolic acid (Dexon) on a cutting needle. Steri-Strips are
10
OTOLOGIC SURGERY
applied, and the wound is covered with a standard mastoid dressing. Some additional instruments that prove to be handy in many chronic ear procedures include an ossicles holder, Crabtree dissectors, Zini mirror, right angle hooks, and the House-Dieter malleus nipper (Fig. 1-25). It is impossible to describe instruments for every conceivable situation, but the foregoing should cover most of the needs of the otologist.
ENDOLYMPHATIC SAC SURGERY There are many well-described procedures on the endolymphatic sac. The purpose of this chapter is not to outline the surgical options, but rather to discuss the
FIGURE 1-23. Suction irrigation setup.
A FIGURE 1-24. A, Surgifoam packing. B, Meatoplasty packing.
methodology for performing sac surgery. The preparation and draping of the patient for endolymphatic sac surgery are essentially the same as for tympanoplasty with mastoidectomy surgery. In the preoperative holding area, the postauricular area is shaved, exposing at least 4 cm of skin behind the sulcus. Plastic adhesive drapes are applied, and the patient is transported to OR. Endolymphatic sac surgery is performed under general anesthesia. The field is scrubbed in the usual manner, and the patient is positioned as described for chronic ear surgery. This is a good time to mention briefly the use of intraoperative facial nerve monitoring and other forms of physiologic monitoring, including eighth cranial nerve and cochlear potentials. Many surgeons use facial nerve monitoring whenever the facial nerve might be in jeopardy. Electrodes for facial nerve monitoring or other forms of monitoring should be positioned before the preparation. After the preparation for endolymphatic sac surgery, the planned incision is injected with 1% lidocaine with 1:100,000 epinephrine. The incision is made 2 to 3 cm behind the sulcus. Periosteal incisions are made sharply or with the electrocautery. A Lempert elevator elevates soft tissues and periosteum up to the level of the spine of Henle. A House narrow (canal) elevator is used to delineate the EAC, and a large self-retaining retractor is inserted. With drill and suction-irrigator, a complete mastoidectomy is performed. The antrum is not widely opened, but is instead blocked with a large piece of absorbable gelatin sponge (Gelfoam) to prevent bone debris from entering the middle ear. Bone over the sigmoid sinus and posterior fossa dura is thinned with diamond burrs. The retrofacial air tract is opened widely to locate the endolymphatic sac. The sac is decompressed with a diamond burr. A stapes curette can be used to remove bone over the proximal sac. The occasional bleeding that occurs over the surface of the sac or surrounding dura is best controlled with bipolar cautery. Alternatively, unipolar cautery at a very low setting can be used. The cautery tip is touched to an insulated Rosen or gimmick that is in contact with the offending vessel (Fig. 1-26). Another
B
Chapter 1 • Otologic Instrumentation
FIGURE 1-25. Additional instruments used in chronic ear procedures
11
FIGURE 1-27. Endolymphatic sac instruments and materials.
(see text).
FIGURE 1-28. Drapes (3M 1000) applied for neurotologic surgery.
NEUROTOLOGIC PROCEDURES FIGURE 1-26. Insulated gimmick (top) and Rosen (bottom).
method used to control small areas of bleeding in endolymphatic sac and chronic ear surgery is to cover the area with pledgets of Gelfoam that have been soaked in topical thrombin. Before opening the sac, the wound is copiously irrigated with saline or bacitracin solution. Fresh towels are placed around the field. The sac is opened with a disposable Beaver ophthalmic blade (No. 59S, 5910, or 5920). The lumen is probed with a blunt hook or gimmick. The shunt tube preferred by the surgeon is now inserted. Thin Silastic sheeting (0.005 inch) can be used to fashion a shunt. Figure 1-27 shows the materials for the latter steps of endolymphatic sac surgery. As with chronic ear procedures, the wound is closed in layers, usually beginning with 2-0 chromic and finishing with 4-0 Vicryl or Dexon. A standard mastoid dressing is applied. This dressing either is prepared in the OR or is obtained as a prepackaged dressing (e.g., Glasscock dressing).
This section describes the OR layout for neurotologic procedures, the only exception being middle fossa surgery, which is discussed separately. For procedures involving intracranial structures, extraordinarily meticulous attention to detail is mandatory. The preparation for neurotologic surgery may begin the evening before surgery by having the patient wash his or her hair and scalp with an antiseptic shampoo. The day of surgery, the patient is seen by the surgeon in the holding area so that the ear to be operated on is positively identified. The surgical site is shaved so that at least 6 cm of postauricular scalp is exposed. The area is sprayed with an adhesive, and the plastic drapes are applied (Fig. 1-28). At the same time, the abdomen is shaved from below the umbilicus to the inguinal ligaments, in preparation for harvesting a fat graft. The fat donor site is surrounded by plastic drapes (Fig. 1-29). After anesthetic induction, a catheter is inserted, and arterial and central venous lines are placed when indicated. Electrodes for monitoring CN VII and VIII
12
OTOLOGIC SURGERY
FIGURE 1-29. Abdominal area prepared.
A
B
C FIGURE 1-30. A-C, Draping sequence for neurotologic surgery.
FIGURE 1-31. SK-100 Surgi-kit for holding instruments.
(and possibly other nerves) are positioned. The patient’s head is supported on towels or a “donut” as needed, and rotated toward the contralateral shoulder. The surgical sites are scrubbed, then blotted dry with a sterile towel. The areas are draped off with towels and then covered with plastic adhesive drapes (e.g., Steri-Drape, Ioban, Cranial-Incise). Some surgeons prefer to include another layer of towels around the cranial site, followed by either sheets or a disposable split sheet. It is important to have several layers of draping to prevent saturation of the drapes with fluids down to the level of the patient (Fig. 1-30). Because the scrub nurse must handle numerous items attached to tubes and cords, it is helpful to have fastened to the field a plastic pouch into which the drill, suction, and cautery tips can be placed (Fig. 1-31). Two Mayo stands are kept near the field: one for the neurotologic instruments and the other for the fat-harvesting tools (Fig. 1-32). The postauricular area is injected with the usual local anesthetic, and the plastic drape is cut away with scissors to expose the mastoid and lateral subocciput. As with other procedures, a skin incision is made, hemostasis is obtained, soft tissues and periosteum are elevated, and a large self-retaining retractor is inserted. Bone removal is accomplished using a drill and suction-irrigation. For neurotologic cases, bone removal is more extensive, exposing the sigmoid sinus and a considerable amount of posterior fossa dura behind the sigmoid. It is imperative that the surgeon has immediate access to bone wax and Surgicel. Many surgeons also insist on having immediate access to hemoclips and thrombin-soaked Gelfoam. The extent of bone removal varies depending on the surgeon’s preference and the nature of the procedure. Some surgeons decompress the sigmoid completely, whereas others leave a thin shell of bone over the sinus (Bill’s island). After appropriate bone removal, the retractor is removed, and the field is vigorously irrigated with bacitracin solution. Bacitracin solution can be prepared
Chapter 1 • Otologic Instrumentation
A
B FIGURE 1-32. A, Mayo stand setup for tumor. B, Mayo stand setup for fat graft.
FIGURE 1-33. Brackmann neurotologic instruments.
by dissolving 50,000 U of bacitracin in 1 L of normal saline. After wound irrigation, fresh towels are placed around the field. With a wound free of bone dust and debris, the dura can now be opened with a No. 11 Bard-Parker scalpel blade or with the tips of Jacobson scissors. The dura can be pulled away from underlying structures by using a corkscrew-like instrument included in some neurotologic instrument sets (Fig. 1-33). The subdural space is entered, taking care not to violate the arachnoid; this helps to avoid injury to vessels before adequate exposure. The dural flap is carefully developed with Jacobson scissors. Hemostasis is controlled with bipolar cautery. The arachnoid is carefully opened with a sharp hook or the
13
FIGURE 1-34. Brackmann fenestrated suction-irrigators.
tips of the scissors, allowing the egress of cerebrospinal fluid. Figure 1-33 shows the instruments for dural and arachnoid opening. After opening the arachnoid, one should switch to fenestrated (Brackmann) suction tips (Fig. 1-34). The cerebellum and other intracranial structures should be protected with moist neurosurgical cottonoids. A variety of cottonoids should always be on the stand. For vestibular neurectomy procedures, the plane between the cochlear and vestibular nerves can be developed with a blunt hook, or the gimmick. The nerve section itself can be completed with a sharp hook or microscissors (Fig. 1-35). The same instruments can be used to define the plane between an acoustic neuroma and the facial nerve. A sharp right angle hook palpates Bill’s bar and sections the superior vestibular nerve fibers along with the vestibulofacial fibers. After establishing the proper plane between the tumor and facial nerve, a blunt hook is used to continue the dissection, avoiding stretching of the facial nerve. Facial nerve monitoring has greatly assisted this part of the dissection. For small tumors, the previously mentioned technique might suffice for total tumor removal. Larger tumors are removed by gutting the tumor extensively, mobilizing the capsule, and removing the capsule in a piecemeal fashion; this is accomplished by morcellizing the tumor with a large crushing forceps, such as the Decker. The Urban rotary suction-dissector is used to extract the pieces (Fig. 1-36). Bayonet forceps direct the tumor into the suction port of the Urban suction-dissector. As the tumor is gutted, the capsule collapses and can be dissected from the brainstem. The Selector ultrasonic aspirator (Fig. 1-37) is another instrument that some surgeons prefer for gutting the tumor. Whatever tool is used, proper use of these sophisticated, potentially dangerous instruments must be learned from user manuals and appropriate training and courses. Hemostasis is vital during neurotologic surgery, and the surgeon must have immediate access to all possible items necessary to control bleeding from whatever the source. In addition to unipolar and bipolar cautery, bone wax and precut pieces of Surgicel should be on the Mayo stand. Microfibrillar collagen (Avitene) is another preferred hemostatic agent to have available. Pledgets
14
OTOLOGIC SURGERY
FIGURE 1-35. Hooks for neurectomy and tumor dissection.
FIGURE 1-36. Urban dissector.
FIGURE 1-37. Selector aspirator.
of Gelfoam soaked in topical thrombin are quite useful. Vascular clips and a reliable clip applicator are useful for controlling bleeding from the petrosal vein and its tributaries (Fig. 1-38). Infratemporal fossa and other approaches to the skull base are set up in much the same manner as has already been discussed. Incisions are generally long and may extend into the upper cervical region to access major
FIGURE 1-38. Clips and clip applicators.
neurovascular structures. Silastic vessel loops should be placed around these structures for control and easy identification. Ligatures of 0 silk and transfixion sutures of 2-0 silk need to be available for jugular vein ligation. Cardiovascular sutures (e.g., 5-0 and 6-0 polypropylene [Prolene]) should also be close by. The self-retaining retractors described earlier are usually insufficient for skull base surgery. The Fisch infratemporal retractor or pediatric rib retractor are better suited to these tasks, which often include anterior displacement of the mandible. If mandibulotomy is indicated, the appropriate oscillating saw needs to be available. Some instruments facilitate work on or near the facial nerve. For rerouting the facial nerve, bone is removed with a drill until an eggshell thickness remains. The remaining bone is gently removed with a stapes curette. The nerve can be mobilized with a dental excavator or microraspatory. If a segment of the nerve is to be excised, as in a facial neuroma, this should be done sharply with a fresh knife blade. Likewise, before any neurorrhaphy, the ends of the nerve and graft should be freshened. A 9-0 monofilament suture is used for nerve anastomosis.
Chapter 1 • Otologic Instrumentation
15
FIGURE 1-39. Nerve anastomosis equipment.
Also under the rubric of neurotologic surgery is cochlear implant surgery. Each presently available cochlear implant device has its own unique set of requirements and, possibly, instruments. The surgeon must have proper training and experience to perform cochlear implant procedures. He or she must have all of the necessary special equipment for electrode placement and internal receiver fixation (Fig. 1-40).
MIDDLE CRANIAL FOSSA SURGERY
FIGURE 1-40. Cochlear implant tools.
Appropriate needle holders and forceps must be available (Fig. 1-39). An alternative or adjunct to suturing is to use NeuraGen nerve guides. Before closing neurotologic and skull base wounds, abdominal fat is removed from the left lower quadrant, most of the dissection being done with electrocautery. The abdominal wound is closed (over a drain if necessary) in layers, with the skin being approximated with a running intradermal 4-0 Vicryl or Dexon suture. The fat is cut into strips and insinuated into the dural defect. Continuous lumbar drainage is rarely necessary to prevent cerebrospinal fluid leakage except in extensive intracranialextracranial resections. If the neck is opened, a suction drain is inserted into the depths of the wound before closure. Wounds are closed as in other otologic procedures and dressed with a standard mastoid dressing.
Middle fossa procedures are discussed separately from other neurotologic procedures because they involve a different OR setup and some different instruments. The most obvious deviation from other procedures is the position from which the surgeon operates. The surgeon and the microscope trade locations, so that the surgeon operates from the head of the bed facing caudally (see Fig. 1-10). As with other neurotologic procedures, middle fossa surgery is performed under general anesthesia. In the preoperative holding area, the ipsilateral scalp is shaved to a distance of 6 cm postauricularly and nearly to the midline of the head above the ear in the temporal fossa. Plastic adhesive drapes are applied, and the patient is taken to the OR. After anesthesia, the surgical site and plastic drapes are scrubbed and blotted dry. The area is covered with another plastic adhesive drape. Towels are positioned to block off the entire temporoparietal scalp, including the auricle and zygomatic arch. Sterile sheets complete the draping (Fig. 1-41). The abdomen is usually prepared as in other neurotologic surgeries. The incision is planned so that it begins in the preauricular incisura below the root of the zygoma. It extends cephalad to the area just above the superficial temporal
16
OTOLOGIC SURGERY
FIGURE 1-41. Patient draped for middle fossa surgery.
FIGURE 1-42. Adson periosteal elevator (“joker”).
line. A gentle curve facilitates exposure. Before the incision, as in other cases, the area is infiltrated with local anesthesia. The plastic drape is cut away to expose the skin. After the skin incision is made, the superficial temporal vessels are identified and ligated. After the temporalis fascia is identified, it is recommended that an inferiorly based temporalis muscle flap be created, instead of splitting the muscle. This flap is centered over the zygoma, is elevated from the calvaria, and is reflected caudally by suturing the end of the flap to the drapes. Preserving the muscle with its neurovascular bundle does not limit the surgeon’s exposure, and allows the use of this muscle if facial reanimation surgery should ever be necessary. The remaining temporalis muscle is reflected laterally, and a self-retaining retractor is inserted. A craniotomy is performed. The size of the bone flap removed is dictated by the amount of exposure necessary. For tumor removal, it is wise to err on the large side. The bone flap is carefully removed from the dura with an Adson periosteal elevator, or “joker” (Fig. 1-42). The bone flap is placed in bacitracin solution. The craniotomy
FIGURE 1-43. House-Urban middle fossa retractor.
edges are smoothed with a rongeur, and bleeding is controlled with bone wax. The joker is used to dissect the dura from the floor of the middle fossa. The surgeon is now ready to insert the House-Urban middle fossa retractor. The surgeon must be familiar with the mechanical workings of this device (Fig. 1-43). The retractor is locked under the bony edges of the craniotomy. The blade housing is positioned so that it allows good visualization of the field without placing excessive traction on the temporal lobe. This usually requires repositioning the retractor several times during the early stages of the dissection. Next, the retractor blade is inserted, and the extradural dissection proceeds. The blade can be tilted with the hand and advanced with the thumb, leaving the other hand free for suctioning. Bleeding can be troublesome from the floor of the middle fossa, especially near the middle meningeal artery. Bipolar cautery, bone wax, Surgicel, and other hemostatic agents should be readily available. The surgeon elevates the dura and temporal lobe until the arcuate eminence, superior petrosal sinus, and greater superficial petrosal nerve are visible. Bone over the internal auditory canal (IAC) and geniculate ganglion is removed with a large diamond burr. When the dura over the IAC has been completely skeletonized as far medially as the porus, the wound is irrigated with bacitracin solution, and fresh towels are placed around the field. The dura over the IAC is opened posteriorly (away from the facial nerve) with a sharp hook. For vestibular neurectomy, Bill’s bar is palpated with the same sharp hook that then transects the superior vestibular nerve. Fine microscissors (e.g., Malis, Jacobson) are used to remove a segment of the nerve in continuity with Scarpa’s ganglion. In a likewise fashion, the inferior vestibular and singular nerves are sectioned. For acoustic tumor removal, significantly more bone removal is required. Having established adequate exposure, the plane between the facial nerve and tumor is developed as in the translabyrinthine approach.
Chapter 1 • Otologic Instrumentation
17
and the middle fossa retractor is removed, allowing the brain to re-expand. The wound is irrigated again with bacitracin. Microplates are used to secure the bone flap in place (Fig. 1-44), and the wound is closed in layers, suturing the temporalis flap back to normal anatomic position. Some surgeons close the skin over a Penrose drain, which is removed the day after surgery. A mastoid dressing completes the closure.
CONCLUSION
FIGURE 1-44.
At the conclusion of the procedure, the defect over the IAC can be reconstructed by filling it with small pieces of muscle or abdominal fat and covering it with a small piece of the bone flap that has been cut and trimmed to an appropriate size. The field is inspected for hemostasis,
This chapter has provided a detailed description of the OR environment and instrumentation for most procedures that the otologist is likely to encounter. Although these descriptions do not exhaust all possibilities, they have proved to be satisfactory for many otologists. Appendix 1 lists instruments and equipment that have been presented in the text.
18
OTOLOGIC SURGERY
APPENDIX 1 INSTRUMENTS AND EQUIPMENT FOR OTOLOGIC SURGERY General Operating Room Equipment
1. 3M 1000 plastic aperture drapes 2. 3M 1020 aperture drapes 3. 3M Steri-Drape, Ioban drape, or Cranial-Incise drape 4. Pharmaseal preoperative skin preparation tray, No. 4480 5. Suction irrigation setup 6. Suction canisters 7. Electrocautery unit 8. Skytron operating table
Stapes Surgery 1. Assorted Farrior specula 2. Finger-control Luer-Lok syringe 3. 11⁄2 inch, 25 or 27 gauge needle 4. Small Weitlaner retractor 5. Sheehy fascia press 6. House cutting block 7. Scalpel, No. 15 Bard-Parker blade 8. Adson tissue forceps 9. Iris scissors 10. House-Baron suction tubes, No. 3 to 7 Fr 11. House suction tube adapter 12. Rosen suction tubes, 18 to 24 gauge 13. Sickle knife (No. 1 knife) 14. Lancet knife (No. 2 knife) 15. Robinson knife 16. Sheehy-House weapon (large and small) 17. Rosen needle 18. House elevator 19. Gimmick annulus elevator 20. House stapes curette 21. Incudostapedial joint knife 22. Bellucci scissors 23. Straight Barbara pick 24. Measuring struts, 4.0 to 5.0 mm 25. Measuring disk, 0.6 mm 26. Hough hoe 27. Obtuse, 30 degree, 0.25 mm hook 28. Pick, 0.3 mm, 90 degree 29. Strut guide 30. Footplate chisel 31. Skeeter drill; 1.0, 0.7, and 0.6 mm burrs 32. House strut forceps (nonserrated) 33. McGee wire closing forceps (crimper) 34. Antibiotic ointment 35. Cotton balls, Band-Aids, mastoid dressing 36. Speculum holder
Chronic Ear Surgery
1. Assorted Farrior specula 2. Finger-control syringe 3. 11⁄2 inch, 25 or 27 gauge needle 4. Small Weitlaner retractor 5. Large self-retaining retractor (Weitlaner, Adson cerebellar) 6. Scalpel, No. 15 Bard-Parker blade 7. No. 64 or 67 Beaver blade 8. House cutting block 9. Sheehy fascia press 10. House-Baron suction tubes, No. 3 to 7 Fr 11. Adson forceps 12. Iris scissors 13. Small Metzenbaum scissors 14. Sickle knife 15. Lancet knife 16. Robinson knife 17. Sheehy-House weapon (large and small) 18. Rosen needle 19. Gimmick 20. Crabtree dissector (large and small) 21. Lempert elevator 22. House narrow elevator 23. Pick, right angle, 0.6 mm 24. Pick, right angle, 1.5 mm 25. Pick, right angle, 3 mm 26. Bellucci scissors 27. Hartmann forceps 28. House alligator forceps 29. House cup forceps 30. House-Dieter malleus nipper 31. Zini mirrors 32. Sheehy ossicles holder 33. Speculum, endaural (or nasal) 34. Drill with cutting and diamond burrs 35. House suction-irrigators, No. 2.5 × 4 Fr through No. 8 × 12 Fr 36. Needle holder, Webster 37. Suture scissors 38. Suture, 2-0 chromic and 4-0 Vicryl (or Dexon) 39. Surgifoam (saline-soaked and antibiotic-soaked) 40. Curity Packing strip gauze 41. Silastic sheeting, 0.005 42. Gelfilm 43. Steri-Strips 44. Mastoid dressing 45. Bone wax 46. Surgicel 47. Sheehy bone dust collector
Chapter 1 • Otologic Instrumentation
Endolymphatic Sac Surgery
1. Finger-control syringe 2. 11⁄2 inch, 25 or 27 gauge needle 3. Scalpel, No. 15 Bard-Parker blade 4. Large self-retaining retractor 5. Lempert elevator 6. House narrow elevator 7. Drill and burrs 8. House suction-irrigators (assortment) 9. Brackmann suction-irrigators, No. 4 × 5 Fr, No. 5 × 7 Fr 10. Stapes curette 11. Gimmick 12. Insulated gimmick 13. Bone wax 14. Surgicel 15. Bipolar cautery 16. Bacitracin irrigation solution 17. Beaver ophthalmic blade (No. 59S, 5910, 5920) 18. Pick, right angle, 1.5 mm 19. Hook, right angle, blunt 20. Rosen needle 21. House alligator forceps 22. Silastic shunt material, 0.005 23. Suture, 2-0 chromic and 4-0 Vicryl (or Dexon) 24. Steri-Strips 25. Mastoid dressing 26. Cranial nerve monitoring equipment
Neurotologic Surgery 1. Finger-control syringe 2. 11⁄2 inch, 25 or 27 gauge needle 3. Scalpel, No. 15 Bard-Parker blade 4. Large self-retaining retractor 5. Lempert elevator 6. House narrow elevator 7. Drill and burrs 8. Assorted House suction-irrigators 9. Assorted Brackmann suction-irrigators 10. Stapes curette 11. Gimmick 12. Insulated gimmick 13. Bone wax 14. Surgicel 15. Bipolar cautery 16. Bacitracin irrigation 17. SK-100 Surgi-Kit (Ethox Corp.) 18. Suture scissors 19. House-Urban dissector 20. Pick, right angle, 1 mm 21. Pick, right angle, 1.5 mm 22. Hook, right angle, blunt, 1.5 mm 23. Bellucci scissors
19
24. House cup forceps 25. Blakesley nasal forceps (No. 1) 26. House alligator forceps 27. Myringoplasty knife 28. Jacobson scissors 29. Malis scissors 30. Allis forceps 31. Bayonet forceps 32. Adson tissue forceps 33. Microclip applicator 34. Assorted hemostats 35. Metzenbaum scissors 36. Senn retractor 37. U.S. Army retractor 38. House-Urban rotary dissector or Selector 39. Fisch infratemporal fossa retractor 40. Woodson elevator 41. Fisch microraspatory 42. Sagittal saw 43. Needle holder, Castroviejo 44. Needle holder, Crile-Wood 45. Needle holder, Webster 46. Fisch microscissors 47. Titanium needle holders, smooth (2) 48. Janetta forceps 49. Gerald forceps, with and without teeth 50. Avitene 51. Drains, Penrose and Jackson-Pratt 52. Vessel loops 53. Suture, 5-0 and 6-0 vascular Prolene 54. Suture, 0 and 2-0 chromic 55. Suture, 0 and 2-0 silk 56. Suture, 9-0 nylon or Prolene 57. Suture, 4-0 Dexon or Vicryl 58. Neurosurgical cottonoids 59. NeuraGen nerve guide 60. Steri-Strips 61. Mastoid dressing 62. Topical thrombin 63. Surgifoam 64. Special neurotologic instrument sets (e.g., Kartush, Benecke) 65. Cranial nerve monitoring equipment
Middle Cranial Fossa Surgery 1. Finger-control syringe 2. 11⁄2 inch, 25 or 27 gauge needle 3. Scalpel, No. 15 Bard-Parker blade 4. Large self-retaining retractor 5. Lempert elevator 6. House narrow elevator 7. Drill and burrs 8. Assorted House suction-irrigators 9. Brackmann suction-irrigators 10. Stapes curette
20
OTOLOGIC SURGERY
11. Gimmick 12. Insulated gimmick 13. Bone wax 14. Surgicel 15. Bipolar cautery 16. Bacitracin irrigation 17. SK-100 Surgi-Kit 18. Pick, right angle, 1 mm 19. Pick, right angle, 1.5 mm 20. Hook, right angle, blunt, 1.5 mm 21. Bellucci scissors 22. Fisch microscissors 23. House cup forceps 24. Metzenbaum scissors
25. House-Urban middle fossa retractor 26. Rongeur, Leksell 27. Adson tissue forceps 28. Microclip applicator 29. Assorted hemostats 30. Avitene 31. Cottonoids 32. Gelfoam 33. Suture, 0 and 2-0 chromic 34. Suture, 4-0 Vicryl (or Dexon) 35. Topical thrombin 36. Mastoid dressing 37. Special neurotologic instrument sets 38. Cranial nerve monitoring equipment
2
Canalplasty for Exostoses of the External Auditory Canal and Miscellaneous Auditory Canal Problems Joseph B. Roberson, Jr. and Rodney Perkins Videos corresponding to this chapter are available online at www.expertconsult.com.
Although clinical disease caused by exostoses of the external auditory canal (EAC) is infrequent, it occurs often enough that a method of surgical management should be in the armamentarium of the otologic surgeon. Because it is not a high-incidence problem or one that is life-threatening, many otolaryngologists use various independent approaches, which frequently result in elimination of or damage to the canal skin. These procedures frequently produce suboptimal results. A well-conceived approach addresses the problem of removal of exostoses, while maintaining the valuable residual skin of the EAC. This chapter begins with clinical observations regarding this condition and then describes an operative procedure that has been very successful in its management. The etiology of these benign growths of the tympanic bone is strongly associated with the frequency and severity of exposure to cold water.1 Frequently, these lesions are found in surfers, swimmers, or other individuals with frequent cold water exposure over several years. A widely held belief based on clinical information is that exostoses occur primarily during the years of growth, with their proliferation being enhanced or perhaps even caused by exposure to cold water during this period. This belief tends to be supported by historical information from patients with exostoses, who almost always indicate that they swam in cold water during their youth.2-4 This historical information is strongly corroborated by the high incidence of exostoses in avid surfers who spend hours in the water almost daily. In our clinical experience, this problem occurs almost exclusively in men, who are more likely than women of the same age to have had frequent cold water exposure during their youth. Most exostoses do not develop to a degree sufficient to cause clinical symptoms. Patients are frequently referred to otologists because the growths are observed, and not understood, by primary care physicians. This is particularly true with exostoses that have a more pedunculated form than the more subtle sessile configuration. When exostoses become more marked, however, they obstruct the natural elimination of desquamated epithelium from the ear canal, and patients usually present with recurrent episodes of external otitis. In their most prolific expression, exostoses can lead to hearing impairment by causing the collection
of epithelial debris that tamponades tympanic membrane movement, by impinging on and limiting the mobility of the malleus, or by markedly narrowing the aperture of the canal. These conditions may manifest as a conductive hearing impairment on audiometric examination. The EAC is part of the hearing pathway. Essentially, the EAC is a tube with resonant characteristics that amplify the incoming sound. The degree of amplification and the frequency at which it occurs are a function of the diameter and the length of the canal. When the diameter becomes small, it can interfere with the passage of sound and cause a hearing impairment. This effect does not become significant, however, until the aperture becomes very small. With apertures less than 3 mm, high-frequency sounds begin to diminish, and further compromise of the channel diameter results in increased impairment and lower frequency loss.
EXOSTOSES OF THE EXTERNAL AUDITORY CANAL Surgical Indications Surgery is indicated when chronic or recurrent external otitis exists, or a conductive hearing impairment develops. The presence of chronic and recurrent infection over an extended period seems to debilitate the canal skin, and can compromise the skin’s ability to re-epithelialize in a robust and healthy manner in the postoperative period. For this reason, surgical therapy should be considered when a pattern of recurrent external otitis has been established in these patients. Patients who have significant external canal exostoses without recurrent infection or hearing impairment should be observed periodically, and surgery should be avoided until these symptoms occur.
Preoperative Preparation Patient Preparation There are two components of patient preparation for otologic surgery performed under local anesthesia: psychological and pharmacologic. 21
22
OTOLOGIC SURGERY
Psychological Preparation To reduce anxiety and create rapport, the surgeon should provide the patient with a full explanation of the procedure and its objectives, benefits, and risks. In addition, a surgical nurse or medical assistant should explain what will happen to the patient in the operating room, and describe such things as the operating room environment, use of an intravenous line for medication delivery, placement of monitor electrodes, and draping. By informing the patient of these things and making him or her part of the process, the clinician reduces the patient’s anxiety, encourages cooperation, and may reduce bleeding. Beyond the technical advantages achieved by such preparation, there is an ethical responsibility to inform the patient. In addition, the likelihood of the patient’s becoming litigious because of a poor result is markedly reduced if he or she has been informed about the procedure and its risks and benefits, and has had an opportunity to discuss the risks and benefits with the surgeon before the surgery. Pharmacologic Preparation The pharmacologic preparation of the patient can be achieved in many ways. In the average adult who selects intravenous sedation with local anesthesia, we give fentanyl, 50 to 100 μg, and midazolam (Versed), 5 to 10 mg intramuscularly 1 hour before the surgical incision. An intravenous catheter is started in the arm opposite the ear to be operated on before the patient arrives in the operating room, and 5% dextrose in Ringer solution is started with a volutrol. Unless the patient appears very sedated, an additional 50 to 100 μg dose of fentanyl is placed in the volutrol and infused slowly over 30 to 45 minutes. As the surgery proceeds, alternating supplements of intravenous midazolam and fentanyl are infused as needed to maintain sedation. In most cases, general anesthesia is selected by either the surgeon or the patient. Patients with a history of claustrophobia, patients with poor language skills in the surgeon’s native tongue, and patients with difficult neck mobility are best approached under general endotracheal anesthesia. One advantage of use of general anesthesia is the ability to use facial nerve monitoring during the procedure. Patients receive cefazolin, 1 g intravenously, or another appropriate antibiotic 1 hour before incision.
Site Preparation The hair is shaved behind the ear to a distance of approximately 1.5 inches posterior to the postauricular fold. The auricle and the periauricular and postauricular areas are scrubbed with povidone-iodine (Betadine) solution or chlorhexidine gluconate (Hibiclens) for iodineallergic patients. A plastic drape is placed over the area with the auricle and the postauricular area exteriorized through the opening in the drape. This drape is placed
over an L-shaped bar that is fixed in the rail attachment of the operating table (Fig. 2-1). For patients under local anesthesia with sedation, a small, low-volume office fan is attached to the bar to provide a gentle cooling breeze to the patient’s face during the procedure. The plastic drape forms a canopy, allowing the patient to see from under the drape and reducing the feeling of claustrophobia. In addition, a foam earpiece from an insert speaker is put into the opposite ear. The earpiece is connected to a compact disk player and input microphone that allows the patient to listen to relaxing music and provides a pathway to converse with the patient, if desired.
Analgesia It is important not only to achieve analgesia, but also to maximize canal hemostasis with injections into the external auditory meatus. Using 2% lidocaine (Xylocaine) with 1:20,000 epinephrine solution in a ringed syringe with a 27 gauge needle, a classic quadratic injection is made such that each injection falls within the wheal of the previous injection. Another useful injection is an anterior canal injection, which is made with the bevel of the needle parallel to the bony wall of the external meatus (Fig. 2-2). In a patient with extensive exostoses, this injection is usually made into the lateral base of a large anterior sessile osteoma. After insinuation of the needle, it is advanced a few millimeters, and a few drops are injected extremely slowly. The solution infiltrates medially along the anterior canal wall and provides some analgesia to the auriculotemporal branch of CN V, which is usually unaffected by the quadratic injection and adds to the hemostasis anteriorly. The postauricular area is infiltrated with 2% lidocaine with 1:100,000 epinephrine solution mixed with equal parts of 0.5% bupivacaine.
Surgical Technique Most surgical approaches for removal of EAC exostoses are through the transmeatal route.5-7 This approach has two disadvantages. It usually results in significant loss of the remaining canal wall skin through damage by the drill, and it does not allow adequate visibility or instrument and drill access to remove the medial portion of the exostotic mass near the tympanic membrane safely. A large sessile anterior exostosis is almost uniformly present in these patients (Fig. 2-3). The approach described here is primarily postauricular and one that maximizes conservation of the canal wall skin and facilitates careful removal of the anterior exostosis, which is usually extremely close to the tympanic membrane. A curvilinear postauricular incision is made approximately 1 cm behind the postauricular fold (Fig. 2-4). The skin and subcutaneous tissues are elevated anteriorly to the area of the spine of Henle and the bony posterior canal, and a toothed, self-retaining retractor is placed (Fig. 2-5).
Chapter 2 • Canalplasty for Exostoses of the External Auditory Canal
Locating this area is facilitated by finding the plane of the lateral surface of the inferior border of the temporalis muscle and dissecting in this plane anteriorly to reach the meatus. When this area is reached, the skin overlying the lateral slope of the posterior exostosis is elevated from its
23
surface, and a Perkins bladed tympanoplasty retractor is inserted to hold elevated skin off the surface of the lateral portion of the bony mass (Fig. 2-6). Although there may be more than one posterior and anterior exostosis, predominant anterior and posterior
24
OTOLOGIC SURGERY
Chapter 2 • Canalplasty for Exostoses of the External Auditory Canal
exostoses are usually present along with others of lesser mass. These secondary masses may be handled similarly to the primary exostoses, or may be removed directly. To simplify the description here, this operation is divided into two major segments: removal of the posterior exostosis, and removal of the anterior exostosis.
Removal of Posterior Exostosis By use of a medium-sized cutting burr and an appropriately scaled suction-irrigator, the posterior exostosis is entered along its lateral sloping edge, and the bony removal is progressed medially, keeping a shell of bone over the area being burred anteriorly (Fig. 2-7). The remaining skin over the exostosis medial to the skin elevated earlier is protected from the burr. As this shell becomes thinner, it is advisable to switch to a diamond burr to prevent a sudden breakthrough to the skin, which might occur if one continues with the cutting burr on the excessively thinned bone. The bone removal is continued medially and posteriorly until the estimated normal posterior canal contour and dimension is achieved. As one approaches a medial depth consistent with the posterior annulus of the tympanic membrane (which usually cannot be seen directly at this point), care must be taken to avoid damage to the chorda tympani nerve and the posterior aspect of the tympanic membrane. The surgeon should also keep in mind that some patients’ facial nerve exists lateral to the tympanic annulus at its posteroinferior border. Facial nerve monitoring reduces the possibility of injury to the nerve in a patient unable to tolerate local anesthesia. The thinned bony shell is collapsed, and a small elevator reveals the inside surface of the posterior canal skin that was over the exostosis (Fig. 2-8). An incision is made midway along the posterior canal skin perpendicular to the long axis of the EAC (Fig. 2-9). The posterior canal skin medial to this incision is positioned onto the new contour of the posterior canal wall (Fig. 2-10). The transmeatal approach is then taken, and incisions are made with a sickle knife superiorly and inferiorly in the canal, extending from the ends of this previous incision laterally to the meatus, and creating a laterally based posterior canal skin flap. This flap is involuted back into the meatal portion of the canal and held there with the Perkins retractor (Fig. 2-11). Attention is turned to the anterior exostosis, which has now been revealed.
Removal of Anterior Exostosis By use of a round knife, an incision is made in the skin overlying the anterior exostosis from superior to inferior over the dome of the exostosis and as far medially as can be seen. This incision is connected to the incisions previously made superiorly and inferiorly in the canal that defined the posterior canal skin flap, and this anterior canal flap is elevated laterally (Fig. 2-12). Frequently,
25
the skin of the vascular strip can be left intact if the exostoses do not involve this portion of the canal. By use of a back-angled Perkins tympanoplasty elevator, this laterally based anterior canal skin flap is elevated further to the cartilaginous portion of the anterior canal and is smoothed so as to lie laterally near the posterior canal flap under the retractor (Fig. 2-13). With a cutting burr and small suction-irrigator, the anterior exostosis is removed in a manner similar to that of the posterior one, and a thin shell of bone that protects the canal skin is left over the anteromedial portion of the exostosis from the burr (Fig. 2-14). This bone removal is continued to the area of the anterior annulus of the tympanic membrane. The bony shell is collapsed and removed, leaving the intact anterior canal skin (Fig. 2-15). Usually, it is necessary to finish up and smooth an edge of bone that remains at the anterior extent of this dissection to have a smooth contour near the annulus area. To protect the elevated anterior sulcus skin from the burr, a small tympanic membrane–sized piece of silicone elastomer (Silastic) or suture packet foil is placed on the inside surface of the anterior canal skin to hold it against the tympanic membrane during drilling. This prevents the skin flap from getting involved with the burr, and prevents damage to the tympanic membrane that might occur with the burr being used in such close proximity to the membrane. Subsequently, the Silastic is removed, the medial anterior canal skin is placed on the bone, and all skin flaps are folded back into position on the new contours of the bony canal (Fig. 2-16). The medial flaps are packed into place with chloramphenicol (Chloromycetin)-soaked absorbable gelatin sponge (Gelfoam) pledgets, and the postauricular incision is closed with interrupted subcuticular 4-0 polyglactin 910 (Vicryl) suture. Through the transmeatal route, the laterally based canal skin flaps are packed into place with Gelfoam pledgets. A cotton ball is placed in the meatus, and a mastoid dressing is applied. The patient is returned to the outpatient recovery area and discharged after appropriate recovery.
Postoperative Care The patient is instructed to remove the mastoid dressing the next morning. The Gelfoam packing is removed using the stereomicroscope on the first office visit 1 week later. Antibiotic-steroid ear drops are prescribed for use twice daily for 1 week and once every 3 days for another 2 to 3 weeks. The second postoperative visit is at 1 month. If there is no evidence of infection, no additional ear drops are recommended. Because most of the patients in whom this procedure is done have had recurrent external otitis, and because time is needed for epithelialization of uncovered bone, the ear canal may remain moist for a longer time than in a typical tympanoplasty. Until the ear canal is completely dry and healed, the patient should be
26
OTOLOGIC SURGERY
Chapter 2 • Canalplasty for Exostoses of the External Auditory Canal
27
28
OTOLOGIC SURGERY
seen every few weeks to inspect and clean debris from the canal as needed. The canal skin has usually been exposed to numerous infections and has been stretched over the exostoses; it may not be as resilient as normal canal skin. Return to water exposure should be avoided until 1 to 2 months after complete healing has occurred. Frequently, avid surfers return to the water much sooner than instructed, however. Antibiotic drops given after water exposure reduce the risk of early postoperative infection. If the patient is still in the growth years, further repeated exposure of the ear to cold water should be moderated. The bone may reproliferate under these conditions, and further surgery may become necessary. In patients who want to return to frequent surfing or similar water exposure, earplugs should be worn to prevent water entrance. This problem lessens in older surfers because they may be beyond their rapid growth phase, and the economic exigencies of life tend to decrease their frequency of exposure. It is advisable to see the patient annually for 2 years to assess the tendency for the problem to recur, although recurrence is infrequent.
Problems and Complications Although this procedure is not fraught with serious complications, complications can occur during several aspects of the operation. As the medial extent of the canal is approached in the removal of the posterior exostosis, the course of the chorda tympani nerve must be kept in mind. This portion of the bone removal is done largely without definite landmarks: the surgeon must rely on mental estimation of the distances in arriving at the posterior annulus. The chorda tympani nerve is beneath the bone near this field of dissection and could sustain damage. Also, it is important to remember the course of the facial nerve, which passes posterior and inferior to the canal, although this area is farther from the immediate area of dissection than the chorda tympani nerve. When a burr is used very near the tympanic membrane and the malleus, a diamond burr should be used because it is less likely to run erratically than the cutting burr.
Summary Exostoses of the EAC usually manifest without attendant compromise in function or clinical disease. When recurrent external otitis or hearing impairment results, however, surgical removal is indicated. Canalplasty has significant advantages over commonly employed transmeatal approaches by maximizing conservation of canal skin and providing surgical access to the anterior medial zone of the canal. Complications are infrequent, but attention to the anatomy of the chorda tympani and facial nerve pathways and careful drill technique in the area of the tympanic membrane are important. Although surgical techniques involving the EAC have had little attention compared with other reconstructive
procedures, they should be in the armamentarium of all otologic surgeons. This technique has proved to be effective for the management of exostoses of the EAC.
MISCELLANEOUS EXTERNAL AUDITORY CANAL CONDITIONS Medial Third Stenosis For unknown reasons, some patients develop weeping epitheliitis over the medial third of the EAC. Treatment consists of antibiotic-steroid ear drops that supply broadspectrum bacterial coverage. Intense treatment, including débridement and the use of topical agents, is usually necessary to bring the process under control. Despite attempts at treatment, progression of the condition may follow a relentless course, resulting in dense fibrosis of the medial segment of the EAC with conductive hearing loss. The mesotympanum and ossicular chain are characteristically spared. Surgical repair may be necessary when conductive hearing loss produces a functionally significant deficit for the patient. Successful repair is frequently possible, although restenosis may occur, and this possibility should be included in the informed consent. Technically, a postauricular approach is used to allow complete resection of the fibrotic segment medial to noninvolved EAC skin where an incision has been previously created working through a transcanal route (Fig. 2-17). Removal of most of the fibrous layer of the tympanic membrane seems to reduce the chance of postoperative restenosis. Tympanoplasty is performed with a lateral graft or fasciaform technique. Coverage of the resultant exposed bone is mandatory and is provided with a free split-thickness skin graft. The posterior surface of the pinna provides skin of appropriate character within the operative field and can be taken with a No. 10 blade. Skin grafts should overlap the fascia used for tympanic membrane replacement, but should not extend to cover the lateral surface of the reconstructed drum. Antibiotic-containing absorbable packing is removed 7 to 14 days later and antibiotic-steroid ear drops are continued for 2 weeks beyond healing to be tapered over time. Close observation postoperatively is necessary to intervene with any signs of restenosis. Recurrent epitheliitis may occur months or years after successful repair.
Collapsing Canal Stenosis of the cartilaginous portion of the lateral EAC may produce symptoms for some patients. In severe cases, conductive hearing loss may result when closure to less than 2 mm occurs. More commonly, accumulation of debris and a warm, moist environment lead to recurrent external otitis. Although this condition occurs naturally, an iatrogenic component is frequently present. After a postauricular incision, the natural tension of the
Chapter 2 • Canalplasty for Exostoses of the External Auditory Canal
Canal incision
29
Tympanic membrane and Fibrosis
Noninvolved canal skin
M I S
FIGURE 2-17.
cartilaginous canal may be unopposed by inadequately reapproximated deep layers such as the mastoid periosteum. Gradual stenosis may occur until symptoms become evident many years after the surgical procedure. Operative repair includes removal of cartilage from the anterior concha and posterior cartilaginous canal from the postauricular area with imbrication of the deep tissue layers overlying the mastoid cortex, similar to imbrication of the subcutaneous musculoaponeurotic system in a facelift. The skin of the ear canal need not be violated in such a procedure. Postoperative stenting for 2 weeks also is helpful in restoring a normal contour to the canal. In some patients with only lateral soft tissue and cartilage involvement, the stenosis may be addressed via a transcanal route, avoiding a postauricular incision.
Keratosis Obturans Exuberant accumulation of desquamated skin may produce bony erosion and gradual expansion of the bony EAC.8 The process may progress to the point of erosion into structures adjacent to the canal, such as the temporomandibular joint or mastoid. Erosion lateral to the eardrum may cause loss of support of the fibrous annulus of the tympanic membrane and a characteristic “jump rope sign” inferiorly (which can also be seen after curetting for a stapes procedure more superiorly). Poor epithelial migration has been proposed as the cause of the disorder. Frequent cleaning may retard the process. Cleaning may be much easier if the typically inspissated and adherent material is softened with mineral oil for several days before the clinical appointment. Surgical intervention is rarely indicated, unless severe erosion exposes vital structures.
Osteonecrosis and Osteoradionecrosis of the Tympanic Bone Radiation and occasionally chronic vasculitis devascularize a portion of the tympanic bone, producing skin loss and bone exposure. The low-grade osteomyelitis can be managed conservatively with topical antimicrobials and mild débridement. Addition of oral antibiotics may improve the chance of healing lesions in the early phases. Frequently, bone involvement progresses, however, and can lead to further skin loss. A culture and sensitivity test is indicated before institution of topicals and later with deterioration of healing to look for resistant organisms. One must always consider malignancy in such a clinical situation, and biopsy is prudent in many cases. Operative repair is indicated for progression of bone exposure or associated cellulitis or both. Removal of all devitalized bone with the postauricular approach is necessary. The margins of the canal skin are freshened similar to what is performed in a tympanoplasty. Autogenous fascia is placed directly on the freshly drilled bone, and the skin is returned to anatomic position overlying it. The external canal is packed with antibiotic-containing absorbable sponge, which is removed in 7 to 10 days when antibiotic drops are initiated, which are continued until complete healing occurs.
Scutum Defects Cholesteatoma of the pars flaccida produces bone erosion in many patients. Repair of the EAC is necessary to prevent re-retraction and cholesteatoma formation through the canal defect. Small defects (24 >24 3-44 12 1-48 6-76 1
27 36 58 26 14 43 44 7 43 48 40
35 50 34 NA NA 36 NA NA 27 16 NA
54 66 64 NA NA 34 NA NA 36 16 NA
Retrosigmoid Vestibular Nerve Section Author, Year Glasscock et al, 199142 Molony, 199647 Pareschi et al, 200248 Fukuhara et al, 200249 Miyazaki et al, 200526
Follow-up Time (mo)
Patients with Hearing Loss (%)
Tinnitus (% Improved)
Fullness (% Improved)
>24 24-42 >48 19-65 3-132
45 10 7 7 0
32 NA 10 NA NA
39 NA 21 NA NA
Combined Retrolabyrinthine/Retrosigmoid Vestibular Nerve Section Author, Year Silverstein and Jackson, 200246 Goksu et al, 200550
Follow-up Time (mo)
Patients with Hearing Loss (%)
Tinnitus (% Improved)
Fullness (% Improved)
1
20
NA
NA
18-24
10
NA
NA
NA, not applicable. *Series with >25 patients; only nonoverlapping data from a single group or institution are included.
approach is reported to have a success rate of 92% to 95% for patients with Meniere’s disease. No data are available for patients with other causes of vertigo. A challenge in interpretation of these data is the natural course of Meniere’s disease, which is often a progres sion to spontaneous complete relief of vertigo even if no treatment is provided.29-32 The advantage of selective vestibular neurectomy over destructive labyrinthine procedures is division of the vestibular fibers with preservation of the cochlear fibers. The ability of the procedure to preserve hearing is an important consideration. As mentioned earlier, ves tibular neurectomy does not change the course of the hearing loss, but is designed only to treat the immedi ately debilitating symptom of vertigo. In addition, ves tibular neurectomy itself introduces a risk of hearing loss secondary to the surgery. Lack of a distinct cleavage plane between the vestibular and cochlear nerve fibers is the primary cause of damage to auditory fibers. This risk may be theoretically decreased using the retrosigmoid
approach, owing to surgical division of the vestibular nerve more laterally, where the cleavage plane becomes more distinct. Medial retraction of the auditory nerve may also lead to a sensorineural hearing loss. In addition, a conductive hearing loss may occur later from fixation of the ossicular chain caused by bone dust that accu mulates in the oval window during the procedure.33,34 A study by Teixido and Wiet34 showed that the most common audiometric abnormality after retrolabyrin thine vestibular neurectomy is low-frequency conductive hearing loss thought to be due to ossicular fixation from this mechanism. As shown in Table 36-2, reports of hearing loss after vestibular neurectomy state that retrolabyrinthine patients have a 7% to 58% incidence of hearing loss, whereas retrosigmoid patients have a 7% to 45% inci dence of hearing loss. These data must be interpreted with caution because the postoperative time points at which the hearing losses were evaluated vary among studies, and are no doubt contaminated by the continued
Chapter 36 • Retrolabyrinthine and Retrosigmoid Vestibular Neurectomy
451
TABLE 36-3 Common Complications* Retrolabyrinthine Vestibular Neurectomy Author, Year McElveen et al, 198838 Monsell et al, 198839 Boyce et al, 198840 Zini et al, 198841 Kemink et al, 19911 Glasscock et al, 199142 Teixido and Wiet, 199234 Ortiz Armenta, 199243 Nguyen et al, 199244 Aristegui et al, 199745 Silverstein and Jackson, 200246
CSF Leak (%)
Infection (%)
Meningitis (%)
Hearing Loss (%)
4 3 6 8 4 12 NA 7 3 0 10
1 0 0 0 6 2 NA 0 4 0 NA
1 0 4 3 0 0 NA 0 0 0 0
0 0 0 3 0 2 0 7 0 0 0
Retrosigmoid Vestibular Nerve Section Author, Year Glasscock et al, 199142 Molony, 199647 Pareschi et al, 200248 Fukuhara et al, 200249 Miyazaki et al, 200526
CSF Leak (%)
Infection (%)
Headache (%)
Hearing Loss (%)
0 0 0 0 4
5 0 0 0 0
9 7 0 7 0
0 4 0 7 0
Combined Retrolabyrinthine/Retrosigmoid Vestibular Nerve Section Author, Year Silverstein and Jackson, 200246 Goksu et al, 200550
CSF Leak (%)
Infection (%)
Headache (%)
Hearing Loss (%)
NA
NA
NA
0
1
0
0
10
CSF, cerebrospinal fluid; NA, not applicable. *Series with >25 patients; only nonoverlapping data from a single group or institution are included.
decline of hearing because of the natural progression of Meniere’s disease. A study by Wazen and coworkers35 showed that serviceable hearing decreased from 81% to 43% of patients at an average of 4 years after vestibular neurectomy. A reasonable conclusion may be drawn that hearing loss continues after vestibular neurectomy, just as it does with untreated Meniere’s disease,29-31 but per haps at a slightly slower rate. The other classic symptoms of Meniere’s disease, tin nitus and aural fullness, also occasionally improve after vestibular neurectomy. As shown in Table 36-2, tinnitus was reduced in half of patients, and aural fullness was reduced in two thirds of patients after the retrolabyrin thine approach.
COMPLICATIONS Table 36-3 summarizes the complications encoun tered with posterior fossa vestibular neurectomy. For the retrolabyrinthine approach, the most common
c omplication is cerebrospinal fluid leak, which occurs in up to 12% of patients. Wound infection and meningitis are also common complications, occurring in up ��������� to���� 6% and 4% of patients. The most commonly cited complica tion with retrosigmoid vestibular neurectomy is signifi cant postoperative headache, sometimes disabling, which can persist for years after surgery. The reported incidence of cerebrospinal leak and wound infection are much lower than for the retrolabyrinthine approach at 4%, and the wound infection rate is similar at 5%. Although facial nerve paresis is a risk, this occurs rarely with the poste rior fossa approaches and is not generally reported in the literature. Other complications reported in the literature for these operations include hydrocephalus, continued imbalance, tinnitus, aseptic meningitis, and abdominal hematoma if an abdominal fat graft is used. Review of the literature shows that the incidence of headache after retrosigmoid vestibular neurectomy approaches 1 in 10. The etiology for such headache syn dromes is still a matter of debate in the otology and neu rosurgery literature. Numerous explanations have been
452
OTOLOGIC SURGERY
proposed, and many strategies have been presented in an effort to reduce this problem. Adhesion of cervical musculature to the dura that is exposed during the cra niectomy can subsequently result in traction on the dura with head movement. Injury to the greater and lesser occipital nerves during the procedure, either by retraction or during the incision itself, has been frequently cited as a possible contributing factor. A third explanation may be that bone dust can be trapped in the subarachnoid space, which can cause chemical meningitis or interfere with cerebrospinal fluid resorption and cause increased intracranial pressure. Other factors including dural tension, cerebellar retraction, and aseptic meningitis have been posited as contributing to postoperative headache. Efforts to prevent the attachment of the deep neck musculature to exposed dura, designing skin incisions to avoid the occipital nerves, and the avoidance or minimization of intradural bone drilling all have been shown to decrease the incidence of postoperative head ache.19 The prevention of cervical muscle adherence to dura has been described using cranioplasty and wound closure modifications. Cranioplasty to replace missing bone volume was initially advocated to decrease the incidence of postoperative headache. Cranioplasty increases operative time, however, and more recent studies have shown that the foreign materials such as methylmethacrylate may lead to a higher incidence of headache, owing to increased local tissue reaction.36 In addition, the data suggest that there is no decrease in the frequency of postoperative headache in cranioplasty patients.36,37
REFERENCES 1. Kemink J L , Telian S A, el-Kashlan H, et al: Retrolaby rinthine vestibular nerve section: efficacy in disorders other than Meniere’s disease. Laryngoscope 101(5):523528, 1991. 2. Jackler R K, Whinney D: A century of eighth nerve sur gery. Otol Neurotol 22(3):401-416, 2001. 3. Frazier C : Intracranial division of the auditory nerve for persistent aural vertigo. Surgery Gynecology and Obstet rics 15:524-529, 1912. 4. McKenzie K : Intracranial division of the vestibular por tion of the auditory nerve for Meniere’s disease. Cana dian Medical Association Journal 34:1127-1152, 1936. 5. Green R E, Douglass CC : Intracranial division of the eighth nerve for Meniere’s disease; a follow-up study of patients operated on by Dr. Walter E. Dandy. Ann Otol Rhinol Laryngol 60(3):610-621, 1951. 6. House WF: Surgical exposure of the internal auditory canal and its contents through the middle, cranial fossa. Laryngoscope 71:1363-1385, 1961. 7. Fisch U: Vestibular and cochlear neurectomy. Trans Am Acad Ophthalmol Otolaryngol 78(4):ORL252-ORL255, 1974.
8. Glasscock M E, 3rd: Vestibular nerve section. Middle fossa and translabyrinthine. Arch Otolaryngol 97(2):112114, 1973. 9. Hitselberger WE, Pulec J L : Trigeminal nerve (posterior root) retrolabyrinthine selective section. Operative pro cedure for intractable pain. Arch Otolaryngol 96(5):412415, 1972. 10. Badke M B, Pyle G M, Shea T, et al: Outcomes in vestib ular ablative procedures. Otol Neurotol 23(4):504-509, 2002. 11. Silverstein H, Norrell H : Retrolabyrinthine surgery: a di rect approach to the cerebellopontine angle. Otolaryngol Head Neck Surg 88(4):462-469, 1980. 12. Silverstein H, Norrell H : Retrolabyrinthine vestibular neurectomy. Otolaryngol Head Neck Surg 90(6):778782, 1982. 13. Silverstein H, McDaniel A, Wazen J, et al: Retrolabyr inthine vestibular neurectomy with simultaneous moni toring of eighth nerve and brain stem auditory evoked potentials. Otolaryngol Head Neck Surg 93(6):736-742, 1985. 14. Silverstein H, Nichols M L , Rosenberg S, et al: Combined retrolabyrinthine-retrosigmoid approach for improved ex posure of the posterior fossa without cerebellar retraction. Skull Base Surg 5(3):177-180, 1995. 15. Silverstein H, Norrell H, Smouha E, et al: Combined retrolab-retrosigmoid vestibular neurectomy. An evolu tion in approach. Am J Otol 10(3):166-169, 1989. 16. Silverstein H, Norrell H, Smouha E, et al: An evolution of approach in vestibular neurectomy. Otolaryngol Head Neck Surg 102(4):374-381, 1990. 17. Silverstein H : Cochlear and vestibular gross and histo logic anatomy (as seen from postauricular approach). Otolaryngol Head Neck Surg 92(2):207-211, 1984. 18. Rasmussen A : Studies of the VIIIth cranial nerve of man. Laryngoscope 50:667, 1940. 19. Silverman D A, Hughes G B, Kinney S E, et al: Techni cal modifications of suboccipital craniectomy for preven tion of postoperative headache. Skull Base 14(2):77-84, 2004. 20. Megerian C A, Hanekamp J S, Cosenza M J, et al: Selec tive retrosigmoid vestibular neurectomy without internal auditory canal drill-out: an anatomic study. Otol Neuro tol 23(2):218-223, 2002. 21. Kartush J M, Telian S A, Graham M D, et al: Anatomic basis for labyrinthine preservation during posterior fossa acoustic tumor surgery. Laryngoscope 96(9 Pt 1):10241028, 1986. 22. Wazen JJ, Sisti M, Lam S M : Cranioplasty in acous tic neuroma surgery. Laryngoscope 110(8):1294-1297, 2000. 23. Feghali JG, Elowitz E H : Split calvarial graft cranioplasty for the prevention of headache after retrosigmoid resec tion of acoustic neuromas. Laryngoscope 108(10):14501452, 1998. 24. Soumekh B, Levine SC, Haines S J, et al: Retrospective study of postcraniotomy headaches in suboccipital ap proach: diagnosis and management. Am J Otol 17(4):617619, 1996. 25. Ozluoglu L N, Akbasak A : Video endoscopy-assisted ves tibular neurectomy: a new approach to the eighth cranial nerve. Skull Base Surg 6(4):215-219, 1996.
Chapter 36 • Retrolabyrinthine and Retrosigmoid Vestibular Neurectomy 26. Miyazaki H, Deveze A, Magnan J: Neuro-otologic sur gery through minimally invasive retrosigmoid approach: endoscope assisted microvascular decompression, ves tibular neurotomy, and tumor removal. Laryngoscope 115(9):1612-1617, 2005. 27. Wackym PA, King WA, Barker FG, et al: Endoscopeassisted vestibular neurectomy. Laryngoscope 108 (12):1787-1793, 1998. 28. Eisenman DJ, Speers R , Telian S A : Labyrinthectomy versus vestibular neurectomy: long-term physiologic and clinical outcomes. Otol Neurotol 22(4):539-548, 2001. 29. Quaranta A, Marini F, Sallustio V: Long-term outcome of Meniere’s disease: endolymphatic mastoid shunt versus natural history. Audiol Neurootol 3(1):54-60, 1998. 30. Quaranta A, Onofri M, Sallustio V, et al: Comparison of long-term hearing results after vestibular neurectomy, endolymphatic mastoid shunt, and medical therapy. Am J Otol 18(4):444-448, 1997. 31. Silverstein H, Smouha E, Jones R : Natural history vs. surgery for Meniere’s disease. Otolaryngol Head Neck Surg 100(1):6-16, 1989. 32. Filipo R , Barbara M : Natural course of Meniere’s dis ease in surgically-selected patients. Ear Nose Throat J 73(4):254-257, 1994. 33. Parikh A A, Brookes G B : Conductive hearing loss fol lowing retrolabyrinthine surgery. Arch Otolaryngol Head Neck Surg 122(8):841-843, 1996. 34. Teixido M, Wiet R J: Hearing results in retrolabyrinthine vestibular neurectomy. Laryngoscope 102(1):33-38, 1992. 35. Wazen JJ, Spitzer J, Kasper C, et al: Long-term hearing results following vestibular surgery in Meniere’s disease. Laryngoscope 108(10):1470-1473, 1998. 36. Lovely TJ, Lowry DW, Jannetta PJ: Functional outcome and the effect of cranioplasty after retromastoid craniec tomy for microvascular decompression. Surg Neurol 51(2):191-197, 1999. 37. Catalano PJ, Jacobowitz O, Post K D: Prevention of head ache after retrosigmoid removal of acoustic tumors. Am J Otol 17(6):904-908, 1996. 38. McElveen JT Jr, Shelton C, Hitselberger WE, et al: Retrolabyrinthine vestibular neurectomy: a reevaluation. Laryngoscope 98(5):502-506, 1988.
453
39. Monsell E M, Wiet R J, Young N M, et al: Surgical treat ment of vertigo with retrolabyrinthine vestibular neurec tomy. Laryngoscope 98(8 Pt 1):835-839, 1988. 40. Boyce S E, Mischke R E, Goin DW: Hearing results and control of vertigo after retrolabyrinthine vestibular nerve section. Laryngoscope 98(3):257-261, 1988. 41. Zini C, Mazzoni A, Gandolfi A, et al: Retrolabyrinthine versus middle fossa vestibular neurectomy. Am J Otol 9(6):448-450, 1988. 42. Glasscock M E 3rd, Thedinger B A, Cueva R A, et al: An analysis of the retrolabyrinthine vs. the retrosigmoid vestibular nerve section. Otolaryngol Head Neck Surg 104(1):88-95, 1991. 43. Ortiz Armenta A.: Retrolabyrinthine vestibular neurec tomy. 10 years’ experience. Rev Laryngol Otol Rhinol (Bord) 113(5):413-417, 1992. 44. Nguyen C D, Brackmann D E, Crane RT, et al: Retro labyrinthine vestibular nerve section: evaluation of tech nical modification in 143 cases. Am J Otol 13(4):328-332, 1992. 45. Aristegui M, Canalis R F, Naguib M, et al: Retrolaby rinthine vestibular nerve section: a current appraisal. Ear Nose Throat J 76(8):578-583, 1997. 46. Silverstein H, Jackson L E : Vestibular nerve section. Otolaryngol Clin North Am 35(3):655-673, 2002. 47. Molony TB : Decision making in vestibular neurectomy. Am J Otol 17(3):421-424, 1996. 48. Pareschi R , Destito D, Falco Raucci A, et al: Posterior fossa vestibular neurotomy as primary surgical treatment of Meniere’s disease: a re-evaluation. J Laryngol Otol 116(8):593-596, 2002. 49. Fukuhara T, Silverman D A, Hughes G B, et al: Vestibular nerve sectioning for intractable vertigo: efficacy of simpli fied retrosigmoid approach. Otol Neurotol 23(1):67-72, 2002. 50. Goksu N, Yilmaz M, Bayramoglu I, et al: Combined retrosigmoid retrolabyrinthine vestibular nerve section: results of our experience over 10 years. Otol Neurotol 26(3):481-483, 2005.
37
Translabyrinthine Vestibular Neurectomy Craig A. Buchman and Oliver F. Adunka
Deafferentation of the peripheral vestibular system continues to play a role in the management of patients with fluctuating or poorly compensated vestibulopathy that remains refractory to medical therapies or vestibular rehabilitation. In various peripheral vestibular disorders, abnormal spontaneous or motion-induced inputs that conflict with normal contralateral responses can create symptoms of dizziness, imbalance, vertigo, motion intolerance, and visual instability (i.e., oscillopsia) with a resulting negative impact on quality of life.1 Removing these dynamic vestibular signals generated from the abnormal ear can create a static vestibular lesion, for which the brain is more easily able to compensate.2 Deafferentation of the peripheral vestibular system can be produced through surgical or chemical labyrinthectomy, vestibular nerve section (i.e., neurectomy or neurotomy), or a combination thereof. Although chemical labyrinthectomy may produce either a partial or a total loss of peripheral vestibular function from the affected ear,3,4 surgical labyrinthectomy or vestibular nerve section reliably produces a complete lesion.5 Chemical labyrinthectomy destroys the sensory hair cells of the semicircular canals (SSC) and otolithic organs, whereas surgical labyrinthectomy comprehensively removes the contents of the vestibular labyrinth. Conversely, vestibular nerve section more proximally creates deafferentation by interrupting the transduction of the abnormal neural impulses from the labyrinth to the brainstem. Both surgical procedures allow for direct pathologic assessment of tissues, which may be necessary in certain instances. Each of these approaches has merits and disadvantages, and must be chosen on an individualized basis for the patient and his or her condition. This chapter focuses on translabyrinthine vestibular nerve section. This procedure, by necessity, combines the advantages of labyrinthectomy and vestibular nerve section, including (1) dual deafferentation of the peripheral vestibular end organs and (2) direct pathologic assessment of the contents of the vestibular labyrinth and the internal auditory canal (IAC). By removing preganglionic and postganglionic neural elements, a more complete vestibular lesion may be produced, especially in
cases where previous labyrinthectomy or vestibular nerve section attempts have failed.6 Examination of the tissues may reveal inflammatory or neoplastic processes that require further medical attention.6,7
HISTORY Charcot, in 1874, and later Frazier8 initially described neurectomy of the vestibulocochlear nerve. Subsequently, in 1928, Dandy9 strongly advocated intracranial sectioning of the eighth cranial nerve for paroxysmal vertigo symptoms. Although he noted that the procedure often was successful for relieving troubling vertigo, patients typically had a complete hearing loss in the operated ear. McKenzie,10 in 1931, developed a selective neurectomy technique, preserving auditory function in some cases. Some 30 years later, House11 developed the middle fossa and translabyrinthine approaches for vestibular nerve sectioning. Later modifications by Fisch and Glasscock and colleagues12-14 refined these approaches. The importance of preserving not only the cochlear nerve, but also the labyrinthine blood supply as a critical premise for hearing preservation was realized. Silverstein and colleagues15,16 later described in detail the complex interrelationship of the vestibular and cochlear nerve fibers in the cerebellopontine angle, allowing for even more selective neurectomy near the root entry zone in the brainstem. Over the last 50 years, Hitselberger and his colleagues can be credited with performing vestibular nerve section procedures on hundreds of patients with peripheral vestibular disorders.17 Today, successful vestibular nerve section relies on the anatomic and surgical principles developed by these pioneers.
DIAGNOSTIC CONSIDERATIONS Selecting an appropriate surgical intervention for patients with peripheral vestibular disorders is challenging. In considering such therapy, a diagnosis must first be rendered or, minimally, an affected ear must be identified. 455
456
OTOLOGIC SURGERY
Although a detailed discussion regarding the diagnosis and management of vestibular disorders is beyond the scope of this chapter, an understanding of the various vestibular disorders is crucial to identifying a correct diagnosis and making an adequate treatment recommendation.18,19 Establishing the correct diagnosis may be difficult sometimes, and can require the input from various professionals in differing disciplines. Vertigo is the cardinal symptom of a vestibular system disorder. It is also important to recognize that many cases of dizziness are not related to the vestibular system at all, and dizziness without vertigo is common. When vertigo is absent, the diagnosis of vestibular disorder should be carefully scrutinized. Although the presence of vertigo can be of either central or peripheral origin, most cases of vertigo arise from the peripheral vestibular apparatus (SSC, otolithic organs, or vestibular nerves). Although most of these peripheral vestibular disorders manifest to the clinician with a clear clinical picture, occasionally differentiation from the common central causes of vertigo is difficult. Differentiation among the peripheral and central varieties of vertigo takes on even greater significance when ear-specific therapies are being contemplated. When peripheral deafferentation is considered in the treatment regimen, defining the peripheral cause and the affected labyrinth is crucial. The most common causes of peripheral (or ear-related) vertigo in adult patients include benign paroxysmal positional vertigo (BPPV), Meniere’s disease, and vestibular neuronitis. More recently, superior semicircular canal dehiscence (SSCD) syndrome has become more frequently recognized.20,21 Common central causes of vertigo include migraine-related dizziness (also known as migraine vestibulopathy22), transient ischemic attack, and demyelinating disease such as multiple sclerosis and stroke. In nearly all patients with vertigo, a careful history, physical examination, and appropriate laboratory and radiologic testing can reveal the correct diagnosis. A minimal test battery usually includes comprehensive audiometric testing, imaging of the temporal bone and brain, and laboratory evaluations, which may include selective vestibular function testing (see section on preoperative testing). Meniere’s disease is characterized by episodic vertigo that lasts minutes to hours with associated, fluctuating aural symptoms, including hearing loss, tinnitus, and pressure. Audiometric testing characteristically reveals a low-frequency sensorineural hearing impairment. In contrast, BPPV is characterized by brief (70% speech discrimination, 3 cm) or in patients with peritumoral brain edema, when dexamethasone (10 mg) is administered intravenously. To reduce the risk of CSF fistulization, an indwelling lumbar CSF drain is used when extensive peri-IAC pneumatization is encountered.
SURGICAL SITE PREPARATION We ask the patient to wash his or her hair thoroughly either the morning of surgery or on the evening before with an antiseptic shampoo. In the operating room, after induction of anesthesia, the hair over the suboccipital area is removed with electric clippers. It is also acceptable to use a racing stripe technique. The upper neck is included in the operative site, allowing potential access to the great auricular nerve in case a graft is required for facial nerve reconstruction. The scalp is washed with povidone-iodine soap, and the clipped area is shaved. To improve attachment of adhesive drapes, the surgical site is defatted with alcohol and dried. Sterile drapes are placed 1 cm from the hair edge around the prepared scalp and held in place with surgical adhesive (e.g., Mastisol). The field is prepared with povidone-iodine solution and dried. Sterile towels are placed around the operative field and held into position by an adhesive plastic sheet.
SPECIAL INSTRUMENTS Various instruments are used for the retrosigmoid approach to the CPA, including craniectomy instruments, retractors, high-speed surgical drill with a selection of cutting and diamond burrs, suction and suction-irrigation tips of the fenestrated and nonfenestrated types, bipolar cautery, microdissection instruments, and a binocular operating microscope. We perform the craniectomy with an Acra-Cut disposable cranial perforator burr-hole maker in a Hudson brace, a system that allows rapid bone removal, while minimizing the chance of dural or venous sinus injury. The craniectomy is completed with rongeurs. To retract the thick suboccipital musculature, a deep-bladed Weitlaner-type retractor is used. For brain retraction, several sizes of malleable blades are used that may be held in position in several ways. We prefer to use the Apfelbaum base, which combines a Weitlaner-type retractor with a movable arm to affix the retractor. Other options for basing the brain retractors during retrosigmoid craniotomy include a C-clamp placed on the head holder frame or a table-based system (e.g., Greenberg). Either an electric or an air-powered drill is suitable to use for this approach. When the exposure is narrow, an angled handpiece is advantageous because it is less
608
OTOLOGIC SURGERY
obstructing to the surgeon’s point of view. An operating microscope with an inclinable optical pathway is desirable to accommodate the various exposure angles required during the procedure while maintaining a comfortable operating position. Insulated bipolar cautery forceps are essential for obtaining hemostasis during CPA tumor surgery. Large tips for handling substantial vessels and slender, fine tips for use when the coagulation must be confined to a narrow region are needed. We have found that a self-irrigating system (e.g., Malis bipolar irrigating system) is valuable because it discourages tissue adhesion to the forceps tips. We use a microsurgical instrument set that includes sharp and blunt dissectors in various shapes and sizes, needles, and small bone curettes (e.g., Rhoton microneurosurgical instruments). A set of sharp scissors of different sizes and angles is also important. Many special tools are available to facilitate rapid intracapsular debulking of the tumor. We prefer to use a Cavitron ultrasonic surgical aspirator (CUSA), which allows debulking without traction or torsion, minimizes hemorrhage, and respects tumor capsular planes, avoiding inadvertent neural or vascular injury. Other options include the surgical laser and a rotatory surgical aspirator (House-Urban). Operating room electric circuitry and neuroanesthesia electric monitoring equipment should be grounded and electronically quiet to minimize 60 Hz noise production, which interferes with the cranial nerve electrophysio logic monitoring setup. The specialized equipment for intraoperative cranial nerve monitoring used in our institution has been described elsewhere in detail.22
SURGICAL TECHNIQUE Acoustic neuroma excision by the retrosigmoid approach to the CPA can be subdivided into seven stages: (1) craniectomy, (2) exposure of the CPA, (3) exposure of the IAC, (4) tumor resection, (5) hemostasis, (6) IAC closure, and (7) craniotomy closure.
Craniectomy A curvilinear paramedian incision 3 cm behind the postauricular sulcus is made down to bone. The cervical muscles are detached anteriorly and posteriorly, exposing the mastoid and suboccipital areas. Emissary venous bleeding is controlled with bone wax. The mastoid tip is exposed, and the posterior belly of digastric muscle is elevated from its groove. Dissection directly on the bone preserves the occipital nerves and vessels. A posterior fossa craniotomy window of approximately 3 × 3 cm is made in the retrosigmoid approach. It is bounded anteriorly by the sigmoid sinus and superiorly by the transverse sinus. The craniectomy begins with two or three closely approximated burr holes. The burr holes are joined up with rongeurs, creating a craniotomy window. The bone fragments are collected and stored in sterile
antibiotic-saline solution for replacement in the cranial defect at the end of the procedure. Development of the craniectomy anteriorly usually opens the mastoid air cell system to a variable degree. When the bony craniectomy is complete, the opened mastoid air cells are sealed with bone wax. Wax is also used to control bleeding from diploic bone at the craniotomy margins. Many styles of dural opening are described in the literature. We use a posteriorly based dural flap to enter the posterior fossa. The posterior fossa dura is opened 2 to 3 mm from its junction with the sigmoid and transverse sinus dura and at a similar distance from the inferior bony margin. The dural flap is reflected posteriorly. Small, relaxing incisions are made superiorly and inferiorly in the marginal dura to create small anterior and superior dural flaps, which are retracted with stay sutures, completing the dural opening.
Exposure of Cerebellopontine Angle When the dural flap has been reflected posteriorly, it and the craniotomy margins are covered with moist Telfa strips. To drain CSF from the cisterna magna, the cerebellum is gently retracted superiorly with a polytef (Teflon)-coated malleable retractor. The arachnoid of the cistern is lanced with a bayoneted suction tip, which decompresses the posterior fossa, relaxes the cerebellum, and allows it to fall away medially. Premature medially directed cerebellar retraction, before draining the cisterna magna, risks inducing massive cerebellar swelling. After this maneuver, the retractor is withdrawn and repositioned anteriorly to develop posteromedial cerebellar retraction. Retraction in this manner, accompanied by division of arachnoid bands and bridging veins, opens the CPA. The degree of CPA exposure required varies with the size and location of the tumor being addressed. Superiorly, the petrosal veins (or Dandy’s veins), which lie just below the tentorium cerebelli and run parallel to the course of the trigeminal nerve, may hinder exposure or appear to be in jeopardy of tearing with retraction. When necessary, these may be coagulated and divided. After their division, the superior pole of the cerebellar hemisphere falls posteromedially away from the tentorium, providing access to the superior aspect of the CPA. The cerebellar flocculus often overlies the brainstem root entry zones of CN VII and VIII, and must be gently mobilized from the cerebellar peduncle and lateral pontine surfaces. A tuft of choroid plexus, emanating from the lateral recess of the fourth ventricle, is also frequently encountered in this area. Mobilizing these structures from the root entry zone need not be performed during hearing conservation procedures when the proximal portion of the nerves are not involved with tumor because this maneuver places the internal auditory artery at risk. The cranial nerve electrophysiologic monitoring circuitry is tested by stimulating CN XI, which is usually readily accessible at the inferior pole of the exposure. Particular attention is
Chapter 50 • Retrosigmoid Approach to Tumors of the Cerebellopontine Angle
paid to the location of the anterior inferior cerebellar artery (AICA) and its branches. Inferiorly, the posterior inferior cerebellar artery may be seen in relation to the lower cranial nerves, and superiorly the superior cerebellar artery may be identified coursing through the region of the tentorial notch. In acoustic neuroma surgery, we prefer to begin the drill excavation of the IAC at an early stage, before extensive opening of the arachnoid planes above and below the CPA component. This method helps reduce bone debris contamination of the subarachnoid space.
Exposure of Internal Auditory Canal Exposure of the IAC and its contents involves removal of the bone surrounding the posterior, superior, and inferior aspects (Figs. 50-3 to 50-6). Optimal canal visualization may be obtained through a combination of rotation of the operating table away from the side of the surgeon and microscope positioning. These maneuvers bring the posterior petrous face into view centered over the region of the IAC. To locate the canal, the opening of the meatus is gently probed with a blunt, right angle hook. Before IAC opening begins, the operative field is set up to contain as much bone debris as possible and to prevent its dissemination into the subarachnoid space. Absorbable gelatin sponge (Gelfoam) pledgets are placed into the superior and inferior portions of the CPA. A rectangular-shaped rubber dam is fashioned from a surgical glove, placed over the occluding pledgets, and held in place with the cerebellar retractor. An H-shaped dural incision, centered on the long axis of the IAC, is outlined on the posterior petrous face by use of a bipolar cautery. When the dura has been incised with the tip of a No. 11 blade, superior and inferior dural flaps are elevated with a small Lempert mastoid elevator. The surgeon should exercise caution when incising inferiorly because the jugular bulb is occasionally dehiscent on the posterior petrous face. Similarly, the incision should not be carried too far laterally because laceration of the sigmoid sinus can occur. Care is taken to identify and preserve the endolymphatic sac and duct, which are located posterolaterally. The dura usually can be elevated off the endolymphatic sac. The entry point of the vestibular aqueduct into bone is a useful anatomic landmark. When the bony dissection of the IAC does not extend lateral to the operculum of the aqueduct, the labyrinth is unlikely to be breached. The posterior IAC wall is rapidly removed by the drilling of a trough over the posterior petrous face. Drilling from medial to lateral in the line of the IAC reduces the risk of the burr slipping into the CPA. The canal should be opened only as much as required to expose the lateralmost aspect of the tumor. Excessive bony opening does not enhance exposure further, but may increase the risk of CSF leak through the opening of additional petrous air cells. Initially, the bone is removed with a cutting burr until the IAC dura is identified through a thin bony
609
plate. To expose the dura of the posterior aspect of the IAC, the dural cuff of the meatus is first elevated from the thin residual plate. Then the remaining bony shell over the posterior aspect of the IAC is drilled away. To reduce the risk of traumatizing the IAC dural lining or its neural structures, removal of the last eggshell of bone is accomplished with diamond burrs. Diamond burrs are more controllable by virtue of their reduced tendency to run, and are less likely to cause injury if they come into contact with soft tissue structures. Bony troughs, 3 to 4 mm in diameter, are developed above and below the canal. These troughs are important for three reasons: (1) to provide working space for the insertion of angled instruments needed to establish a plane of dissection between the tumor and the facial and cochlear nerves, (2) to permit visualization of the facial nerve when it is acutely angled superiorly or inferiorly as a result of tumor displacement, and (3) to enhance exposure of the anterior aspect of the CPA. In preparation for the drilling of bony troughs around the canal, the IAC dura is elevated from the upper and lower canal walls with a blunt dissector. The troughs, which should be widest at the level of the porus, are excavated with a cutting burr. As the troughs are developed, a thin shell of bone is left over the dura of the superior and inferior walls of the IAC. When the troughs are fully developed, the remaining bony shells are progressively thinned with the side of the diamond burr until the dura is exposed. Copious irrigation is used to prevent thermal injury to neural structures. Often, the IAC dura can be gently retracted with a fenestrated suction, permitting completely atraumatic removal of the remaining bony eggshell fragments, which can be elevated from the exposed IAC dura. This technique exposes 180 to 270 degrees of the IAC circumference. Caution must be exercised in development of the superior and inferior troughs because of the proximity of the facial nerve and the jugular bulb. The width of the inferior trough varies with the location of the jugular bulb. When the jugular bulb is unusually high, creating an inferior bony trough at the level of the meatus may be impossible, although exposure of the fundus is typically unhindered. Compensating for the limited inferior access associated with a high jugular bulb is usually possible through creation of an unusually wide and deep superior trough. Additional exposure of the IAC from above may also be gained through retraction of the tentorium. In hearing conservation attempts, the lateral extent of the IAC opening should be restricted to approximately the medial two thirds of the IAC because opening of the lateral one third to expose the fundus may result in a breach of the vestibule or crus commune, militating against hearing conservation. The decision as to how far laterally the IAC is opened depends on the lateral intracanalicular extent of the tumor, which may be accurately predicted from the preoperative gadolinium-enhanced MRI.16,23 Alternatively, the lateral opening can be limited
610
OTOLOGIC SURGERY
Chapter 50 • Retrosigmoid Approach to Tumors of the Cerebellopontine Angle
611
FIGURE 50-3. Step 1 of exposure of internal auditory canal (IAC) during retrosigmoid approach. After localization of porus acusticus through palpation with a ball hook, an H-shaped dural incision is created over the long axis of IAC. Dural flaps are reflected anteriorly and posteriorly to expose posterior aspect of petrous pyramid. Dura usually can be dissected from posterior surface of endolymphatic sac. To maximize the possibility of hearing preservation, care must be exercised to avoid avulsion of sac from its aqueduct. Before drilling, Gelfoam pledgets are positioned above and below 7 to 8 neurovascular bundle in posterior fossa in an effort to minimize the spread of bone dust onto arachnoidal surfaces. FIGURE 50-4. Step 2 of exposure of internal auditory canal (IAC) during retrosigmoid approach. A cutting burr is used to excavate rapidly bone overlying IAC. When canal dura is encountered, a diamond burr is used. Drilling is carried out from medial to lateral (from porus to fundus), in part to minimize the possibility that drill could accidentally run into posterior fossa.
FIGURE 50-5. Step 3 of exposure of internal auditory canal (IAC) during retrosigmoid approach. A diamond burr is used to create deep troughs around IAC that should extend well deep to canal plane to provide adequate room for microdissection with the angled instruments needed to remove facial nerve from tumor safely. Particular care should be exercised while superior trough is developed because facial nerve may lie immediately beneath dura in this location.
FIGURE 50-6. Step 4 of exposure of internal auditory canal (IAC) during retrosigmoid approach. In preparation for tumor removal, dura of IAC is incised. After an incision along the length of the canal is created with upbiting scissors, two small relaxing incisions are created at porus and fundus to develop dural flaps. These are reflected anteriorly and posteriorly to expose canal contents.
on the premise that an indirect inspection and clearance of the tumor from the lateral IAC can be satisfactorily achieved. This method has the attendant risk of leaving residual tumor in the lateral IAC, however. To avoid this problem, some surgeons have advocated blind curettage using special right angle curettes followed by inspection of the fundus with a small mirror or endoscope to validate the extent of tumor resection.24,25 With these methods, distinguishing residual tumor from the transected vestibular nerves and traumatized dura is sometimes difficult. Dissection of tumor from the fundus without direct visualization risks leaving wellvascularized residual tumor with the potential for clinically significant recurrence.26,27 We advocate exposure of the IAC laterally to a point beyond the tumor interface, where the naked CN VII and residual CN VIII may be visualized. This process sometimes may require opening the canal to the fundus, with resultant entry into labyrinthine structures and sacrifice of residual hearing. It has been proposed that enhanced visualization of the fundus can be achieved by skeletonization of the posterior and superior semicircular canals.28 Similarly, it has been shown that partial resection of the posterior semicircular canal may be helpful in augmenting fundal exposure.29 After completion of the IAC exposure, the rubber dam and Gelfoam pledgets are removed. The dura of the IAC is opened along the long axis of the canal with sharp, upturned, right angle microscissors working from a medial-to-lateral direction. This incision is placed slightly eccentrically and is biased to the superior side to avoid the creation of a long flap over the facial nerve course. The dural flaps are reflected superiorly and inferiorly, exposing the IAC contents.
Acoustic Neuroma Resection Attention is now turned to planning the actual resection of the tumor; the size of the tumor largely dictates the actual sequence and pattern of removal (Figs. 50-7 to 50-9). We prefer to dissect the IAC initially because this step helps ascertain the probable course of the facial
nerve outside of the porus into the CPA, and allows early identification of the facial nerve. A test run of the neural monitoring system can be performed in which positive identification of the nerve by its anatomic relationships is possible. In many cases, ascertaining whether the tumor has arisen from the superior or inferior vestibular nerve is possible. When only one of these nerves is visible on the posterior surface of the tumor, it may be assumed that the other was the nerve of origin. Dissection is begun laterally by identification of the plane between the facial nerve and the tumor. A fine-tipped dissector is insinuated between the superior dural leaf of the canal and the tumor while gentle downward pressure and a rotating motion are applied. Gradually, this process brings into view the interface between the facial nerve and the lateral end of the tumor. When this plane has become established, a sharp, right angle instrument is used to dissect the tumor from the posterior surface of the facial and cochlear nerves. All tissue superficial to this plane, including the tumor and both vestibular nerves, is transected either with curved microscissors or through an upward motion with the sharp edge of the dissector. When the intracanalicular tumor component is bulky, it may require debulking to a variable degree to permit microdissection of the capsular peel from the facial and cochlear nerves. This initial dissection of the intracanalicular portion should proceed only to the lip of the porus acusticus or just beyond it, to avoid dissection of the typically most adherent section at this stage. It is important to avoid inducing neurapraxic injury, which might impair later electric identification of the facial nerve medially at its brainstem exit. After removal of the intracanalicular portion of the tumor, the CPA component is addressed. The posterior capsule is swept with the neural monitoring probe to ensure that the facial nerve is not on this surface (a rarity in acoustic neuroma). A rectangular incision is made in the posterior capsule with the point of a No. 11 blade, and the peel is resected with scissors. Intracapsular debulking may be done with cupped forceps, sharp dissection with scissors, an ultrasonic aspirator (e.g., Cavitron), a rotatory aspiration device (e.g., House-Urban), or the
612
OTOLOGIC SURGERY
Chapter 50 • Retrosigmoid Approach to Tumors of the Cerebellopontine Angle
613
FIGURE 50-7. Step 1 of removal of acoustic neuroma via retrosigmoid approach. After intracanalicular portion of tumor is debulked, lateralmost extension of tumor is reflected medially, and plane between tumor capsule and facial nerve is developed. This maneuver ensures complete removal of tumor from fundus. It also affords an early opportunity to confirm function of cranial nerve monitoring system by stimulation of distal facial nerve under direct vision in a region where it is characteristically not especially adherent to tumor surface.
FIGURE 50-8. Step 2 of removal of acoustic neuroma via retrosigmoid approach. Main portion of tumor in cerebellopontine angle is rapidly debulked. To facilitate rapid and safe tumor removal, we use a Cavitron ultrasonic aspirator. Tumor capsule is first liberated from cerebellum and middle cerebellar peduncle. Pontine surface, including root entry zones of CN VII and VIII, can be exposed. In larger tumors, lesion also must be microdissected from trigeminal and lower cranial nerves (IX and X). FIGURE 50-9. Step 3 of removal of acoustic neuroma via retrosigmoid approach. Characteristically, facial nerve is most adherent to tumor capsule between brainstem surface and anterior lip of porus acusticus. Liberation of nerve from tumor surface in this location often requires particularly delicate microdissection techniques.
FIGURE 50-10. Closure of internal auditory canal defect at completion of retrosigmoid craniotomy. After waxing of cut bony walls to seal any transected air cells, muscle graft harvested from nuchal area is mortised into bony defect. Graft is retained in position by sutures, which are anchored in dural flaps previously developed from posterior petrous surface.
surgical laser. We favor using the CUSA because it efficiently removes the tumor core while respecting its capsule, avoiding potential injury of adherent nerves and vessels. Tumor resection proceeds with alternate intracapsular debulking, followed by microdissection of the thin capsule from the brain surface and cranial nerves, and, ultimately, resection of the liberated capsular segment. The most crucial aspect of CPA tumor removal is identification and preservation of the cranial nerves and blood vessels that lie draped on the capsular surface. In larger tumors, the medial tumor brain dissection plane begins posteriorly along the middle cerebellar peduncle. When this arachnoid plane has become established, it is gradually developed onto the lateral surface of the pons. Attention is turned inferiorly to the probable root entry zone of the seventh and eighth cranial nerve complexes. As an aid to facial nerve identification, electric stimulation is periodically performed along the meniscus of dissection. The course and appearance of the nerve vary depending on its displacement by the tumor. It may be thinned and fanned to a variable degree, making it difficult to delineate from surrounding thickened arachnoid tissue without the use of the microneural stimulator. The brainstem entry of CN VIII is usually encountered lateral to and immediately above the seventh cranial nerve entry zone. A small branch of AICA typically passes between the two nerves and may be a useful guide in orienting the surgeon. In hearing conservation approaches, the vestibular fibers must be separated from the cochlear fibers and divided proximally to establish a tumor dissection plane. When no effort is being made at hearing preservation, CN VIII may simply be transected, a maneuver that simplifies identification of the proximal seventh cranial nerve. When the proximal plane over these two nerves is established, an arachnoid plane can be developed between them and the tumor capsule. While the tumor capsule is dissected, large and small arteries, potential AICA branches, are meticulously preserved. Vessels directly entering the tumor capsule can generally be safely coagulated and divided at the capsular surface without adverse consequences.
While the tumor neural plane is dissected, use of the microneural stimulator (e.g., Xomed Treace-Yingling) with a curved, pliable wire allows blind stimulation of the yet undissected anterior capsule. By localizing the facial nerve course before dissecting the tumor nerve interface, the surgeon may rapidly resect uninvolved capsule and direct meticulous efforts along the actual course of the nerve. Although the course of the facial nerve varies, it characteristically lies anterior to the tumor, occasionally with an anterosuperior or anteroinferior bias. In small tumors, the entire dissection may be accomplished from a medial-to-lateral direction. In larger tumors, medialto-lateral dissection becomes difficult when the nerve is anteriorly angulated toward the porus acusticus. When this occurs, we return to the lateral tumor nerve interface at the end of the IAC and work medially. Alternatively, the anterior tumor capsule with attached nerve may be lifted and rotated to bring the facial nerve course into the surgeon’s view. This action is quite traumatic to the facial nerve, however, and risks disruption of its attenuated fibers. We prefer to dissect the tumor from the facial nerve in situ without mobilizing it from its bed, where it lies supported by an arachnoidal mesh. When the facial nerve is splayed and tightly adherent to the tumor capsule, removing the last remnant of capsule may be impossible without disruption of the nerve.30 In such cases, we prefer to perform a near-total removal, leaving a thin velum of capsule, only 1 to 2 mm thick, attached to the nerve. We believe that this minuscule residual capsule, hanging free in the CPA, is unlikely to generate a recurrent tumor.26 By contrast, tumor left in the distal IAC or in contact with brainstem possesses a vascular supply, and the possibility of regrowth is greater. Several modifications in the strategy of tumor removal are used during hearing conservation approaches. The direction of dissection should be from medial to lateral, whenever possible, to reduce the risk of traumatic avulsion of the delicate cochlear nerve fibers from their entry into modiolus. Throughout the cochlear nerve dissection, changes in auditory brainstem responses relative to the previously recorded baseline waveforms obtained at the start of the procedure are reported. Continuity of
614
OTOLOGIC SURGERY
the cochlear nerve is maintained if possible; however, tumor adherence to it may necessitate its resection. Even when the cochlear nerve is well preserved during dissection, hearing is often lost because of interruption of the cochlear blood supply. This may occur either in the CPA, where the labyrinthine artery branches from a loop of AICA, or in the IAC, where it courses between the inferior vestibular and cochlear nerves. After tumor resection, anatomic and electrical continuity of the cochlear and facial nerves are checked. The facial nerve stimulation threshold voltage at the root entry zone and the intraoperative auditory brainstem response waveform pattern and latencies are recorded. We believe that electrophysiologic monitoring of the auditory nerve is not clearly beneficial, other than in the prognostic sense, in the maintenance of hearing. Monitoring of the facial nerve is indispensable, however, if an optimal outcome is to be obtained.
sutures. At closure of the dura, bacitracin-saline solution is instilled into the subarachnoid space. The margins of the craniectomy are reinspected for opened air cells and smeared with bone wax as indicated. When the transected air cells are large, rather than merely impacting the wax into the exposed cavities, a thin sheet of wax is applied. A Gelfoam pad cut in the shape of the craniectomy defect is placed over the dura, and the previously preserved bone chips are replaced. In our experience, the bone regenerates over several months into a strong, bony plate that restores the cranial contour. After removal of the remaining retractors, the soft tissues of the neck are closed in a series of layers with interrupted 2-0 Surgilon sutures, closing any potential dead space, after the wound is irrigated with antibiotic solution. The skin is sutured with interrupted 4-0 nylon sutures. Electromyography electrodes, ground pads, and the external auditory canal earphone are removed after completion of wound closure and dressing.
Hemostasis After the tumor resection is completed, the wound is irrigated with bacitracin-saline solution, the blood clot is removed, and all bleeding points are identified and controlled with bipolar cautery or by application of thrombin-soaked Gelfoam. As a means of detecting subtle or intermittent bleeding, the anesthesiologist gives the patient a Valsalva maneuver for 20 seconds. Because postoperative hemorrhage into the CPA is a potentially devastating complication, hemostatic efforts should be diligent.
DRESSING After closure of the wound, it is cleaned, dried, and covered with a Telfa strip, to which a sterile adhesive, Op-Site, or similar dressing is applied. To discourage subcutaneous accumulation of CSF, a mastoid-type padded pressure bandage is applied. The dressing is removed 48 hours after surgery, and the wound is inspected and left open to the air. The skin sutures are removed 7 to 10 days after surgery.
Internal Auditory Canal Closure At the time of IAC closure (Fig. 50-10), the bony troughs developed for the IAC exposure are inspected for opened air cells by palpation with a ball hook. Inspection of the cut bony edge may also be carried out through use of a 90 degree angled rigid endoscope. Bone wax is applied to a small Cottonoid and smeared over the exposed bony trough surfaces to seal overtly and covertly opened air cells to prevent CSF leakage. A small fat graft is harvested from the abdomen and is used to seal the IAC. We prefer fat to muscle as an IAC sealant because, with fat-suppressed MRI, fat creates less obscuration of the tumor bed on follow-up imaging. A 7-0 monofilament nylon suture may be placed through the dural flaps of the posterior petrous face, although this is not always needed. The fat is positioned in the IAC, and the suture is tied. Auditory and facial nerve monitoring are maintained until the fat is secured in place so that any possible neural irritation induced by its placement can be identified.
Craniotomy Closure After removal of all the Cottonoids and Telfa strips, the dural flap is sutured back into place with multiple, closely positioned, interrupted 4-0 braided nylon (Surgilon)
POSTOPERATIVE CARE The anesthesiologist awakens the patient, ideally with a smooth extubation that avoids straining and coughing. Antiemetics are given prophylactically to prevent vomiting, which could cause aspiration and associated pneumonitis during recovery from anesthesia. Postoperative monitoring is carried out initially in the postanesthesia care unit and then in the neurosurgical intensive care unit for 24 hours after surgery. After the initial 24-hour period, patients spend an average of 5 to 6 days on a hospital unit staffed by nurses experienced in postcraniotomy care. In addition to the monitoring of temperature, cardiorespiratory status, consciousness level, and fluid balance, the nursing staff and patients are instructed to identify and report any CSF wound leakage or rhinorrhea. If a postoperative facial palsy is present, its grade is recorded according to the House-Brackmann scale, and preventive eye care is instituted. When eye closure is incomplete, artificial tears are applied hourly, or more often as needed, while the patient is awake. During sleep, a plastic eye shield is placed to prevent drying and development of corneal abrasion. Special attention needs to be directed toward patients with dysfunction of the facial
Chapter 50 • Retrosigmoid Approach to Tumors of the Cerebellopontine Angle
and trigeminal nerves. When the cornea is dry, exposed, and insensitive, early gold weight placement is performed even when facial nerve recovery is expected. Moderate to severe headache for several days is typical and may require narcotic analgesia for a variable period. Global headache that is delayed in onset by several days may signify the evolution of aseptic or bacterial meningitis and is discussed later. We try to wean patients from narcotics quickly and try to get their headaches under control with nonsteroidal anti-inflammatory preparations. In the few cases in which corticosteroids have been used, they are tapered over a 7 to 10 days. Vertigo can be controlled with parenterally administered antivertiginous agents, if the condition is severe, and with oral agents, if it is mild. We generally avoid vestibular suppressants in the postoperative period because they may retard vestibular compensation. Diet and increasingly independent mobilization are encouraged under the guidance of a dietitian and physical therapist. We usually restrict the fluid intake for 3 days to a total of 1.5 L/24 hour period. Most patients are able to start a light diet 24 to 48 hours after surgery. Constipation and straining are avoided by the administration of stool softeners to prevent aggravation of headache and possible development of CSF leakage. Most patients begin mobility around 48 hours after surgery, although they have been encouraged to exercise their legs actively while they are recumbent in bed to reduce the risk of deep venous thrombosis. Antiembolism stockings are used until the patient is mobile. Mobilization usually takes the form of initially sitting at the bedside chair, followed by accompanied walks to the bathroom and then farther afield to the hospital corridors, and then onto a trial of practice on the stairs. Walking aids are provided by the physical therapist as required by the patient, depending on his or her progress. We usually discourage hair washing until 1 week after surgery to prevent the wound from getting wet and macerated. Patients may use a dry shampoo if desired. Most patients are usually ready for discharge 5 to 7 days after surgery. Even when all other functions have recovered fully, easy fatigability often persists for 1 to 3 months postoperatively. The convalescent period required before returning to full-time employment and all the previous activities of daily living varies, but is usually 2 to 3 months.
RESULTS Historically, the primary issue in acoustic neuroma surgery was the survival of the patient. With the evolution of microsurgical techniques, mortality from acoustic neuroma surgery has become very low—less than 2% in most recent series. Contemporary emphasis includes tumor control and, particularly, functional preservation. Before the data from our own experience and the data published
615
in the literature are addressed, however, it is important to appreciate that limited international standardization exists in the criteria used for reporting results on degree of resection,31 facial nerve function,32 and hearing preservation.33 In our opinion, the goal of acoustic neuroma resection should be tumor control and not complete resection in every case. Nevertheless, we perform a complete removal in most cases. Incomplete removal can be considered in two categories: subtotal and near-total excision. Subtotal removal, in which a substantial bulk of tumor remains, used to be reserved for elderly or infirm patients with a short anticipated life span in whom shortening of the operative procedure is thought to be in the patient’s interest. In recent years, it is increasingly used in large tumors, backed up by stereotactic radiation if the remnant grows. As previously discussed, near-total excision, in which a thin peel of capsule is left on the most adherent portion of the facial nerve, is occasionally used. Although few data are published on the recurrence risk for this group of patients, we have observed many individuals with serial gadolinium-enhanced MRI and have found a 3% risk of recurrence. The decision to undertake a near-total resection depends on the patient’s age (i.e., less desirable in a younger individual) and preference as to whether the slightly higher risk of recurrence is justified by the improved facial nerve outcome. There are numerous articles in the literature on the subject of facial nerve preservation in acoustic neuroma surgery citing varying degrees of success. Because these results are difficult to compare and draw conclusions from, we confine our commentary to our own series. In our experience, facial nerve outcome from the retrosigmoid approach is similar to that from the other methods of removing acoustic neuromas for tumors of similar size.34 In our acoustic neuroma patients, anatomic continuity of the facial nerve was maintained in 99.2% of cases. Anatomic continuity does not imply functional integrity. The probability of a grade I or II facial function at 1 year after surgery in the context of tumor size was 100% for tumors less than 1 cm; 90% for tumors 1 to 3 cm; and 82% for tumors greater than 3 cm. With regard to hearing preservation, most published series address residual “measurable” hearing in contrast to the much more relevant concept of “useful” hearing.35 For a patient with a unilateral acoustic neuroma, it could be argued that unless the conserved hearing maintains an interaural difference of less than 30 dB hearing loss with good speech discrimination (>50%), it would be likely to be beneficial. Preservation of useful hearing has been reported to be achieved in 25% to 58% of hearing conservation candidates.13 Very little information is available on the long-term follow-up of patients with preserved hearing. In two published series, significant late decline occurred in 22% to 56% of ears with successful hearing conservation.36,37
616
OTOLOGIC SURGERY
Factors relevant to success in hearing conservation approaches to acoustic neuroma include tumor size in the CPA, the depth to which the tumor penetrates the IAC, pure tone hearing level, and auditory brainstem response results. A full discussion of these criteria is beyond the scope of this chapter. It is not yet well established whether intraoperative auditory monitoring materially improves hearing conservation results. In one study using auditory brainstem response monitoring, it was found to be of marginal benefit overall, with the possible exception of tumors less than 1 cm in diameter.38 Several retrospective studies have compared hearing preservation rates after the retrosigmoid and middle fossa approaches.39-41 In each study, the middle fossa approach yielded significantly better hearing results. Although that choice has not yet been universally accepted, the trend among centers undertaking a large volume of acoustic neuroma surgical procedures is to use the middle fossa approach as the preferred means of attempting hearing conservation. In our institution, the upper limit on the use of the extended middle fossa approach is a tumor in the 15 mm range of extracurricular diameter. The retrosigmoid approach is reserved for acoustic neuromas when three conditions are met: (1) excellent hearing, (2) a cisternal component 15 to 20 mm, and (3) no tumor involvement of the distal one third of the IAC. The retrosigmoid approach is still used for selected nonacoustic tumors of the CPA (e.g., meningiomas and epidermoids). Incomplete resection also has a potential role in hearing preservation, particularly in patients with neurofibromatosis type 2 or in patients with a tumor in an only hearing ear.12,42
COMPLICATIONS The common complications of the retrosigmoid approach to the CPA are persistent headache and CSF leakage.11,43,44 Less common complications include (aseptic or bacterial) meningitis, hydrocephalus, cerebellar dysfunction, vascular compromise (thrombosis and hemorrhage), and problems associated with patient malpositioning during surgery. Medical complications, such as pulmonary thromboembolism and pneumonia, may also occur, but are not specific to surgery of this region. Although the potential complications of acoustic neuroma surgery are similar among the various operative approaches, their relative incidence varies considerably. In the retrosigmoid approach, persistent headache and CSF leakage occur more frequently than with the other technique used in approaching CPA tumors.
Vascular Complications Hemorrhage Vascular complications may be extra-axial or intra-axial. The main extra-axial problem is bleeding into the CPA. CPA hematomas may cause brainstem compression
and acute obstructive hydrocephalus. The incidence of acute CPA hematomas has been reported to be 0.5% to 2%; however, with modern hemostatic techniques, the incidence is probably considerably less frequent.11 This diagnosis should be suspected when a patient does not promptly awaken after surgery or has a delayed deterioration in the level of consciousness. The diagnosis may be made by noncontrast CT scan, in which fresh blood appears as a hyperdense mass in the CPA and extrinsic pontine compression is noted. If serious neurologic sequelae and death are to be avoided, prompt surgical evacuation of the hemorrhage is essential. Intra-axial pontine hemorrhage may occur, particularly after removal of very large tumors that have greatly deflected the brainstem. Although major parenchymal hemorrhage is rare, minor amounts of intrinsic pontine bleeding are often evident radiographically after extirpation of giant tumors. Presumably, these bleeds result form the sudden re-expansion of the deeply compressed parenchyma. Supratentorial intra-axial hemorrhages have been reported after retrosigmoid approaches performed with the patient in the sitting position. These hemorrhages were associated with hypertension and may have resulted from subcortical venous tearing resulting from mechanical stress induced by the sitting position.45-47 Extradural hematoma formation, a concern in the middle fossa approach, is uncommon after the retrosigmoid approach.
Anterior Inferior Cerebellar Artery Syndrome Brainstem infarction may occur after damage to the AICA, the vascular supply to the pons and cerebellar peduncle. Mechanisms of injury include disruption, cauterization, and arteriospasm with thrombosis. A full-fledged AICA syndrome is extremely serious and is often fatal because it results in the loss of respiratory center control.48 Partial interruption of flow in the AICA system, avulsion of one or more of its branches, or obstruction of a nondominant AICA may result in an incomplete AICA syndrome.17 More recently, we have recognized several patients operated on for acoustic neuromas greater than 3 cm in diameter in whom gadolinium-enhanced MRI detected an infarction in the region of the middle cerebellar peduncle. These patients had unilaterally impaired cerebellar function and required prolonged physical therapy rehabilitation.49
Nonvascular Complications Complications from Patient Positioning As with any craniotomy, air embolism through breach of the major venous sinuses is a potential hazard. This risk is minimal, however, when a supine or lateral patient position is used.50 Air embolism is the main complication of the sitting position and has been reported in 30% of cases.51 When the sitting position is used, intraoperative monitoring with precordial Doppler ultrasonography alerts the anesthesiologist to venous air entry.
Chapter 50 • Retrosigmoid Approach to Tumors of the Cerebellopontine Angle
The initial maneuvers to perform when air embolism has been detected are to flood the field with fluid and lower the head of the bed. Quadriplegia (in four cases) and paraplegia have also been reported after acoustic neuroma resection in the sitting position. The degree of cervical flexion in the absence of protective spinal reflexes during anesthesia was thought to have caused spinal cord compression and infarction.52,53 In the supine and lateral supine positions, the unconscious patient must be handled carefully, especially when the head holder is positioned. Excessive head rotation risks cervical injury and may obstruct vertebral venous drainage and contribute to cerebellar swelling. Excessive downward displacement of the shoulder risks traction injury on the brachial plexus. As with any prolonged surgical procedure, adequate padding under pressure points is important to avoid pressure ulceration. Despite the best of precautions, patients frequently complain of discomfort over the ischium or other bony prominences for a few weeks postoperatively.
Cerebrospinal Fluid Leakage CSF leakage is the most common postoperative complication, occurring in approximately 15% of patients who undergo retrosigmoid approaches for acoustic neuroma.54-56 The patient must be counseled to recognize and report CSF leakage so that steps can be taken to control it rapidly to prevent infectious meningitis. CSF leak occurs either directly through the wound or indirectly through the ear and auditory tube to the nasopharynx, where it manifests as a watery rhinorrhea or salty postnasal discharge. CSF escape into the ear may occur through opened and unsealed mastoid air cells in the region of the craniectomy or through air cells opened and unsealed in the bony IAC dissection.57 CSF drainage often stops spontaneously with simple fluid restriction and avoidance of straining. The use of acetazolamide, a carbonic anhydrase–inhibiting diuretic, may also be beneficial. Alternatively, the early use of a lumbar CSF drain for 48 to 72 hours may halt the drainage. Some authors have advocated (1) wound reexploration with rewaxing of the bone to close covert open air cells, (2) replacement of the muscle graft plug to close CSF leakage, and (3) continued lumbar drainage.50 We prefer to address persistent, intractable CSF otorhinorrhea transtemporally. When useful residual hearing is present, a canal wall up mastoidectomy is performed, perilabyrinthine cells are copiously waxed, the fossa incudis is occluded with a fascia graft, and fat is used to obliterate the cavity. When the operated ear is deaf, a canal wall down mastoidectomy is performed. The external auditory canal is sutured closed, and the auditory tube is sealed under direct vision with bone wax and muscle. The mastoid air cells are also waxed, and the cavity is obliterated with fat. Lumbar CSF drainage is maintained for approximately 72 hours after surgery.
617
Aseptic and Bacterial Meningitis Entry of blood and bone dust into the subarachnoid space can result in aseptic meningitis. Care is taken during the drilling of the posterior petrous face during the IAC exposure to prevent contamination of the subarachnoid space with bone dust. Gelfoam is placed in the CPA superior and inferior to the tumor and CN VII-VIII complex, and a rubber dam is placed over the cerebellum. After completion of the bone work, the wound is thoroughly irrigated, and the bone debris is removed. Similarly, throughout the tumor dissection and at its completion, a combination of suction and irrigation is used to prevent the buildup of blood and clots because blood and bone debris produce an irritative or chemical aseptic meningitis.11 To reduce the risk of bacterial meningitis, intravenous prophylactic antibiotics are administered at the start of surgery, and bacitracin is added to the irrigant solution used to flush the CPA at the end of the procedure. This complication should be suspected if the patient develops headache, fever, and malaise in the first postoperative week. Nuchal rigidity, usually considered a sign of meningeal irritation, is not a useful sign after retrosigmoid craniotomy because the neck muscles may be in spasm owing to direct surgical trauma. Bacterial meningitis may also occur in the late postoperative period, particularly when a CSF leak is present. The clinician should maintain a high degree of suspicion about bacterial meningitis and, when in doubt, should obtain a sample of CSF via lumbar puncture for analysis. In patients in whom the clinical picture is suggestive, intravenous antibiotics should be instituted pending results of culture and sensitivity testing.
Hydrocephalus Hydrocephalus can occur as a result of blood and bone debris contamination of the posterior fossa subarachnoid space. Particulate and proteinaceous debris becomes ingested by arachnoid granulations, impairing their absorptive capabilities, which results in raised intracranial pressure. Cerebellar retraction with subsequent swelling at release may also result in the development of hydrocephalus.11
Cerebellar Dysfunction Prolonged cerebellar retraction may result in edema and swelling and possibly contusion, with resultant dysmetria and impaired balance in the postoperative period.
Persistent Headache Headache is encountered more frequently after the retrosigmoid approach than after other types of posterior fossa craniotomy.58 In our experience, nearly all retrosigmoid patients have substantial headache during the first
618
OTOLOGIC SURGERY
postoperative month. By 3 months after surgery, approximately one third continue to complain of this symptom. By 1 year, approximately 15% of patients continue to have chronic moderate to severe headaches compared with very few headaches for patients who underwent the translabyrinthine procedure. Some individuals are unable to return to work or resume other life activities because of this symptom. The highest incidence of persistent headache in our series has been in patients with small tumors who underwent the retrosigmoid approach in an effort to preserve hearing. Although the headache may have myriad presentations, it is most commonly either frontal or referred to the area of surgery and is often triggered by cough. Numerous potential underlying causes exist for chronic headache after retrosigmoid craniotomy, including aseptic meningitis, coupling of the suboccipital dura to the nuchal musculature, occipital neuralgia, and exacerbation of an underlying headache tendency, such as migraine. Although numerous mechanisms are possible, we believe that most headaches are a result of chronic arachnoiditis incited by contamination with bone dust and blood at the time of surgery. One apparent risk factor for the development of chronic headaches is retrosigmoid craniectomy, in which the calvarial bony defect is left unreconstructed.59,60 Replacement of the retrosigmoidal bone with a flap, bone chips, or even alloplastic material diminishes the incidence of persistent headache.61,62
Residual or Recurrent Tumor We prefer to revise recurrent tumors after retrosigmoid craniotomy using a translabyrinthine approach. This method avoids the previously scarred dural areas and tends to present more favorable arachnoid dissection planes during the early portion of the procedure.63
Acknowledgment Figures 51-1 to 1-10 were adapted from artwork produced by the authors from Jackler RK: Atlas of Skull Base Surgery and Neurotology, 2nd ed. New York, Thieme. 2009. The original drawings in this chapter were produced by Christine Gralapp, MA.
REFERENCES 1. Krause F: Zur Freilegung der hinteren Felsenbeinflache und des Kleinhirns. Beitr Klin Chir 37:728-764, 1903. 2. Rhoton A L Jr, Tedeschi H : Microsurgical anatomy of acoustic neuroma. Otolaryngol Clin North Am 25:257294, 1992. 3. Lang J: Clinical Anatomy of the Posterior Cranial Fossa and its Foramina. New York, Thieme, 1991. 4. Lang J Jr, Samii A : Retrosigmoidal approach to the posterior cranial fossa: An anatomical study. Acta Neurochir (Wien) 111:147-153, 1991.
5. Camins M B, Oppenheim J S : Anatomy and surgical techniques in the suboccipital transmeatal approach to acoustic neuromas. Clin Neurosurg 38:567-588, 1992. 6. Silverstein H, Morrell H, Smouha E, Jones R : Combined retrolabretrosigmoid vestibular neurectomy: An evolution in approach. Am J Otol 10:166-169, 1989. 7. Jackler R K, Pitts L H : Selection of surgical approach to acoustic neuroma. Otolaryngol Clin North Am 25:361387, 1992. 8. Selesnick S H, Jackler R K : Clinical manifestations and audiologic diagnosis of acoustic neuromas. Otolaryngol Clin North Am 25:521-551, 1992. 9. Cohen N L , Hammerschlag P, Berg H, Ransohoff J: Acoustic neuroma surgery: An eclectic approach with an emphasis on hearing preservation. Ann Otol Rhinol Laryngol 95:21-27, 1986. 10. Cohen N L : Retrosigmoid approach for acoustic tumor removal. Otolaryngol Clin North Am 25:295-310, 1992. 11. Wiet R J, Teixido M, Liang JG: Complications in acoustic neuroma surgery. Otolaryngol Clin North Am 25:389412, 1992. 12. Glasscock M E III, Hart M J, Vrabec JT: Management of bilateral acoustic neuroma. Otolaryngol Clin North Am 5:449-469, 1992. 13. Shelton C : Hearing preservation in acoustic tumor surgery. Otolaryngol Clin North Am 25:609-621, 1992. 14. Yates PD, Jackler R K, Satar B, et al: Is it worthwhile to attempt hearing preservation in larger acoustic neuromas? Otol Neurotol 24:460-464, 2003. 15. Nassif PS, Shelton C, Arriaga M M : Hearing preservation following surgical removal of meningiomas affecting the temporal bone. Laryngoscope 102:1357-1362, 1992. 16. Blevins N, Jackler R K : Exposure of the lateral extremity of the internal auditory canal via the retrosigmoid approach: A radioanatomic study. Otolaryngol Head Neck Surg 11:81-90, 1994. 17. Hegarty J L , Jackler R K, Rigby PL , et al: Distal AICA syndrome following acoustic neuroma surgery. Otol Neurotol 23:560-571, 2002. 18. Bloch D, Oghalai J S, Jackler R K, Pitts L H : The role of less-than-complete resection of acoustic neuroma. Otolaryngol Head Neck Surg 130:104-112, 2004. 19. Cheung SW, Jackler R K, Pitts L P, Gutin PH : Interconnecting the posterior and middle fossa for tumors which traverse Meckel’s cave. Am J Otol 16:200-208, 1995. 20. Seoane E, Rhoton A L Jr: Suprameatal extension of the retrosigmoid approach: Microsurgical anatomy. Neurosurgery 44:553-560, 1999. 21. Samii M, Tatagiba M, Carvalho G A : Retrosigmoid intradural suprameatal approach to Meckel’s cave and the middle fossa: Surgical technique and outcome. J Neurosurg 92:235-241, 2000. 22. Yingling C D, Gardi J N: Intraoperative monitoring of facial and cochlear nerves during acoustic neuroma surgery. Otolaryngol Clin North Am 25:413-448, 1992. 23. MacDonald C B, Hirsch B E, Kamerer D B, Sekhar L : Acoustic neuroma surgery: Predictive criteria for hearing preservation. Otolaryngol Head Neck Surg 104:128, 1991. 24. Goksu N, Bayazit Y, Kemaloglu Y: Endoscopy of the posterior fossa and dissection of acoustic neuroma. J Neurosurg 91:776-780, 1999.
Chapter 50 • Retrosigmoid Approach to Tumors of the Cerebellopontine Angle 25. Wackym PA, King WA, Poe DS, et al: Adjunctive use of endoscopy during acoustic neuroma surgery. Laryngoscope 109:1193-1201, 1999. 26. Lye R H, Pace-Balzan A, Ramsden RT, et al: The fate of tumour rests following removal of acoustic neuromas: An MRI Gd-DTPA study. Br J Neurosurg 6:195-201, 1992. 27. Thedinger B S, Whittaker C K, Luetje C M : Recurrent acoustic tumor after a suboccipital removal. Neurosurgery 29:681-687, 1991. 28. Mazzoni A, Calabrese V, Danesi G: A modified retrosigmoid approach for direct exposure of the fundus of the internal auditory canal for hearing preservation in acoustic neuroma surgery. Am J Otol 21:98-109, 2000. 29. Arriaga M, Gorum M : Enhanced retrosigmoid exposure with posterior semicircular canal resection. Otolaryngol Head Neck Surg 115:46-48, 1996. 30. Kemink J L , Langman AW, Niparko J K, Graham M D: Operative management of acoustic neuromas: The priority of neurologic function over complete resection. Otolaryngol Head Neck Surg 104:96-99, 1991. 31. Moffat D A : Synopsis on near-total, subtotal, or partial removal: Acoustic neuroma. In Tos M, Thomsen J (eds): Proceedings of the First International Conference on Acoustic Neuroma, Copenhagen, August 25-29, 1991. New York, Kugler Publications, 1992, pp 983-984. 32. Baer S, Tos M, Thomsen J, Hughes G: Synopsis on grading of facial nerve function after acoustic neuroma treatment: Acoustic neuroma. In Tos M, Thomsen J (eds): Proceedings of the First International Conference on Acoustic Neuroma, Copenhagen, August 25-29, 1991. New York, Kugler Publications, 1992, pp 993-995. 33. Sanna M, Gamoletti J, Tos M, Thomsen J: Synopsis on hearing preservation following acoustic neuroma surgery: Acoustic neuroma. In Tos M, Thomsen J (eds): Proceedings of the First International Conference on Acoustic Neuroma, Copenhagen, August 25-29, 1991. New York, Kugler Publications, 1992, pp 985-987. 34. Lalwani A, Butt FY, Jackler R K, et al: Facial nerve outcome after acoustic neuroma surgery: A study from the era of cranial nerve monitoring. Otolaryngol Head Neck Surg 111:561-570, 1994. 35. Hinton A E, Ramsden RT, Lye R H, Dutton J E : Criteria for hearing preservation in acoustic schwannoma surgery: The concept of useful hearing. J Laryngol Otol 106:500503, 1992. 36. Shelton C, Hitselberger WE, House WF, Brackmann D E : Long-term results of hearing after acoustic tumor removal: Acoustic neuroma. In Tos M, Thomsen J (eds): Proceedings of the First International Conference on Acoustic Neuroma, Copenhagen, August 25-29, 1991. New York, Kugler Publications, 1992, pp 661-664. 37. McKenna M J, Halpin C, Ojemann RG, et al: Long-term hearing results in patients after surgical removal of acoustic tumors with hearing preservation. Am J Otol 13:134136, 1992. 38. Slavit D H, Harner SG, Harper C M Jr, Beatty CW: Auditory monitoring during acoustic neuroma removal. Arch Otolaryngol Head Neck Surg 17:1153-1157, 1991. 39. Staecker H, Nadol J B, Ojeman R , et al: Hearing preservation in acoustic neuroma surgery: Middle fossa versus retrosigmoid approach. Am J Otol 21:399-404, 2000.
619
40. Irving R M, Jackler R K, Pitts L H : Hearing preservation in patients undergoing vestibular schwannoma surgery: Comparison of middle fossa and retrosigmoid approaches. J Neurosurg 88:840-845, 1998. 41. Arriaga M A, Chen D A, Fukushima T: Individualizing hearing preservation in acoustic neuroma surgery. Laryngoscope 107:1043-1047, 1997. 42. Wigand M E, Haid T, Goertzen W, Wolf S : Preservation of hearing in bilateral acoustic neurinomas by deliberate partial resection. Acta Otolaryngol (Stockh) 112:237-241, 1992. 43. Mangham C A : Complications of translabyrinthine versus suboccipital approach for acoustic tumor surgery. Otolaryngol Head Neck Surg 99:396-400, 1988. 44. Ebersold M J, Harner SG, Beatty CW, et al: Current results of the retrosigmoid approach to acoustic neurinoma. J Neurosurg 76:901-909, 1991. 45. Haines J H, Maroon JC, Janetta PJ: Supratentorial intracerebral hemorrhage following posterior fossa surgery. J Neurosurg 49:881, 1978. 46. Harders A, Gilbach J, Weigel K : Supratentorial spaceoccupying lesions following infratentorial surgery: Early diagnosis and treatment. Acta Neurochir (Wien) 74:57, 1985. 47. Seiler RW, Zurbrugg H R : Supratentorial intracerebral hemorrhage after posterior fossa operation. Neurosurgery 18:472, 1986. 48. Atkinson J: The anterior cerebellar artery: Its variations, pontine distribution, and significance in the surgery of cerebellopontine angle tumours. J Neurol Neurosurg Psychiatry 12:137-151, 1949. 49. Hegarty JL, Jackler RK, Rigby PL, Pitts LP, Cheung SC: Distal AICA syndrome following acoustic neuroma surgery. Otol Neurotol 23:560-571, 2002. 50. Harner SG, Beatty CW, Ebersold M J: Retrosigmoid removal of acoustic neuroma: Experience 1978-1988. Otolaryngol Head Neck Surg 103:40-45, 1990. 51. Duke D A, Lynch JJ, Harner SG, et al: Venous air embolism in sitting and supine patients undergoing vestibular schwannoma resection. Neurosurgery 42:1282-1286, 1998. 52. Hitselberger WE, House WF: A warning regarding the sitting position for acoustic tumor surgery. Arch Otolaryngol Head Neck Surg 106:69, 1980. 53. Samii M, Turel K E, Penker G: Management of seventh and eighth nerve involvement by cerebellopontine angle tumors. Clin Neurosurg 32:242, 1985. 54. Becker S S, Jackler R K, Pitts L P. CSF Leak after acoustic neuroma surgery: A comparison of the translabyrinthine, middle fossa, and retrosigmoid approaches. Otol Neurotol 24:107-112, 2003. 55. Baird C J, Hdeib A, Suk I, et al: Reduction of cerebrospinal fluid rhinorrhea after vestibular schwannoma surgery by reconstruction of the drilled porus acusticus with hydroxyapatite bone cement. J Neurosurg 107:347-351, 2007. 56. Falcioni M, Romano G, Aggarwal N, Sanna M : Cerebrospinal fluid leak after retrosigmoid excision of vestibular schwannomas. Otol Neurotol 29:384-386, 2008. 57. Smith PG, Leonetti J P, Grubb R L : Management of cerebrospinal fluid otorhinorrhea complicating the retrosigmoid approach to the cerebellopontine angle. Am J Otol 11:178-180, 1990.
620
OTOLOGIC SURGERY
58. Schessel D A, Nedzelski J M, Rowed Feghali JG: Headache and local discomfort following surgery of the cerebellopontine angle: Acoustic neuroma. In Tos M, Thomsen J (eds): Proceedings of the First International Conference on Acoustic Neuroma, Copenhagen, August 25-29, 1991. New York, ������������������������������������������������������������ Kugler Publications, 1992, pp 899-904. 59. Koperer H, Deinsberger W, Jodicke A, Boker D K : Postoperative headache after the lateral suboccipital approach: Craniotomy versus craniectomy. Minim Invasive Neurosurg 42:175-178, 1999. 60. Feghali JG, Elowitz E H: Split calvarial graft cranioplasty for the prevention of headache after retrosigmoid resection of acoustic neuromas. Laryngoscope 108:1450-1452, 1998.
61. Silverman D A, Hughes G B, Kinney S E, Lee J H : Technical modifications of suboccipital craniectomy for prevention of postoperative headache. Skull Base 14:77-84, 2004. 62. Schaller B, Baumann A : Headache after removal of vestibular schwannoma via the retrosigmoid approach: A long-term follow-up-study. Otolaryngol Head Neck Surg 128:387-395, 2003. 63. Beatty CW, Ebersold M J, Harner SG: Residual and recurrent acoustic neuromas. Laryngoscope 97:11681171, 1987.
51
Transotic Approach Ugo Fisch and Joseph M. Chen
The transotic approach to the cerebellopontine angle (CPA) was first introduced in 1979 by one of us (U.F.) in response to the limitations of the translabyrinthine technique. The objective of this approach is to obtain a direct lateral exposure and the widest possible access to the CPA through the medial wall of the temporal bone, from the superior petrosal sinus to the jugular bulb, and from the internal carotid artery to the sigmoid sinus. The tympanic and mastoid portions of the fallopian canal are left in situ. This transtemporal access is achieved at the expense of bony exenteration, rather than cerebellar retraction. Despite well-documented technical details,1 there is a general misconception equating the transotic approach with the transcochlear approach of House and Hitselberger.2 Significant differences exist between the two approaches in the extent of exposure, the management of the facial nerve, and the obliteration of the surgical cavity. As a natural extension of subtotal petrosectomy, which forms the basis of lateral and posterior skull base surgery at the University of Zurich,1 the trans otic approach was initially designed for acoustic neuromas and has since expanded to include other pathology. Several modifications were also made over the years to optimize its use.3-5
INDICATIONS Acoustic Neuroma Although the transotic approach, similar to the translabyrinthine approach, can be used for tumors of all sizes, it is ideal for tumors 2.5 cm or less in their mediolateral extent, in patients with no serviceable hearing. In this clinical setting, the transotic approach offers the best possible exposure for tumor extirpation and preservation of facial nerve with minimal morbidity. At the University of Zurich, tumors larger than 2.5 cm that cause significant brainstem compression are managed by the neurosurgery department as a matter of departmental policy. Small intracanalicular tumors in patients with good hearing (using the 50/50 rule of at least
50 dB hearing loss and 50% discrimination score) are managed through a middle cranial fossa (transtemporalsupralabyrinthine) approach (see Chapter 35).
Other Lesions Other lesions involving the CPA or the temporal bone with invasion of the internal auditory canal (IAC) or the otic capsule could also be approached via the transotic technique. These lesions include the following:
• Epithelial cysts (congenital cholesteatoma) • Arachnoid cysts • Hemangiomas • Giant cholesterol and mucosal cysts • Jugular foramen schwannomas • Temporal paragangliomas (glomus tumors) These lesions can be quite extensive and may require a combined infratemporal fossa type A or B approach for added exposure.
PREOPERATIVE EVALUATION The evaluation for retrocochlear lesions, such as acoustic neuromas, is standard at the University of Zurich and includes routine audiometry, auditory brainstem response, electronystagmography, and magnetic resonance imaging (MRI) with gadolinium enhancement. High-resolution computed tomography (CT) is still performed for bony assessment of lesions within or invading the temporal bone. Facial nerve status is recorded clinically using the Fisch grading system6 and quantified by electroneuronography before surgery. In addition to a candid discussion of surgical and postoperative complications, patients are informed of the advantages of the transotic approach specifically with regard to the preservation of the facial nerve and the complete obliteration of the surgical cavity, with blind sac closure of the external auditory canal to minimize cerebrospinal fluid (CSF) leak. The option 621
622
OTOLOGIC SURGERY
of conservative management by close monitoring with serial MRI to follow tumor growth is presented to all patients and is recommended for patients with nonprogressive long-standing symptoms (especially elderly patients), patients with small tumors and normal hearing, patients with significant medical illnesses, or patients who refuse to undergo surgery. These patients are told that rapid tumor growth ultimately requires surgical attention, and that facial nerve function and hearing preservation may be compromised as a result of the delay in surgery.
Skin Incision
SURGICAL TECHNIQUE
A mastoid periosteal flap is developed while the postauricular skin flap is elevated. The external auditory canal is transected, and its skin is elevated, everted externally, and closed as a blind sac. A second layer of closure using the mastoid periosteal flap ensures a complete seal (Fig. 51-2).
Preoperative Preparation The patient is premedicated with clonidine, metoclopramide, and midazolam before surgery. A perioperative antibiotic, ceftriaxone (Rocephin), 2 g intravenously for 24 hours, is given at the time of surgery until the removal of intravenous infusion, usually by the 3rd day after surgery.
Surgical Site Preparation, Positioning, and Draping The surgical site is prepared in the operating room after induction of anesthesia. Hair over the temporal area is shaved 9 cm above and 5 cm behind the pinna. The skin is washed with povidone-iodine. The abdomen and the contralateral leg are also shaved and prepared for fat harvesting and the possible need of a sural nerve graft. The positioning and draping for this procedure are similar to positioning and draping described in Chapter 1, with some minor differences. The patient is secured in supine position on the Fisch operating table (see Chapter 35), with the head turned away from the surgeon. A large plastic bag is incorporated into the draping to catch excess irrigation and blood.
Intraoperative Monitoring and Concerns Intraoperative facial nerve monitoring using the Xomed nerve integrity monitor (NIM-II) and percutaneous electromyography needles is standard with this approach. Intracranial pressure is controlled by deep anesthesia induced intravenously before introduction of inhalational anesthetics. Partial pressure of carbon dioxide is maintained at 30 to 40 mm Hg. Pharmacologic manipulation with dexamethasone (Decadron), 4 mg every 8 hours perioperatively and 4 days postoperatively, and mannitol, 0.5 mg/kg intravenously intraoperatively, is also standard. Furosemide is added when necessary. Lumbar CSF drainage is not routinely performed. Hypotensive anesthesia with nitroglycerin or clonidine (Catapres), or both, is used in most cases to maintain a systolic blood pressure of 80 to 100 mm Hg.
A postauricular incision is placed along the hairline to keep it behind the operative cavity (Fig. 51-1). The incision is made from the mastoid tip to the temporal region for the surgical approach; its superior extension is made at the time of wound closure for the exposure of the temporalis muscle flap.
Blind Sac Closure of External Auditory Canal
Subtotal Petrosectomy: Exposure of Jugular Bulb and Petrous Carotid A complete mastoidectomy is performed, and the remaining external auditory canal skin, tympanic membrane, and ossicles are removed in a stepwise fashion. The tympanic bone is progressively thinned out, and a complete exenteration of the pneumatic spaces (retrofacial, retrolabyrinthine, supralabyrinthine, hypotympanic, infralabyrinthine, and pericarotid) is carried out. Figure 51-3 shows the surgical cavity at the completion of this step. The middle fossa dura, sigmoid sinus, and jugular bulb are blue-lined; the fallopian canal and the vertical portion of the petrous carotid artery are skeletonized. The mastoid tip is removed to reduce the depth of the surgical cavity.
Obliteration of Eustachian Tube The mucosa of the membranous eustachian tube orifice is coagulated, and the bony canal is obliterated with bone wax at the isthmus. An additional muscle plug is used before closure.
Exenteration of Otic Capsule With the completion of subtotal petrosectomy, the surgical cavity is divided into two compartments by the fallopian canal (Fig. 51-4). Because the enlarged IAC lies mostly deep within the anterior compartment, the advantage of the transotic approach to access this region fully is clear. To begin this step, the semicircular canals are removed, and the vestibule is opened as in the translabyrinthine approach. The posterior aspect of the IAC is exposed from the fundus to the porus, leaving a thin layer of bone over the meatal dura. The posterior fossa dura of the posterior compartment is exposed caudal to the
623
Chapter 51 • Transotic Approach BLIND SAC CLOSURE OF THE EXTERNAL AUDITORY CANAL AFTER EVERSION OF THE CANAL SKIN
Vertical internal carotid a.
FIGURE 51-1. Skin incision. FIGURE 51-2. A and B, Blind sac closure of external auditory canal. FIGURE 51-3. Subtotal petrosectomy. EAC, external auditory canal. FIGURE 51-4. Position of internal auditory canal in relation to anterior and posterior compartments of operative cavity.
624
OTOLOGIC SURGERY
superior petrosal sinus and anterior to the sigmoid sinus. Retrofacial cells are subsequently removed to gain access over the inferior aspect of the IAC. Attention is now focused on the anterior compartment. The cochlea is drilled away to expose the enlarged IAC, which lies predominantly within this compartment, and as the dissection is carried forward, the dura anterior to the porus is also exposed to the level of the vertical segment of the ICA. Bony reduction between the jugular bulb and the inferior aspect of the IAC requires working beneath and over the fallopian canal, which is left in its anatomic position across the surgical field. Sufficient bone is left surrounding the canal to prevent accidental fracture. The cochlear aqueduct is identified between the jugular bulb and the IAC. The arachnoid of the aqueduct is opened to allow the outflow of CSF, decompressing the lateral cistern before the posterior fossa dura is opened. The tensor tympani muscle and bone medial to it are removed to gain more anterior access; likewise, bone medial to the vertical carotid artery is removed as much as possible.
Unroofing of Labyrinthine Portion of Facial Nerve Since 1988, the unroofing of the labyrinthine segment of the facial nerve from the meatal foramen to the geniculate ganglion has been incorporated as a standard step in the transotic approach. The meatal foramen can be found approximately 2 mm anterior and superior to the meatal fundus. This exposure provides additional room for the portion of the facial nerve most likely to sustain traction injury and edema subsequent to tumor manipulation. Also, the labyrinthine portion of the facial nerve serves as an important landmark while further access over the porus is gained along the superior petrosal sinus. The completed exposure is shown in Figure 51-5. The posterior fossa dura surrounding the porus is circumferentially exposed from the carotid artery to the sigmoid sinus, and from the jugular bulb to the level of the superior petrosal sinus.
Tumor Removal A few instruments are required for tumor removal. Bayonet and angled bipolar forceps, cup forceps, microraspatories, and a long suction with finger control are the most essential instruments. The intrameatal portion of the tumor is approached first and is separated from the facial nerve until the level of the porus. Figure 51-6 illustrates the advantage of the additional space obtained with the transotic exenteration, whereby the intrameatal portion of the tumor can be easily displaced and mobilized during its removal. The posterior fossa dura is incised between the sinodural angle and the posterior edge of the porus. The incision is extended superiorly and inferiorly along the
porus (Fig. 51-7). It is important to elevate the dura with a hook before making an incision to prevent the inadvertent injury of vessels over the cerebellum. The dural edges must be cauterized before extending the incision to facilitate hemostasis. One must also be acutely aware of the variations of the course of the anterior inferior cerebellar artery (AICA) and its branches. The superior and inferior dural flaps are retracted with 4-0 polyglactin 910 (Vicryl) sutures, which are clipped to the wound edges (Fig. 51-8). The full extent of the tumor usually can be shown. The posterior pole of the tumor abuts against the cerebellum and the petrosal vein, and the AICA courses anteroinferior to the tumor. Intracapsular reduction of the tumor can now begin and is continued until tumor margins can be seen without tension being placed on the facial nerve. During this step, the meatal dura at the superior pole of the porus is not detached to render some stability to the tumor. It is crucial to handle the tumor meticulously; manipulations should be carried out with suction over a Cottonoid, and the displaced facial nerve should always be in view to avoid undue traction (Fig. 51-9). Bleeding is diminished by coagulation of all visible vessels over the tumor capsule. The main blood supply to the tumor generally runs along CN VIII, and some may come from branches of the AICA. These vessels should be coagulated, cut on the tumor, and gently pulled away. With sufficient reduction, separation of the facial nerve can now be attempted. The advantage of the transotic approach is now easily appreciated because the displaced facial nerve can be followed in its entirety. The dural attachments of the tumor at the porus are cut, and the nerve can be gently grasped with bipolar forceps and teased away from the tumor (Fig. 51-10). Likewise, the AICA can be separated from the nerve by using the tips of the forceps or by pulling on the coagulated branches. At the inferior pole of the tumor, the origin of CN VIII and the course of the AICA looping around it are identified. In many instances, the root exit zone of the facial nerve, always anterior to CN VIII, is identified only after CN VIII is cut. The anterior access of the transotic approach offers an unparalleled view to an area that is usually partially hidden from the surgeon during suboccipital or translabyrinthine surgery.7,8 In this exact region, the facial nerve is most tenuous and frequently appears as a thin, transparent band. Any manipulation not under direct vision can easily rupture the nerve. When it is completely detached from all vital structures, the tumor can now be removed. The CPA and all its structures are exposed in Figure 51-11. The facial nerve is stimulated electrically to obtain a threshold response. Despite a normal response intraoperatively, the patient may still show an immediate or delayed facial paralysis because of impaired vascular supply and the inevitable trauma to the nerve during dissection. If stimulation fails to produce a response or facial contraction, and if the anatomic integrity of the nerve is precarious,
Chapter 51 • Transotic Approach
625
FIGURE 51-5. A and B, Exposure of internal auditory canal and posterior fossa dura. FIGURE 51-6. Intrameatal tumor dissection.
it is best to proceed with nerve grafting immediately. Failure to do so while waiting for the improbable return of facial function may delay reinnervation for 2 years. The details of intracranial/intratemporal and hypoglossal/facial crossover grafting techniques are beyond the scope of this chapter and are discussed elsewhere.1,9
Wound Closure A musculofascial graft that is slightly larger than the dural defect is taken from the temporalis muscle. It is placed under the dura and fixed in place with the two 4-0 Vicryl sutures used previously as stay sutures (Fig. 51-12). These sutures are passed through the edges of the graft and secured to the dura. A second temporalis fascial graft
is used to cover the opened IAC. A small muscle graft is also used as a plug to supplement the prior wax obliteration of the eustachian tube. Both grafts are stabilized with fibrin glue. A second layer of closure with abdominal fat grafts is to follow. A large piece of fat is first passed under the fallopian canal and firmly anchored (Fig. 51-13). Several small pieces of fat are used to fill out the surgical cavity and are stabilized with fibrin glue. The posterior half of the temporalis muscle is now transposed and sutured in place with 2-0 Vicryl sutures. Additional fat is placed under the muscle flap to create a slight compressive tension (Fig. 51-14). This type of closure has consistently minimized the incidence of postoperative CSF leaks, which is another advantage of
626
OTOLOGIC SURGERY DURAL INCISIONS
Facial nerve
INITIAL CPA EXPOSURE
V Suction
FIGURE 51-7. Dural incisions. CSF, cerebrospinal fluid; PFD, posterior fossa dura. FIGURE 51-8. Initial cerebellopontine angle (CPA) exposure. AICA, anterior inferior cerebellar artery. FIGURE 51-9. Intracapsular tumor reduction. FIGURE 51-10. Intracranial facial nerve dissection. AICA, anterior inferior cerebellar artery. FIGURE 51-11. View of cerebellopontine angle after tumor removal. AICA, anterior inferior cerebellar artery.
Chapter 51 • Transotic Approach
627
DURAL CLOSURE FASCIA
TEMPORALIS MUSCLE FLAP FAT OBLITERATION OF THE SURGICAL CAVITY
FIGURE 51-12. Dural closure. IAM, internal auditory meatus. FIGURE 51-13. Fat obliteration of surgical cavity. FIGURE 51-14. Temporalis muscle flap. SCM, Sternocleidomastoid. FIGURE 51-15. Cross-sectional views of transotic approach versus translabyrinthine approach. AN, acoustic neuroma; C, carotid; CER, cerebellum; CVN, cochleovestibular nerve; EAM, external auditory meatus; FN, facial nerve; ME, middle ear; SS, sigmoid sinus.
628
OTOLOGIC SURGERY
the transotic approach. A small plastic suction drain is inserted over the muscle flap while the skin incision is closed in two layers with 2-0 polyglycolic acid (Dexon) and 3-0 nylon sutures.
DRESSING AND POSTOPERATIVE CARE The suction drain is removed as soon as a compression dressing is applied. The dressing is left in place for 5 days, and if there is any evidence of CSF leak or subcutaneous CSF accumulation, the compression dressing is reapplied. The patient is transferred to the postanesthesia care unit extubated and fully awake. Routine and neurologic vital signs are closely monitored every 30 to 60 minutes. Adequate analgesics and antiemetics are ordered to keep the patient comfortably at bed rest, usually for 72 hours after surgery. Ambulation and oral intake are started slowly thereafter. Subcutaneous heparin is often given during the early convalescent period. An oral antibiotic, trimethoprim and sulfamethoxazole (Bactrim Forte) or ciprofloxacin (Ciproxin), is prescribed for at least 5 days after intravenous fluid and ceftriaxone therapy are discontinued. Head and abdominal wound sutures are removed on day 12, and leg sutures are removed after 2 weeks. The patient is discharged from the hospital at this time, barring any complication.
TIPS AND PITFALLS The transotic approach is more than a combination of the translabyrinthine and transcochlear approaches. It uses the complete infralabyrinthine compartment of the temporal bone, from the carotid artery to the sigmoid sinus, and from the jugular bulb to the superior petrosal sinus. It provides the largest possible transtemporal access to the CPA, which can be best appreciated by comparing the cross-sectional surgical exposure of the transotic approach with the translabyrinthine approach shown in Figure 51-15. The preservation of the facial nerve in its anatomic position within the fallopian canal does not limit the visibility or illumination. Enough bone must be kept surrounding the canal initially during subtotal petrosectomy and subsequently while the otic capsule is exenterated. Skeletonization of the fallopian canal must be done progressively as the surgical cavity enlarges, and with only diamond burrs. Bone surrounding the proximal tympanic segment of the facial nerve and the superior aspect of the IAC should be left intact to support the facial nerve. If the fallopian canal is inadvertently fractured during dissection, it is likely to remain undisplaced and not to impede surgery. If no significant torsion or traction has occurred, no major adverse effects should result,
rovided that no further manipulation occurs. The fracp tured edges can be supported with fibrin glue, and at the time of closure, abdominal fat adequately renders support from beneath. If the fracture is displaced and unstable, a small, malleable aluminum strip can be used as a retractor and can maintain the fallopian canal in position. If this is impossible, the nerve may have to be fully unroofed and transposed anteriorly as in the infratemporal fossa type A approach. This situation has not occurred in our hands, however. The three fundamental principles for the removal of acoustic neuromas are as follows: 1. Perform intracapsular reduction of the tumor to gain better visibility progressively around the circumference of the tumor 2. Expose tumor from lateral to medial, coagulate all visible vessels over the tumor capsule, and remove the devascularized portion in a piecemeal fashion 3. Separate the facial nerve from the tumor and not vice versa CSF outflow after the opening of the cochlear aqueduct decompresses the lateral cistern before the dural incision is made. It also indicates that the pars nervosa of the jugular foramen and the lower cranial nerves have not yet been reached by the tumor. Even in the presence of a high jugular bulb, a few millimeters of exposure can be obtained between it and the IAC to allow adequate access to the inferior pole of the tumor. Unroofing and compressing the jugular bulb to gain more exposure are unnecessary and dangerous. If access to the CPA is severely limited by the jugular bulb, the facial nerve can be unroofed and transposed anteriorly, as in the infratemporal fossa type A approach. Bony exenteration to expose the posterior fossa dura should be done in a stepwise manner, gaining as much exposure of the posterior fossa dura around the porus as possible. The time spent in the initial bony exenteration may be tedious at first, but it is well rewarded by much expanded access and improved illumination, which facilitate tumor removal dramatically. The dura should not be opened until all bony work has been completed, and hemostasis has been perfectly controlled. While incising the dura, the surgeons should beware of the AICA, which may loop underneath, and make the initial cut in the center of the exposure to avoid this artery, which usually lies in the inferior half of the CPA. Intracapsular reduction of the tumor is performed while it is still attached to the meatal dura at the superior pole of the porus to prevent excessive traction on the facial nerve. It also stabilizes the tumor during reduction. The major blood supply of acoustic tumors runs along CN VIII; one should always check for vessels on the undersurface of the tumor before removal. The most delicate portion of the facial nerve is just proximal to the acoustic porus, where the nerve can be flattened to a thin
629
Chapter 51 • Transotic Approach
COMPLICATIONS AND MANAGEMENT
transparent band. It is most frequently pushed anteriorly and superiorly. The surgeon must keep an eye on this area while working deeper in the CPA. If the facial nerve appears to be significantly traumatized and cannot be stimulated at the end of surgery, one must proceed directly to an intracranial-intratemporal grafting or CN XII-VII cross-innervation procedure, depending on the clinical setting. Spontaneous return of function with conservative treatment is not likely to occur. Posterior fossa dura should not be resected to gain exposure; the dural edges are retracted with stay sutures. One should not forget to obliterate the eustachian tube with bone wax and a muscle plug to prevent CSF rhinorrhea. Fat graft anchored under the fallopian canal supports the musculofascial repair and prevents lateralization of these autogenous tissues.
The obliteration of the surgical cavity and eustachian tube, along with the blind sac closure of the external auditory canal, has significantly reduced the incidence of CSF leak. In the series mentioned earlier, 4% of patients developed a subcutaneous CSF collection without leakage, and the collections usually resolved within 3 to 4 weeks with conservative management, including bed rest and prolonged compressive dressing over the surgical site. Three patients (4%) developed either immediate or delayed CSF leaks and were treated with lumbar drainage and bed rest; only one required surgical revision. One patient had meningitis and responded to antibiotic treatment without sequelae. Most notably is the lack of any other central nervous system complication in this series. One death occurred as a result of postoperative pulmonary embolism and cardiorespiratory failure. Wound infection with necrosis of the abdominal fat graft or temporalis muscle flap was a rare complication and was thought to be the inciting cause in the case of meningitis.
RESULTS Between 1979 and 1990, 147 consecutive transotic app roaches were performed for the removal of unilateral acoustic neuromas by the senior author (U.F.). Tumors in this series were limited in size from 1 to 2.5 cm in mediolateral extension. Tumors may fill the lateral cistern, abutting but without significantly compressing the brainstem. Complete tumor removal was achieved in all cases. Optimal visualization of the facial nerve was obtained, and the anatomic preservation of the nerve was possible in 139 cases (94.6%). In eight cases in which facial nerve integrity could not be preserved, intracranialintratemporal nerve grafting resulted in an average of 66% return of facial function by the Fisch facial nerve grading system (or House-Brackmann grade III or better), during a mean follow-up of 4 years. In this series, 66 patients were available for followup of at least 2 years. Tumors 1 to 1.4 cm were removed with no incidence of permanent facial injury, and 80% of patients with tumors 1.5 to 2.5 cm had normal or near-normal facial function (Table 51-1). Since 1988, the unroofing of the labyrinthine portion of the facial nerve has been a standard step in the transotic approach, which is believed to be a major contributing factor in the diminished incidence of delayed facial palsy in acoustic neuroma surgery.
ALTERNATIVE TECHNIQUES When and how acoustic neuromas should be operated on are issues of ongoing and often emotional debates. If surgery is contemplated, the aim is to try to obtain the safest and best possible exposure that would allow complete tumor extirpation and the preservation of facial nerve. Hearing preservation is of secondary concern if the opposite ear is functional. Translabyrinthine and suboccipital approaches are perhaps the most established and popular techniques, whereas the middle fossa approach has traditionally been reserved for small tumors in patients with serviceable hearing; an extended version of the middle fossa approach has gained popularity in some centers to remove tumors measuring 4.5 cm10,11 despite a seemingly high morbidity.12 The relative efficacy of each of these approaches is difficult to quantify without a randomized multi-institutional study. Our own experience with the translabyrinthine removal of acoustic neuromas before 1979 was unsatisfactory in many respects, and the problems were subsequently rectified with the transotic approach.1
TABLE 51-1 Facial Function Two Years Postoperatively % RECOVERY Tumor Size (cm) 1-1.4 1.5-2.5* Total *No
No.
100
80-99
60-79
40-59
0-39
14 52 66
100 61 70
— 19 15
— 12 9
— 4 3
— 4 3
difference in facial function after removal of tumors of 1.5-1.9 cm versus 2-2.5 cm.
630
OTOLOGIC SURGERY
There are three advantages of the transotic approach over the translabyrinthine approach, as follows: 1. A wider surgical access with a near-circumferential exposure of the IAC and the porus acusticus; this added exposure is particularly important in the presence of a high-riding jugular bulb and an anteriorly positioned sigmoid sinus 2. The direct visualization and access to the anterior CPA where the facial nerve is most tenuous and vulnerable 3. A much reduced rate of CSF leakage as a result of permanent closure of the ear canal and eustachian tube and complete obliteration of the surgical cavity
REFERENCES 1. Fisch U, Mattox D: Microsurgery of the Skull Base. New York, Thieme Publishers, 1988. 2. House WF, Hitselberger WE : The transcochlear approach to the skull base. Arch Otolaryngol Head Neck Surg 102:334-342, 1976. 3. Jenkins H A, Fisch U: The transotic approach to resection of difficult acoustic tumors of the cerebellopontine angle. Am J Otol 2:70-76, 1980.
4. Gantz B J, Fisch U: Modified transotic approach to the cerebellopontine angle. Arch Otolaryngol Head Neck Surg 109:252-256, 1983. 5. Chen J M, Fisch U: The transotic approach in acoustic neuroma surgery. J Otolaryngol 22:331-336, 1993. 6. Burres S, Fisch U: The comparison of facial grading systems. Arch Otolaryngol Head Neck Surg 112:755-758, 1986. 7. Whittaker C K, Leutje C M : Translabyrinthine removal of large acoustic neuromas. Am J Otol 7(Suppl):155-160, 1985. 8. Gardner G, Robertson J H : Transtemporal approaches to the cranial cavity. Am J Otol 7(Suppl):114-120, 1985. 9. Fisch U, Lanser M J: Facial nerve grafting. Otolaryngol Clin North Am 24:691-708, 1991. 10. Wigand M E, Haid T: Extended middle cranial fossa approach for acoustic neuroma surgery. Skull Base Surg 1:183-187, 1991. 11. Kanzaki J, Ogawa K, Yamamoto M, et al: Results of acoustic neuroma surgery by the extended middle cranial fossa approach. Acta Otolaryngol Suppl (Stockh) 487:1721, 1991. 12. Kanzaki J, Ogawa K, Tsuchihashi N, et al: Postoperative complications in acoustic neuroma surgery by the extended middle cranial fossa approach. Acta Otolaryngol Suppl (Stockh) 487:75-79, 1991.
52
Transcochlear Approach to Cerebellopontine Angle Lesions Antonio De la Cruz and Karen B. Teufert Videos corresponding to this chapter are available online at www.expertconsult.com.
The transcochlear approach was developed to treat midline intracranial lesions arising from the clivus, and cerebellopontine angle (CPA) masses arising anterior to the internal auditory canal (IAC), without requiring the use of brain retractors. These lesions may extend around the vertebrobasilar arteries. Because in the 1970s traditional surgical approaches were limited by the cerebellum and the brainstem, these lesions had been considered inoperable by many surgeons. The transcochlear approach does not have these limita tions, and was designed primarily for meningiomas arising from the petroclinoid ridge, intradural clivus lesions, chordomas, congenital petrous apex cholestea tomas, and primary intradural epidermoids anterior to the IAC. The transcochlear approach evolved from the inabil ity to excise the base of implantation and control the blood supply of these near-midline and midline tumors with other surgical approaches. Total removal of these lesions through a suboccipital approach is often impos sible because of the interposition of the cerebellum and the brainstem.1,2 The transpalatal-transclival approach was attempted for these intradural midline lesions, with little success, during the early 1970s.3 The exposure was often inadequate; the field is relatively far from the surgeon; the blood supply is lateral, away from the sur geon’s view; and the risk of intracranial complications caused by oral contamination is increased. The retro labyrinthine approach is limited in its forward exten sion by the posterior semicircular canal. Tumor access with the translabyrinthine approach is limited anteri orly by the facial nerve, which impedes removal of the tumor’s base of implantation, which is anterior to the IAC, around the intrapetrous carotid artery, or ante rior to the brainstem. The development of the extended middle fossa approach and combined transpetrous approach enables complete removal of petroclinoid meningiomas, and is used in patients with useful hear ing.4-6 The primary limitation with this approach is poor access to tumors with inferior or midline extensions.7,8 The endoscopic-assisted transsphenoidal approach has
been gaining acceptance as an alternative approach to access midline intracranial lesions arising from the clivus and petrous apex lesions (Stamm A, personal communication, 2008). The transcochlear approach was developed by House and Hitselberger2,3 in the early 1970s as an anterior extension of the translabyrinthine approach. It involves rerouting of the facial nerve posteriorly and the removal of the cochlea and petrous apex, which exposes the area of the intrapetrous internal carotid artery. This approach affords wide intradural exposure of the anterior CPA; CN V, VII, VIII, IX, X, and XI; both sixth cranial nerves; the clivus; and the basilar and vertebral arteries, without using any brain retractors. The contralateral cranial nerves and the opposite CPA are also visible.9 This wide exposure affords removal of the tumor base and its arterial blood supply from the internal carotid artery, which is particu larly important in the treatment of meningiomas.2 Adding excision and closure of the external auditory canal (EAC), as advocated by Brackmann (personal communication, 1987), increases further the anterior exposure for lesions of the petroclival regions and prepontine cistern. Smaller lesions can be reached without rerouting the facial nerve (transotic approach).
ADVANTAGES OF TRANSCOCHLEAR APPROACH This approach requires no cerebellar or temporal lobe retraction. Exposure and dissection of the petrous apex and clivus allows excellent exposure of the midline and complete removal of the tumor, its base of implantation, and its blood supply. This is of particular importance in meningiomas.10,11 Careful handling and constant monitoring of the facial nerve during rerouting prevent injury to the intra temporal portion of the nerve. Cholesteatomas tend to wrap themselves around it. If the facial nerve is lost dur ing tumor removal, we recommend immediate repair by end-to-end anastomosis or nerve graft interposition. 631
632
OTOLOGIC SURGERY
DISADVANTAGES OF TRANSCOCHLEAR APPROACH The main disadvantages of this approach are sacrifice of residual hearing in the operated ear and risk of temporary facial palsy. This technique is indicated when no service able hearing exists in the involved ear, or when the tumor is too far anterior for the extended middle fossa crani otomy approach or transpetrous approach. With the use of continuous facial nerve monitoring, the incidence of permanent facial nerve paralysis is low.
PATIENT EVALUATION AND PREOPERATIVE COUNSELING Individuals with tumors that require transcochlear surgery may have minimal symptoms, with tumors that can be quite large at the time of diagnosis.1,12 Petrous apex epidermoids manifest with unilateral hearing loss and tin nitus in 80% of the cases. Facial twitch is also common. Imbalance, ataxia, and parietal or vertex headaches may be the only complaints in 20% of patients.13 Patients with meningiomas and intradural epidermoids may be nearly symptom-free until they present with CN V findings and signs of increased intracranial pressure.14,15 There is a high rate of jugular foramen syndrome in patients with meningiomas.1 Seizures, dysarthria, and late signs of dementia from hydrocephalus were common presenting symptoms in the past.13 Hearing and vestibular functions are frequently normal, and acoustic reflex decay or abnor mal auditory brainstem response audiometric results may be the only anomalies.1 Imaging using high-resolution computed tomography (CT) with contrast enhancement, magnetic resonance imaging (MRI), and magnetic resonance angiography (MRA) is essential for diagnosis and surgical planning.16 Petrous apex and intradural epidermoids are expansile, spherical, or oval lesions, with scalloped bone edges on CT. They are isodense to cerebrospinal fluid on CT, with capsular enhancement. On MRI, they are hypoin tense on T1-weighted images and hyperintense on T2weighted images. Meningiomas enhance with contrast on MRI, and manifest with a “dural tail.” Evaluation of blood supply of some tumors may also require MRA. In tumors surrounding or invading the intrapetrous carotid artery, patency of the circle of Willis is accessed, and carotid residual pressure is measured; preopera tive balloon occlusion of the carotid artery and selec tive embolization are performed 1 day before surgery. Radioisotope, xenon, or positron emission tomography studies are used to assess cerebral perfusion during occlusion studies. The natural history of petrous apex epidermoids is that they grow slowly and may produce cranial neu ropathies; they may also become infected. Treatment is difficult when infection occurs, and meningitis, sepsis,
and death may result. Intracranial epidermoids spread through the cisterns and subarachnoid planes to neigh boring regions, including the opposite CPA. Petrous ridge meningiomas grow and are space-occupying lesions that increase intracranial pressure. After surgery, intracranial pressure is reduced, and cranial nerve symptoms tend to improve. Risks and complications in the immediate postoperative period include transient vertigo; com plete hearing loss; temporary CN VII, IX, X, XI, and XII paresis; infection; bleeding; swallowing difficulties; aspiration pneumonia; cerebrovascular accidents; and, rarely, death.
SURGICAL ANATOMY Intracranial structures that may be exposed by the trans cochlear approach include the entire lateral aspect of the pons and upper medulla, CN V through XI, and the mid basilar artery. Posterior fossa exposure is extensive except inferiorly, where it is limited in the area of the jugular foramen and foramen magnum. The degree to which the neural compartment of the jugular foramen is visible depends on the height of the jugular bulb. Modifications to the transcochlear approach permit identification of the anterior aspect of the pons and both sixth cranial nerves, and improved identification of the basilar artery and vertebrobasilar junction.
SURGICAL TECHNIQUE A wide mastoidectomy and labyrinthectomy are per formed, exposing the IAC. When first described by House and Hitselberger in 1976,3 the tympanic ring was not removed, and only the facial recess was opened to permit anterior exposure. Brackmann, after Fisch, modified the approach by removing the entire tympanic ring, malleus, incus and stapes, and blind-sac closing the EAC.17,18 The facial nerve is completely skeletonized, with transection of the greater superficial petrosal and chorda tympani nerves, and is rerouted posteriorly out of the fallopian canal. The cochlea and the fallo pian canal are completely drilled out, and the internal carotid artery is skeletonized. A large triangular win dow is created into the skull base. Its superior bound ary is the superior petrosal sinus; inferiorly, it extends below and medial to the inferior petrosal sinus into the clivus. Anteriorly lies the region of the intrapetrous internal carotid artery, and the apex of the triangle is just beneath Meckel’s cave. When the dura is opened, this window gives excellent direct access to the midline without need of any retraction (Fig. 52-1). After tumor removal, the dura is reapproximated with dural silk, the eustachian tube is packed with absorbable knitted fab ric (Surgicel) and muscle, and abdominal fat is used to
Chapter 52 • Transcochlear Approach to Cerebellopontine Angle Lesions
633
I.C.A.
T. B.a.
EAC
C.N. VII, VIII
FIGURE 52-1. Transcochlear approach. Exposure obtained at skull base at comple tion of approach. Ba, basilar artery; EAC, external auditory canal; ICA, internal carotid artery; SPS, superior petrosal sinus; SS, sigmoid sinus; T, tumor.
S. S. SPS
fill the dura and mastoidectomy defects and to cushion the facial nerve.
Setup General endotracheal anesthesia with direct arterial blood pressure monitoring is used, and a urinary catheter and a nasogastric tube are inserted. Long-acting muscle relaxants are avoided. Intraoperative nerve monitoring is used in all cases. Anesthesia is kept light so that changes in blood pres sure and pulse brought about by tumor manipulation are not masked. Prophylactic third-generation cephalosporin antibiotics and steroids are used routinely before the skin incision is made. Venous antiembolism compression boots are placed on the patient’s legs before the procedure begins. The patient is placed supine on the operating room table, with the head turned to the opposite side, and is maintained in a natural position without fixation. This position avoids air embolization, minimizes surgeon fatigue, and allows stabilization of the surgeon’s hands during the microsurgical procedure.
Incision A postauricular suboccipital incision is made 5 cm behind the postauricular fold, starting 1 cm above the ear, extend ing through the occipital bone and ending at the level of the mastoid tip. This incision can be extended inferiorly into the neck to provide control of the great vessels and of the lower cranial nerves, if necessary. The scalp flap is lifted anteriorly uncovering the temporalis fascia. The periosteum is incised just above the linea temporalis from the zygomatic root anteriorly to a level posterior to the sigmoid sinus. A second periosteal incision perpendicular to the previous one is carried inferiorly in the direction of the mastoid tip. The periosteal flap is elevated forward to the spine of Henle and to the level of the EAC. The skin of the EAC is usually left in place. In some cases
of lesions placed far anteriorly, removal of the tympanic bone and blind closure of the EAC are necessary. In this case, the skin, tympanic membrane, malleus, and incus are removed, and the meatus is closed in three layers.
Mastoidectomy An extended mastoidectomy (Fig. 52-2) is carried out with microsurgical cutting and diamond burrs and con tinuous suction-irrigation. Bone removal is started along two lines: one along the linea temporalis and another tangential to the EAC. The mastoid antrum is opened, and the lateral semicircular canal is identified. The lat eral semicircular canal is the most reliable landmark in the temporal bone, and allows the dissection to proceed toward delineating the vertical fallopian canal and the osseous labyrinth. The opening of the mastoid cavity must be as large as possible and is extended posterior to the sigmoid sinus, exposing 1 to 2 cm of suboccipital dura. The larger the tumor, the further back the posterior fossa dura is exposed, to a maximum of 2 to 3 cm. Mastoid emissary veins are dissected, and bleeding is controlled using bipolar cautery and Surgicel packing. Removal of bone over the sigmoid sinus is performed with diamond burrs, and an island of bone (Bill’s island) may be left over the dome of the sinus initially. This eggshell of bone protects the sinus from being injured by the shaft of the burr. Bone is removed from the sinodural angle along the superior petrosal sinus. The mastoid air cells are exenterated from the sinodural angle, skeletonizing the dura of the posterior and the middle fossae.
Closure of External Auditory Canal When further anterior exposure is required, removal and three-layer closure of the EAC are included. The canal skin is transected at the bony-cartilaginous junction
634
OTOLOGIC SURGERY Facial recess
EAC skin
MFD
PCW .
Fn
B. I.
FIGURE 52-3. Exposure of internal auditory canal, skeletonization of facial nerve from internal auditory canal to stylomastoid foramen, and extended facial recess. Labyrinthectomy has been completed. PFD bone Dura
FIGURE 52-2. Mastoidectomy. There is wide exposure of poste rior and middle fossa dura, with identification of bony labyrinth and s keletonization of sigmoid sinus. Bill’s island (BI) is preserved initial ly, then removed after internal auditory canal skeletonization. EAC, external auditory canal; MFD, middle fossa dura; PCW, posterior canal wall; PFD, posterior fossa dura; FN, facial nerve.
and is undermined laterally. Extra cartilage is removed, and the skin is closed with interrupted nylon sutures in a dimple-like fashion at the external auditory meatus. A flap of mastoid periosteum is developed on a pedicle just posterior to the EAC. This flap is rotated anteriorly and secured as a second layer of closure for the meatus. After removal of all of the canal skin, tympanic mem brane, and malleus, and incus, the bony EAC is drilled out and excised circumferentially. An extended facial recess is created before the posterior bony wall is removed. The eustachian tube is curetted and packed with Surgicel and temporalis muscle. Care is taken to avoid entering the glenoid fossa.
Labyrinthectomy and Skeletonization of Internal Auditory Canal Dissection of the perilabyrinthine cells down to the lateral semicircular canal is completed (Fig. 52-3). The facial nerve is identified in its vertical portion between the nonampullated end of the lateral semicircular canal and the stylomastoid foramen. At this stage, exposing the perineurium of the nerve is unnecessary, but it should be clearly and unmistakably identified in its vertical course.
The lateral semicircular canal is fenestrated superi orly, and the membranous portion is identified and fol lowed anteriorly to its ampullated end and posteriorly to the posterior semicircular canal. All three membranous and bony semicircular canals are removed, and the saccule and utricle in the vestibule are identified and removed. The dissection proceeds along the sinodural angle and the superior petrosal sinus. The dura of the posterior fossa is exposed anteriorly. The cells over the jugular bulb are removed, skeletonizing it, and the cochlear aqueduct is removed. To obtain a better control of the lower cra nial nerves and the vertebrobasilar junction, the jugular bulb is completely exposed and, if needed, compressed. The IAC is skeletonized, beginning inferiorly and then around the porus acusticus. The falciform crest (trans verse) and vertical crest (Bill’s bar) are used as identifying landmarks for the superior and inferior vestibular nerves and the facial nerve. The bone of Bill’s island over the sigmoid sinus is removed. The superior and inferior ves tibular and cochlear nerves are excised.
Facial Nerve Rerouting (Figs. 52-4 and 52-5) After removal of the incus, if the bony EAC was not removed, an extended facial recess opening is created. The facial nerve is completely skeletonized from the IAC to the stylomastoid foramen, including the genicu late ganglion, with diamond burrs. An area comprising 180 degrees of the bony fallopian canal is uncovered. The greater superficial petrosal nerve is cut at its origin from the geniculate ganglion. The facial nerve is reflected posteriorly out of the fallopian canal with care taken to avoid traction or bending on the nerve, especially near the geniculate ganglion, where the nerve is reduced in size and forms an acute angle, and near the mastoid genu, which is the site of several branches to the stapedius muscle.
635
Chapter 52 • Transcochlear Approach to Cerebellopontine Angle Lesions
G.G. F.C. IAC
VII
FD
MFD
M
JB
D PF
Fn. S. S.
D
S. S.
PF
FIGURE 52-4. Bone over facial nerve is removed. At this point, the
SDA
g reater superficial petrosal nerve is sectioned. GG, geniculate ganglion; IAC, internal auditory canal; MFD, middle fossa dura; PDF, posterior fossa dura; SS, sigmoid sinus.
Care is also taken to avoid kinking of the nerve near the stylomastoid foramen when reflecting it posteriorly. The facial nerve is protected at all times and kept wet. The IAC dura is preserved, and CN VII and VIII, with all the protecting dura, are rerouted posteriorly.
FIGURE 52-5. Location of facial nerve after it is completely reflected posteriorly out of bony fallopian canal. FC, fallopian canal; Fn, facial nerve; JB, jugular bulb; MFD, middle fossa dura; PFD, posterior fossa dura; SDA, sinodural angle; SS, sigmoid sinus.
Tumor Removal (Fig. 52-7) With meningiomas, arterial feeder vessels from the inter nal carotid artery are encountered and eliminated during the approach. The diamond burr is used to excise these vessels and the base of implantation at the petroclival area. The dura is opened anterior to the IAC, and the open ing is extended as far forward as necessary for complete
.
Co.
ICA
Fn
The fallopian canal and the ossicles have been removed, and the promontory is now exposed. Starting with the basal coil, the cochlea is completely drilled out. Bone removal is carried forward around the internal carotid artery, and inferiorly the bone removal extends to the inferior petrosal sinus and jugular bulb. Superi orly, the superior petrosal sinus is followed to Meckel’s cave. Medially, bone removal extends to the clivus. At this stage, a large triangular window, covered by dura, has been created into the midline of the skull base. Its boundaries are the superior petrosal sinus superiorly; below and medial to the inferior petrosal sinus into the clivus inferiorly; the region of the internal carotid artery anteriorly; and the lateral clivus medially. The apex of the triangle is just beneath Meckel’s cave.
MFD
Transcochlear Drill-out (Fig. 52-6)
JB
MFD
PFD
. S.S
FIGURE 52-6. Cochlear drill-out. Co, cochlea; Fn, facial nerve; ICA, internal carotid artery; JB, jugular bulb; MFD, middle fossa dura; PFD, posterior fossa dura; SS, sigmoid sinus.
636
OTOLOGIC SURGERY
tumor exposure. The dural opening extends from the superior petrosal sinus superiorly to the inferior petrosal sinus inferiorly. CN V, VII, and VIII are identified. The facial nerve is kept on the posterior surface of the tumor. The junction of the intracranial portion of the facial nerve and the skeletonized intratemporal por tion is now identified. The nerve is protected and kept moist.
Tumor
B.A .
V
ICA
Fn.
VI
S. B.
The tumor pseudocapsule is opened, and the center of the main mass of the tumor is removed with the HouseUrban rotatory dissector or with an ultrasonic aspira tor. As the dissection proceeds forward and medially, the basilar artery and CN VI are identified anterosu periorly. The vertebral arteries appear posteroinferiorly. The tumor is removed from these vessels and their major tributaries under direct vision. In tumors extend ing across the midline, the basilar artery and its major branches can be dissected posteriorly off the tumor capsule. When the lesion is removed in this fashion, the cranial nerves and the IAC in the opposite CPA come into view. No brain retractors are needed to allow for this exposure. For dumbbell-shaped tumors, the tentorium can be opened to excise the part of the tumor that is lying in the middle fossa. Care is taken not to injure the vein of Labbé posteriorly or CN IV at the edge of the tento rium. Figures 52-8 to 52-12 illustrate the transcochlear approach with removal of the tympanic bone and blind closure of the EAC, which is done when further anterior exposure is necessary.
VIII
. S.S
FIGURE 52-7. Tumor removal. Tumor is removed with a HouseUrban rotatory dissector or with an ultrasonic aspirator. BA, basilar artery; BS, brainstem; Fn, facial nerve; ICA, internal carotid artery; SS, sigmoid sinus.
Closure After the tumor has been removed, hemostasis is secured. The dura is reapproximated, and the facial nerve is replaced forward. The eustachian tube orifice is plugged with Surgicel, bone wax, and muscle. Abdominal fat strips are used to fill the dural defect, and the mastoid and skull base defect, and to form a bed for the facial nerve. A titanium mesh cranioplasty is performed. The postauricular incision is closed in three layers, and a compressive dressing is placed securely around the head.
E.
FIGURE 52-8. Increased anterior exposure obtained with removal of bony external auditory canal. E, exposure.
Chapter 52 • Transcochlear Approach to Cerebellopontine Angle Lesions
637
Lumbar drainage may be initiated and continued for 4 to 5 days.
POSTOPERATIVE CARE The patient is observed in the intensive care unit for 24 hours after surgery and remains in the hospital for 4 to 5 days. Steroids are continued for 48 hours. Antibiotics are routinely used after the perioperative period. With early mobilization and ambulation, thromboembolism is avoided, and there is a speedy return of balance.
RESULTS JB
F.C. one
Db
PF
B.I
Dura
FIGURE 52-9. Mastoidectomy. Meatus is closed in three layers; skin, tympanic membrane, malleus, and incus are removed; and bony exter nal auditory canal is drilled out. BI, Bill’s island; FC, fallopian canal; JB, jugular bulb; PFD, posterior fossa dura.
In 1982, De la Cruz1 reviewed the results of 16 patients in whom the transcochlear approach was used. A com bination transcochlear/middle fossa approach was used in the three cases involving dumbbell-shaped tumors in the posterior and middle cranial fossae. Total tumor removal was possible in 13 of the 16 patients. Each of the other three patients had had surgery elsewhere and presented with large recurrent meningiomas and exten sive neurologic deficits preoperatively. During the trans cochlear approach, scraps of tumor were left behind on the vertebral artery in two of these cases. None of these patients had tumor recurrence. Four patients had facial paresis or twitch preoperatively; of the other 12, 4 had permanent facial paralysis because of tumor
ICA
ICA
P.A bone.
Co.
JB
JB Fn.
Fn.
S. S.
A
B
FIGURE 52-10. A and B, Cochlea and adjacent part of petrous apex (PA) are drilled away. Internal carotid artery (ICA) is exposed at anterior limit of dissection. Co, cochlea; Fn, facial nerve; JB, jugular bulb; SS, sigmoid sinus.
638
OTOLOGIC SURGERY
Dorsal incision
ICA
JB
MFD PFD
Fn.
. S.S
FIGURE 52-11. Final dural exposure and outlining of dural incision. Fn, facial nerve; ICA, internal carotid artery; JB, jugular bulb; MFD, middle fossa dura; PFD, posterior fossa dura; SS, sigmoid sinus.
involvement of the nerve, and 7 had temporary paresis with good recovery of facial function. This paresis was attributed primarily to removal of blood supply to the nerve, and these patients were operated on before facial nerve monitoring was available. Two deaths occurred in this series. One patient had bleeding from the ver tebral artery 1 week postoperatively, requiring clipping of the vessel, with subsequent infarction of the brain stem; the other patient was diabetic and died 1 month postoperatively because of gram-negative shock from pyelonephritis. He was one of the patients reported as having a “permanent” facial paralysis. Autopsy revealed no evidence of residual tumor on the facial nerve or elsewhere. In 1989, Yamakawa and colleagues15 published their results with the suboccipital approach, reporting subto tal tumor removal in 17 of 29 patients with intracranial epidermoids and tumor recurrence in 7 patients. One of 14 patients with CPA tumors had postoperative CN VII paralysis, 6 had CN VI palsy, 4 had dysphagia, and 2 had hearing loss. The report by Yasargil and associates19 pub lished in the same year analyzed results in 43 patients; 35 had epidermoid tumors, whereas others had intracranial dermoid tumors. There were no recurrences. Aseptic meningitis and transient cranial nerve palsies were the most common complications.
In 2001, Angeli and coworkers20 reviewed 24 cases operated on between 1985-1995 at the House Ear Clinic using the transcochlear approach or 1 of its modifica tions. In 1 case, a modified transotic approach was used (small melanoma); in 2 other cases, the transcochlear approach was extended inferiorly with an infratemporal and upper neck dissection (1 glomus and 1 meningi oma). Most of the tumors (16 of 24) were meningiomas. The other tumors included 4 cholesteatomas, 2 mela nomas, 1 glomus, and 1 ependymoma. The EAC was closed in 12 patients. Complete removal was achieved in 82% of tumors (average follow-up time 36 months). In 2 cases (8%), it was elected intraoperatively to do a subtotal tumor resection owing to excessive blood loss (intracranial glomus jugulare tumor) in 1 and unresect ability (melanoma invading the brainstem) in 1. Most patients had some degree of facial nerve dysfunction immediately after surgery, and 12 of 20 patients sub sequently improved to House-Brackmann grade III or better. A significant incidence of temporary facial weak ness was expected as a result of posterior facial nerve transposition. 59% of patients had permanent neuro logic sequelae because of either the surgery or their dis ease. The most common neurologic deficit was diplopia (27%). Other complications included dysphagia, facial numbness, unsteadiness, hoarseness, hemiparesis, and dysarthria. In 2004, Gonzalez and associates21 reported 32 patients with 34 anterior inferior cerebellar artery aneu rysms. Surgical approaches included retrosigmoid, trans cochlear, translabyrinthine, and orbitozygomatic. The transcochlear approach was used in 4 patients; 1 patient had small bilateral aneurysms that were approached, with good results, from the same side by taking advantage of the wide corridor obtained with a transcochlear approach. Complications reported were involvement of CN VI, VII, and VIII, and CSF leak. Siwanuwatn and coworkers22 quantitatively assessed the working areas and angles of attack associated with retrosigmoid, combined petrosal, and transcochlear cra niotomies, using silicone-injected cadaveric heads. They reported that the transcochlear approach provided sig nificantly greater working areas at the petroclivus and brainstem than the combined petrosal and retrosigmoid approaches (P < .001). The horizontal and vertical angles of attack achieved using the transcochlear approach were wider than the angles of the combined petrosal and retro sigmoid at the Dorello canal and the origin of the anterior inferior cerebellar artery (P < .001). They concluded that the transcochlear approach provides the widest corridor, improving the working area and angle of attack to both areas. In 2006, Leonetti and colleagues24 reported 29 patients with large meningiomas of the CPA surgically treated through a combined retrosigmoid/transpetrosal/ transcochlear approach. Total tumor removal was achieved in 19 of 29 (67%) of the patients, and the facial
Chapter 52 • Transcochlear Approach to Cerebellopontine Angle Lesions
639
ICA VI T.
JB
BA.
V I
VI
SPS
. S.S
FIGURE 52-12. Tumor removal and structures seen at skull base at completion of approach. Basilar artery (BA) and CN VI are seen. ICA, internal carotid artery; JB, jugular bulb; SPS, superior petrosal sinus; SS, sigmoid sinus; T, tumor.
nerve was anatomically preserved in 26 of 29 (89%) of the cases. Cerebrospinal fluid leakage was seen in 3.5% of the patients, and additional transient cranial nerve deficits were noted in 14% of the cases, but no signifi cant neurologic sequelae occurred. Of the 10 patients with residual tumor, 6 have been stable without growth, 2 were treated with reoperation for regrowth of disease, and 2 were controlled with localized radiotherapy. These investigators concluded that this combined lateral trans temporal approach provided wide exposure to the CPA and optimized the surgical extirpation of the 29 menin giomas presented in their series.
COMPLICATIONS AND THEIR MANAGEMENT Temporary facial nerve paresis is the most common complication. If facial paresis occurs, prompt eye care is essential; adequate lubrication with drops, nighttime ointments, and a moisture shield prevent corneal compli cations. Unless the facial nerve has been severed, surgical intervention for facial reanimation is not indicated. The best approach in individuals with even complete paralysis after transcochlear surgery, if the facial nerve is anatomi cally intact, is eye care that includes soft lenses, spring and gold weights, sometimes canthoplasty, and “watch ful waiting” because most of these patients recover to an
acceptable grade of facial function within the first year after surgery. Other cranial nerve palsies may occur and should be addressed individually. The neurotologist must attain a good working relationship not only with a neurosur geon, but also with an ophthalmologist and a laryngolo gist to help in the management of these cranial nerve deficits. Intracranial bleeding is controlled at the time of surgery. Close observation of the patient in the intensive care unit for the initial 24 hours postoperatively allows early recognition of delayed postoperative intracranial hemorrhage. In these cases, treatment consists of imme diate reopening of the surgical wound and removal of the fat in the intensive care unit while the operating room is being prepared, and then operative evacuation of the hematoma and control of the bleeding site or sites. Meningitis may occur after complete excision of intracranial epidermoids, and the incidence increases when the tumor capsule is left in place.19,24 Meningitis may be fatal if it is infectious and requires early, aggressive antibiotic therapy. More commonly, however, it is a chemical aseptic meningitis, and the patient is treated with dexamethasone. Postoperative pain is not as severe as that seen with the suboccipital approach and is managed adequately with analgesics. With this approach, tumor recurrence is rare when all visible tumor has been removed. Patients
640
OTOLOGIC SURGERY
with recurrences do not present typically and may have vague complaints of unsteadiness or trigeminal symp toms several years after the initial resection.1 Annual follow-up with gadolinium-enhanced and fat-suppression MRI is necessary. In cases of suspected tumor regrowth or recurrence, complete re-evaluation is performed, and removal of the recurrent tumor is advised. Gamma knife stereotactic radiosurgery and stereotactic radiotherapy are also treatment options for residual or recurrent meningiomas.26
SUMMARY Access to midline intradural lesions, intradural petroclival tumors, and CPA tumors arising anterior to the IAC has traditionally been difficult. With the transcochlear approach, the facial nerve is mobilized, the cochlea is removed, and the petrous apex is dissected around the internal carotid artery, allowing direct exposure of these lesions and of midline and contralateral CPA structures, without using retraction. Total removal of the tumor and its base and blood supply is possible with this approach. The transcochlear approach is recommended for these lesions in patients with poor hearing. Its safety and efficacy encourage its use.
REFERENCES 1. De la Cruz A: The transcochlear approach to meningio mas and cholesteatomas of the cerebellopontine angle. In Brackmann D E (ed): Neurological Surgery of the Ear and Skull Base. New York, Raven Press, 1982, pp 353-360. 2. House WF, De la Cruz A: Transcochlear approach to the petrous apex and clivus. Trans Am Acad Ophthalmol Otolaryngol 84:927-931, 1977. 3. House WF, Hitselberger WE: The transcochlear approach to the skull base. Arch Otolaryngol Head Neck Surg 102:334-342, 1976. 4. Hitselberger WE, Horn K L, Hankinson H, et al: The middle fossa transpetrous approach for petroclival meningiomas. Skull Base Surg 3:130-135, 1993. 5. Spetzler RF, Daspit CP, Pappas CT: The combined supratentorial and infratentorial approach for lesions of the petrous and clival regions: Experience with 46 cases. J Neurosurg 76:588-599, 1992. 6. Daspit CP, Spetzler RF, Pappas CT: Combined approach for lesions involving the cerebellopontine angle and skull base: Experience with 20 cases-preliminary report. Otolaryngol Head Neck Surg 105:788-796, 1991. 7. Shiobara R, Ohira T, Kanzaki J, Toya S: A modified extended middle cranial fossa approach for acoustic nerve tumors: Results of 125 operations. J Neurosurg 68: 358-365, 1988. 8. Wigand M E, Haid T, Berg M: The enlarged middle cranial fossa approach for surgery of the temporal bone and the cerebellopontine angle. Arch Otol Rhinol Otolar yngol 246:299, 1989.
9. Jackler R K, Sim DW, Gutin PH, Pitts L H: Systematic approach to intradural tumors ventral to the brain stem. Am J Otol 16:39-51, 1995. 10. Arriaga M, Shelton C, Nassif P, Brackmann D E: Selection of surgical approaches for meningiomas affect ing the temporal bone. Otolaryngol Head Neck Surg 107: 738-744, 1992. 11. Thedinger B A, Glasscock M E III, Cueva R A: Transco chlear transtentorial approach for removal of large cer ebellopontine angle meningiomas. Am J Otol 13:408-415, 1992. 12. Brackmann D E, Anderson RG: Cholesteatomas of the cerebellopontine angle. In Silverstein H, Norrell H (eds): Neurological Surgery of the Ear. Birmingham, Aescula pius, 1979, pp 340-344. 13. De la Cruz A, Doyle KJ: Epidermoids of the cerebello pontine angle. In Jackler R A, Brackmann D E (eds): Neu rotology. York, PA, Spectrum, 1994, pp 823-834. 14. Nager GT: Epidermoids involving the temporal bone: Clinical, radiological, and pathological aspects. Laryngo scope 2(Suppl):1-22, 1975. 15. Yamakawa K, Shitara N, Genka S, et al: Clinical course and surgical prognosis of 33 cases of intracranial epider moid tumors. Neurosurgery 24:568-573, 1989. 16. Mafee MF: MRI and CT in the evaluation of acquired and congenital cholesteatomas of the temporal bone. J Otolaryngol 22:239-248, 1993. 17. Brackmann DE: Translabyrinthine Transcochlear ap proaches. In Sekhas LN, Janecka IP (eds): Surgery of Cranial Base Tumors. New York, Raven Press, 1993, pp 351-365. 18. Friedman RA, Brackmann DE: Transcochlear Approach Operative Techniques in Neurosurgery Vol 2, No 1, pp 39-45, 1999. 19. Yasargil MG, Abernathy C D, Sarioglu AC: Microneu rosurgical treatment of intracranial dermoid and epider moid tumors. Neurosurgery 24:561-567, 1989. 20. Angeli S I, De la Cruz A, Hitselberger WE: The trans cochlear approach revisited. Otol Neurotol 22:690-695, 2001. 21. Gonzalez L F, Alexander M J, McDougall CG, Spetzler R F: Anteroinferior cerebellar artery aneurysms: Surgical approaches and outcomes—a review of 34 cases. Neuro surgery 55:1025-1035, 2004. 22. Siwanuwatn R, Deshmukh P, Figueiredo EG, et al: Quantitative analysis of the working area and angle of at tack for the retrosigmoid, combined petrosal, and trans cochlear approaches to the petroclival region. J Neuro surg 104:137-142, 2006. 23. Leonetti J P, Anderson D E, Marzo S J, et al: Combined transtemporal access for large (>3 cm) meningiomas of the cerebellopontine angle. Otolaryngol Head Neck Surg 134:949-952, 2006. 24. Cantu RC, Ojemann RG: Lucosteroid treatment of kera tin meningitis following removal of a fourth ventricle epidermoid tumor. J Neurol Neurosurg Psychiatry 31:75, 1968. 25. Guidetti B, Gagliardi FM: Epidermoid and dermoid cysts. J Neurosurg 47:12-18, 1977. 26. Mattozo C A, De Salles A A, Klement I A, et al: Stereotactic radiation treatment for recurrent nonbenign meningiomas. J Neurosurg 106:846-854, 2007.
53
Extended Middle Cranial Fossa Approach Rick A. Friedman
Gaining access to the cerebellopontine angle (CPA) and prepontine cisterns has presented a formidable challenge. Approaches designed to expose infraclinoid basilar tip aneurysms, petroclival meningiomas, chondromas, chondrosarcomas, and chordomas involving the petrous apex and clivus must take into consideration the vital neighboring neurovascular structures.1,2 The risks encountered in the region of the prepontine cistern during the management of aneurysms of the posterior circulation were best described by Drake3 in 1961, when he said, “the upper clival region is to be considered no-man’s land.” Several conventional neurosurgical approaches to this region have been described, including the subtemporal and trans-sylvian and a combination half-andhalf approach incorporating both techniques. Despite advances in microsurgical techniques and neuroanesthesia, the petrous bone has previously been an impediment to satisfactory exposure in this anatomically complex region. Modern skull base approaches, including the middle cranial fossa and the middle fossa transpetrous or extended middle fossa (EMF), have been instrumental in removing the petrous barrier, minimizing temporal lobe retraction, and improving the line of sight for the neurosurgeon. The middle fossa approach was first described in the literature in 1904.4 The sentinel work of House in 19615 led to a revitalization and refinement of this approach to the internal auditory canal (IAC), CPA, and prepontine cisterns. By extending the traditional middle fossa dissection anteriorly to the clivus, the EMF approach provides complete exposure of the IAC and the prepontine cisterns and the mid to upper clivus.6-11 Posteriorly, the approach allows access to tumors approaching, but not entering the jugular foramen (Table 53-1).
TECHNIQUE We administer preoperative antibiotics and continue them for 24 hours postoperatively. Intraoperative furosemide and mannitol are given to allow easier temporal
lobe retraction. We administer dexamethasone intravenously during the procedure and continue this for 24 hours postoperatively. Long-acting muscle relaxants are avoided during surgery so as not to interfere with facial nerve monitoring.
SURGICAL ANATOMY The surgical anatomy of the temporal bone from the middle fossa approach is complex (Fig. 53-1).12 Landmarks are not as apparent as with other approaches through the temporal bone. Laboratory dissection is essential so that the surgeon may become familiar with the anatomy from above. Anteriorly, the limit of the dissection is the middle meningeal artery, which is anterior and lateral to the greater superficial petrosal nerve. Excessive anterior retraction can lead to postoperative paresthesias in CN V3. The arcuate eminence roughly marks the position of the superior semicircular canal. The relationship between the arcuate eminence and the superior semicircular canal is inconstant.13 The superior semicircular canal tends to be perpendicular to the petrous ridge. Medially, the superior petrosal sinus runs along the petrous ridge. Surgical tolerances are tight in the area of the lateral IAC. The labyrinthine portion of the facial nerve lies immediately posterior to the basal turn of the cochlea. Bill’s bar separates the facial and superior vestibular nerves. Slightly posterior and lateral to this area is the vestibule and ampullated end of the superior semicircular canal. Identification of the geniculate ganglion can be accomplished by tracing the greater superficial petrosal nerve posteriorly. The ganglion is dehiscent in approximately 16% of patients. The IAC lies roughly on the same axis as the external auditory canal; this relationship is useful in orienting the surgical field. The area around the porus acusticus medially is a “safe zone” compared with the lateral or fundal region, and provides an excellent place to begin IAC dissection. We begin our dissection medially, by 641
642
OTOLOGIC SURGERY
SURGICAL TECHNIQUE
drilling in the meatal plane in the area of the bisection of the angle formed by the superior semicircular canal and the greater superficial petrosal nerve. The IAC can be located initially in this medial area of the temporal bone and followed laterally.
The patient is placed in the supine position with the head turned to the side opposite the lesion (Fig. 53-2). The surgeon is seated at the head of the table, and the anesthesiologist is seated at the foot. An incision is made in the preauricular area and extended superiorly in a gently curving fashion. Care must be taken near the anterior extension of the incision to avoid injury to the temporal branch of the facial nerve (see Fig. 53-2). The temporalis muscle is incised, beginning at the zygomatic root, along the linea temporalis, and the muscle is elevated from the temporal fossa and reflected anteroinferiorly. The temporal squama is exposed. Using cutting and diamond burrs, a temporal craniotomy is performed. The craniotomy measures approximately 5 × 5 cm, and is two thirds anterior and one third posterior to the zygomatic root (Fig. 53-3). The inferior
TABLE 53-1 Indications for Extended Middle Fossa
Approach
Vestibular schwannoma (35 21-35
Low Moderate
≤20
High
Carotid may be sacrificed Patient would tolerate occlusion under controlled circumstances; reconstruction is recommended Patient would not tolerate occlusion of internal carotid artery
regarding collateral cerebral blood flow include single photon emission CT (SPECT) with balloon occlusion and transcranial Doppler. Histologic diagnosis should be obtained before the extirpative surgery whenever possible. Tumors amenable to a punch or open biopsy are approached in this manner. Tumors that are in deeper planes may be sampled by fine-needle aspiration biopsy. Rarely, a histologic diagnosis cannot be obtained before the approach because of the intrinsic limitations of fine-needle aspiration biopsy. Under these circumstances, a frozen section analysis, obtained via a skull base approach, may be sufficient to justify the resection of the tumor. Vital neurovascular structures, such as the ICA, the eye, and cranial nerves, should not be sacrificed, however, based on a frozen section analysis. The extent of the evaluation to rule out regional or distant metastasis or to determine that the ITF tumor is a metastasis is dictated by the histologic type and stage of the tumor. CT scan of the neck is more sensitive than physical examination for the detection of regional lymphadenopathy. Patients presenting with tumors that metastasize hematogenously (sarcoma, melanoma) should undergo CT scan of the chest and abdomen and a bone scan. Cerebrospinal fluid (CSF) cytology is advised for patients with tumors that have invaded the dura. These patients are also at risk for “drop metastasis,” which should be ruled out by spinal MRI.
Rehabilitation Considerations Functional or neurologic deficits that are identified preoperatively should be taken into consideration during the surgical planning and during postoperative care. These deficits often have a significant impact on the recovery and functional rehabilitation of the patient. Dysfunction of the trigeminal nerve or the masticator muscles or both is commonly underdiagnosed. Cutaneous and corneal sensation should be assessed preoperatively. Corneal anesthesia associated with concomitant facial nerve palsy requires aggressive measures to prevent corneal injury.
Chapter 54 • Anterior and Subtemporal Approaches to the Infratemporal Fossa
Lateral deviation of the jaw on opening may reflect weakness or paralysis of the ipsilateral pterygoid muscles, invasion of the muscles, or dysfunction of the temporomandibular joint (TMJ). Likewise, trismus may be due to mechanical restriction caused by the bulk of the tumor, ankylosis of the TMJ, scarring, tumor tethering, or pain. The nature of the trismus is an important consideration in the perioperative management of the airway. Trismus secondary to pain resolves with the induction of general anesthesia, allowing safe oral endotracheal intubation. In patients with mechanical trismus, an awake nasotracheal intubation may be performed if it is anticipated that surgery would correct the trismus. Otherwise, a tracheostomy, performed under local anesthesia, is the safest perioperative airway. Neoplastic invasion of the facial nerve may manifest with facial weakness or paralysis, facial spasms, epiphora, facial spasms, and dysgeusia. Significant destruction of the facial nerve by tumor may occur before the patient develops these clinical signs. A gold weight, implanted in the upper eyelid, or surgical tightening of the lower lid may be necessary to protect the cornea. Hearing loss caused by a tumor of the ITF may be either conductive, resulting from eustachian tube dysfunction, or sensorineural, resulting from tumor involvement of the temporal bone or posterior cranial fossa. A myringotomy or amplification or both facilitate communication with the patient. Deficits of the lower cranial nerves (CN IX, X, XI, and XII) are associated with tumors that originate in the parapharyngeal space or tumors that extend to the jugular foramen, or both. Patients with deficits of CN IX, X, and XII present with varying degrees of swallowing or speech problems, such as hypernasal or slurred speech, nasal regurgitation, dysphagia, aspiration, and dysphonia. Findings on physical examination reflect the involvement of specific cranial nerves, and include decreased elevation of the palate, decreased mobility and strength of the tongue with deviation to the involved side on protrusion, decreased supraglottic sensation, pooling of secretions in the hypopharynx, ipsilateral vocal cord paralysis, and decreased bulk and strength of the sternocleidomastoid and trapezius muscles. Patients with partial deficits of the lower cranial nerves (paresis) often experience a complete deficit (paralysis) after surgery, resulting in increased dysphagia and aspiration. Consequently, a tracheostomy for tracheal toilet and a gastrostomy tube for nutrition and hydration are often necessary during the perioperative period. Laryngeal framework surgery (thyroplasty) performed during the extirpative surgery or the early postoperative period improves the glottic closure and decreases the risk for aspiration, often obviating the need for a tracheotomy for the sole purpose of tracheopulmonary toilet.10-12 Laryngeal framework surgery allows the patient to compensate for the deficits using the remaining function (contralateral side) more effectively. Laryngeal
651
f ramework surgery does not restore the motor or the sensory function. These patients remain at a higher risk for aspiration and nutritional deficiencies. Collaboration with an experienced speech-language pathologist, who can assist with the monitoring of the patient and the diet modifications and provide intensive swallowing therapy, is crucial to prevent the pulmonary and nutritional complications of aspiration. In patients with severe deficits or in patients with cognitive problems, strong consideration should also be given to placement of a gastrostomy tube to facilitate postoperative feeding and decrease the risk of prandial aspiration. Velopharyngeal insufficiency may be ameliorated by a palatal lift prosthesis that pushes the soft palate against the posterior pharyngeal wall. Alternatively, a pharyngeal flap or a palatopexy may be performed in patients who do not tolerate the prosthesis.
Reconstructive Considerations Most commonly, a temporalis muscle transposition flap is adequate to separate the cranial cavity from the upper aerodigestive tract and obliterate the dead space. Microvascular free flaps, such as rectus abdominis flap (for soft tissue defects), latissimus dorsi flap (for myocutaneous or massive defects), and iliac composite flap (for defects requiring bone reconstruction), are indicated when the temporalis muscle or its blood supply is sacrificed as part of the oncologic resection, when the patient requires a complex resection involving composite tissue flaps with skin or bone or both, or when the extirpative surgery leads to a massive soft tissue defect and dead space. These needs are usually anticipated during the surgical planning, and the patient and consultants (e.g., the microvascular surgeon) are informed accordingly. Ideally, functional and cosmetic deficits created by the tumor or the surgery should be addressed in a single stage, concomitant with the oncologic resection. When a temporary facial palsy is anticipated, corneal protection using lubricants or a temporary lateral tarsorrhaphy or both is usually adequate. Grafting of the facial nerve involves a longer recovery period, however. Insertion of a gold weight implant into the upper eyelid is advisable. When an immediate reconstruction of the facial nerve is impossible, static fascial slings or muscle transpositions are indicated. Lower cranial nerve deficits may be ameliorated by laryngeal framework surgery, tracheotomy, or laryngotracheal separation, as previously discussed.
Other Perioperative Considerations Preoperatively, the patient’s blood is typed and crossmatched for 2 to 6 U of packed red blood cells, according to the extent and nature of the tumor and surgery. Autologous blood banking is used when feasible, although it is frequently impractical. A Cell Saver autologous transfusion device may be used during the resection of benign vascular tumors.
652
OTOLOGIC SURGERY
Perioperative antibiotic prophylaxis with a wide spectrum against the flora of the skin and upper aerodigestive tract and that exhibits good penetration of the bloodbrain barrier is administered before the surgery and is continued for 48 hours after the surgery. The use of a broad-spectrum cephalosporin with good CSF penetration (e.g., ceftriaxone) seems to be as effective as multiple antibiotic regimens. Somatosensory evoked potential monitoring using the median nerve is indicated whenever surgical manipulation of the ICA is anticipated. Lower cranial nerve monitoring is not routinely employed. It may be useful for the identification and preservation of nerve function when the tumor is in close proximity to these nerves. Conversely, facial nerve monitoring is routinely used for transparotid or transtemporal approaches. Monitoring electrodes and lines for vascular access should be secured with sutures, staples, or adhesive dressings. The choice of anesthetic agent is influenced by the extent of intracranial dissection, potential for brain injury, systemic hemodynamics, the need for monitoring of cortical and brainstem functions (e.g., brainstem-evoked response, somatosensory evoked potentials, electroencephalography), and the need for cranial nerve monitoring (CN VII, and X to XII). All these factors should be thoroughly discussed with the anesthesiologist. When changes of head position during surgery are anticipated, the endotracheal tube should be secured with a circumdental or circum-mandibular wire ligature (No. 26 stainless steel wire). The operating table is positioned perpendicular to the anesthesia staff, and if intradural dissection is anticipated, a spinal drain is inserted and secured with sutures and adhesive dressing (e.g., Tegaderm, OpSite). Other measures to diminish the intracranial pressure, such as hyperventilation, osmotic diuresis, and corticosteroids, are used as needed throughout the surgery. A nasogastric tube and Foley catheter are passed and secured after adequate placement is corroborated. Antiembolic sequential compression stockings are recommended to prevent deep venous thrombosis.
SURGICAL APPROACHES The head of the patient is positioned on a horseshoe headrest or, if necessary for intracranial neurovascular or neurosurgical work, on a three-pin head fixation system. When the horseshoe headrest is used, it is important to use additional “egg-crate” foam padding because the scalp may develop a decubitus ulcer during prolonged surgery. If the ICA is at risk, the head should be positioned in slight extension to provide access to the neck for proximal control of the ICA. Tarsorrhaphy sutures are placed for protection of the eyes. The scalp is shaved following the planned incision line (e.g., bicoronal), and the incision line is infiltrated with a solution of lidocaine and epinephrine (1:100,000 to 1:400,000).
Preauricular (Subtemporal) Approach The preauricular approach is suited for tumors that originate in the ITF and intracranial tumors that originate at the anterior aspect of the temporal bone, or greater wing of the sphenoid bone, and that extend into the ITF.5,13,14 It may also be combined with other approaches, such as a subfrontal approach to expose massive tumors that extend to the anterior and middle skull base. The preauricular approach does not provide an adequate exposure for the resection of tumors that invade the tympanic bone, however, and does not provide control of the intratemporal facial nerve or jugular bulb. An incision, following a hemicoronal or bicoronal line, is carried through the subcutaneous tissue, galea, and pericranium (Fig. 54-2). Over the temporal area, the incision extends down to the deep layer of the temporal fascia. The anterior branches of the superficial temporal artery are preserved, ensuring adequate blood supply to the scalp flap. Ipsilateral to the tumor, the incision is extended following into the preauricular crease down to the level of the tragus. When proximal control of the ICA is warranted, the incision is extended into the neck using a lazy S pattern, or, alternatively, a separate cervical incision is performed. The scalp is dissected following a subpericranial plane, separating the attachments of the pericranium to the deep layer of the temporal fascia. The scalp flap is elevated from the deep temporal fascia using a broad periosteal elevator. Above the zygoma, the deep temporal fascia splits into superficial and deep layers, which attach to the lateral and medial surfaces of the zygomatic arch. To continue the surgical exposure, the superficial layer of the deep temporal fascia is incised following an imaginary line that joins the superior orbital rim to the zygomatic root. The dissection continues deep to this plane, elevating the superficial layer of the deep temporal fascia off the zygomatic arch (see Fig. 54-2). Fascia and periosteum are reflected anteriorly with the scalp flap. This maneuver protects the frontal branches of the facial nerve that are just lateral to the superficial layer of the deep temporal fascia. Elevation of the periosteum from the lateral surface of the zygomatic arch and malar eminence completes exposure of the orbitozygomatic complex. The periorbita is elevated from the lateral orbit using a Penfield No. 1 dissector, exposing the roof and lateral wall of the orbit down to the inferior orbital fissure. The fascial attachments of the temporalis and masseter muscles to the zygomatic arch are transected using electrocautery. The attachments of the temporalis muscle to the cranium are transected with the electrocautery, and the muscle is elevated off the temporal fossa. If the temporalis muscle is to be returned to its original position at the completion of the surgery, a curved titanium plate (1.5 to 1.7 mm) is screwed at the temporal line, leaving some screw holes empty to facilitate suturing from the plate to the muscle (Fig. 54-3). Then the masseteric
Chapter 54 • Anterior and Subtemporal Approaches to the Infratemporal Fossa
653
Hairline
Fat pad
B A
Parotid gland
FIGURE 54-2. A, Bicoronal scalp incision is extended along preauricular skin crease. This incision may be continued into upper cervical region as a lazy S incision, or a separate cervical incision may be made for exposure of vessels and nerves. B, Scalp flap is elevated from underlying cranium, fascia, lateral orbital rim, zygomatic arch, and masseteric fascia. Plane of dissection is deep to superficial layer of deep temporal fascia (incised) and deep to parotid masseteric fascia.
Inferior orbital fissure
FIGURE 54-3. Areas of bone removal are noted with dark stippling. Temporal craniotomy is performed in conjunction with orbitozygomatic osteotomy. Additional exposure of infratemporal skull base may be achieved by removal of subtemporal cranium (striped area) and resection of mandibular condyle (lightly stippled area).
fascia is dissected from the masseter muscle, elevating the overlying parotid gland with a broad periosteal elevator (see Fig. 54-2). To increase the arc of rotation of the scalp flap, any soft tissue anterior to the tympanic bone can be transected from superior to inferior, down to the
level of the facial nerve. The facial nerve is identified and preserved using a standard technique. It is helpful to preserve a cuff of soft tissue around the main trunk of the facial nerve to prevent a traction injury to the main trunk of the facial nerve.
654
OTOLOGIC SURGERY
Using the caudal limb of the incision, the sternocleidomastoid muscle is dissected laterally, and the carotid sheath is exposed. When necessary, the contents of the carotid sheath, including the ICA and common and external carotid arteries, and the internal jugular vein are exposed, dissected, and controlled. CN X to XII are also identified and preserved. Vessel loops are placed around these structures and secured with hemoclips rather than hemostats to avoid inadvertent traction. Orbitozygomatic osteotomies are performed at the zygomatic root posteriorly, the zygomaticofrontal suture superiorly, and the zygomaticomaxillary buttress at the level of the zygomaticofacial nerve medially (see Fig. 54-3). Prior periorbital elevation off the lateral and inferior walls is necessary to identify the inferior orbital fissure and to complete the osteotomies of the orbitozygomatic complex. The tip of the reciprocating saw is placed in the most lateral aspect of the inferior orbital fissure; an osteotomy is performed through the malar eminence following a vertical imaginary line medial to the zygomaticofacial foramen. This osteotomy separates the zygoma from the maxilla. Accidental entry into the maxillary sinus may occur, requiring closure of the defect using fascia or pericranium free grafting or both. These free tissue grafts are held in place by compression against the opening when orbitozygomatic bone graft is replaced and plated at the completion of the surgery. All osteotomies are completed with a reciprocating saw transecting the bone in beveled or V-shaped manner in such a way that maximizes the exposure and facilitates replacement of the bone graft at the completion of the surgery. If tumor involvement of the orbit is present, the osteotomies are modified to secure a complete resection. In cases requiring intracranial and extracranial exposure, the superior and lateral osteotomies are made through the superior and lateral orbital walls after the craniotomy is completed, and the brain is separated from the skull base. This way, the orbital walls can be incorporated in the orbitozygomatic graft. Using intracranial and extracranial exposures, osteotomies are made through the superior and lateral orbital walls to remove the orbitozygomatic bone segment. This approach provides excellent access to the infratemporal skull base, orbital apex, and lateral maxilla. The temporalis muscle is reflected inferiorly until the infratemporal crest is fully visualized. A subperiosteal plane is followed to dissect the soft tissues from the infratemporal cranium. Bleeding from the underlying bone is controlled by the application of bone wax. Fracturing or removal of the coronoid process increases the arc of rotation of the temporalis muscle. Care should be exercised when dissecting the medial aspect of the temporalis muscle, especially near its insertion (coronoid process) because the blood supply to the muscle (deep temporal artery from the internal maxillary artery) penetrates the muscle at this area. Likewise, the soft tissue at the sigmoid notch should be dissected carefully to prevent accidental injury to the internal maxillary artery that travels adjacent to the medial surface of the mandibular ramus.
Dissection of the soft tissues from the infratemporal skull base is usually associated with troublesome bleeding arising from the pterygoid plexus. Bleeding is controlled with the use of bipolar cautery or Cottonoids moistened in oxymetazoline 0.05%, or both. Unipolar cautery is seldom used because it stimulates V3, causing contraction of the mastication muscles and occasional cardiac arrhythmias. A subtemporal craniectomy may aid in the identification of neurovascular structures piercing the infratemporal skull base and to augment the exposure. The lateralmost bone is removed using rongeurs. The origin of the lateral pterygoid plate at the skull base is identified anteriorly. Anatomic relationships that are useful for the identification of infratemporal skull base structures are shown in Figure 54-4, including the posterior curve of the attachment of the lateral pterygoid plate that is in alignment with the foramen ovale, foramen spinosum, and the spine of the sphenoid bone. These structures lie in a straight “line of sight” that is lateral to the canal of the ICA. The inferolateral aspect of the sphenoid sinus may be accessed by removing the bone (i.e., pterygoid plates) between the second and third divisions of the trigeminal nerve. The extirpation of the tumor can now proceed, including the involved soft tissue and bone. To continue the subtemporal exposure, the middle meningeal artery is clipped or cauterized using bipolar electrocautery and transected. Bleeding from the venous plexus that accompanies V3 through the foramen ovale may be controlled with absorbable knitted fabric (Surgicel) packing. Lesions that do not involve the temporal bone or petrous portion of the ICA are adequately exposed with this stepwise approach. Dissection of the petrous ICA is necessary, however; the glenoid fossa is removed as part of the orbitozygomatic bone graft. It is first necessary to perform a temporal craniotomy for exposure of the superior aspect of the glenoid fossa (Fig. 54-5). The capsule of the TMJ is dissected free from the fossa and displaced inferiorly. If possible, the capsule and meniscus are preserved. With use of a reciprocating saw, osteotomies are made through the glenoid fossa, incorporating the lateral two thirds of the fossa (see Fig. 54-5). This maneuver avoids potential injury to the ICA that is located medial to the fossa. In addition, this modification provides stability for the mandibular condyle after reconstruction, although it can be prone to anterior dislocation. Injury to the cochlea is possible if the osteotomies are made too posteriorly. If additional exposure is necessary (i.e., carotid canal and extratemporal ICA), the condylar neck and contents of the condylar fossa can be transected at the level of the sigmoid notch and removed (Fig. 54-6). To dissect the petrous segment of the ICA, it is necessary to transect the mandibular division of the trigeminal nerve at the foramen ovale (see Fig. 54-6). When the ICA is mobilized from its horizontal canal, it can be transposed or retracted to facilitate the resection of tumor, or to gain access to the petrous apex.
Chapter 54 • Anterior and Subtemporal Approaches to the Infratemporal Fossa Foramen spinosum
655
Spine of sphenoid bone
Lateral plate of sphenoid Foramen ovale Glenoid fossa
Scaphoid fossa
Eustachian tube Carotid canal
EAC
B Carotid canal and artery
A
Jugular fossa
FIGURE 54-4. A, Base of skull showing anatomic relationships of carotid canal. Foramen ovale and foramen spinosum are in a direct line from lateral pterygoid plate to spine of sphenoid.
Burr hole
Inferior orbital fissure
FIGURE 54-5. Using intracranial and extracranial approaches, osteotomies of superior orbital roof, lateral orbital wall, and glenoid fossa are performed.
656
OTOLOGIC SURGERY
Zygomaticoorbital osteotomy
Tumor
Internal carotid artery V2 V3
VII Jugular vein XI X
Temporalis muscle Parotid gland IX Glossopharyngeal XII Stump of mandible
FIGURE 54-6. Tumor medial to mandibular division of trigeminal nerve and in close proximity to petrous portion of internal carotid artery. Additional exposure of internal carotid artery and removal of tumor often necessitate transection of mandibular division of trigeminal nerve.
Approaches to the ITF are modified according to the extent of the tumor and other clinical circumstances. Tumors that invade the mandible mandate a partial mandibulectomy to obtain negative margins. In a pediatric patient, the distance from the body of the mandible to the infratemporal skull base is greatly foreshortened. Adequate exposure of the infratemporal skull base can often be achieved using a transcervical approach with superior transposition of the facial nerve. After extirpation of the tumor, it is necessary to close any communication with the upper aerodigestive tract. If viable, a temporalis muscle flap is used to obliterate the dead space and protect the ICA (Fig. 54-7). Because of the branching pattern of the blood supply to the temporalis muscle, the muscle can be divided vertically, and the anterior half of the muscle may be transposed with an intact blood supply. The remaining posterior half of the muscle is transposed anteriorly to fill the temporal fossa defect. Defects of the orbital floor may be reconstructed with titanium mesh, which is then covered with a temporalis muscle transposition flap or temporoparietal fascia flap.
Likewise, defects of the lateral orbital wall can be reconstructed with titanium mesh. In selected patients, anteriorly or posteriorly based pericranial scalp flaps may be elevated to provide protection of the infratemporal skull base. When the temporalis muscle is unavailable, massive soft tissue defects are best reconstructed with microvascular free tissue flaps. The orbitozygomatic bone graft is replaced and fixated in its original position with titanium alloy adaptation plates, wire, or braided nylon sutures. Plating of the bone grafts is preferred because it provides greater stability. To avoid compression of reconstructive flaps, it is sometimes necessary to remove a portion of the zygomatic arch. If resection of the mandibular condyle was necessary to expose the petrous ICA, reconstruction of the TMJ is not attempted. Reconstruction of the TMJ after oncologic exenteration of the ITF does not improve the postoperative function significantly, and may lead to scarring, ankylosis, and trismus. Periosteal and muscular attachments to the craniofacial skeleton must be repaired to prevent retraction or
Chapter 54 • Anterior and Subtemporal Approaches to the Infratemporal Fossa
657
Temporalis muscle in defect Temporalis muscle
FIGURE 54-7. Anterior portion of muscle may be transposed to fill infratemporal skull base defect, as illustrated with this temporalis muscle. Right temporalis muscle has been split vertically with preservation of its axial blood supply.
sagging, or both, of the muscles and other soft tissues. The skin and mucosal incisions are closed using a multilayered technique
Postauricular (Transtemporal) Approach The postauricular approach is designed to expose and resect lesions involving the temporal bone and extending into the ITF.3,14 A question mark–shaped or C-shaped incision is started in the temporal area and extended, postauricularly, into the mastoid region, curving down to follow one of the midneck horizontal skin creases (Fig. 54-8). If the middle ear is to be sacrificed as part of the approach or the tumor resection, and there is a risk of a postoperative CSF leak (intradural work), the external auditory canal (EAC) is closed permanently to prevent CSF otorrhea. The EAC is divided at the bonycartilaginous junction and then closed using everting stitches. This closure is reinforced with a myoperiosteal U-shaped flap based on the posterior margin of the EAC. Alternatively, if the middle ear is to be spared, the canal may be preserved by placing the incisions in the conchal area (Fig. 54-8B). The incision follows the margin of the conchal bowl and tragus so that the scar would be hidden. In the conchal area, the skin, cartilage, and perichondrium are incised to communicate with the retroauricular plane of dissection. These incisions, placed laterally, facilitate the anastomosis of the EAC to the pinna at the end of the extirpative procedure. An incision inside the EAC is not recommended because it is
difficult to suture in a watertight manner, and tends to stenose. A Penrose drain can be inserted through the conchal defect in the skin-auricle flap to facilitate its retraction. Elevation of the cervicofacial flap is carried in a subplatysmal plane in the cervical area, suprasuperficial musculoaponeurotic system plane over the parotid area, and following the deep layer of the deep temporal fascia over the cranium. The main trunk of the facial nerve is identified anterior to the EAC just distal to the stylomastoid foramen, as is described for a parotidectomy. If circumferential mobilization of the main trunk is unnecessary, a cuff of soft tissue is preserved around its main trunk to minimize the possibility of a traction injury when the facial flap is retracted anteriorly. In selected cases, a “tail” parotidectomy may enhance the access to the retromandibular area. A total parotidectomy is indicated when facing an epithelial malignancy of the parotid gland. Skeletonization of the main trunk of the facial nerve and its branches facilitates their retraction and the access to the ITF. Resection of the main trunk of the facial nerve and its branches (radical parotidectomy) is indicated when the nerve is invaded by the tumor. Attention is then directed to the cervical exposure to obtain proximal control of the common, internal, and external carotid arteries, and the internal jugular vein. CN X to XII are identified and preserved. The sternocleidomastoid and digastric muscles are transected at their insertion to the mastoid bone. The stylohyoid and stylopharyngeus muscles are transected, and the styloid process is removed. CN IX usually can be identified at this time, as it crosses lateral to the ICA.
658
OTOLOGIC SURGERY Curvilinear incision
Conchal incision
Conchal incision
A B
FIGURE 54-8. A, Curvilinear incision is made from temporal area to mastoid bone and upper cervical region. Flap is elevated superficial to deep temporal fascia, deep to mastoid periosteum, and deep to platysma muscle. B, Conchal incision is preferable to incision of external auditory canal when permanent obliteration of the ear is not indicated.
A mastoidectomy and dissection of the vertical portion of the facial nerve allows the transposition of the facial nerve, providing a wider access to the infratemporal fossa (see Fig. 54-9). In patients who require a radical parotidectomy, a mastoidectomy provides the means to obtain proximal control of the neural margins and to graft the nerve. It also provides access to the jugular bulb and adjacent lower cranial nerves. Orbitozygomatic osteotomies may be performed as previously described (preauricular approach). After the orbitozygomatic complex is removed, the anterior, superior, medial, and posterior boundaries of the infratemporal fossa are well exposed, and all major vessels are “controlled.” Completion of the infratemporal skull base approach, including a temporal craniotomy, is performed as described in the previous section. The extirpation of the tumor can now proceed, including the involved soft tissue and bone. Reconstruction of the defect follows the principles outlined in previous sections.
Lateral Fisch Infratemporal Fossa Approaches Fisch has described an array of lateral infratemporal fossa approaches that are the prototypic otologic approaches to the ITF (Fig. 54-8,9,10)3. The hallmark of these approaches
is temporal bone management emphasizing facial nerve rerouting and subtemporal dural exposure for wide access to the lateral skull base. Figure 54-10 illustrates the anatomic regions appropriate for the Fisch type A, B, and C approaches. The Fisch type A approach has been described in detail in Chapter 46 regarding its application in glomus jugulare surgery. The Fisch type B and C approaches are designed to approach more anterior pathology involving the petrous apex and clivus. The type C approach is an extension of the type B approach, and is used for lesions of the anterior ITF, sella, and nasopharynx. The Fisch type D is a preauricular ITF approach using an orbitozygotomy and resection of the floor of the middle fossa for medial dural exposure without a lateral temporal craniotomy.
Fisch Type B The principal exposure maneuver in the Fisch type B ITF approach is reflection of the zygomatic arch and temporalis muscle inferiorly and removal of the bone of the skull base floor to provide access to the ITF. The incision is wide and C-shaped beginning at the angle of the mandible and extending retroauricularly and anterior laterally to the eyebrow. The ear canal is transected and closed in the same manner as described in Chapter 47 in a two-layer technique.
Chapter 54 • Anterior and Subtemporal Approaches to the Infratemporal Fossa
659
Raised fascia (deep)
1
Fat pad 2 Facial flap V3
Parotid gland
Ear canal sewn Facial nerve isolated
Temporalis muscle
3
A
B
Temporalis muscle Stump of mandible
FIGURE 54-9. Fisch B approach. A, The external ear canal has been divided and sewn, the facial nerve is exposed, the internal carotid artery is exposed after a TMJ exenteration. B, V3 is exposed after a subtemporal craniotomy. Surgical exposure obtained after temporal craniotomy and reflection of temporalis muscle inferiorly. 1, temporal fat pad; 2, facial nerve.
The main trunk of the facial nerve is identified using standard landmarks at the stylomastoid foramen. The superior division is followed to the level of the frontalis branch. With this direct visualization, the periosteum attached to the zygomatic arch is reflected down to protect the frontalis branch of the facial nerve. At this point, the osteotomies of the zygomatic arch can be completed anteriorly as close as possible to the orbital rim and posteriorly at the root of the zygoma. The masseter muscle is left attached at the zygomatic arch to be reflected inferiorly. The temporalis muscle is completely reflected inferiorly, carefully protecting its blood supply. A key to this extradural exposure is the subtotal “petrosectomy.” This step includes a canal wall down mastoidectomy including complete skeletonization of the labyrinth, facial nerve, sigmoid sinus, middle fossa, and posterior fossa dura and the jugular bulb, and exenteration of all hypotympanic air cells and skeletonization of the ICA. The unique aspect is that by removing the ossicular chain and middle ear structures, the carotid artery can be skeletonized completely beyond the genu to the foramen lacerum. The TMJ is disarticulated by incising the capsule and removing the articular disc. At this point, the bone of the glenoid fossa and the root of the zygoma are completely removed with cutting and diamond burrs. Middle fossa dura in the subtemporal region is completely skeletonized. By placing the infratemporal fossa retractor over the mandibular condyle, additional skeletonization of the
middle fossa dura can be accomplished medially until reaching the middle meningeal artery and V3 at the foramen ovale. Cauterization of the middle meningeal artery and transection of the mandibular nerve permit greater exposure. At this point, further skeletonization of the carotid artery is possible along the lateral and anterior wall of the ICA. Complete exposure of the carotid artery permits its mobilization out of the carotid canal, providing free access to the petrous apex and clivus. The eustachian tube must be sutured closed to prevent infection of the nasal cavity. Although the bone defect is filled with abdominal fat, temporalis muscle is used to cover the fat and is placed inferior to the skeletonized middle fossa dura and mandibular condyle. The zygomatic arch can be secured with microplates, and the skin can be closed in a standard fashion.
Fisch Type C The principal distinguishing feature of the Fisch type C approach compared with the type B approach is resection of the pterygoid plates. This resection permits exposure of the lateral wall of the nasopharynx, eustachian tube orifice, posterior maxillary sinus, and posterior nasopharyngeal wall past the midline. After completion of the type B approach, the lateral surface of the pterygoid process is identified, and soft tissues are elevated. In this manner, the base of medial and lateral plates of the pterygoid processes can be drilled away, exposing the lateral wall of the nasopharynx. The
660
OTOLOGIC SURGERY
FIGURE 54-10. Different exposures obtained with Fisch A, B, and C approaches to infratemporal fossa. EAC, external auditory canal; IAC, internal auditory canal; ICA, internal carotid artery.
nasal cavity can be entered. The exposure permits full visualization of the peritubal area, which can be resected en bloc. Inferiorly, the excision can extend to the upper surface of the palate. Superiorly, the dissection can extend to the carotid artery and the cavernous sinus. The wide communication between the nasopharynx and the operative field makes closure in type C ITF surgery more difficult than in type B. Although mobilization of the entire temporalis muscle into the wound is one technique, vascularized free flaps are often necessary to provide adequate closure. Abdominal fat should be avoided because of the possibility of contamination.
Fisch Type D The Fisch type D ITF exposure is a preauricular modification of the Fisch infratemporal approach. Type D1 addresses tumors of the anterior infratemporal fossa, whereas type D2 is designed for lateral orbital wall lesions and high pterygopalatine fossa tumors. The distinguishing feature of the type D approach from the B and C approaches is that the middle ear and eustachian tube area is not obliterated, and conductive hearing is not sacrificed. In addition, the intratemporal facial nerve
is not rerouted, and the petrous ICA is not fully exposed. Although these preauricular approaches do not include a temporal craniotomy, the floor of the skull base can be drilled away to allow full access to the ITF.
Anterior Transfacial Approach (Facial Translocation) The anterior transfacial technique is best used to approach sinonasal tumors invading the ITF, the masticator space, or the pterygomaxillary fossa, and for tumors of the nasopharynx extending into the ITF (Fig. 54-11).7,13,14 A bicoronal incision with an ipsilateral preauricular extension is performed and extended through the subcutaneous tissue (see the section on preauricular approach). A Weber-Fergusson incision is completed and extended down to the periosteum of the maxilla, nasal bones, and orbital rim. During a “traditional” translocation approach, a horizontal incision is carried over the superior edge of the zygomatic bone, extending into the lateral canthus, to meet the Weber-Fergusson incision (see Fig. 54-11). The frontal branches of the facial nerve are identified and dissected as they cross over the zygomatic arch. They are then entubulated with silicone tubing and transected.
Chapter 54 • Anterior and Subtemporal Approaches to the Infratemporal Fossa
A
Masseter muscle
B
661
C
FIGURE 54-11. Incisions and osteotomies for facial translocation approach as described by Janecka.13,14
These nerve branches are reanastomosed at the end of the case, using an entubulation technique. Subperiosteal dissection of the anterior maxilla exposes the infraorbital nerve, which is then transected and tagged to facilitate its identification and reanastomosis at the end of the case. Then, an inferiorly based flap including the upper third of the upper lip, entire cheek, lower eyelid, parotid gland, and facial nerve is reflected inferiorly. The frontotemporal scalp flap is elevated in a subpericranial plane. This flap is reflected anteriorly, exposing the superior orbital rims (see Fig. 54-11). Alternatively, the exposure can be achieved without the temporal incision by combining the preauricular approach with the anterior exposure provided by the WeberFergusson incision. Orbitozygomatic osteotomies are performed and joined with the maxillary osteotomies to free the anterior face of the ipsilateral maxilla en bloc with the orbitozygomatic complex. Alternatively, the maxillary bone graft can be elevated as a vascularized graft attached to the cheek flap, as described by Catalano and Biller.8 The temporalis and masseter muscles are dissected from the zygomatic bone with electrocautery. Osteotomies are completed, and the bone graft is removed. The temporalis muscle is reflected inferiorly. Removal of the coronoid process increases the caudal arc of rotation of the temporalis muscle. After completion of these steps, the anterior, medial, and lateral boundaries of the ITF are well exposed. In selected cases, the pterygoid plates can be excised to provide further access to the medial ITF or nasopharynx. A temporosubtemporal craniotomy provides additional exposure superiorly and allows dissection of intracranial structures (Fig. 54-12). After the tumor resection, the temporalis muscle may be used to obliterate the surgical defect and provide separation of the cranial cavity from the upper aerodigestive tract, as previously described.
Periosteal and muscular attachments are repaired, and the incisions are closed using a multilayer technique. The conjunctiva is repaired with running 6-0 fastabsorbing suture. The lacrimal canaliculi are stented with Crawford silicone tubing that is tied to itself in the nasal cavity. The eye is closed with a temporary tarsorrhaphy for 10 to 14 days to prevent a lower lid ectropion.
Transorbital Approach In selected cases, a transorbital approach may be used to complement the exposures obtained with one of the previous approaches enhancing the exposure of the orbital apex and cavernous sinus. This approach consists of transection of the orbital tissues posterior to the globe with preservation of the attachments of the orbital soft tissues, including the globe, to the scalp flap. The orbital apex is removed to provide direct anterior access to the cavernous sinus and cavernous ICA. This approach is reserved for patients with benign tumors of the orbita apex and cavernous sinus who have lost vision as a result of tumor growth. It may also be employed for low-grade malignant neoplasms with minimal involvement of the orbital soft apex or optic nerve to obtain complete tumor removal. Extensive involvement of the orbital soft tissues requires an orbital exenteration. The advantages of this approach include improved cosmesis, a result of the preservation of the globe, and excellent anterior and lateral exposure of the cavernous sinus and its associated structures. A preauricular infratemporal skull base approach is performed, as previously described. The periorbita is elevated from the superior, lateral, and inferior walls of the orbit. The periorbita is then incised, and the orbital tissues are transected posterior to the globe using bipolar electrocautery and sharp dissection. A cuff of tissue remains at the orbital apex to provide an adequate tumor margin.
662
OTOLOGIC SURGERY
FIGURE 54-12. Subtemporal craniectomy is performed to provide additional exposure and adequate resection margin of extracranial tumors. Maximal exposure of infratemporal and central skull base is achieved.
The remaining periorbital attachments are elevated medially to allow complete displacement of the globe from the orbital cavity. Over the medial wall, the neurovascular bundles are clipped or cauterized and transected. The lacrimal duct is transected, and the sac is marsupialized. Using rongeurs, bone is removed from the lateral wall of the orbit to the superior orbital fissure. The contents of the superior orbital fissure and optic canal are transected to provide additional exposure of the orbital apex. Because of the loss of orbital bone, enophthalmos results unless the orbital defect is reconstructed with bone grafts or titanium mesh. A temporalis transposition or free tissue transfer provides soft tissue augmentation and protection of the carotid artery.
POSTOPERATIVE CARE After surgery, the patient is transferred to the intensive care unit for continuous cardiovascular and neurologic monitoring. Laboratory tests to rule out postoperative anemia and electrolyte imbalance are performed. Patients who required multiple blood transfusions should be screened for transfusion-induced coagulation disorders. Mild narcotic analgesia is provided, avoiding sedation that could interfere with a detailed neurologic evaluation.
If the ICA is dissected, ligated, or grafted, close monitoring of the patient’s hemodynamic status and fluid balance is essential. When grafting of the ICA is performed, an angiogram is obtained in the early postoperative period to assess the patency of the graft and detect pseudoaneurysm formation. A CT scan of the brain without contrast medium is performed on the 1st or 2nd postoperative day to screen for intracranial complications, such as cerebral contusion, edema or hemorrhage, fluid collections, or pneumocephalus. A compressive dressing is maintained for 24 to 72 hours. When the dressing is removed, the wound is cleaned with normal saline solution and covered with antibiotic ointment three to four times a day. The scalp and other wound drains are kept to bulb suction until the drainage is less than 30 mL/day. The drain is then removed, and the wound is closed using an encircling stitch placed at the time of surgery. If the cranial cavity is entered, wall suction is never used because of the risk of direct negative pressure on the central nervous system. In most cases, the spinal drain is needed only during the surgery and is removed when the procedure is completed. If there is a significant risk of postoperative CSF leak, the spinal drain is kept at the level of the patient’s shoulder, and 50 mL is removed every 8 to 12 hours. The lumbar drain is removed 3 to 5 days after surgery,
Chapter 54 • Anterior and Subtemporal Approaches to the Infratemporal Fossa
and the lumbar puncture site is closed with an encircling stitch (e.g., 2-0 nylon), placed at the time of surgery. Lagophthalmos from weakness or paralysis of the facial nerve may lead to exposure keratoconjunctivitis. Initially, an exposed eye can be protected using artificial tears every 1 to 2 hours, lubrication ointment at bedtime, eye patching, or a moisture chamber. Taping of the eyelids or a temporary tarsorrhaphy is advised if rapid recovery is anticipated. If prolonged paralysis is expected, we prefer a gold weight implant. This procedure can be performed at the time of the original surgery, using a 0.1 to 0.12 g weight (No. 10 or 12), or it may be performed during the early postoperative period. Except in selected cases, we favor the latter because it provides the advantage of being able to establish the exact weight that is needed by the patient. In most cases, the airway can be secured for the short-term using an endotracheal tube (high-volume/ low-pressure cuff). Nevertheless, a tracheotomy is indicated for patients in whom significant edema of the upper aerodigestive tract is anticipated, or if a prolonged mechanical ventilation is anticipated. A tracheotomy also provides better access to the airway for pulmonary toilet for patients with an ineffective cough or severe aspiration. Patients with high vagal lesions or any combination of deficits of CN IX, X, or XII experience severe swallowing difficulty and aspiration. These patients can be assisted with a medialization laryngoplasty, an arytenoid adduction procedure, or an arytenoidpexy with or without a cricopharyngeal myotomy. Patients who continue to aspirate despite all these measures and who develop repeated aspiration pneumonias are managed by a laryngotracheal separation procedure.
PITFALLS AND COMPLICATIONS The most common morbidity associated with surgery of the infratemporal fossa is related to deficits of the trigeminal nerve. Sacrifice of the third, sometimes the second, and rarely the first divisions of the trigeminal nerve may be necessary for surgical exposure or to obtain adequate clear margins of resection. Facial anesthesia may predispose the patient to self-inflicted injuries, including neurotrophic ulcers. The loss of corneal sensation, especially in a patient with paresis of the facial nerve, greatly increases the risk of a corneal abrasion or exposure keratitis. The loss of motor function of the mandibular nerve causes asymmetry of jaw opening and decreased force of mastication on the operated side. Mastication may be impaired further by resection of the TMJ or mandibular ramus. Whenever feasible, sensory and motor divisions of the trigeminal nerve are repaired or grafted after transection for surgical exposure. Permanent deficits (accidental) of the facial nerve or its branches are uncommon. The frontal branches of the
663
facial nerve are at risk of injury during elevation of the temporal scalp flap. Injury is usually the result either of a dissection in a plane that is superficial to the superficial layer of the deep temporal fascia or of compression during the retraction of the flap. To avoid a traction injury, a cuff of soft tissue is preserved around the main trunk of the facial nerve when a preauricular approach is employed. The facial nerve can also sustain an ischemic injury that occurs as a result of devascularization on mobilization of its infratemporal segments or skeletonization of its extratemporal segments. A temporary paresis of the facial nerve is to be expected with mobilization of the mastoid segment of the facial nerve. Close attention to postoperative eye care is necessary in patients with combined deficits of the trigeminal and facial nerves. In most patients, surgical resection of the TMJ is not a major factor in the development of postoperative trismus or difficulties with mastication. Rather, mastication seems to be most affected by loss of function of the mandibular division of the trigeminal nerve. Nevertheless, every effort is made to preserve the TMJ. If resection of the glenoid fossa is necessary, the capsule of the TMJ is displaced inferiorly. If resection of the TMJ is necessary, no attempt is made to reconstruct the joint. These patients experience deviation of the jaw to the unaffected side. This is usually of no major consequence, but some patients may need an occlusal guide to help them when chewing. Postoperative trismus is also a common occurrence because of postoperative pain and scarring of the pterygoid musculature and TMJ. Trismus improves dramatically if patients regularly perform stretching exercises for the jaw. Devices such as the TheraBite appliance are helpful in stretching the scar tissue and forcefully opening the mouth. In severe cases, a dental appliance may be fabricated that is gradually opened by a screw. Infectious complications are rare. Predisposing factors include communication with the nasopharynx, seroma or hematoma, and a CSF leak. Generally, the dead space should be obliterated to prevent fluid collection that subsequently can be infected, and the cranial cavity should be separated from the sinonasal tract. The use of vascularized tissue flaps is preferred, especially when there has been dissection of the ICA or resection of dura. Necrosis of the scalp flap is uncommon because of its excellent blood supply. Poorly designed incisions may result, however, in areas of ischemia, particularly around the auricle, which can make the tissue susceptible to secondary infection. Prolonged use of hemostatic clamps can also lead to necrosis of the wound edges. Neurovascular complications are of the greatest concern. Postoperative cerebral ischemia may result from surgical occlusion of the ICA, temporary vasospasm, and thromboembolic phenomena. Surgical dissection of the ICA can injure the vessel walls, resulting in immediate or delayed rupture and hemorrhage. The ICA is particularly
664
OTOLOGIC SURGERY
vulnerable to injury where it enters the cranial base. Injuries to the ICA should be repaired primarily (or using a vein graft). An angiogram is obtained in the early postoperative period to assess the adequacy of the repair. If a repair of the ICA is impossible, it should be permanently occluded by ligation or by the placement of a detachable balloon or vascular coil. When the artery is to be permanently occluded, the occlusion is performed as distal as possible (near the origin of the ophthalmic artery). The potential for thrombus formation is less with a short column of stagnant blood above the level of occlusion. After occlusion of the ICA, there is a significant risk of immediate and delayed stroke in patients who do not have more than 35 to 40 mL of blood flow per 100 g of brain tissue per minute by ABOX-CT testing. After reconstruction of the ICA with a vein graft, there is a risk of postoperative occlusion because of thrombus formation at the suture line, and torsion or kinking of the graft. Pseudoaneurysm formation and delayed blowout of the graft are also risks, especially in the presence of infection. For this reason, reconstruction of the ICA is usually not indicated in a contaminated field with communication to the upper aerodigestive tract. In such cases, permanent occlusion of the ICA or rerouting of a vein graft posterior to the surgical field is performed. An extracranial-intracranial bypass graft to the middle cerebral artery may be performed before tumor resection when sacrifice of the ICA is anticipated. Patients who undergo surgical manipulation of the ICA may also develop cerebral ischemia at the margins of the vascular territories of the cerebral vessels (watershed areas). This ischemia is of particular concern when there is sacrifice of extracranial-intracranial collateral blood vessels, which are not routinely assessed by ABOX-CT as part of the surgical approach. Decreased oxygen delivery because of hypoxic postoperative anemia or hypotension can result in a cerebral infarct in these watershed areas. A watertight dural closure may be difficult to achieve with large infratemporal skull base defects, particularly around nerves and vessels. An epidural fluid collection may result. In most cases, this fluid collection is contained by the soft tissues and slowly resolves without further intervention. Occasionally, the CSF collection may communicate with the exterior through the EAC, the scalp incision line, or along the eustachian tube to the nasopharynx. Most CSF leaks can be managed nonsurgically by placement of a pressure dressing and a spinal drain to diminish the CSF pressure. Surgical exploration and repair of the dural defect may be necessary if the CSF leak does not resolve within 1 week. A middle ear effusion is often apparent after infratemporal skull base approaches because of dysfunction or interruption of the eustachian tube. Tympanostomy tubes are not placed for at least 6 weeks postoperatively, however, because there is always a risk of CSF communication. We have encountered patients who developed profuse unilateral rhinorrhea in the postoperative period that
was misinterpreted as a CSF leak. These cases all were associated with surgical dissection of the petrous ICA and are probably due to loss of the sympathetic fibers that travel along the ICA in their route to the nasal mucosa. This loss produces vasomotor rhinitis that may be treated with the use of anticholinergic nasal sprays. Testing of the fluid for β2-transferrin is mandatory, however, to rule out a CSF leak. Cosmetic deformities may result from the loss of soft tissue and bone. Transposition of the temporalis muscle results in a depression in the temporal area. This depression can be lessened by placement of a free-fat graft or hydroxyapatite cement in a secondary surgery. If the temporalis muscle is not transposed, the anterior margin of the muscle should be resutured anteriorly and superiorly to prevent its retraction and a resulting depression lateral to the orbital rim. The use of microvascular free muscle flaps, such as the rectus abdominis flap, for reconstruction may necessitate sacrifice of the zygomatic arch to accommodate the additional bulk. As the muscle atrophies, significant depression may occur. It is important to repair all periosteal and muscle attachments around the maxilla, orbital rim, and zygomatic arch to avoid a “cadaveric” look that occurs when the soft tissues over these areas atrophy or retract. Large muscle flaps, such as a latissimus dorsi flap, may swell and compress the brain if the cranial base is not reconstructed.
REFERENCES 1. Barbosa J F: Surgery of extensive cancer of paranasal sinuses: Presentation of a new technique. Arch Otolaryngol 73:129-138, 1961. 2. Terz JJ, Young H F, Lawrence W Jr: Combined craniofacial resection for locally advanced carcinoma of the head and neck, II: Carcinoma of the paranasal sinuses. Am J Surg 140:618-624, 1980. 3. Fisch U: The infratemporal fossa approach for the lateral skull base. Otolaryngol Clin North Am 17:513-552, 1984. 4. Biller H F, Shugar J M A, Krespi YP: A new technique for wide-field exposure of the base of the skull. Arch Otolaryngol 107:698-707, 1981. 5. Sekhar L N, Schramm VL , Jones N F: Subtemporalpreauricular infratemporal fossa approach to large lateral and posterior cranial base neoplasms. J Neurosurg 67:499, 1987. 6. Cocke EW Jr, Robertson J H, Robertson JT, Crooke J P Jr: The extended maxillotomy and subtotal maxillectomy for excision of skull base tumors. Arch Otolaryngol Head Neck Surg 116:92-104, 1990. 7. Janecka I P, Sen C N, Sekhar L N, Arriaga M : Facial translocation: A new approach to the cranial base. Arch Otolaryngol Head Neck Surg 103:413-419, 1990. 8. Catalano PJ, Biller H F: Extended osteoplastic maxillotomy: A versatile new procedure for wide access to the central skull base and infratemporal fossa. Arch Otolaryngol Head Neck Surg 119:394-400, 1993.
Chapter 54 • Anterior and Subtemporal Approaches to the Infratemporal Fossa 9. Snyderman C H, Carrau R L , de Vries E J: Carotid artery resection: Update on preoperative evaluation. In Johnson JT, Derkay C S, Mandell-Brown M K, Newman R K (eds): AAO-HNS Instructional Courses, 6. Chicago, IL, Mosby Year Book, 1993, pp 341-346. 10. Netterville J L , Jackson G, Civantos F: Thyroplasty in the functional rehabilitation of neurotologic skull base surgery patients. Am J Otol 14:460-464, 1993. 11. Carrau R L , Pou A, Eibling D E, et al: Laryngeal framework surgery for the management of aspiration. Head Neck 21:139-145, 1999.
665
12. Pou A, Carrau R L , Eibling D E, Murry T: Laryngeal framework surgery for the management of aspiration in high vagal lesions. Am J Otolaryngol 19:1-8, 1998. 13. Nuss DW, Janecka I P, Sekhar L N, Sen C N: Craniofacial disassembly in the management of skull-base tumors. Otolaryngol Clin North Am 24:1465-1497, 1991. 14. Sekhar L N, Sen C, Snyderman C H, Janecka I P: Anterior, anterolateral, and lateral approaches to extradural petroclival tumors. In Sekhar L N, Janecka I P (eds): Surgery of Cranial Base Tumors. New York, Raven Press, 1993, pp 157-223.
55
Endoscopic Endonasal Approaches to the Skull Base and Paranasal Sinuses Paul A. Gardner, Amin B. Kassam, Carl H. Snyderman, Ricardo L. Carrau, and Daniel M. Prevedello Videos corresponding to this chapter are available online at www.expertconsult.com.
INTRODUCTION/BACKGROUND Approaches to the skull base via the paranasal sinuses were introduced in the late 19th and early 20th century. Tumors of the sellar region were first approached through various incisions in the forehead to gain access to the sphenoid sinus via the ethmoid sinuses. Cushing introduced what became the standard for transsphenoidal surgery in 1910 when he first used a sublabial incision to gain access to the sinuses.1 coincidentally, on the exact same day, June 4, Oscar Hirsch used an endonasal approach to gain access to the sphenoid sinus. This would eventually replace the sublabial approach, but Cushing’s influence delayed this for decades. This transsphenoidal approach to the sella turcica was conceptually unchanged for approximately 70 years. However, the operation and its outcomes were changed greatly during this period by advances in technology which provided improved visualization, intraoperative image-guidance and instrumentation. Dott, Guiot and Hardy added pneumoencephalocisternography, radiofluoroscopic guidance, and the operative microscope. All of this revolutionized the approach and improved outcomes. A similar change in technology began in the 1980s with the development of early intraoperative CT and MR image-guidance systems and the introduction of the endoscope by Carrau.2,3 At the same time, otolaryngologists were developing and refining functional endoscopic sinus surgery (FESS).4-9 It was the collaboration between otolaryngologists and neurosurgeons that led to the expansion of the standard transsphenoidal approach to include the rest of the anterior skull base and beyond.
BASIC ENDOSCOPIC ENDONASAL CONCEPTS The application of endoscopy to the endonasal approach has allowed the expansion of this approach while improving visualization. This is due to one basic optic difference
when compared to a microscope. A microscope visualizes from a distance and delivers light in a cone whose apex is at the target. This requires a superficial exposure which is wider than the deep exposure (“ice cream cone” effect). An endoscope delivers light and provides a view in a cone whose apex is at the tip of the scope. This allows a smaller exposure superficially while allowing a much wider working field in the depth (“flashlight” effect). This becomes a distinct advantage when approaching deep lesions with complex surroundings, such as those involving the skull base and paranasal sinuses. This does create a problem with instrumentation and the modification of existing and development of new instrumentation was necessary and is still ongoing. The loss of three-dimensional visualization is easily overcome using an active, handheld endoscope and proprioceptive cues obtained by keeping one instrument on or near the object of focus. Three-dimensional sensation is recreated via propioception and triangulation of instruments. This is a part of the learning curve, but one which is relatively easy to overcome. Despite these differences, endonasal techniques should not differ from standard microsurgical techniques. Two-handed dissection by the operating surgeon is required and therefore two surgeons are needed. Identification of critical structures, central debulking followed by extracapsular dissection and fine, sharp dissection are all critical components of microsurgery and should be directly translated to endoscopic neurosurgery (endoneurosurgery). One of the main reasons these approaches hold such promise is that many skull base lesions are medial and anterior to the surrounding neurovascular structures. This provides a distinct advantage to a direct anterior, midline approach, as these structures do not have to be displaced or transgressed in order to access them. This principle guides the selection of tumors for EEA. Vascularity, tumor consistency, and size represent important surgical considerations but do not constitute contraindications to an EEA. 667
668
OTOLOGIC SURGERY
approaches. The middle fossa is the most complex and is broken into five “transpterygoid” anatomic zones of approach: medial pterygoid (petrous apex), petroclival junction, quadrangular space or Meckel’s cave, superior cavernous sinus, and the infratemporal fossa (Fig. 55-3). These are all critically dependent upon their relationship with the internal carotid artery (ICA). Finally, the posterior fossa or inferior expanded approaches consist of the transcondylar and parapharyngeal space approaches.
EXPANDED ENDONASAL APPROACHES: ANATOMIC MODULES The key to the development of endonasal approaches to skull base pathology was the collaboration between otolaryngologists and neurosurgeons. The knowledge of the paranasal sinuses developed by otolaryngologists melded with the work neurosurgeons had done in the sella turcica for pituitary tumors. Both were supplemented by knowledge of skull base anatomy as learned through performing traditional, open approaches. Endoscopic endonasal approaches (EEAs) can be divided into the sagittal or rostral-caudal plane (between the carotids) and coronal or paramedian plane (lateral to the carotid). The sagittal plane can be divided from rostral to caudal, anterior to posterior into the transfrontal, transcribriform, transplanum/transtuberculum, transsellar, transclival and transodontoid approaches (Fig. 55-1). The coronal plane is somewhat more complex in that the lateral expanded approaches vary based upon which fossa is involved (Fig. 55-2). The anterior fossa, lateral approaches consist of the supra- and transorbital
Sagittal Plane This represents the region between the internal carotid arteries (ICA) as it rises and courses along the ventral skull base from a caudal to rostral direction as well as the rostral extension of this midline region. The sagittal plane is also referred to as the rostral-caudal plane.
Transfrontal Approach The transfrontal approach represents the most anterior expanded endonasal approach. A transfrontal approach starts with identification of the nasofrontal recesses and
Modular approaches
I II P. 1.
S. S.
V CN 7, 8 CN 9, 10, 11
S.C.
2. 3.
M.C.
4.
I.C.
12
5.
P.
1. 2. 3. 4.
Transcribriform Transplanum Transsellar Transclival: superior (S.C.) middle (M.C.) inferior (I.C.) 5. Transodontoid
S.S .
Tumor
A Figure 55-1. A, Lateral view of the sagittal plane modular endoscopic endonasal approaches (EEAs) as shown in a rostral-caudal axis.
Chapter 55 • Endoscopic Endonasal Approaches to the Skull Base and Paranasal Sinuses
669
1. C.N.I
2. 3.
4. 5. 6.
7. V
8.
B.A.
S. S.
C.E.
Tumor locations 1. Near cribriform plate (ethmoid) 2. Suprasellar near A.C.P. 3. Optic chiasma, and C.N. II 4. Pituitary gland
5. 6. 7. 8.
Dorsum sellae P.C.P. (post.) clinoid process Cavernous sinus Clivus
B Figure 55-1—cont’d B, View of the anterior skull base from above showing sample tumor locations for the various rostral-caudal modules.
continues until bilateral frontal sinusotomies are performed with creation of a superior septal defect and removal of the floor of the frontal sinuses (Draf 3 or Lothrop procedure) if needed. The anterior attachment of the middle turbinate provides a posterior landmark to preserve olfaction and avoid violation of the cribriform plate with resultant cerebrospinal fluid (CSF) leak. This approach can be used in isolation for rare lesions such as nasal dermoid cysts or fibro-osseous tumors and more commonly for frontal sinusitis or mucoceles. In pediatric patients with dermoid cysts, the cyst is drained endonasally but the septum should be resected up to the skull base. The transfrontal approach is also used in conjunction with the transcribriform approach for accessing the anterior extent of lesions such as olfactory groove meningiomas. A key anatomic landmark for this region is the frontoethmoidal recess.
Transcribriform Approach The transcribriform approach is a commonly used approach for anterior skull base tumors. Most often used in our practice for resection of olfactory groove
eningiomas and esthesioneuroblastomas, it is also used m for repair of post-traumatic and iatrogenic CSF leak repairs and has potential for any subfrontal lesion. A complete sphenoethmoidectomy is performed on each side and the nasofrontal recesses are visualized. If needed, exophytic tumor within the nasal cavity can be debulked. The superior nasal septum is transected from the crista galli to the sphenoid rostrum as needed. As mentioned above, a transfrontal approach can be performed to establish an anterior margin. The transcribriform approach provides direct access to the vascular supply of tumors such as olfactory groove meningiomas, allowing early devascularization. The anterior and posterior ethmoidal arteries should be identified early and cauterized or clip ligated. The lateral bony margins are drilled to form “gutters” or osteotomies for the length of the tumor or planned resection as needed. If necessary, the lamina papyracea can be removed as well to allow retraction on the periorbita, thereby displacing the orbital contents for even more lateral access. In fact, the lateral access can extend all the way to the midorbital line at the level of the superior rectus muscle. After the lateral
670
OTOLOGIC SURGERY
argins are drilled, the planum or tuberculum can be m drilled as needed for a posterior margin. Even if necessary for tumor resection, it may be safest to leave the bone of the optic canals intact as long as possible. Finally, the dura is dissected from the crista galli to allow its removal. The remaining bone of the cribriform plate can now be dissected free (Fig. 55-4). If necessary for purely extradural lesions, the dura should be left intact. This may not be possible over the olfactory filaments, which can be cauterized to minimize CSF leakage, but should nonetheless undergo formal repair (see below). Obviously, olfaction (when present) is sacrificed if a bilateral approach is undertaken. The dura is opened in the same fashion as with microsurgery, with a fine blade and scissors. The durotomies are paramedian on both sides of the falx. This is done in order to minimize bleeding from branches of the anterior falcine artery and the inferior sagittal sinus. Bilateral durotomies are made either at the lateral extent of desired resection or overlying tumor. The durotomy is extended to the midline both anteriorly and posteriorly. The falx and inferior sagittal sinus must be systematically addressed next. Pistol-grip, endonasal bipolars are used with one blade on either side of the falx/sinus to coagulate it prior to transecting it to allow anterior tumor or dural release. In addition, anterior falcine branches may be encountered during tumor resection for access to the falx, providing additional devascularization. Dura can be resected en bloc as needed for tumors such as esthesioneuroblastoma. All tissues in such cases must be
1.
2.
3. B.S. S. S.
Ce.
Rectangles designate areas of approach by fossa 1. Anterior 2. Middle fossa 3. Posterior
�
Figure 55-2. Superior view of skull base with general regions of approach by fossa. 1) anterior, 2) middle, 3) posterior.
S.T., S.S.
C.S.
T.L. 4. V3 3.
1. V.N.
C.S.–Cavernous sinus L.P.M. V.N.–Vidian nerve V3–V3 in foramen ovale M.P.M. S.T.–Sella turcica S.S.–Sphenoid sinus T.L.–Temporal lobe L.P.M./M.P.M.–Lat and Medial pterygoid m’s. Np.–Nasopharynx
Np.
T.L. 5.
2.
. I. J . V C. A.
Lateral pterygoid plate Medial pterygoid plate
Figure 55-3. Coronal plane, lateral EEAs by anatomic zones. Zone 1 is the petrous apex, Zone 2 the petroclival junction, Zone 3 the quadrangular space/Meckel’s cave, Zone 4 the superior lateral cavernous sinus, and Zone 5 the infratemporal fossa.
Chapter 55 • Endoscopic Endonasal Approaches to the Skull Base and Paranasal Sinuses
evaluated both intraoperatively and postoperatively for margins. Intradural tumor resection must be performed with caution, especially when approaching the interhemispheric fissure, as there are frequently anterior cerebral artery branches or even the anterior communicating artery on the surface of the tumor (Fig. 55-5). An endonasal ultrasonic aspirator or two suctions can be used depending upon tumor consistency and proximity of involved structures. Microsurgical concepts of preserving
CG
OFim PO
ON
PO
PI tub I C mOCR A sella
ON I C A
I OCR
Figure 55-4. (Reprinted with permission Gardner PA, Kassam AB, Thomas A, Snyderman CH, Carrau RL, Mintz AH, Prevedello DM. Endoscopic endonasal resection of anterior cranial base meningiomas. Neurosurgery 2008;63:36-52.Copyright 2008, Lippincott, Williams & Wilkins.) Intraoperative, endoscopic view of the entire anterior skull base following exposure and removal of necessary bone for approach to an anterior base lesion. (CG = crista galli; ICA = internal carotid artery; lOCR = lateral opticocarotid recess; mOCR = medial opticocarotid recess; Oflm = olfactory filament; ON = optic nerve protuberance; Pl = planum; PO = periorbita; tub = tuberculum.
671
arachnoid planes when possible and sharp dissection of critical structures is maintained throughout. All of this may require the use of angled scopes (45° or 70°).
Transplanum/Transtuberculum Approach Also described as an “extended” approach, the transplanum approach was the first expansion of a traditional transsphenoidal approach.10-13 This approach provides a natural corridor for many suprasellar tumors such as craniopharyngiomas, tuberculum meningiomas and large pituitary adenomas. It may also be used for biopsy or resection of infundibular lesions such as metastases or hypophysitis. This approach is often an integral addition to transsellar and transcribriform approaches. A transsphenoidal exposure (see transsellar approach below) is augmented by a posterior ethmoidectomy. The posterior ethmoidal arteries are a good landmark for anterior extent to preserve olfaction while providing adequate access. The optic canals are obviously critical to identify in order to avoid damage to the nerves. Whenever drilling over or near the optic canals, it is important to constantly irrigate to avoid heat transmission from the drill to the nerves. After the sella is exposed, the planum can be drilled and thinned. This can require an angled endoscope for adequate visualization depending upon the slope of the anterior skull base. Though somewhat counterintuitive, the planum is most easily removed in an anterior to posterior direction after the bone has been adequately thinned, exposing dura at the anterior extent and laterally. The lateral margins are actually the optic nerves which form the sides of a trapezoid which include the anterior planum and tuberculum sellae. Often there is no need to expose the optic nerves. However, in many tuberculum meningiomas, there is extension of disease into the medial optic canals (Fig. 55-6). This disease is
ACA
ACA
OGM
A
B
Figure 55-5. A, Preoperative CT angiography sagittal reconstruction showing the anterior cerebral artery (ACA) involvement with an olfactory groove meningioma (OGM). B, Postoperative, sagittal MRI following endoscopic endonasal approach (EEA) showing complete resection of the tumor with patent and intact ACA.
672
OTOLOGIC SURGERY
A
B
Figure 55-6. (Reprinted with permission Gardner PA, Kassam AB, Thomas A, Snyderman CH, Carrau RL, Mintz AH, Prevedello DM. Endoscopic endonasal resection of anterior cranial base meningiomas. Neurosurgery 2008;63:36-52.Copyright 2008, Lippincott,Williams & Wilkins.) A, Axial, post contrast MRI showing extension of meningioma into the left medial optic canal (arrow). B, Axial, post-contrast MRI following EEA (endoscopic endonasal approach) for meningioma showing resection of tumor from the medial optic canal (arrow).
not easily accessed via a craniotomy and requires release and retraction of an often already compromised optic nerve. In these cases, the bone overlying the tumor in the optic canal should be thinned (“blue-lined”) with a drill and carefully removed with a dissector. When approaching suprasellar lesions, the superior intercavernous sinus (SIS) can often merely be retracted inferiorly rather than being transected. If, however, it must be traversed, it is useful to open the dura of the sella below the SIS in the midline as well as the dura of the planum above the SIS. This provides access on both sides of the sinus to perform a controlled ligation, either with a bipolar or clips. The dura can be reduced bilaterally with bipolar coagulation, using care to not damage the optic nerves. It is also important not to coagulate the superior hypophyseal artery as this can lead to hypopituitarism. Once the dura is opened, tumor resection proceeds carefully, identifying critical structures systematically. First, one supraclinoid internal carotid artery (ICA) is identified. Resection proceeds until the ipsilateral optic nerve is identified, followed by the chiasm, then the contralateral optic nerve and supraclinoid ICA (Fig. 55-7). In order to gain direct and unencumbered access to the paraclinoid ICA it is imperative to remove the medial optico-carotid recess (mOCR). This represents the lateral extension of the tuberculum and the pneumatization of the middle clinoid.14 The mOCR is the key anatomic landmark for this module.
Transsellar Approach Transsellar approaches are well known for access to pituitary adenomas as well as the myriad of sellar pathologies which exist, such as Rathke’s cleft cysts (RCCs), arachnoid cysts and rare craniopharyngiomas. In addition,
ON
ch
ON
Inf
ICA
Figure 55-7. Endoscopic, endonasal view following resection of a s uprasellar meningioma. The tumor is debulked and then the following structures identified in succession: optic nerve (ON), chiasm (ch) and infundibulum (Inf), contralateral optic nerve (ON) and supraclinoid internal carotid artery (ICA).
by utilizing a pituitary transposition, retroinfundibular lesions, such as granular cell tumors and some craniopharyngiomas, can be easily accessed while potentially preserving pituitary function. Endoscopic approaches to the sella are different from standard microscopic approaches in that they both provide more lateral access and require more exposure. Indeed, tumor which extends into the medial cavernous sinus can be accessed effectively using an EEA (Fig. 55-8). The bone overlying the cavernous ICA which guards the medial cavernous sinus must be carefully thinned and removed to allow some displacement of the ICA. However, it is intimate and direct visualization which allows successful resection of tumor
Chapter 55 • Endoscopic Endonasal Approaches to the Skull Base and Paranasal Sinuses
Gland
673
Res Cav
ICA Adenoma
ICA MCW
A
B
Figure 55-8. A, Preoperative, contrast-enhanced coronal MRI showing recurrent pituitary adenoma largely involving the medial cavernous sinus, as evidenced by the enhancing medial cavernous wall (MCW). (ICA = cavernous internal carotid artery). B, Postoperative, contrast-enhanced, coronal MRI showing complete resection of the tumor, including the portion in the medial cavernous sinus. Reminaing pituitary gland and normal cavernous sinus contents are more apparent following tumor resection. (ICA = cavernous internal carotid artery; Res Cav = tumor resection cavity)
in the cavernous sinus. Lateral sphenoidal exposure will be covered in detail in the coronal plane section. Greater exposure is required because room must be made for instruments to work around the endoscope while maintaining visualization. As a result, the sphenoid must be opened completely to work unencumbered in the sella. For example, lesions such as RCCs which are commonly in the pars intermedia can be approached from below, under the anterior gland (“subsellar approach”) rather than through it. This requires drilling the floor of the sphenoid sinus flush with the clival recess to allow this inferior access and visualization that the endoscope provides. The guiding anatomic principle for this module is complete bony removal and exposure of the sella from “blue to blue”; that is, laterally from cavernous sinus to cavernous sinus and from SIS superiorly to the inferior intercavernous sinus (IIS) inferiorly.
Transclival Approaches The transclival approaches may be easier to understand if broken down into superior, middle and inferior clival approaches. The superior clivus extends from the level of the posterior clinoid to Dorello’s canal. The middle clival segment continues from Dorello’s canal to the jugular foramen and the lower third extends from the jugular fossa to the basion. The superior clival approach is usually combined with a transsellar approach and involves a pituitary transposition (see below) and posterior clinoidectomy. The middle clival approach is essentially the bulk of the clivus and has its own potential pitfalls and nuances. The inferior clivus is the approach to the foramen magnum.
Superior Clivus The pituitary transposition is a newly described technique for access to lesions directly behind the pituitary gland or infundibulum, such as retroinfundibular craniopharyngiomas, granular cell tumors and chordomas.15 In addition, clival lesions, such as “chondroid” tumors (e.g. chordomas and chondrosarcomas) often involve this portion of the clivus (Fig. 55-9). While conceptually simple, this is technically demanding. The pituitary gland is dissected free from the sella and lifted to allow access to the space behind it. However, there are many bands of fibrous connective tissue which anchor the gland in the sella, mostly along the lateral walls to the cavernous sinus. These bands must be sharply dissected to free and preserve the gland. There is often some venous bleeding during this portion of the dissection and a two-surgeon, three of four-hand technique becomes critical. The inferior hypophyseal arteries may need to be sacrificed. In our experience, as long as the superior hypophyseal arteries (SHAs) are preserved, gland function will be preserved. After all of the lateral connective bands are released, the aperture in the sellar dura through which the stalk enters should be opened to allow release of the stalk and complete freedom for transposition. This should be done carefully with a fine, pistol-grip scissor with visualization of the SHAs. At this point, the gland can be finally raised from the sella and held superiorly with fibrin glue. This gives direct access to the dorsum sellae and posterior clinoids which can then be drilled and removed for access to the interpeduncular and basilar cisterns. There is more brisk venous bleeding during this drilling, as the clival venous plexus extends into the clinoids and dorsum sellae. This tight space, surrounded by critical structures with often brisk bleeding, requires an experienced team
674
OTOLOGIC SURGERY
scar
gl ch gl
NSF
A
B
Figure 55-9. A, Sagittal post-contrast MRI showing a superior clival chordoma (ch) with extension behind the pituitary gland (gl) into the dorsum sellae. B, Postoperative MRI following EEA for the lesion in (A) showing complete resection of the superior clival lesion which involved a pituitary transposition, as evidenced by the scar tissue posterior to the gland (gl). There is a brightly enhancing nasal septal flap (NSF) in place for reconstruction.
PG MC
MC
A
B
Figure 55-10. A, Preoperative, post-contrast, sagittal MRI showing a large, clival chordoma involving all of the upper and middle clivus (MC) as well as some of the anterior skull base. B, Post-contrast, sagittal MRI of the same patient following EEA resulting in complete resection, including the clival component. (PG = intact pituitary gland; MC = middle clivus)
for a safe and controlled approach. In our opinion, this may be the most complex EEA procedure.
Middle Clivus This approach accesses the majority of the clivus. Tumors such as chordomas and chondrosarcomas can often be relatively easily accessed endonasally (Fig. 55-10).
In addition, we have used EEAs to address large portions of petroclival meningiomas, neurenteric cysts and a clival arteriovenous malformation (AVM). As with any approach it is necessary to understand the anatomic structures involved. The paraclival (vertical portion of the cavernous ICA) creates the lateral boundaries for this approach. It is therefore essential to stay medial to the pterygoid base, beneath which runs the paraclival
Chapter 55 • Endoscopic Endonasal Approaches to the Skull Base and Paranasal Sinuses
675
Kassam line
A
B
Figure 55-11. A, Preoperative, sagittal CT reconstruction showing the ‘Kassam line’ (dotted white line) which is used to determine inferior extent of endonasal resection. The line extends from the tip of the nasal bones to the posterior hard palate and into the depth as needed. B, Postoperative, sagittal CT reconstruction following endoscopic endonasal resection of the odontoid process and pannus. The patient also underwent posterior occipitocervical fusion.
ICA. At times, it is necessary to work behind the paraclival ICA to completely resect clival tumors. In addition, the abducens nerve is always a concern in this area and its course must be understood. The abducens nerve emerges at the level of the vertebrobasilar junction (VBJ), making preferable to open dura or tumor as caudal as possible. Abducens nerve monitoring plus slow, careful dissection toward the superior end of the clivus is critical. In addition, tumors such as petroclival meningiomas, which often have their origin posterior to the VIth nerve can displace the nerve anteriorly, even pressing it against the dura. Careful opening of only the dura when approaching these tumors can prevent inadvertent injury. Perhaps the most challenging aspect of these approaches is the clival venous plexus. Sometimes, the clivus and plexus are full of tumor, thereby thrombosing the plexus. However, it is sometimes the case that the tumor has merely caused venous congestion, engorging the already significant plexus. As a result, the bleeding encountered during this approach can be copious. There are times that the blood loss can be prohibitive and staging of the case may be required after the sinus has been packed, hopefully resulting in thrombosis before the second stage.
Foramen Magnum The foramen magnum is an uncommon location for lesions which require an anterior approach. Foramen magnum meningiomas are the most common tumor we have addressed via this approach. It is often used in conjunction with other approaches for chordomas. We have also resected a recurrent posterior fossa
emangioblastoma and performed a vertebral artery h aneurysmorrhaphy via this approach. The medial condyles and hypoglossal canals create the lateral borders for this approach. Once again, routine monitoring of the XII nerve is valuable. In our experience, resection of the medial half of the condyle (as is the case with the lateral half) does not result in occipitocervical instability. Nevertheless, these patients should be followed long term to ensure that they do not develop delayed instability.
Transodontoid Approach The most common pathology treated in this location is basilar invagination/pannus associated with longstanding degenerative arthritis. Additional pathology includes craniocervical meningiomas and chordomas. The anatomic landmarks for this module are the floor of the sphenoid above and the soft/hard palate below. Laterally, the region is bounded by the Eustachian tubes (ETs) that guard the parapharyngeal ICAs (internal carotid arteries). Inferior extent of endonasal resection can be determined preoperatively by using the ‘Kassam line’, which is drawn from the tip of the bony nasal bridge to the posterior hard palate and extended to the pathology in the depth as needed (Fig. 55-11). The nasopharyngeal mucosa and the longus capitus muscles are resected to expose the basion, arch of C1 and base of the odontoid process. The arch of C1 is removed and the dens resected using an 18 cm highspeed drill. If the lesion extends intradurally, the circular sinus is packed and the dura opened. Intradural resection requires respecting the lateral plane of the vertebral arteries as all of the lower cranial nerves are located laterally at this level.
676
OTOLOGIC SURGERY
Coronal Plane Anterior Fossa Supraorbital Approach An endonasal supraorbital approach (like all of the coronal plane approaches) is a lateral extension of another approach, in this case, a transfrontal or transcribriform approach. Lesions such as fibro-osseous tumors and fibrous dysplasia are often approached with a limited supraorbital approach. As an addition to a midline transcribriform approach, supraorbital approaches provide the lateral EEA exposure for olfactory groove meningiomas, esthesioneuroblastomas and other nasal carcinomas. Many orbital approaches can potentially be performed via only one nostril, but the exposure should not be compromised in order to avoid a binarial approach. The limit for lateral extent is the midorbital line. Reaching this limit requires removal of the lamina papyracea to allow for gentle lateral retraction of the periorbita and orbital contents. Transorbital Approach The orbit can be accessed endonasally via a maxillectomy and ethmoidectomy. ��������������������������������������� Applications include orbital and optic nerve decompressions for Graves’ disease, fibrous dysplasia and optic gliomas as well as resection of intraorbital tumors such as meningiomas and hemangiomas. Imageguidance can be critical for some complex decompressions. In general, for very complex cases such as circumferential fibrous dysplasia, only one nerve is decompressed in each operation. Orbital tumors which are medial to the optic nerve can be addressed with the addition of an ophthalmologist to the team via a corridor between the medial and inferior rectus muscles which are suspended and retracted via an external canthal approach. Transpterygoid Approach These approaches are lateral extensions from the sella and sphenoid. They provide access to five anatomical zones: I) petrous apex, II) petroclival junction, III) quadrangular space (Meckel’s cave), IV) superior lateral cavernous sinus, and V) infratemporal fossa. In general, adequate access to any of these zones requires at least a maxillary antrostomy and may require an extended endoscopic medial maxillectomy (endoscopic Denkers). I. Petrous Apex
Lesions of the petrous apex may be approached medially through the sphenoid sinus. Cholesterol granuloma of the petrous apex with medial expansion into the sphenoid sinus is the most common pathology. Other indications include selected benign and malignant tumors, such as juvenile nasal angiofibromas (JNAs) and chondrosarcomas (which usually require other modules for complete resection). A wide sphenoidotomy is performed and the usual anatomic landmarks identified. Exposure of the sphenoid sinus must extend laterally to include the lateral
recess. Often, this lateral recess is not pneumatized but is nonetheless key to this approach. If the lesion is expansile it may have remodeled the petrous apex thereby protruding into this lateral recess medial to the paraclival ICA. In these cases, the thin cortical bone overlying the lesion can be drilled providing direct access to the target. However, if remodeling has not occurred, the lesion will be located deep and lateral to the paraclival ICA. In these circumstances, the ICA may need to be mobilized laterally to give adequate access. After removal of the sphenoid floor (to the level of the clival recess) and intrasinus septations, the bone overlying the inferior sella and petrous apex is thinned with the drill using vertical strokes, parallel to the paraclival ICA. The ICA can then be skeletonized and mobilized. It may prove useful to thin or remove the clivus medially in order to enlarge the opening into the petrous apex. Tumor or granuloma contents can be removed carefully with two suctions. Angled endoscopes, in addition to introducing instrumentation from the contralateral naris utilizing a binarial approach, help access all of the lesion. In the case of a granuloma, a Silastic stent can be placed into the cavity for to help maintain drainage. II. Petroclival Junction
1. An extension of the petrous apex approach, this zone is usually involved with similar tumors. In addition, debulking of petroclival meningiomas can be achieved via this approach, often in combination with a clival approach. Occasionally, lateralization of the ICA is necessary to increase exposure when there is minimal medial expansion of the lesion. In such cases, the vidian canal is carefully drilled and followed to the 2nd (anterior) genu of the ICA, starting inferomedially to prevent carotid injury. After the bone overlying the paraclival (vertical cavernous) carotid artery is thinned, it can be carefully removed, allowing the artery to be displaced laterally without compressing it on an edge of bone. The relationship between this portion of the carotid artery and the vidian canal16,17 is critical to understand even if a transpterygoid approach is not needed (Fig. 55-12). In addition, the oblique course of the abducens nerve, running from the vertebrobasilar junction (VBJ) to Dorello’s canal, just lateral to the superior-most aspect of the paraclival ICA, to the cavernous sinus, is critical to fully understand in order to prevent injury during these transpterygoid approaches. III. Quadrangular Space (Meckel’s Cave)
The cranial nerves which are contained in the cavernous sinus are crowded into the superolateral cavernous sinus. This allows relatively safe access to the medial cavernous sinus via the previously described transsellar route. Tumor in Meckel’s cave can also be relatively safely accessed, but much greater care must be taken to avoid ophthalmoplegia. Tumors such as pituitary adenomas rarely are found in this location. However, tumors such
Chapter 55 • Endoscopic Endonasal Approaches to the Skull Base and Paranasal Sinuses
677
B Plane of x-section (sagittal) A.C.P.
V2 1.
9.
2.
8. 3.
B Sagittal section
V3 P.C.P.
4A 4B
5.
7.
CN3 CN4 CN6 CN5
9.
CN 3, 4
6.
A 1. 2. 3. 4A 4B 5. 6. 7. 8. 9. 10.
G.g.
5.
CN6 Cavernous sinus Pterygopalatine ganglion Vidian canal Vidian artery Vidian nerve Vertical paraclival I.C.A. Petrous I.C.A. G.P.N. to geniculate ganglion Ophthalmic segment I.C.A. Cavernous I.C.A. segment (parasellar) Maxillary sinus
V V3
1.
To geniculate ganglion
V2
10.
B 6.
4A
4B
3.
2.
4B
Figure 55-12. Illustration demonstrating the relationship of the vidian canal (with nerve and sometimes artery) with the anterior genu of the ICA (internal carotid artery) as it turns from the horizontal (petrous) portion to the paraclival (vertical cavernous) segment. This relationship is critical for transpterygoid approaches.
as adenoid cystic carcinoma and other nasopharyngeal carcinomas can extend via neural spread along trigeminal branches (usually the maxillary [V2] or mandibular [V3]) directly into this space. In addition, this space provides good endonasal access to schwannomas of the cranial nerves in the cavernous sinus (Fig. 55-13). The “quadrangular space” (Fig 55-2) is an area of the skull base which contains the Gasserian ganglion and is bounded inferiorly by the petrous ICA, medially by first the vertical segment of the cavernous ICA (also know as the paraclival ICA), laterally by the maxillary branch of the trigeminal nerve (V2) and superiorly by the abducens nerve (VI). The abducens nerve runs directly under the ophthalmic branch (V1) of the trigeminal nerve and can be easily damaged. Therefore, the ideal way to avoid this is to not cross the superior plane of V2. Again, the vidian canal is the key landmark for accessing this area as it leads to the anterior genu of the ICA which forms the inferomedial corner of the quadrangle. Unless there is wide pneumatization of the lateral sphenoid sinus, bone between the vidian canal and V2 must be removed carefully after the inferomedial aspect of the vidian canal is exposed as described above.
IV.Superior Lateral Cavernous Sinus
In our opinion, there are few indications for addressing tumor in the superior lateral cavernous sinus. With good control of small tumor residuals with other treatments such as radiosurgery, it is uncommon for the risk of ophthalmoplegia to be outweighed by the potential benefits of chasing tumor into this region. However, in the setting of pre-existing ophthalmoplegia, especially when addressing hormonally active pituitary adenomas, it is both reasonable and feasible to access the superior lateral cavernous sinus. Superior extension is usually bounded by the optic nerve and optic strut. Surgical approach is merely an extension of the transsellar and quadrangular approaches discussed above.
Infratemporal Fossa Medial temporal fossa lesions are usually an extension of tumors such as medial sphenoid wing, petroclival meningiomas, and schwannomas as well as nasopharyngeal carcinomas which are addressed as an extension of the approach chosen for these tumors. There are only rare tumors which occur in this region in isolation. However,
678
OTOLOGIC SURGERY
PG ICW schwannoma
PG
QS MCW ICA
A
B
Figure 55-13. A, Preoperative, post-contrast, coronal MRI image showing a schwannoma filling Meckel’s cave (AKA quadrangular space). (ICA = internal carotid artery (cavernous portion); PG = pituitary gland) B, Post-contrast, coronal MRI image following EEA with complete resection of the quadrangular space (QS) schwannoma. (ICW = intercavernous wall, between the medial and lateral cavernous sinuses; MCW = medial cavernous wall; PG = pituitary gland).
ICA ICA
ss
SS
sch
A
TC
B
Figure 55-14. A, Preoperative, post-contrast, coronal MRI showing a giant, recurrent infratemporal fossa trigeminal schwannoma (sch). ICA = internal carotid artery; SS = sphenoid sinus. B, Post-contrast coronal MRI of the same patient as (A) immediately following removal via EEA (endoscopic endonasal approach). ICA = internal carotid artery; SS = sphenoid sinus (filled with packing); TC = tumor cavity.
we have addressed neurenteric cysts and schwannomas in this area effectively via an EEA (Fig 55-14). Often, the inferior tail of the cavernous sinus dura as it ensheathes the trigeminal ganglion and its branches must be transgressed. This is approached initially via a quadrangular region approach. Care should be taken upon opening dura to determine if the ganglion is medial (and
therefore between the surgeon and the tumor) or lateral (and therefore displaced away from the operative trajectory) to the lesion. If the ganglion is medial, a lateral approach such as an anterior transpetrosal could be considered. However, the fibers of the trigeminal nerve may be thinned and splayed thus allowing one to work between them to at least biopsy the lesion. Tumors which
Chapter 55 • Endoscopic Endonasal Approaches to the Skull Base and Paranasal Sinuses
displace the nerves and/or ganglion laterally are ideally suited for an endonasal approach which will provide direct access. A complete understanding of the course of the parapharyngeal, petrous and cavernous ICA is a necessity both for injury avoidance as well as control.
Posterior Fossa The caudal lateral expanded endonasal approaches consist of the transcondylar and infratemporal fossa/ parapharyngeal space approaches. These essentially represent lateral extensions from the lower third of the clivus extending through the medial occipital condyle, the hypoglossal canal and into the jugular foramen.
Transcondylar Approach The medial transcondylar approach (the “far medial” approach) employs the limits of an endonasal approach. Chordomas, meningiomas, schwannomas and even a vertebral aneurysm have all been approached endonasally in part with this corridor. The hard palate becomes a limiting factor with all caudal approaches. An endoscopic medial maxillectomy is also needed to work unencumbered in this location. The complete exposure of the medial condyle is one of the most challenging exposures. In order to access the condyle, the torus of the Eustachian tube must be removed. Appropriate vascular imaging should be used pre- and intraoperatively (image-guidance) to verify the position of the parapharyngeal ICA to avoid injury during this dissection. The cartilage of the Eustachian tube blends with the foramen lacerum and the dissection of this tough material which is intimate with the petrous ICA can be very tedious and potentially dangerous. It may be helpful to identify the ICA in as many areas as possible prior to this dissection. Naturally, hypoglossal nerve monitoring should be employed, especially during drilling. It has been our experience that (as with a lateral approach) half of the condyle can be removed without harming stability (in this case, the medial half). The key to this is to avoid violating the atlanto-occipital joint, thereby preserving stability. Long term follow up is needed to ascertain the durability of this stability.
679
medial maxillectomy. The Eustachian tube must be transected and cartilage of the tarus which blends with foramen lacerum very carefully resected. The IMA branches are identified, coagulated or clip ligated. These can also be coiled using endovascular techniques preoperatively. The parapharyngeal ICA is identified following removal of the pterygoid plates. The lateral pterygoid plate points to the ICA in the depth. Careful, blunt dissection allows identification of the fat pad which protects the ICA. Once the parapharyngeal ICA and petrous bone are identified, resection can proceed. This exposure can be carried further laterally to the level of the posterior ICA canal, providing direct access to the jugular foramen.18
COMPLICATIONS AND LEARNING CURVE As is the case with any new approach, the endoscopic endonasal approaches have a learning curve. It is important to select cases appropriately based upon experience. The senior authors have performed over 1000 cases over the past 10 years. They were able to maintain a low complication rate by very gradually increasing the complexity of cases they attempted. As a result, a set of levels has been developed as a guide. Level I cases are basic sinonasal procedures, which represent an opportunity for the neurosurgeon to gain familiarity with the working corridors and endoscope. Level II cases include simple pituitary adenomas, confined to the sella, and CSF leaks. Neurosurgeons have traditionally done their pituitary tumors without an otolaryngologist and otolaryngologists have always done CSF leaks on their own. However, this is a unique opportunity to learn to work as a team and develop the skills needed to progress to higher level cases. Level III procedures are those which involve the extradural ventral skull base between the plane of the ICAs. Level IV procedures progress to intradural surgery. Level V procedures are those lateral to the plane of the ICA, requiring complete ICA control. CSF leak has been by far the most frequent complication of the expanded endonasal approach. Despite high rates initially, bacterial meningitis rates were low, largely due to rapid reoperation in those patients with identified CSF leaks. The incidence of meningitis in the first 700 patients was 1.2%
Lower Infratemporal Fossa/Parapharyngeal Space 2. Perhaps the most challenging endonasal approach is the most inferior and lateral, the approach to the infratemporal fossa. Filled with muscle and soft tissue interspersed with large vessels including the ICA and IMA (internal maxillary artery) and lacking in clearly defined anatomical landmarks, this region is challenging to navigate. As with the other EEAs, control of the ICA is critical. Tumors such as schwannomas, nasopharyngeal carcinomas, JPAs, paragangliomas meningiomas, and lesions such as meningoencephaloceles can all involve this area. Once again, the majority of the work is done through a
RECONSTRUCTION Initial rates using techniques developed for simple CSF leak repair were as high as 58% for certain tumor types (craniopharyngiomas).19 This prompted us to pursue the development of vascularized reconstructive techniques. As a result, the pedicled, vascularized nasal septal mucosal flap (NSF) was adopted for reconstruction.20 Rates since the adoption of this flap have been reduced to 5.4% (in press), well within the range of traditional approaches. This has been a critical component of the development
680
OTOLOGIC SURGERY
of EEA and has allowed it to become a feasible approach for many tumors of the skull base. This flap is durable, effective and has even proven to be reusable in reoperations, an extremely valuable characteristic. There are patients in whom nasal septal mucosa is not available, either due to prior surgery, radiation therapy, other septal pathology, or tumor involvement. There are also options for vascularized flaps other than the nasal septal flap in those patients who need it . We have used turbinate flaps for repair of small defects in close proximity to a turbinate.21 We have also used a vascularized temporoparietal fascial flap (TPFF) which can be tunneled through the pterygomaxillary fissure via a small lateral canthal incision using a percutaneous tracheostomy tube.22 The advent of these vascularized reconstruction techniques has been critical for endoscopic endonasal skull base surgery.
CONCLUSIONS Endoscopic endonasal surgery for lesions of the paranasal sinuses and skull base has developed over the past decade into a feasible approach. Almost the entire anterior skull base as well as some portions of the middle and posterior fossae are accessible endonasally. The advantage for many lesions is the relationship of critical neurovascular structures, with these approaches providing excellent direct medial corridors to lesions which have the critical neurovascular structures located along their perimeter. These techniques show promise in short term studies and may provide lower morbidity than standard approaches. We are optimistic that these short-term data will be translated into durable long-term results. We believe that the EEA corridors provide an important armamentarium to complement existing lateral external corridors, thereby providing the contemporary skull base surgeon with 360° access to this compact and complex region.
REFERENCES 1. Rosegay H : Cushing’s legacy to transsphenoidal surgery. J Neurosurg 54:448-454, 1981. 2. Carrau R L , Jho H D, Ko Y: Transnasal-transsphenoidal endoscopic surgery of the pituitary gland. Laryngoscope 106:914-918, 1996. 3. Jho H D, Carrau R L , Ko Y, et al: Endoscopic pituitary surgery: Early experience. J Neurosurg 84:744, 1996. 4. Friedrich J P, Terrier G: Chirurgie endoscopique de la sinusite par voie endonasale. Med Hyg 41:3722-3726, 1983. 5. Kennedy DW, Zinreich S J, Rosenbaum A E, et al: Functional endoscopic sinus surgery: theory and diagnostic evaluation. Arch Otolaryngol 111:576-582, 1985. 6. Jankowski R , Auque J, Simon C, et al: Endoscopic pituitary tumor surgery. Laryngoscope 102:198-202, 1992. 7. Messerklinger W: Endoscopy of the Nose. Urban & Schwarzenberg, Baltimore, 1978.
8. Wigand M E : Transnasale endoscopishe chirurgie der nasennebenhohlen bei chronischer sinusitis. HNO 29:215-221, 1981. 9. Stammberger H : Endoscopic endonasal surgery for mycotic and chronic recurring sinusitis. Ann Otol Rhinol Laryngol 119(Suppl):1-11, 1985. 10. Weiss M H : Transnasal transsphenoidal approach. In Apuzzo M L J (ed): Surgery of the Third Ventricle. Baltimore: Williams & Wilkins, 1987, pp 476-494. 11. Mason R B, Nieman L K, Doppman J L , et al: Selective excision of adenomas originating in or extending into the pituitary stalk with preservation of pituitary function. J Neurosurg 87:343-351, 1997. 12. Kouri JG, Chen MY, Watson JC, et al: Resection of suprasellar tumors by using a modified transsphenoidal approach. Report of four cases. J Neurosurg 92:1028-1035, 2000. 13. Kaptain GJ, Vincent D A, Sheehan J P, et al: Transsphenoidal approaches for the extracapsular resection of midline suprasellar and anterior cranial base lesions. Neurosurgery 49:94-100; discussion 100-101, 2001. 14. Rhoton A L Jr: The supratentorial cranial space: microsurgical anatomy and surgical approaches. Neurosurgery 51:S1-385, 2002. 15. Kassam A B, Prevedello D M, Thomas A, Gardner P, Mintz A, Snyderman C, Carrau R : Endoscopic endonasal pituitary transposition for transdorsum sellae approach to the interpeduncular cistern. Neurosurgery. 62(ONS Suppl 1):ONS57-74, 2008. 16. Kassam A B, Vescan A D, Carrau R L , Prevedello D M, Gardner P, Mintz A H, Snyderman C H, Rhoton A L Jr: Expanded endonasal approach: vidian canal as a landmark to the petrous internal carotid artery. J Neurosurg 108:177-183, 2008. 17. Vescan A D, Snyderman C H, Carrau R L , Mintz A, Gardner P, Branstetter B 4th, Kassam AB: Vidian canal: analysis and relationship to the internal carotid artery. Laryngoscope 117:1338-1342, 2007. 18. Kassam A B, Snyderman C, Carrau R , Gardner P, Hirsch B, Mintz A : Endoscopic, expanded endonasal approach to the jugular foramen. Operative Techniques in Neurosurgery 8(1):35-41, 2005. 19. Gardner P, Kassam A, Snyderman C, Carrau R , Mintz A, Grahovac S, Stefko ST: Outcomes following endoscopic, expanded endonasal resection of suprasellar craniopharyngiomas: a case series. J Neurosurg, in press, 2008. 20. Hadad G, Bassagasteguy L , Carrau R L , Mataza JC, K assam A, Snyderman C H, Mintz A : A novel reconstructive technique following endoscopic expanded endonasal approaches: Vascular pedicle nasoseptal flap. Laryngoscope 116(10):1881-1885, 2006. 21. Fortes FS, Carrau R L , Snyderman C H, Prevedello D, Vescan A, Mintz A, Gardner P, Kassam A B : The posterior pedicle inferior turbinate flap: a new vascularized flap for skull base reconstruction. Laryngoscope 117(8):13291332, 2007. 22. Fortes FS, Carrau R L , Snyderman C H, Kassam A, Prevedello D, Vescan A, Mintz A, Gardner P: Transpterygoid transposition of a temporoparietal fascia flap: a new method for skull base reconstruction after endoscopic expanded endonasal approaches. Laryngoscope 117:970976, 2007.
56
Petrosal Approach Todd A. Hillman
Petroclival tumors arise from or involve the petroclival junction cephalad from the jugular tubercle, medial to the trigeminal nerve, and anteromedial to the internal auditory canal (IAC).1,2 Tumors of the petroclival area are a particular challenge because these tumors often involve the middle and posterior cranial fossae, cause significant brainstem compression, invade the cavernous sinus, and abut or surround the upper cranial nerves and basilar artery. The complex anatomy requires an individualized surgical approach for each patient based on tumor origin, area of tumor extension, and preoperative neural function.3 The term combined petrosal approach actually describes numerous surgical approaches to the petroclival area well suited for these tumors. The term combined refers to the fact that this approach combines the middle fossa approach with a variation of a posterior fossa approach. The combination of these approaches allows for excellent exposure of the medial petrous bone and clivus from the cavernous sinus to the foramen magnum. First described by Decker and Malis,4 this combination allows the surgeon to take advantage of the strengths of each approach to minimize brain retraction. The middle fossa approach gives the surgeon good exposure of the petrous bone and clivus superior to the IAC, whereas the posterior approach provides surgical exposure inferior to the IAC. A tumor that is superior to the IAC can be accessed by a middle fossa approach alone. A tumor that is inferior to the IAC and tentorium cerebelli can be addressed with a posterior approach alone. It stands to reason that tumors involving both areas are best treated with a combination of these surgical procedures. The most common tumor of the petroclival area is the meningioma. Less common lesions include chordomas, chondrosarcomas, epidermoids, and vascular abnormalities such as aneurysms and arteriovenous malformations of the basilar system. Tumors that remain intracranial can be treated with a combined petrosal approach. Tumors that extend into the infratemporal fossa require a lateral approach.
COMBINED PETROSAL VARIATIONS All of the combined petrosal approaches include similar middle fossa exposure as a component of the procedure. The variations of the combined petrosal approach are classified by the type of posterior approach used. Traditionally, three variations of the combined petrosal approach were described: the retrolabyrinthine, translabyrinthine, and transcochlear approaches.5,6 The fourth variation described by Sekhar and colleagues,7 which provides exposure in between the retrolabyrinthine approach and the translabyrinthine approach, uses a partial labyrinthectomy for the posterior approach. This approach removes most of the labyrinth, while preserving functional hearing in 80% of patients. These four approaches give different degrees of anterior petrous bone exposure, which changes the level of brainstem and clivus exposure. The narrow angle between the anterior brainstem and clivus is the major factor influencing the level of visualization across both of these structures. Generally, further anterior removal of the otic capsule allows a more direct angle across the clivus, allowing improved medial exposure to the brainstem, basilar artery, and central clival depression. More aggressive anterior otic capsule removal secondarily provides improved access to the ipsilateral medial petrous bone as well (Fig. 56-1A). The choice of posterior approach depends on tumor location and size, preoperative hearing level, and extent of brainstem compression. The translabyrinthine and transcochlear approaches sacrifice residual hearing. The transcochlear approach also includes posterior facial nerve transposition, leading to a temporary facial paralysis and risking permanent injury. The translabyrinthine approach is used when preoperative hearing is poor, and the transcochlear approach is reserved for the largest of tumors that cross the midline of the clivus. Tumor effect on the brainstem is also an important consideration. Sometimes larger tumors need a less radical posterior approach because the posterior compression of the brainstem opens the angle between the clivus and brainstem. 681
682
OTOLOGIC SURGERY
CN 4 Basilar artery
V
CN 5 Tumor
Transcochlear
CN’s 7, 8
Translabyrinthine
CN’s 9, 10, 11
Sig
CN 12
mo
id s
inu
Partial Labyrinthectomy
s
Retrolabyrinthine
A
Cerebellum
FIGURE 56-1. A, Four variations of posterior approach provide unique exposure of anterior and medial petrous bone, and change angle of visualization across brainstem. Retrolabyrinthine variation allows access to petroclival junction, but limited medial access. Partial labyrinthectomy allows exposure up to lateral aspect of clival depression. Total labyrinthectomy allows exposure of anterior brainstem and central clival depression. Transcochlear approach allows access to contralateral clivus as well. B, Large petroclival meningioma. Posterior compression of brainstem keeps angle of approach flat and allows translabyrinthine approach despite contralateral clival involvement. (B Courtesy of Dr. Clough Shelton.)
The tumor in Figure 56-1B was addressed with a translabyrinthine variation of the combined petrosal approach despite involvement of the contralateral clivus. The posterior displacement and contralateral displacement of the brainstem kept the angle between the tumor-involved clivus and brainstem open, obviating the need for a transcochlear approach.
PREOPERATIVE EVALUATION Most patients who present to a neurotologist with a petroclival tumor have had either a computed tomography (CT) scan or magnetic resonance imaging (MRI) of the brain for evaluation of cranial nerve findings or nonspecific complaints. MRI and CT imaging modalities are complementary and obtained for every patient with these complex lesions. MRI needs to be a contrast-enhanced study, but the CT scan does not. MRI
B
is superior for determining the extent and character of the tumor, whereas CT scan gives the bony detail necessary for surgical planning. The surgeon determines from these studies whether the combined petrosal approach is appropriate, and which variation of the combined petrosal approach is needed. High-quality images are required with thin cuts (3 mm for MRI, 1 mm for CT) through the skull base. All images are reviewed with a neuroradiologist and the operating neurosurgeon before any planned intervention. Important temporal bone arterial and venous variations can often be detected from MRI; however, angiography with venous phase should be considered for each patient in whom a combined petrosal approach is being contemplated. Important venous anomalies exist that may result in catastrophe if not recognized before surgery.8 The main venous drainage of the temporal lobe is through the vein of Labbé, a single tributary that runs along the inferior surface of temporal lobe and typically
Chapter 56 • Petrosal Approach
anastomoses into the transverse sinus. The vein of Labbé may run through the tentorium, however, and insert into the superior petrosal sinus, rather than the transverse sinus. Because superior petrosal vein and tentorium transection is a key component of the combined petrosal approach, the vein of Labbé would be at risk if this anatomic variation is present. Accidental transection can lead to venous infarction of the temporal lobe with particularly grave consequences to the patient, especially if it occurs in the dominant hemisphere. A dominant sigmoid sinus should also be recognized before surgery. The surgeon must exercise extra caution not to injure a dominant sigmoid sinus because the contralateral venous drainage may be insufficient or absent, leading to a high risk of venous infarction. A final concerning anatomic variation is a transverse sinus that does not connect at the torcular Herophili. This situation leads to one side of the venous system draining the sagittal sinus system. The sigmoid on the side draining the sagittal system must be preserved to prevent stroke. Angiography easily detects these variations so that the surgeon can optimize the treatment plan. Angiography also gives the surgeon the option of preoperative embolization for particularly vascular tumors.
PREOPERATIVE PREPARATION A neurosurgeon is part of the surgical team in all cases. Patients are given preoperative antibiotics to help prevent postoperative wound infections. A first-generation cephalosporin such as cefazolin is used for non–penicillin-allergic patients, and clindamycin is for patients who have a known penicillin allergy. The patient is anesthetized with the head on the foot of the bed to allow adequate leg clearance for the surgeon. The anesthesiologist is instructed to use short-acting or no paralysis so that detection of facial nerve stimulation and somatosensory evoked potentials are not compromised during the case. Because these cases often take many hours, extra care is taken to pad the patient and bed appropriately to prevent pressure points that can lead to skin breakdown or peripheral neuropathy. The author uses a customized padding system developed by his circulating nurse to keep the legs in a slightly bent position and to pad the elbows generously. The arms are crossed over the patient’s chest, which is a more relaxed arm position than tucking the arms at the side and prevents pressure at the medial epicondyle of the humerus, a common point of ulnar nerve injury. An arterial line and double venous access is placed by the anesthesiologist. The blood pressure cuff is preferably placed on the upper extremity opposite the tumor side. The patient is double or triple strapped to the bed because significant bed tilting is necessary at times. The patient is kept in a supine position with the head turned so that the operative side is up and parallel to the floor. If there is limitation to neck rotation, a shoulder roll may
683
be placed under the shoulder to roll the torso away. The head is placed in a Mayfield head holder and secured to the bed. All retractors are secured to the head holder. The bed is turned 180 degrees away from the anesthesiologist to allow adequate room for the scrub nurse, surgeon, and microscope. A wide head shave is performed with an electric razor, and a small amount of alcohol is used to remove oil from the skin, which is allowed to dry. Benzoin is placed and clear adhesive drapes are placed around the periphery of the planned surgical site. The abdomen should be prepared so that a fat graft can be harvested. CN VII, IX, X, XI, and XII are monitored during the case. For large tumors with brainstem compression, somatosensory evoked potentials are also monitored. For cases in which hearing preservation is being attempted, auditory brainstem responses are monitored in both ears.
SURGICAL TECHNIQUES A large-diameter C-shaped incision is made starting just above the level of zygoma, 2 cm anterior to the zygomatic root. The incision is curved around, staying three fingerbreadths above the auricle, two fingerbreadths posterior to the sulcus, and ending posterior to the mastoid tip. The mastoid incision is kept posterior to allow enough posterior skin retraction for full sigmoid sinus exposure. The temporalis muscle is freed from its temporal bony attachment and reflected anterior and inferior over the zygoma. Skin hooks on rubber bands attached with clamps to the drapes are helpful to provide scalp retraction while maintaining a low profile. An alternative skin incision is a combination of the preauricular incision typically used for a straight middle fossa approach and the incision used for a translabyrinthine approach. Although this incision ends up with a 90 degree angle at the intersection, the author has not noted wound complications with this method, and the scar is less conspicuous. The entire mastoid and squamosal temporal bone must be exposed before proceeding with drilling.
Posterior Exposure The middle fossa craniotomy or the posterior temporal bone approach may be started first; however, the author finds that beginning with the posterior approach facilitates the middle fossa craniotomy by identifying the level of the middle fossa and limiting the number of burr holes required to turn the middle fossa flap (Fig. 56-2A). A complete mastoidectomy is performed with an electric high-speed drill, and all of the bone covering the middle and posterior fossa dura is removed. Suction irrigation is used, and the irrigant should contain bacitracin to prevent infection and the incidence of cerebrospinal fluid
684
OTOLOGIC SURGERY Temporalis m. Craniotomy
Retracted skin dissection over intact canal skin (EAC)
Mastoidectomy
P.C.W. CN VII in Fallopian canal Sigmoid sinus Retrosigmoid bone dissection
B
Superior petrosal sinus
Burr hole
A
Dural incision Partial Labyrinthectomy
Middle fossa dura incised
VII
S
s
inu
id s
o igm
C Posterior fossa dura incised Superior petrosal sinus
(CSF) leak.9 It is important to remove the bone over the sigmoid sinus and at least 5 mm of bone posterior to the sigmoid to aid with posterior retraction of the sinus. The mastoid emissary vein is divided to facilitate the retrosigmoid bone removal. The bone from the distal transverse sinus should be removed for identification, but the surgeon should use caution to preserve the insertion of vein of Labbé. Wide exposure laterally at the sigmoid greatly facilitates instrumentation medially when the exposure is complete. If a retrolabyrinthine approach is chosen, the labyrinth should be skeletonized (Fig. 56-2B). The horizontal canal is identified and followed to the posterior canal.
FIGURE 56-2. Combined retrolabyrinthine petrosal approach. A, Middle fossa and posterior approach outlined. Either approach can be performed first. Burr holes made for middle fossa flap need to be large enough to fit a side-biting drill with footplate attachment. B, Combined retrolabyrinthine approach shown with completed posterior exposure and craniotomy flap turned. Labyrinth is skeletonized to maximize exposure. C, Dural incisions shown with superior petrosal sinus divided. Tentorium is identified and divided. EAC, external auditory canal; PCW, posterior canal wall.
The bone over the superior canal is also removed. The bone covering the posterior fossa dura is removed from the petrosal vein to the jugular bulb. The bone over the middle fossa dura is also removed. The neurosurgeon can perform the middle fossa craniotomy, make the dural incisions, divide the greater petrosal vein, and begin to divide the tentorium to connect the middle and posterior fossae (Fig. 56-2C). If more exposure is needed, a partial labyrinthectomy can be performed by carefully plugging the semicircular canals as they are drilled away (Fig. 56-3). The key is to avoid exposure and traumatic suctioning of the labyrinthine fluids by keeping the labyrinth sealed while removing
Chapter 56 • Petrosal Approach Partial Labyrinthectomy
P.C.W.
M.F.D.
VII
P.F.D.
. S.S
Superior petrosal sinus
FIGURE 56-3. Combined partial labyrinthectomy approach. Semicircular canals are drilled away slowly while advancing bone wax into labyrinth to prevent communication with fluid spaces of membranous labyrinth. MFD, middle fossa dura; PCW, posterior canal wall; PFD, posterior fossa dura; SS, sigmoid sinus.
bone. The superior, horizontal, and posterior canals are skeletonized and blue-lined, but not entered. Bone wax is placed on the end of a 3 mm smooth diamond bit and advanced through the thin bone of the posterior canal to seal it while drilling at slow speed. More wax is placed on the drill bit and advanced inferiorly toward the vestibule, but halted posterior to the facial nerve. The drilling continues superiorly to the common crus. The wax is advanced while drilling at slow speed to the superior semicircular canal. The horizontal canal is also sealed in this fashion. The wax that is placed advances through the membranous labyrinth keeping it sealed as bone is removed. Drilling is stopped short of entering the vestibule, or hearing would be compromised. This technique can give almost the same exposure to the brainstem and clivus as a translabyrinthine approach, although it does not provide access to the IAC. If hearing is poor, or the IAC is significantly involved with tumor, the translabyrinthine approach is the preferable posterior exposure (Fig. 56-4A). Hearing is sacrificed as a result of this procedure. The semicircular canals are removed starting with the horizontal canal, which is followed posteriorly to identify the posterior canal. The anterior half of the horizontal canal is left to protect the second genu of the facial nerve. The common crus is followed to identify the superior semicircular canal. The semicircular canals are followed into the
685
vestibule, which forms the lateral extent of the IAC. The posterior fossa bone is removed off the dura until the IAC is identified. The bone surrounding the IAC is removed with a diamond drill. The IAC should be exposed about 270 degrees, just as for a translabyrinthine approach for an acoustic tumor. The middle fossa craniotomy is performed, and the tentorium divided as shown in Figure 56-4B. Exposure across the brainstem to the contralateral clivus can be maximized by extending the petrosal bone dissection anteriorly with a transcochlear approach. The external auditory canal is transected at the level of the bony-cartilaginous junction, and the canal skin is everted, and then oversewn in two watertight layers. The posterior external auditory canal wall is removed, as are the remaining canal skin, tympanic membrane, and ossicles. The inferior tympanic ring is also removed with the drill. The facial nerve must be mobilized by removing the bone of the IAC, tympanic fallopian canal, and mastoid segment of the fallopian canal. Bone removal should be greater than 180 degrees at all parts of the fallopian canal. The greater superficial petrosal nerve is sectioned anterior to the geniculate ganglion, and the facial nerve is mobilized posteriorly. There is usually bleeding at the canal of the greater superficial petrosal nerve, which must be packed with bone wax. The dura of the IAC is incised whether or not tumor involves the IAC to allow full posterior facial nerve mobilization. The cochlea is drilled away, and the carotid artery is identified (Fig. 56-5). When the carotid canal is exposed, the anterior petrous bone can be drilled away medially. One advantage of the combined petrosal approach is that transcochlear exposure is rarely needed except for the largest tumors. Transposition of the facial nerve has an inherent risk of facial nerve injury.10 This variation is carefully considered only when the translabyrinthine exposure is inadequate.
Middle Fossa Exposure The neurosurgeon usually performs the middle fossa exposure. To facilitate temporal lobe retraction, furosemide, 10 mg, followed by mannitol, 0.5 g/kg, is given intravenously 20 minutes before the middle fossa craniotomy. Burr holes are made as shown in Figure 56-2A, and a side cutting burr with a footplate is introduced into the defect. The craniotomy should be large, following the squamosal suture line. A large craniotomy helps to minimize the temporal lobe retraction required. The craniotomy is centered over the zygomatic arch to ensure adequate anterior exposure. The dura is elevated from posterior to anterior and medially over the remaining bone of the petrous ridge. The middle meningeal artery is coagulated to allow anterior dural elevation. The bone between the trigeminal nerve and IAC is removed (Kawase’s triangle). The dura is divided in the posterior fossa from the jugular bulb and in the middle fossa dura
686
OTOLOGIC SURGERY
M.F.D. Bone over IAC removed and posterior fossa dura exposed
CN. VII in fallopian canal
P.F.D.
ce Displa id o m sig sinus
A Translabyrinthine approach
Superior petrosal sinus
S.P.S.
Infundibulum Tumor CN 5
Tentorium (cut) CN’s 9, 10, 11
CN 4 Brainstem
Cerebellum Sigmoid sinus
CN’s 7, 8
B
FIGURE 56-4. Combined translabyrinthine approach. A, Labyrinth has been removed, and internal auditory canal (IAC) has been exposed. Facial nerve distal to geniculate remains under bone. Dural incisions are made as shown. B, Tentorium is divided parallel to superior petrosal sinus (SPS). Caution needs to be taken medially to recognize and preserve CN IV. Tumor is exposed on clivus and can be removed with careful dissection. MFD, middle fossa dura; PFD, posterior fossa dura.
Chapter 56 • Petrosal Approach
687
Transcochlear approach
MFD Co
VII P.F.D.
S.P.S. . S.S
B.a.
FIGURE 56-5. Combined transcochlear approach. Exter V
VIII
9 10 11 VII Ce
S.S.
to the superior petrosal vein. The superior petrosal vein is divided connecting the middle and posterior compartments, and the tentorium is sectioned from posterior to anterior. CN IV runs with the tentorium medially, and caution is needed to preserve it. When the tumor is exposed, excision proceeds with microdissection technique, debulking the tumor until the capsule can be visualized and safely dissected from cranial nerves, the brainstem, and arteries. The most challenging areas of dissection are the posterior cavernous sinus, tumor-involved upper cranial nerves, and tumor-involved basilar artery.
CLOSURE The middle fossa and presigmoid dura are reapproximated as well as possible. Typically, the middle fossa dura is able to be closed adequately enough to prevent CSF leak, but suturing the presigmoid dura is difficult, and the closure needs to be augmented with fat. Abdominal fat is used for this purpose and is placed though the dural defects in a dumbbell fashion. Prevention of CSF
nal canal is transected and oversewn. Posterior external auditory canal wall is removed. Facial nerve is removed from fallopian canal, and greater superficial petrosal nerve is sectioned to allow nerve to be transposed posteriorly. Removal of anterior and medial petrosal bone facilitates exposure of central clival depression and contralateral clivus. Ba, basilar artery; Ce, cerebellum; Co, cochlea; MFD, middle fossa dura; PFD, posterior fossa dura; SPS, superior petrosal sinus; SS, sigmoid sinus.
leak through the middle ear and eustachian tube varies by what posterior approach was used for the case. Hydroxyapatite cement has been helpful in preventing CSF leak in acoustic tumor surgery, and is used when appropriate for the combined approaches as well.11 If a retrosigmoid posterior approach has been used, temporalis fascia is placed over the antrum, and 5 to 10 mL of hydroxyapatite cement is placed in the posterior defect lateral to the fat graft. The cement should be placed over the antrum and against remaining bone of the posterior external auditory canal and jugular bulb. If a translabyrinthine approach has been used, the incus is removed, the eustachian tube is plugged with muscle, and hydroxyapatite cement is used to fill the antrum and posterior defect. If the transcochlear approach has been used, the use of cement is not advised because there is little bone to hold it in place, the eustachian tube can be aggressively sealed off with soft tissue as is easily seen with this exposure, and the facial nerve is uncovered in the dissection field. When cement is used, a JacksonPratt drain should be placed superficial to the cement and removed overnight because of the higher tendency to
688
OTOLOGIC SURGERY
form seromas. This drain is removed in 24 hours, and the drain hole is sutured closed.
RESULTS Results vary depending on the pathology of the tumor. Meningiomas constitute most of these tumors, and outcomes are well described in the literature. Gross total excision can be accomplished in most cases; however, there are a few reports of good long-term results with subtotal excision to preserve cranial nerves followed by stereotactic irradiation if necessary.12 Gross total excision is preferable if possible because of the difficulty in managing recurrent disease in this area. If tumor invades the cavernous sinus, a small amount can be left because of the morbidity of surgery in this area. If tumor is firmly attached to a cranial nerve, but separated from the brainstem, clivus, and petrous bone, a small piece of the capsule may be left with little concern about growth. Stereotactic irradiation can be used for small residual disease or recurrences that show growth during follow-up.
The most common complication of the combined petrosal approach is cranial nerve injury. The most commonly injured cranial nerves are CN V and VII. CN IV and VI can also be injured resulting in diplopia. Lower cranial nerve injury can result in dysphagia postoperatively. Rarely, preoperative deficits may improve after surgery, typically in nerves with partial dysfunction caused by nerve traction that is relieved by tumor excision. Generally, preoperative deficits do not improve, however. In the University of Utah series, 5 of 19 patients (26%) had at least one new, long-standing postoperative cranial nerve deficit.13 Most patients had preoperative cranial nerve function preserved, however. Gross total excision was accomplished in 86% of all patients. Functional hearing was preserved in 85% of patients. CSF leak is also a common complication of this approach. The rate of CSF leak varies by report, but is approximately 15% to 19%.5,6,13 Most of these cases resolve after placement of a lumbar drain for 3 days. There are not enough data to determine if the success of hydroxyapatite closure in preventing CSF leak after acoustic tumor surgery will translate to the combined petrosal approach.
Venous Drainage 1.
2. 3.
4.
12. 9. 5. 3.
6.
8.
2.
9. 1.
10. 7.
5. 11.
4.
8. 1. 2. 3. 4. 5. 6.
Cavernous sinus 7. Transverse sinus Basilar plexus 8. Confluence of sinuses Superior petrosal sinus 9. Vein of Labbé Inferior petrosal sinus 10. Jugular bulb 11. Mastoid emissary v. Sigmoid sinus 12. Superior sagittal sinus Tentorium (cut) FIGURE 56-6. Typical venous drainage is shown. Variations exist and should be identified preoperatively to prevent accidental trauma and venous infarction.
Chapter 56 • Petrosal Approach
Cerebrovascular accident is a rare, but serious complication of the combined petrosal approach. Venous infarction is a specific complication that can occur if the sigmoid is injured on a dominant or noncommunicating side, or if the vein of Labbé is injured. Avoidance of these complications is maximized by knowledge of the cerebral venous drainage system (Fig. 56-6) and with preoperative imaging to assess the patient’s specific vascular anatomy.
SUMMARY The combined petrosal approach is a safe and effective method to access and remove petroclival tumors and vascular lesions that extend above and below the tentorium cerebelli. Variations of the combined petrosal approach allow the surgeon to tailor the surgical approach for each patient to minimize morbidity, while maximizing postoperative neural function. The exposure obtained from these approaches allows access to an area of the skull base that is otherwise difficult to treat. Selection of the surgical approach is based on the extent of the tumor, predicted morbidity, and status of the patient’s preoperative hearing. The combined petrosal approaches provide a favorable tumor cure or control rate, while maintaining acceptable levels of postoperative morbidity.
REFERENCES 1. Al-Mefty O: Operative Atlas of Meningiomas. Philadelphia, Lippincott-Raven, 1998. 2. Abdel Aziz K M, Sanan A, van Loveren H R , et al: Petroclival meningiomas: Predictive parameters for transpetrosal approaches. Neurosurgery 47:139-152, 2000.
689
3. Arriaga M A, Shelton C, Nassif P, Brackmann D E : Selection of surgical approaches for meningiomas affec ting the temporal bone. Otolaryngol Head Neck Surg 107:738-744, 1992. 4. Decker R E, Malis L I : Surgical approach to midline lesions at base of skull. J Mt Sinai Hosp 37:84-102, 1970. 5. Spetzler R F, Daspit C P, Pappas CT: The combined supra- and infratentorial approach for lesions of the petrous and clival regions: Experience with 46 cases. J Neurosurg 76:588-599, 1992. 6. Daspit C P, Spetzler R F, Pappas CT: Combined approach for lesions involving the cerebellopontine angle and skull base: Experience with 20 cases—preliminary report. Otol Head Neck Surg 105:788-796, 1991. 7. Sekhar L N, Schessel D A, Bucar S D, et al: Partial labyrinthectomy petrous apicectomy approach to neoplastic and vascular lesions of the petroclival area. Neurosurgery 44:537-552, 1999. 8. Erkmen K, Pradvdenkova S, Al-Mefty O: Surgical management of petroclival meningiomas: Factors determining the choice of approach. Neurosurg Focus 19:1-12, 2005. 9. Kartush J M, Cannon SC, Bojrab D I, et al: Use of bacitracin for neurotologic surgery. Laryngoscope 98:10501054, 1988. 10. Selesnik S H, Abraham MT, Carew J F: Rerouting of the intratemporal facial nerve: An analysis of the literature. Am J Otol 17:793-809, 1996. 11. Arriaga M A, Chen D A, Burke E L : Hydroxyapatite cement cranioplasty in translabyrinthine acoustic neuroma surgery-update. Otol Neurotol 28:538-540, 2007. 12. Natarajan S K, Sekhar L N, Schessel D, Morita A : Neuro surgery 60:965-979, 2007. 13. Baugh A, Hillman TA, Shelton C : Combined petrosal approaches in the management of temporal bone meningiomas. Otol Neurotol 28:236-239, 2007.
57
Neurofibromatosis 2 William H. Slattery III
Neurofibromatosis 2 (NF2) is a rare syndrome characterized by bilateral vestibular schwannomas, multiple meningiomas, cranial nerve tumors, spinal tumors, and eye abnormalities. NF2 presents unique challenges to the otologist because hearing loss may be the presenting complaint leading to the diagnosis of the disorder. NF2 is quite invasive, requiring a multispecialist team approach for the evaluation and treatment of the disorder. The primary impairment is hearing loss resulting from bilateral vestibular schwannomas. NF2 must be characterized from neurofibromatosis 1 (NF1); although the names are linked, the disease entities are distinctly different. This chapter reviews the clinical characteristics of NF2, and current recommendations for evaluation and treatment.
NEUROFIBROMATOSIS 2 DIFFERENTIATED FROM NEUROFIBROMATOSIS 1 NF1 has distinctly different clinical characteristics from NF2. NF1 and NF2 have been differentiated as completely different genetic diseases based on the chromosome responsible for the disease. NF1 has been localized to chromosome 17, and NF2 has been localized to chromosome 22. NF1 is a multisystem disorder in which some features may be present at birth and others are age-related manifestations. A National Institutes of Health (NIH) Consensus Development Conference identified the following seven features of the disease, of which two or more are required to establish the diagnosis of NF1: 1. Six café au lait spots equal to or greater than 5 mm in longest diameter in prepubertal patients and 15 mm in longest diameter in postpubertal patients 2. Two or more neurofibromas of any type or one plexiform neurofibroma 3. Freckling in the axilla or inguinal regions 4. Optic glioma (optic pathway glioma) 5. Two or more Lisch nodules (iris, hamartomas)
6. Distinct osseous lesion, such as sphenoid wing dysplasia or cortical thinning of the cortex of long bones with or without pseudarthrosis 7. First-degree relative (parent, sibling, or child) with NF1 according to the above-listed criteria Some patients also manifest learning disabilities or language disorders. A careful examination and a detailed history of the patient’s symptoms help distinguish NF1 and NF2.
CLINICAL CHARACTERISTICS OF NEUROFIBROMATOSIS 2 Definition The NIH Consensus Development Conference also developed guidelines for the diagnosis of NF2. NF2 is distinguished by bilateral vestibular schwannomas with multiple meningiomas, cranial tumors, optic gliomas, and spinal tumors. A definite diagnosis is made on the basis of the presence of bilateral vestibular schwannomas or developing a unilateral vestibular schwannoma by age 30 and a first-degree blood relative with NF2, or developing at least two of the following conditions known to be associated with NF2: meningioma, glioma, schwannoma, or juvenile posterior subcapsular lenticular opacity/juvenile cortical cataract (Table 57-1).1 There may be significant heterogeneity in the presentation of the disease from one individual to the next. Some individuals may have a very mild form of the disease with small vestibular schwannomas manifesting in an older individual. Meanwhile, some children present with multiple intracranial tumors at a very young age. Despite the heterogeneity of the disease within a family, the expression of NF2 tends to be very similar.2 There is a significant genetic component to the disease with much variability within the parameters of the observed phenotype. Studies have shown that a truncating mutation (nonsense and frame shift) may be linked with a more severe form of NF2.3-5 The more severe form of 691
OTOLOGIC SURGERY 80
TABLE 57-1 Neurofibromatosis 2 (NF2) Diagnostic
70
Individuals with the Following Clinical Features Have Confirmed (Definite) NF2 Bilateral VS or family history of NF2 (first-degree family relative) plus Unilateral VS 50% had PPV 93% for HB grade II or better immediately postoperative Stimulation threshold of 0.01-0.04 mA had PPV 94% for HB grade II or better 180 days postoperative using four-channel EMG For 0.240 V, authors could predict 85% of patients with HB grade II or better 1 yr postoperative Using proximal-distal amplitude and stimulation threshold, authors developed regression function with sensitivity 89%, specificity 83%, PPV 94% in predicting HB grade III or worse in immediate postoperative period Using stimulation current and tumor size, authors developed regression function correctly describing 93% of patients at 2 yr follow-up With proximal-distal amplitude >80%, 98.4% had HB grade I at 6 mo Median threshold for HB grade II or better group was 0.100 V versus 0.725 V for HB grade III or worse group HB grade III or worse long-term was related to higher minimum stimulation current At 1 yr after surgery, 90% of patients stimulating at 0.1 mA had HB grade II or better 100% HB grade III or better initial and grade I long-term (≤28 mo) function occurred if proximal-distal amplitude >67% Of patients with stimulus of 0.2 V When threshold after tumor removal was ≤0.2 V, 93% patients had early, and 85% had late postoperative HB grade I or II 83% HB grade I 1 wk postoperative if proximal-distal amplitude = 1 with 88% of these patients HB grade I 1 yr after surgery
HB, House-Brackmann; PPV, positive predictive value.
A similar study by Isaacson and colleagues18 looked at two independent intraoperative monitoring parameters in predicting long-term facial nerve function in 60 patients undergoing resection of vestibular schwannomas. They found that 5 of 60 (8.3%) patients showed significant long-term weakness (i.e., House-Brackmann grade III or worse). Intraoperative monitoring parameters (proximal stimulation threshold, proximal-to-distal response amplitude ratio) were accurate in predicting increased risk of long-term facial nerve dysfunction when used in a logistic regression model. Table 64-1 summarizes a selection of studies using facial nerve monitoring to evaluate postoperative prognosis in acoustic neuroma surgery38–47.
Artifactual Responses Artifacts (i.e., monitor activity that is not due to facial nerve stimulation) are common and may cause confusion. Electrocautery obliterates facial nerve responses by saturating the monitor with electric noise. Consequently, monitors have a muting function while electrocautery is in use. During muting, monitoring is disengaged. Ultrasonic aspirators also cause a large electric artifact, and may trigger a muting circuit.14 When accurate monitoring cannot be performed, many surgeons engage a member of the team to observe visually or palpate the patient’s face.
Troubleshooting the Facial Nerve Monitor Although the facial nerve monitor can give the surgeon confidence to work safely and expeditiously by verifying anatomy, when the monitor does not respond as expected, the surgeon should proceed cautiously and initiate a sequence of steps systematically to ensure that the equipment is functioning and providing adequate monitoring sensitivity. Manufacturer manuals serve as a resource for troubleshooting specific to the equipment. Direct communication with the anesthesiologist to confirm complete neuromuscular reversal is warranted. Checking along the electric circuit should be done, including electrode placement in the skin, electrode connections to the monitor, and probe connections. The volume of the monitor might have been turned down. A typical event threshold setting is 100 μV. Event threshold settings set too high may lead to a lack of a response. Conversely, false-positive results can be reduced by increasing the event threshold. The stimulus intensity should be confirmed and can be incrementally increased by 0.05 to 0.1 mA. In the middle ear, high sensitivity can be achieved with stimulus up to 0.7 mA.9 If a continuous current is used, stimulus should be changed to pulsed mode. Selesnick19 calculated the optimal stimulus duration of 50 μs, although many commercial monitors use a default pulse of 100 μs.
778
OTOLOGIC SURGERY
Fixed voltage stimulation is an option when the surgical field is not dry to compensate for current leak from the stimulator. The stimulator probe can be replaced with a stand-alone nerve stimulator. When the face is not covered, visual verification of stimulation is possible and can help in troubleshooting. Alternatively, the face may be manually felt to move under surgical drapes during stimulation. Final options include rebooting the monitor or changing the monitor counsel. Ultimately, surgical judgment should prevail; it behooves the surgeon not to perform irreversible steps, unless these potential sources of error and corrective measures are considered.
Antidromic Facial Nerve Monitoring When a nerve is stimulated, in addition to conduction distally toward the neuromuscular junction (orthodromic), an action potential is conducted proximally (antidromically). The orthodromic wave produces the M wave on muscle contraction. The antidromic electric activity can be measured and used to assess neural function as the signal is reflected back toward the muscle resulting in a more subtle F wave. Wedekind and Klug20 prospectively evaluated F wave monitoring comparing it with intraoperative EMG, and found it useful monitoring facial nerve injury. They found that transient loss of F waves portends imminent severe facial dysfunction, and reported a 100% positive predictive value for unfavorable facial outcome with a permanent loss of F waves. A major advantage of antidromic monitoring is that it may be performed under NMB. Arriaga and associates21 described another use for antidromic potentials to locate the geniculate ganglion during middle fossa craniotomy.
MONITORING HEARING Hearing depends on the integrity of the peripheral and central auditory structures and their vascular supplies. The goal of intraoperative monitoring of auditory function is to preserve hearing, not just anatomy.
Pathophysiology Operating in the internal auditory canal and CPA poses two potential mechanisms for hearing loss: interruption of blood flow to the cochlea and the cochlear nerve, and injury to neural structures of the auditory pathway.22 In some cases, preoperative audiometric tests suggest a site of the lesion for the hearing loss. An inappropriately large reduction in speech discrimination score with good pure tone average suggests tumor effect on CN VIII or brainstem, rather than on the cochlea. Intact otoacoustic emissions (OAE) with poor auditory brainstem response (ABR) suggest the same. Retraction of the cerebellum during the early stages of the operation may stretch and damage CN VIII, which
is particularly vulnerable in the CPA. Surgeons need an objective measure of hearing preservation because visual confirmation of intact anatomic structures alone is inadequate to ensure postoperative auditory function.
Preoperative Testing A standard audiogram displays auditory thresholds at frequencies important to language (500 to 8000 Hz). Various word list tests may be administered to corroborate pure tone threshold averages and to quantify speech discrimination ability. Acoustic reflex tests examine the integrity of the CN VII and VIII reflex arc. Cochlear potentials, ABR, and OAE are useful to evaluate the integrity of the auditory pathways further and to establish a baseline for intraoperative monitoring. It is important to know preoperatively whether the physiologic measures to be monitored are abnormal.
Indications Selecting appropriate cases for monitoring involves analysis of preoperative hearing, disease prognosis, and surgical approach. The role of monitoring hearing is to decrease the risk to the auditory nerve in CPA operations. Hearing can be monitored directly from the auditory nerve, and auditory evoked potentials (electrocochleography [ECoG] potentials) can be used to check the endolymphatic system in endolymphatic sac decompression operations. Operations in which monitoring of auditory function has been reported include microvascular decompression of cranial nerves, vestibular nerve section,21 and removal of vestibular schwannomas and other CPA masses. The most common application for monitoring of auditory function is the removal of vestibular schwannomas. Hearing preservation rates for surgical extirpation of vestibular schwannomas vary inversely with tumor size. There seems to be little reason to monitor hearing during planned total removal of a tumor 4 cm or larger. Conversely, auditory monitoring is ideal when removing smaller tumors or sectioning the vestibular nerve in the face of serviceable hearing. Many other factors may suggest a better prognosis for hearing preservation, including lack of tumor extension into the lateral internal auditory canal and erosion of bony walls, low behavioral thresholds (i.e., good hearing), normal ABR, and reduced caloric response on electronystagmography (i.e., superior vestibular tumor). The generally accepted limits of serviceable hearing have been 50 dB HL pure tone average threshold and 50% speech discrimination score. Tumors in only hearing ears and bilateral tumors warrant special consideration. Worse hearing may indicate a more severe insult of auditory structures by disease and poor prognosis for hearing conservation. Exceptions exist, however, for hearing improvement when tumor removal alleviates a CN VIII conduction block.
Chapter 64 • Intraoperative Neurophysiologic Monitoring
779
In 2006, Samii and coworkers23 published their outcomes in a retrospective review of 200 consecutive vestibular schwannoma resections. They found that anatomic preservation of the facial nerve was possible in 98.5% of patients. By the last follow-up examination, excellent or good facial nerve function had been achieved in 81% of the cases. In patients with preserved hearing, the rate of anatomic preservation of the cochlear nerve was 84%. The overall rate of functional hearing preservation was 51%.
General Considerations Many lateral skull base approaches involve temporal bone dissection. Fluid and bone dust may accumulate in the middle ear and external auditory canal, resulting in a conductive hearing loss. ABR, ECoG, and direct CN VIII potentials are delayed in the presence of conductive hearing loss. OAE generally cannot be measured in the presence of significant conductive hearing loss. Whenever possible, the middle ear and external canal should be isolated from manipulation when monitoring auditory function. None of the methods for measuring auditory function during surgery should be affected by the use of routine anesthetics, including NMB. Patient factors, such as core temperature, muscle activity, blood pressure, and oxygenation, may alter electrophysiologic responses, however. Small decreases in core body temperature to 34.5° C result in delays of ABR waves.23 Similarly, increased muscle activity (e.g., fasciculations in a nonparalyzed patient) or hypotension and hypoxia may interfere with accurate measurements.
Auditory Evoked Brainstem Response ABR is considered a far-field response because it represents activity measured between scalp electrodes that are placed at relatively large distances from CN VIII and brainstem generator sites. Brainstem auditory evoked potentials consist of five to seven peaks representing electric activity of auditory nerve, nuclei, and fiber tracts of the ascending auditory pathways. Peak I refers to the distal portion of the auditory nerve. Peak II represents the central portion of the auditory nerve. Peak III represents the cochlear nucleus. Peak V represents the termination of the lateral lemniscus in the inferior colliculus. Waves III, IV, and V are created by multiple generators. Intraoperative ABR monitoring setup is similar to the setup for routine office measurement (Fig. 64-3). Subdermal needle electrodes are used for recording brainstem auditory evoked potentials. Insert ear plugs are suitable for delivering sound for the recording of auditory evoked potentials. Recorded potentials should be compared with baseline recording before the beginning of the operation. To reduce the time to obtain interpretable evoked potentials, the following actions are recommended:
FIGURE 64-3. Setup montage for auditory brainstem response, including ear canal inserts, active earlobe electrode, reference electrode on low forehead, and ground electrode on high forehead.
1. Reduce electric interference from reaching recording electrodes 2. Filter recoded potentials to attenuate background noise 3. Optimize stimulus repetition rate and strength 4. Optimize electrode placement of recording electrodes to decrease electrode impedance 5. Use methods for quality controls that do not require record replication In pathologic ears, ABR morphology is likely to eteriorate and become even more variable. Amplid tude is likely to decrease, especially in the presence of peripheral hearing loss, and latencies may increase based on the location and size of the lesion. Consequently, during monitoring of a previously impaired ear, changes from a well-established baseline are generally of more use than comparison with a norm. Because wave I is less likely to be present in an impaired ear, monitoring decisions may often depend on wave V. Enhancement of the wave I response can be achieved through ECoG recordings. Selection of testing parameters for intraoperative ABR must be made with the goal of optimizing the recording of the desired response, while minimizing the interference inherent in the electrically hostile operating room environment. Click stimuli should be present at sufficiently high rates and intensities. Wherever possible, background noise must be eliminated at the source. Finally, state-of-the-art signal processing techniques should be employed to enhance the signal-to-noise ratio and minimize the number of responses averaged to obtain an adequate waveform.
780
OTOLOGIC SURGERY
Interpretation of Results During posterior fossa surgery, damage is most likely to occur at the cochlea, CN VIII, or its root entry zone at the brainstem. An increase in peak V latency with or without a decrease in peak V amplitude and no change in peak III amplitude indicates the brainstem has been manipulated. Changes in interpeak latency I-III indicate stretching or compression of the auditory nerve. Disruption of the cochlear blood supply produces a quick and complete loss of ABR. Rapid response by the surgeon to restore the blood supply, if possible, is vital to prevent permanent damage. Other types of insults, such as stretching, compression, manipulation, or heat from the drill, can affect CN VIII and its root entry zone. ABR responses to these insults are neither as quick nor as easily interpretable as the responses produced by the loss of blood flow. Much depends on obtaining an optimal preoperative baseline response before entering the operating room and again after induction of anesthesia before initiation of the procedure. Criteria for judging that a change has occurred include increases in absolute latency of wave V from 0.5 to 2 ms.23 The strongest change is the loss of a repeatable wave V after all potential sources of interference have been ruled out. Harper and Slavit24 used criteria of greater than a 0.1 ms increase in latency or greater than a 50% decrease in amplitude to identify a change in the response.
Electrocochleography ECoG is a method of measuring the most peripheral of the auditory evoked responses. ECoG response consists of three primary components: cochlear microphonic, summating potential, and action potential. The action potential is an alternating current potential that is associated with the synchronous discharge of numerous neural fibers located in the basal region of the cochlea. The action potential represents the activity similar to wave I of ABR. The action potential component is most useful for intraoperative monitoring. ECoG has the advantage of being a near-field recording, and as such requires fewer averages and less time to obtain a response. In addition, the response obtained is larger and easier to interpret. Noninvasive extratympanic recordings require 250 to 1200 samples over a 12 to 60 second time course, whereas transtympanic needle electrodes require only 40 to 100 sweeps over a 2 to 5 second time period. The stimulus should be a broad-band rarefaction click of high intensity (85 to 95 dB normalized HL) with a rate of 21.1/second. An impedance of 100 kΩ may be acceptable in a transtympanic montage compared with the need for extremely low impedance of less than 5 kΩ required for surface electrodes. Recording parameters differ from those of ABR primarily in that the positive recording site is the ipsilateral promontory as opposed to a distant surface placement. Negative and
ground electrodes remain the same. Another difference is that the number of samples may be reduced to less than 100 compared with the 1500 suggested for ABR.
Interpretation of Results As with ABR, an adequate preoperative baseline response must be obtained against which to judge changes observed during the procedure. ECoG has been used to ascertain whether the goal of decompression operation of the endolymphatic shunt has been achieved. It has been thought that summating potential potentials normalize when pressure imbalances of the cochlea have been eliminated. Zappia and colleagues25 suggested that latency changes of greater than 1 ms and an amplitude decrease of greater than 50% should be considered significant changes in the response, although any measurable deviation should be reported immediately to the surgeon. Significant changes in the action potential indicate either direct or ischemic cochlear injury. After interruption of cochlear blood flow, 20 seconds or more may elapse before changes in ECoG response can be detected. This delay presumably occurs as a result of metabolic reserves that sustain cochlear function until their depletion from prolonged ischemia causes failure of electrophysiologic activity. It has been suggested that simultaneous recording of ABR and ECoG would result in the most effective monitoring system. ECoG provides an interpretable wave I in cases where it cannot be identified in ABR. Using the responses in combination, it would be possible to monitor changes in interpeak latencies, which could not be done using either alone. ECoG alone is not reflective of activity in the proximal region of CN VIII or the brainstem. It is possible to record a normal-appearing ECoG in the presence of significant CN VIII dysfunction. Monitoring of ABR wave V provides information related to the more central areas of auditory processing.
Direct Eighth Cranial Nerve Recording It was found that potentials measured directly from the cochlear nerve required significantly fewer averages than those of an ABR (0 to 100 versus hundreds to thousands) to display a recognizable wave pattern. This finding suggested that responses could be updated every few seconds under ideal circumstances. In addition, cochlear nerve action potentials were found to be present when ABR and ECoG recordings had been eroded by tumor or other pathology. The disadvantage of this type of recording in vestibular schwannoma surgery became evident during procedures to treat larger tumors that extended to or into the brainstem. There must be some portion of uninvolved cochlear nerve on which to place the electrode. Practically, many larger tumors are unsuitable for hearing conservation surgery anyway. Recording electrodes are most commonly placed on or around the intracranial segment of the cochleovestibular
Chapter 64 • Intraoperative Neurophysiologic Monitoring
nerve. A cotton or fibrous wick sutured to the tip of a polytef (Teflon)-insulated wire secures the electrode to the nerve atraumatically. The locations of the reference and ground electrodes are similar to those described for recording ABR. The response waveforms obtained from direct recordings of the cochlear nerve typically exhibit triphasic patterns.26 The initial positive deflection (downward) represents nerve activity approaching the recording site. The generally larger negative (upward) deflection occurs as the impulse passes under the electrode. As the depolarization moves away (more centrally) from the recording electrode, another positive wave results and completes the triphasic complex. In the case of CN VIII monitoring, baseline recordings are obtained after craniotomy, but before tumor dissection.
Interpretation of Results Tumor dissection causing pressure of the cochlear nerve results in immediate loss of compound action potentials (CAP) wave. Stretching the cochlear nerve causes increased latency and broadening of the response waveforms. Partial block of conduction, as may occur with direct pressure by a surgical instrument on the nerve, results in lower amplitude of the prominent negative peak. Only the initial positive deflection is seen after complete conduction block or transection of the nerve.
Otoacoustic Emissions OAE are low-level sounds measured from the ear canals of humans and animals with intact cochlear function. Transient-evoked OAE and distortion-product OAE may be useful in differentiating sensory (cochlear) from neural (retrocochlear) hearing losses. Poor behavioral hearing thresholds and good emissions suggest a retrocochlear site of lesion. Reduced OAE indicates at least cochlear dysfunction. Distortion-product OAE have been shown to be very sensitive to hypoxia and interruption of inner ear blood flow. Being sound waves, OAE are described by amplitude, frequency, and phase measures. Amplitude and phase are affected by manipulations that alter cochlear (and presumably outer hair cell) function. Subtle early changes in amplitude were measurable in some subjects within the first 10 seconds after internal auditory artery (IAA) occlusion. More recently, phase changes have been noted within 2 to 3 seconds of IAA compression, suggesting that phase measures may be the most sensitive OAE parameter for monitoring cochlear function during surgery.27 Emissions were stable during various procedures over many hours. Inhalation and intravenous (narcotic) anesthetics did not seem to affect the recording of emissions. Acoustic noise has the potential to undermine the recording of OAE. This effect was particularly prominent when measuring emissions at frequencies less than 2 kHz.28
781
Based on numerous recordings from various operating rooms, most background acoustic noise energy seemed consistently concentrated at less than 2 kHz. The authors recommended choosing patients for surgical OAE monitoring with preoperatively intact hearing and emissions greater than 2 kHz.28 Some of the noise can be eliminated from the ear canal by sealing the probe and the meatus. A surgical scrub sponge has been used with some success.29 Silicone, wax, and other substances also have been employed in an attempt to solve this problem. Distortion-product OAE have been monitored during loud suctioning of cerebrospinal fluid (relatively high frequency) from the operative field. Emissions could not be monitored during drilling (large-amplitude noise of low and high frequency). Mechanical interference with the conductive hearing mechanism would affect measurement of OAE. Fluid or debris such as bone dust in the external auditory canal or middle ear would interfere with emissions. For this reason, OAE monitoring is unsuitable for many transmastoid surgical approaches. Transient-evoked OAE result in an amplitudeweighted frequency spectrum that may be repeated throughout the critical portion of the operation.28 Generally, reliable transient-evoked OAE can be obtained in 30 to 40 seconds in ideal conditions and longer intervals in noisy situations. Using a customized software PC-based system, distortion-product OAE have been monitored during surgery every 2 seconds. Distortion-product OAE seem to be more robust than other types of emissions and are expected to be present in ears with mild to moderate sensory hearing loss (pure tone thresholds ≤45 dB). Also, monitoring programs can be designed to monitor a single or numerous frequency locations of the cochlea. The best frequency for monitoring may change during the case, and the program can be modified to reflect this. Because of these properties, distortion-product OAE at this time seem to be the best OAE suited for intraoperative monitoring. Because OAE reflect only cochlear function. ABR or direct CN VIII techniques would be appropriate to combine with emissions monitoring. Distortion-product OAE and ABR have been measured during vestibular schwannoma surgery using the same acoustic probe (ER 10-B, Etymotic Research Corp., Elk Grove Village, IL), which contained two speakers and a microphone.29 Rapid switching between the two recording instruments allowed comparisons of the responses. The probe should be secured in the ear canal at the level of the meatus. The pinna, probe, and tubing may be prepared out of the field by folding and securing (with tape or suture) the pinna anteriorly. The tubing should be secured to the patient or operating table in a location to prevent pulling or other disturbance. Baseline measurements obtained after probe insertion and after sterile draping of the operative field help ensure successful monitoring.
782
OTOLOGIC SURGERY
Laser-Doppler Cochlear Blood Flow Cochlear blood flow can be measured using laser Doppler flowmetry.30 The technique involves positioning the end of a needle probe, housing emitter and collector optical fibers, against the cochlear promontory in the middle ear. The angle of the probe is adjusted to obtain maximal flow readings. Although the potential application of this technology is exciting, transferring laser Doppler flow measurement techniques developed in the animal model to human surgery is being done in research trials and is to be studied further before it can be practically applied during tumor removal operations.
Monitoring Other Cranial Nerves Skull base tumors may involve many cranial nerves. CN III, IV, and VI, which control extraocular muscles, are at risk in tumors of the cavernous sinus. Motor portions of trigeminal nerve may be involved, and CN IX, X, XI, and XII can be involved in large skull base lesions. Nerves are localized using the same technique as used for the facial nerve electrode, employing subdermal needle electrodes placed with great care. Subdermal needle electrodes can be placed percutaneously in extraocular muscles that are innervated by a respective cranial nerve. Great care is needed to prevent injury to the globe. It is important to secure the electrodes well so that the location of the needle does not change when the patient is repositioned. The opposite forehead serves as a good location for reference electrode to avoid contamination with EMG potentials from ipsilateral facial muscles.31 EMG from soft palate (CN IX),32 false vocal cords or laryngeal muscles (CN X),33 sternocleidomastoid or trapezius muscle (CN XI), or lateral tongue (CN XII) can be used for the lower cranial nerves. CN XII monitoring can be done by placing recording electrodes spaced 1 cm apart in the lateral tongue.34 Each EMG potential is monitored on separate channels and clearly labeled so that the oscilloscope loudspeaker can be set to monitor the channel of importance at a particular time in a case.
CONCLUSION Intraoperative monitoring of CN VIII has become the standard of care for skull base and CPA surgery. Costeffectiveness analysis supports facial nerve monitoring in mastoid and middle ear surgery.35 Innovative methods of cranial nerve monitoring using techniques such as intraoperative F wave20 and transcranial electric motor evoked potential measurement36,37 are being investigated, but have yet to be widely adopted in otolaryngology. The latter technique is used in neurosurgery and vascular surgery, and provides the advantage of not relying on irritating stimulus of the nerve, but rather stimulation of the
c erebral cortex. Their use may be limited by the requirement for only intravenous anesthetic agents because of suppression of the signal when inhalational agents are used. For monitoring hearing, the monitoring paradigm must provide the surgeon with timely and accurate information to prevent, or repair, reversible injuries to auditory structures and to identify the maneuvers that cause hearing loss so that they may be avoided or modified in the future. Generally, the loss of a physiologic response correlates with poor postoperative hearing. Regardless of the monitoring technique used, the presence of an unchanged or reduced response is not predictive of hearing, which limits the prognostic accuracy of current monitoring techniques. There seems to be a consensus to use auditory monitoring during nerve section and microvascular decompression procedures where hearing preservation rates are quite high. Refining techniques of intraoperative auditory monitoring may ensure better hearing outcomes after acoustic neuroma removal. It is important to separate and distinguish changes in sensory and neural auditory responses to understand how a particular surgical manipulation would affect hearing. In the future, probably a combination of two or more of the methods described in this chapter will be the standard. Ultimately, facial nerve function and hearing outcome depend on many factors, including the surgeon’s experience, tumor size and location, tumor invasiveness, surgical approach, and preoperative function. Refining techniques of CN VIII monitoring will ensure better outcomes in the future.
REFERENCES 1. Krause F: Surgery of the Brain and Spinal Cord. New York, Rebman, 1912. 2. Hilger J A : Facial nerve stimulator. Trans Am Acad Ophthalmol Otolaryngol 68:74-76, 1964. 3. Delgado TE, Buchheit WA, Rosenholtz H R , Chrissian S: Intraoperative monitoring of facial muscle evoked res ponses obtained by intracranial stimulation of the facial nerve: A more accurate technique for facial nerve dissection. Neurosurgery 4:418-421, 1979. 4. Moller AR, Jannetta PJ: Preservation of facial function during removal of acoustic neuromas: Use of monopolar constant-voltage stimulation and EMG. J Neurosurg 61:757-760, 2984. 5. Pratt R L : Iatrogenic facial nerve injury: The role of facial nerve monitoring. Otolaryngol Clin North Am 29:265275, 1996. 6. Hughes G, Bottomy M, Dickins J, et al: A comparative study of neuropathologic changes following pulsed and direct current stimulation of the mouse sciatic nerve. Am J Otolaryngol 1:378-384, 1980. 7. Hughes G B, Bottomy M B, Jackson CG, et al: Myelin and axon degeneration following direct current peripheral nerve stimulation: A prospective controlled experimental study. Otolaryngol Head Neck Surg 89:767-775, 1981.
Chapter 64 • Intraoperative Neurophysiologic Monitoring 8. Chase SG, Hughes GB, Dudley AW Jr: Neuropathologic changes following direct-current stimulation of the rat sciatic nerve. Otolaryngol Head Neck Surg 92:615-617, 1984. 9. Choung YH, Park K, Cho M J, et al: Systematic facial nerve monitoring in middle ear and mastoid surgeries: “Surgical dehiscence” and “electrical dehiscence.” Otolaryngol Head Neck Surg 135:872-876, 2006. 10. Nakao Y, Piccirillo E, Falcioni M, et al: Electromyographic evaluation of facial nerve damage in acoustic neuroma surgery. Otol Neurotol 22:554-557, 2001. 11. Prell J, Rampp S, Romstock J, et al: Train time as a quantitative electromyographic parameter for facial nerve function in patients undergoing surgery for vestibular schwannoma. J Neurosurg 106:826-832, 2007. 12. Kizilay A, Aladag I, Cokkeser Y, et al: Effects of partial neuromuscular blockade on facial nerve monitorization in otologic surgery. Acta Otolaryngol 123:321-324, 2003. 13. Blair E A, Teeple E Jr, Sutherland R M, et al: Effect of neuromuscular blockade on facial nerve monitoring. Am J Otol 15:161-167, 1994. 14. Lacombe H, Keravel Y, Eshraghi AA : Interest of monitoring of facial nerve on facial function in translabyrinthine surgery of acoustic neuroma. Ann Otolaryngol. Chir Cervicofac 111:89-93, 1994. 15. Selesnick S H, Carew J F, Victor J D, et al: Predictive value of facial nerve electrophysiologic stimulation thresholds in cerebellopontine-angle surgery. Laryngoscope 106: 633-638, 1996. 16. Grayeli A B, Guindi S, Kalamarides M, et al: Fourchannel electromyography of the facial nerve in vestibular schwannoma surgery: Sensitivity and prognostic value for short-term facial function outcome. Otol Neurotol 26:114-120, 2005. 17. Neff B A, Ting J, Dickinson S L , Welling D B : Facial nerve monitoring parameters as a predictor of postoperative facial nerve outcomes after vestibular schwannoma resection. Otol Neurotol 26:728-732, 2005. 18. Isaacson B, Kileny PR , El-Kashlan H K : Prediction of long-term facial nerve outcomes with intraoperative nerve monitoring. Otol Neurotol 26:270-273, 2005. 19. Selesnick S H : Optimal stimulus duration for intra operative facial nerve monitoring. Laryngoscope 109: 1376-1385, 1999. 20. Wedekind C, Klug N: Facial F wave recording: A novel and effective technique for extra- and intraoperative diagnosis of facial nerve function in acoustic tumor disease. Otolaryngol Head Neck Surg 129:114-120, 2003. 21. Arriaga M, Haid R , Masel D: Antidromic stimulation of the greater superficial petrosal nerve in middle fossa surgery. Laryngoscope 105:102-105, 1995. 22. Levine R A, Ojemano RG, Montgomery WV, McGaffigan PM : Monitoring auditory evoked potentials during acoustic neuroma surgery: Insights into the mechanism of the hearing loss. Ann Otol Rhinol Laryngol 93:116123, 1984. 23. Samii M, Gerganov V, Samii A : Improved preservation of hearing and facial nerve function in vestibular schwannoma surgery via the retrosigmoid approach in a series of 200 patients. J Neurosurg 105:527-535, 2006.
783
24. Harper C M, Harner SG, Slavit D H, et al: Effect of BAEP monitoring on hearing preservation during acoustic neuroma resection. Neurotology, 42(8):1551-3, 1992. 25. Zappia JJ, Wiet R J, O’Connor C A : Intraoperative monitoring in acoustic neuroma surgery. Otolaryngol Head Neck Surg 115:99-106, 1996. 26. Schwartz D M, Bloom M I, Dennis I M : Perioperative monitoring of auditory brainstem responses. Hear J 38: 9-13, 1985. 27. Telischi FF, Stagner B B, Widick M P, et al: Distortionproduct otoacoustic emission monitoring of cochlear blood flow. Laryngoscope 108:837-842, 1998. 28. Telischi FF, Widick M P, Lonsbury-Martin B L , McCoy M I : Monitoring cochlear function intraoperatively using distortion product otoacoustic emissions. Am J Otol 16: 597-607, 1995. 29. Cane M A, O’Donoghue U M, Lutman M E : The feasibility of using oto-acoustic emissions to monitor cochlear function during acoustic neuroma surgery. Scand Audiol 21:173-176, 1992. 30. Nakashima T, Naganawa S, Sone M, et al: Disorders of cochlear blood flow. Brain Res Brain Res Rev 43:17-28, 2003. 31. Sekiya T, Hatayama T, Iwabuchi T, Maeda SH: A ring electrode to record extraocular muscle activities during skull base surgery. Acta Neurochir (Wien) 117:66-69, 1992. 32. Yingling C D: Intraoperative monitoring in skull base surgery. In Jackler R K, Brackmann D E (eds): Neurotology. St Louis, Mosby–Year Book, 1994, pp 967-1002. 33. Stechison MT: Vagus nerve monitoring: A comparison of percutaneous versus vocal fold electrode recording. Am J Otol 16:703-706, 1995. 34. Moller AR: Intraoperative monitoring of evoked potentials: An update. In Wilkins RH, Rengachary SS (eds): Neurosurgery Update I: Diagnosis,��������������������������� Operative Technique, and Neuro-Oncology. New York, McGraw-Hill, 1990, pp 169-176. 35. Wilson L , Lin E, Lalwani A : Cost-effectiveness of intraoperative facial nerve monitoring in middle ear or mastoid surgery. Laryngoscope 113:1736-1745, 2003. 36. Akagami R , Dong CC, Westerberg B D: Localized transcranial electrical motor evoked potentials for monitoring cranial nerves in cranial base surgery. Neurosurgery 57 (1 Suppl):78-85, 2005. 37. Liu BY, Tian YJ, Liu W, et al: Intraoperative facial motor evoked potentials monitoring with transcranial electrical stimulation for preservation of facial nerve function in patients with large acoustic neuroma. Chin Med J 120: 323-325, 2007. 38. Lin VY, Houlden D, Bethune A, et al: A novel method in predicting immediate postoperative facial nerve function post acoustic neuroma excision. Otol Neurotol 27:10171022, 2006. 39. Isaacson B, Kileny PR , El-Kashlan H, Gadre A K : Intraoperative monitoring and facial nerve outcomes after vestibular schwannoma resection. Otol Neurotol 24:812817, 2003. 40. Fenton J E, Chin RY, Fagan PA, et al: Predictive factors of long-term facial nerve function after vestibular schwannoma surgery. Otol Neurotol 23:388-392, 2002.
784
OTOLOGIC SURGERY
41. Goldbrunner R H, Schlake H P, Milewski C, et al: Quantitative parameters of intraoperative electromyography predict facial nerve outcomes for vestibular schwannoma surgery. Neurosurgery 46:1140-1146, 2000. 42. Nissen A J, Sikand A, Curto FS, et al: Value of intraoperative threshold stimulus in predicting postoperative facial nerve function after acoustic tumor resection. Am J Otol 18:249-251, 1997. 43. Zeitouni AG, Hammerschlag PE, Cohen N L : Prognostic significance of intraoperative facial nerve stimulus thresholds. Am J Otol 18:494-497, 1997. 44. Taha J M, Tew J M Jr, Keith RW: Proximal-to-distal facial amplitude ratios as predictors of facial nerve function after acoustic neuroma excision. J Neurosurg 83:994998, 1995.
45. Lalwani A K, Butt FY, Jackler R K, et al: Facial nerve outcome after acoustic neuroma surgery: A study from the era of cranial nerve monitoring. Otolaryngol Head Neck Surg 111:561-570, 1994. 46. Prasad S, Hirsch B E, Kamerer D B, et al: Facial nerve function following cerebellopontine angle surgery: Prognostic value of intraoperative thresholds. Am J Otol 14:330-333, 1993. 47. Niparko J K, Kileny PR , Kemink J L , et al: Neurophysiologic intraoperative monitoring, II: Facial nerve function. Am J Otol 10:55-61, 1989.
65
Stereotactic Radiosurgery of Skull Base Tumors P. Ashley Wackym, Christina L. Runge-Samuelson, and David R. Friedland
Management of many skull base tumors has shifted in recent years away from surgical resection and towards control of growth. This is particularly true for vestibular schwannomas, i.e., acoustic neuromas, and is increasingly applicable to glomus jugulare tumors. The principal modality for such treatment is gamma knife surgery although other conformal radiation treatment systems are available. Gamma knife surgery is advantageous in requiring a single session for treatment of most skull base lesions, which increases its appeal to both surgeon and patient. This chapter will focus primarily on the methods used in treating skull base tumors with gamma knife surgery. Gamma knife surgery, similar to microsurgery, has advantages and disadvantages which must be thoroughly discussed with the patient.1,2 For the patient it is alluring to undergo an outpatient procedure rather than microsurgical management that requires a much longer period of care. Further, gamma knife outcomes show excellent tumor control and, with current methods, low cranial nerve morbidity. Gamma knife surgery is a viable treatment modality for the appropriate patient as defined by age, medical history, tumor characteristics and physical findings. As such, many neurotologists now offer gamma knife surgery as part of their armamentarium for managing vestibular schwannomas and glomus tumors.3 Several institutions world-wide offer training courses for physicians and radiation physicists at centers having the Leksell Stereotactic System or Leksell Gamma Knife. To date more than 1500 neurotologists, neurosurgeons, physicists, and radiation oncologists have received such training. In addition, the parent company, Elekta Instrument AB (Stockholm, Sweden), offers basic and advanced training courses and workshops. Courses typically consist of didactic lectures, observation of patient treatment, and practical hands-on training. Further, all new installations of Leksell Gamma Knife are accompanied by a one-week on-site start-up training for the neurotologists, neurosurgeons, radiation oncologists, and physicists comprising the gamma knife treatment team.
PATIENT SELECTION Opting for gamma knife surgery over observation or microsurgical resection is a complex decision. There are the preferences of the informed patient, the comfort and experience of the surgeon, the patient’s medical history and condition, and the characteristics of the tumor. While there are no definitive measures defining or restricting the use of gamma knife surgery, particular guidelines can inform the decision making process. Although a tissue diagnosis is not typically acquired prior to gamma knife treatment, radiographic and clinical diagnoses of vestibular schwannoma and glomus jugulare are sufficient to initiate a discussion of gamma knife surgery. Other potential neoplasms amenable to gamma knife treatment by the neurotologist are cerebellopontine angle meningiomas, posterior fossa and jugular foramen non-vestibular schwannomas, temporal bone metastatic lesions and primary vascular neoplasms. An absolute contraindication to gamma knife treatment would be tumors extending too far inferiorly to enable placement into the centrum of the collimator helmet. Gamma knife surgery is also contraindicated in large tumors causing life-threatening brainstem and central aqueduct compression. Such large tumors, in the absence of clinically significant problems, provide a relative contraindication to gamma knife surgery as post-treatment swelling may cause obstructive hydrocephalus requiring emergent intervention. Typically, vestibular schwannomas greater than 2.5 cm in the cerebellopontine angle should be cautiously approached if gamma knife proves the best option given other medical concerns. Most surgeons will not treat vestibular schwannomas greater than 3.0 cm in maximum axial dimension within the cerebellopontine angle because of the risk of post-treatment obstructive hydrocephalus. Other guidelines for gamma knife surgery require clinical judgment as to the medical condition of the patient, the expected growth and potential morbidity of the tumor, the functional status of the patient, audiometric and vestibular performance, age and expected life-span of the patient. Individualized treatment plans 785
786
OTOLOGIC SURGERY
depend on a frank and thorough dialogue between physician and patient as to the options available, risks and benefits of each approach, and expected outcomes based upon evidence-based reviews or an analysis of each institution’s outcomes.
PREOPERATIVE COUNSELING Informed consent for gamma knife surgery requires the surgeon to discuss alternative options such as observation and microsurgical resection.2 The risks and benefits of these alternatives should be frankly described and compared to gamma knife treatment. Many patients have received information from the Internet or from physicians with limited experience with gamma knife and may have erroneous information. Common misconceptions include the expectation that gamma knife surgery completely removes the tumor and that hearing will improve, or conversely that cranial nerve morbidities are significant. These need to be addressed with evidence-based reports and information. One statistic, which is particularly alarming to patients considering gamma knife surgery is that there have been eight cases of malignancy within vestibular schwannomas (as of 2002).4 Four of these patients had been previously treated with radiosurgery. While it remains possible that these four malignancies developed after the radiation treatment, it is more likely that these malignant tumors were misdiagnosed as benign at the outset of evaluation and treatment. Delayed development of radiation-induced neoplasms was addressed by Pollock and colleagues in 1998.5 They reviewed more than 20,000 patients treated with radiosurgery worldwide and found no increased incidence of new neoplasm development (i.e., benign or malignant). A retrospective cohort study comparing the Sheffield, England radiosurgery patient database with the national mortality and cancer registries identified a single new astrocytoma among those treated.6 Based on their national incidence figures, 2.47 cases would have been predicted. The risk of radiosurgery induced malignancy in patients with neurofibromatosis type 2 (NF2) and von Hippel-Lindau disease was similarly studied.7 Of 118 NF2 and 19 von Hippel-Lindau disease patients, totaling 906 and 62 patient-years of follow-up data, respectively, only two cases of intracranial malignancy were found. Both of these were in NF2 patients. One was thought to have arisen before the radiosurgery; the other was a glioblastoma diagnosed three years after radiosurgery. Gliomas may occur in as many as 4% of NF2 patients and the single case may not represent an increased risk. It was suggested that the late risk of malignancy arising after irradiation must be put in the context of the condition being treated, the treatment options available to these individuals, and their life expectancy.
Despite the findings of the studies just reviewed, it is important to counsel patients about the possibility of malignant transformation or induction. A handful of tumors suggestive of radiation induced malignancy have been reported among the tens of thousands who have undergone gamma knife treatment. Lustig and colleagues reported the development of a squamous cell carcinoma following radiation treatment of vestibular schwannoma.8 Hanabusa and colleagues reported the malignant transformation of a vestibular schwannoma following gamma knife surgery.9 There was histologic evidence of vestibular schwannoma following a retrosigmoid resection. Four years after this resection, recidivistic tumor was identified, and the patient was subsequently treated with gamma knife surgery. Six months post-treatment, the tumor had grown, and the patient underwent surgical resection via a combined retrosigmoid-translabyrinthine approach. Abnormal mitotic figures were observed on histologic sections, and the diagnosis of malignancy was assigned.
SURGICAL TECHNIQUE The Gamma Knife Unit The first gamma knife unit (Elekta Instrument AB, Stockholm, Sweden) was installed in Stockholm, Sweden in 1968, and it was not until 1987 that the first gamma knife (model U) was installed in the United States at the University of Pittsburgh. The gamma knife model B (1996) is the unit currently most used throughout the United States. The gamma knife model C was introduced three years later and the major upgrade consisted of an automatic positioning system (APS). The unit is otherwise quite similar to the model B and both contain 201 radioactive isotope cobalt 60 (60Co) sources and beam channels. Due to physical restraints these units can only treat lesions intracranially or along the skull base. During 2008, a completely redesigned gamma knife unit, named Perfexion, is being introduced. It uses 192 60Co sources, has a single collimator helmet with variable diameters, and can treat lesions within the entire head and neck, down to the level of the clavicles. The basic principle of gamma knife surgery is to provide focused radiation to the tumor while minimizing radiation delivery to surrounding tissues. As such, a semicircular shield called the collimator helmet is used to generate approximately 200 individual gamma radiation “beams.” In the center of the helmet, where the beams meet, radiation delivery is maximal, but along each individual radiation tract tissue exposure is relatively low. When the collimator helmet is locked into position, the 201 openings of the collimator helmet coincide with the cobalt sources. There is a shielded chamber within which the 60Co sources are contained, and stainless steel shielding doors protect the treatment room from the 60Co sources. There is a treatment couch with an adjustable mattress that slides into the gamma knife unit together
Chapter 65 • Stereotactic Radiosurgery of Skull Base Tumors
787
Automatic positioning system
Helmet with collimators (outside treatment position)
Helmet supports
Cobalt 60 sources
Treatment couch with adjustable mattress Beam channel Shielding Plastic cover
Protective panels
Helmet in treatment position
Shielding doors
Figure 65-1. Gamma knife surgery. Schematic illustration of the Leksell Gamma Knife 4C which utilizes the automatic positioning system. (Published with permission, copyright © 2008, Elekta Instrument AB [Stockholm, Sweden].)
irradiation shot. There are four interchangeable helmets by means of which the size of the collimator (that part of the treatment unit that shapes the beam) can be changed between 4 mm, 8 mm, 14 mm and 18 mm. The combination of four different sized collimators and repositioning the patient in the three-dimensional space defined by the stereotactic headframe are effective to deliver the radiation dose selectively and conformally to radiosurgical targets of any shape.
Frame Attachment
Figure 65-2. Leksell Gamma Knife 4C. (Published with permission, copyright © 2008, Elekta Instrument AB [Stockholm, Sweden].)
with the collimator helmet and the patient. Figure 65-1 schematically shows the orientation of the components of the gamma knife model, Leksell Gamma Knife® 4C and Figure 65-2 shows the overall appearance of the gamma knife model, Leksell Gamma Knife® 4C. When treatment is initiated, the treatment couch is automatically moved from its idle position into the treatment unit together with patient and helmet. Once the couch is docked in its treatment position, the helmet collimator and corresponding collimators in the unit form a beam channel, allowing the radiation that is continuously emitted by the sources to reach the patient. At the end of each irradiation “shot,” the couch is automatically withdrawn, either to its idle position or to a position outside the radiation focus to reposition the patient for the next
The stereotactic head frame is used to coordinate the location of the tumor within the collimator helmet. As such, proper placement is of utmost importance to providing adequate treatment. There are two general principles guiding head frame placement for gamma knife surgery. First, the target should be as close to the center of the frame as possible. This prevents possible collisions of the frame with the sides of the collimator helmet especially when trying to align laterally extended tumors in the center of the unit. Second, the frame attachment should be stable. This prevents movement and ensures accuracy and correlation among the pre-treatment imaging study, workstation treatment plan, and delivery of focused radiation. These principles should be addressed at the time of frame attachment. In lateral targets, such as vestibular schwannomas or glomus tumors, the frame should be shifted toward the tumor side. In skull base tumors the frame should also be positioned lower than for treatment of more superior intracranial lesions. Anteriorposterior alignment should also be accounted for and can be adjusted by varying the lengths of the pins used to
788
OTOLOGIC SURGERY
A
B
C
D
Figure 65-3. A, Stereotactic headframe at the time of assembly. Pins used for fixation prior to imaging, treatment planning, and gamma knife radiosurgery are seen in the foreground. B, With a towel placed on the vertex of the head, the magnetic resonance imaging (MRI) fiducial box is balanced on the head while topical anesthetic is infiltrated at the pin sites. C, While the headframe and attached plastic MRI fiducial box is secured to the skull using the pins, an assistant stabilizes the assembly in place. Note the tightening of pins in opposing vectors. D, After the frame has been secured to the skull, measurements are taken through defined entrance points in a plastic dome representing the size and position of the collimator helmet relative to the frame, head and face. These values are incorporated into the treatment planning software to create a wire grid representing the patient’s head. The post height and pin length are also measured and are inputted into the treatment planning software. These values are used in the calculations used to predict collisions between the collimator helmet and the stereotactic headframe. (Published with permission, copyright © 2008, P. A.Wackym, MD.)
secure the frame. To ensure stability, avoid screw fixation in bone flaps, cranioplasty materials, burr holes, or skull defects. The method of anesthesia used during frame placement is surgeon and patient dependent. In our program, either sedation with versed and fentanyl, or monitored anesthesia with propofol, followed by injection of local anesthetic at the pin sites is used. Figure 65-3A shows the typical array of tools used for the frame attachment. A variety of screw lengths allow the surgeon to choose those ideally suited for the individual location of the posts and tumor. The placement of the frame should begin with an accurate orientation of the location of the target within the patient’s head. Ideally, the target should be located within the fiducial range and placed centrally within the frame thereby avoiding later collisions with the collimator helmet and granting sufficient accuracy for the stereotactic target definition. The stereotactic frame is assembled and preliminarily supported by using external auditory canal support pins, a Velcro band, or a stereotactic fiducial box. When using a fiducial box to facilitate frame placement, it is important to use the MRI fiducial box, rather than the CT or
angiography fiducial box, since this is the smallest of the three plexiglass fiducial boxes (Figs. 65-3B and 65-3C). Asymmetric frame placements are possible and do not impair the accuracy of imaging. The frame can be shifted from side to side or can be moved as far as possible to the front or back to facilitate centering of the tumor. The frame is stabilized against the patient by an assistant and the surgeon should adjust the lengths of the posts to maintain relative tumor position. A low position of the anterior posts can help avoid anterior collisions with the collimator helmet for skull base posterior fossa tumors. In critical positions, collisions can sometimes be avoided by using the curved posts in the anterior position. Once post position is determined the screws can be inserted. The surgeon and assistant should work on diagonally opposing screws to provide the best stability without changing the desired frame position. For asymmetric frame placement apply the longest screws first, thereby defining the desired distance of the target to the frame. Protrusion of the screws from the posts should be kept to a minimum to avoid collisions. Approximately 8 to 10 mm is considered to be sufficient but at our institution we prefer to limit this projection to 4 to 6 mm.
Chapter 65 • Stereotactic Radiosurgery of Skull Base Tumors
789
If a screw extends further it should be exchanged for a shorter screw. Measurements of the frame and placement are then performed to allow the computer to identify any potential collisions after the plan is formulated. These measurements are required for the frame and skull section in Leksell GammaPlan treatment planning software. Measurements include the length of the four posts and the length of the screws that protrude from the posts. Additionally, the volume of the head is measured using the plastic collimator bubble, simulating the relationship of the frame to the treatment collimator helmet (Fig. 65-3D). This concludes frame placement and the patient may proceed to imaging.
Imaging Treatment planning requires imaging of the tumor with respect to the frame as determined by specific fiducial boxes. The MRI fiducial box clips to the frame and care should be taken to ensure that it is flush and square during imaging. The MRI fiducial box has a Z-shaped channel on each side filled with copper sulfate to generate position markers for each axial slice. The box should be checked prior to each use to ensure the channels are filled with solution and no air bubbles are present. The patient, with head frame and fiducial box, is secured into the head holder on the MRI sliding table. For imaging acoustic neuromas and glomus tumors we typically order axial 3D SPGR (spoiled gradient recalled) acquisition with T1 weighting and double dose IV contrast. Before the patient leaves the scanner images are reviewed and the distance between fiducial registration markers is validated for accuracy. Many centers acquire only MRI scans for treatment planning. We prefer to also acquire a non-contrast CT scan through the temporal bone to aid in planning. There is evidence of distortion of MR images and correlation with CT scans at the time of planning can aid in reducing radiation delivery to critical structures such as the cochlea and facial nerve.10 A CT fiducial box is affixed to the frame, the patient secured in the holder attached to the table, and an axial scan through the temporal bone and skull base acquired. Both CT and MR images are imported into the Gamma Knife workstation. Axial scans are defined, and coronal and sagittal reconstructions generated for each.
Treatment Planning Leksell GammaPlan is the dedicated software treatment planning system for Leksell Gamma Knife. Dose planning for gamma knife surgery means precisely conforming the isodose distribution to the target. The isodose distribution is built up by a number of individual shots or isocenters. The Leksell GammaPlan software is designed to help the operator as much as possible to perform this procedure and is quite straightforward to use.
Figure 65-4. Initial treatment planning at the gamma knife workstation involves building a three-dimensional model of the tumor. Determination of the conformation of the treatment plan follows placement of the shots and assignment of the radiation dose delivered to the specified isodose line. (Published with permission, copyright © 2008, P. A.Wackym, MD.)
Currently, for vestibular schwannomas, the routine prescription is 12 to 14 Gy delivered to the 50% isodose line. The 50% isodose line shows where 50% of the prescribed dose lies. In the case of gamma knife treatments the dose is frequently prescribed to the 50% isodose line. This ensures that the periphery of the tumor will receive at least the prescribed dose, that the dose will be higher than the prescribed dose inside the tumor, and that the dose will fall off rapidly outside the tumor thus sparing critical structures. Dose planning using Leksell GammaPlan involves composing shots to develop a conformal isodose. By definition, this includes the whole target but spares the surrounding healthy tissue. Figure 65-4 shows an example of a vestibular schwannoma. The target is well positioned on the screen and magnified for good visibility. When the shot menu is opened, one can select the size of the collimators. The size of the collimator is selected based on the tumor shape and the gaps in coverage of the 50% isodose line displayed over the tumor. Shots are placed sequentially to cover the target as effectively as possible. Changing the position of the shots, adding additional shots, and adjusting the relative weight of shots quickly leads to a conformal dose plan. The dose plan can be checked using Leksell GammaPlan with the three-dimensional (3D) image or the measurement tools, such as dose volume histograms. While the subject of conformity index is beyond the scope of this chapter, an excellent review of available methods has been published.11 Leksell GammaPlan indicates the point in the stereotactic space where a global maximal dose can be found. Leksell GammaPlan also calculates the individual shot times. Once the treatment plan has
790
OTOLOGIC SURGERY
been determined to be appropriate by the gamma knife team (surgeon, radiation oncologist, and radiation physicist), the stereotactic coordinates and irradiation times are printed and used during the gamma knife treatment. An automated approach to initial treatment planning has been developed by Elekta Instruments AB. This software assists the treatment planner in generating a good initial dose plan quickly and is termed “the Wizard.” This is an interactive tool that helps the operator develop the dose plan. The operator first selects the shot size and the degree of density with which the Wizard should fill the target. A mouse click instructs the Wizard to fill the target with shots. If the initial dose distribution is not sufficient, a mouse click on the run button instructs the Wizard to optimize the plan by moving and weighting the shots. This interaction results in a better dose plan, and after a few more changes a satisfactory dose plan can be created. However, in this our experience, manual placement of the shots, particularly for vestibular schwannomas, has always resulted in a better treatment plan. Fine-tuning is made with small adjustments in shot position and weight, allowing optimization of the dose plan. Leksell GammaPlan allows the creation of different plans for the same target. This allows the surgeon and oncologist to follow different strategies and later compare plans and select the best plan for the actual treatment. Treatment plans can utilize as few as one or two shots, such as when treating trigeminal neuralgia, or over 10 shots when treating a large vestibular schwannoma within the cerebellopontine angle and filling the internal auditory canal. With the enhanced capabilities of Leksell Gamma Knife C, plans with 20 shots or more can easily be implemented in a timely manner, since the model C does not require manual adjustments of coordinates in between each shot by the gamma knife treatment team. This allows improved conformity and selectivity of gamma knife surgery, potentially reducing the risk of complications. To shape the dose distribution to avoid critical structures, one or more of the 201 collimators can be replaced with a closed shield called a plug. One can select spherical areas called shields with different diameters and place them over risk centers in the brain, cranial nerves, or cochlea. Once the shields are put in place, the Leksell GammaPlan software closes off all beams that would irradiate through the shielded area. The result is a modified dose plan in the low isodose lines with only little effect on the target peripheral isodose. The beam channels that need to be plugged can be seen in the plug pattern. The plug patterns can be merged for all shots of the same size so that the operator only has to plug the helmets for the treatment once. In the final plan, the peripheral dose is set to a value, which is assessed as optimal for a particular patient. Indication, size, and location of the target are taken into account, as well as clinical experience. The peripheral isodose is usually set to the 50% isodose line. This is exactly
CutBox (MRI_axial)
Sagittal (MRI_axial) Reference Dose for Plan ‘Final_plan’
Absolute Dose Levels Normalize to point Prescription dose
[Gy]
14.00
Prescription dose
[%]
50.00
Maximum dose [Gy] 28.00 OK
Coronal (MRI_axial) Display Isodose
Selected Level Display volume Wire 50 Levels 90 80 45 20 Plot
Set
Exit
Help
Help
OpenBook (CT_axial)
Figure 65-5. Gamma knife surgery. Selecting the Absolute Dose Level and Display Isodose options allows verification that the maximal radiation dose is not delivered near critical structures, such as the facial nerve. In this example, 14 Gy delivered to the 50% isodose line was prescribed. As shown, the maximum dose (28 Gy) is delivered to the center of the tumor (smallest circle). The largest circle represents the 20% isodose line where 6 Gy of radiation is delivered. (Published with permission, copyright © 2008, P. A.Wackym, MD.)
half the maximum dose in the target, referred to as the hot spot (Fig. 65-5). Along the 50% isodose line the dose gradient is usually the steepest ensuring sufficient dose within the target, while the dose level outside falls steeply, sparing the surrounding healthy tissue. Leksell Gamma Plan can also display the absolute dose values if desired. It will show the point in the stereotactic space where the global maximum dose can be found. With vestibular schwannoma it is valuable to complete this exercise, as the maximal dose at the “hot spot” should be positioned well away from the facial nerve and cochlea. In addition, plotting the absolute dose lines will help in determining the actual level of radiation delivered to surrounding structures. When the dose planning is completed, Leksell GammaPlan checks the shots for collisions with the collimator helmet and sorts the plan according to collimator size. The team performs quality assurance steps to check the accuracy of the X, Y, and Z coordinates and to ensure the plan treats the correct side. All relevant data are documented including details of the treatment plan, targets, dose volume histograms, snap shots and images. When the treatment set-up has been finalized the treatment protocol is exported to Leksell Gamma Knife. This is via a special secured direct serial connection. Leksell GammaPlan only accepts valid and verified treatment plans for export. In addition, a protective design limits the transfer of a treatment plan to the Leksell Gamma Knife to one patient at a time. Once the data have been transferred to the operator’s console, it is verified, and the patient can be treated. For the model C unit, the operator does not have to enter the treatment room during a run. However, with the
Chapter 65 • Stereotactic Radiosurgery of Skull Base Tumors
model B the treatment team enters the treatment room after each shot is delivered and manually adjusts the X, Y, and Z coordinates, as well as the gamma angle, i.e., the pitch of the head, if necessary. With both the model B and model C, the team has to change the collimator helmet manually when necessary, as dictated by the treatment plan. Detailed treatment and physics protocols are viewed and printed out.
Treatment Treatment can be performed automatically using the automatic positioning system or manually using trunnions. For the model B, manual setting of the X, Y, and Z coordinates as well as the gamma angle if necessary is accomplished by the treatment team. The Y and Z coordinates are set with the Y, Z slides on the y-bar attached to the coordinate frame, whereas the X coordinate and the gamma angle are set with the trunnions. It is imperative to have a check and balance in place that consists of visual verification of each coordinate by a different team member. Y coordinates need to be verified prior to setting the Z coordinate as the latter will obscure the scale on the Y axis. It is preferable to set the X coordinate of the trunnion on the shorter side first as this will provide more room to manipulate the patient and head frame within the collimator helmet. These coordinates need to be manually changed between each shot on the model B unit. With the automatic positioning system, the treatment is controlled from the operator console. Once the treatment starts, the selected run is carried out automatically. Before repositioning, the couch will move out a short distance to bring the patient out of treatment focus. At this point, the APS will move the patient’s head to the next target position. A run consists of all shots for a specific collimator helmet size. Additional runs are performed after manually changing the collimator helmet. After all runs have been completed the head frame is removed. The anterior fixation sites are dressed with antibiotic ointment and adhesive bandages. The posterior sites are dressed with antibiotic ointment. Often pressure needs to be held to control bleeding and occasionally a staple may need to be used on the posterior sites. Typically patients will experience a transient headache after removal of the frame and some develop nausea and emesis. We typically pre-medicate with decadron and ondansetron prior to frame removal. Patients are observed for several hours post-treatment and discharged home with pain medication and follow-up appointments.
GAMMA KNIFE SURGERY OUTCOMES Just as is the case with other forms of medical and surgical therapy, the techniques and outcomes of gamma knife surgery for vestibular schwannomas have evolved
791
and improved over time. Tumor control and facial nerve motor preservation occurs with virtually all vestibular schwannoma patients treated with current gamma knife protocols. Areas of continued focused investigation include the effects of radiosurgery on hearing and balance, and methods of improving outcomes. The University of Pittsburgh group has the largest clinical experience in treating vestibular schwannomas with gamma knife surgery. Lunsford and colleagues summarized their experience with 829 vestibular schwannomas treated between 1987 and 2002.12 This extensive clinical experience included an average tumor volume of 2.5 cm3 and a median margin dose to tumor of 13 Gy. They reported tumor control in 97% of patients at 10 years, and facial nerve (motor) dysfunction in < 1% of patients. Trigeminal nerve symptoms occurred in < 3% of patients and typically occurred with large tumors reaching the level of the trigeminal nerve. No reporting of balance function was included in their analyses. The reporting of hearing preservation has limited representation in the entire 829 patients. Hearing outcomes data were presented in only 267 patients and “5-year actuarial rates of hearing level preservation and speech preservation” were reported in 103 patients. They reported “unchanged hearing preservation” in 50����� %���� to 77% of these patients, and this method of reporting auditory performance points to the difficulty in interpreting the outcome of most of the studies reporting hearing outcome in patients with vestibular schwannoma who have been treated with gamma knife surgery. They also stated that “for patients with intracanalicular tumors, hearing preservation rates in those treated with 12.5 to 14 Gy at the margin showed 90% preservation of serviceable hearing.”13 Unfortunately, pretreatment and longitudinal data are not available in these reports. In an earlier series of 190 patients the average dose to the tumor margin was reduced to 13 Gy, and excellent tumor control was achieved at 97.1%.14 In this study, issues highlighted earlier with reporting of hearing outcome are equally apparent. They reported “hearing-level preservation” in 71 ± 4.7% of patients. They also reported a “preservation of testable speech discrimination ability” in 91 ± 2.6% of subjects. Obviously, testable speech discrimination ability is far different than useful hearing, and it is unfortunate that these authors did not report the actual auditory thresholds or speech discrimination ability. Most importantly, these were not reported as a function of time post-gamma knife surgery. In addition, they reported that “hearing levels improved” in 10 (7%) of 141 patients who exhibited decreased hearing defined as Gardner-Robertson grades II to V before undergoing gamma knife surgery. Based on our clinical observations and those of other centers, this picture is far more complex over time than is represented in these publications. Prasad and colleagues from the University of Virginia reported their series of 200 vestibular schwannomas treated with gamma knife surgery over a 10-year interval
OTOLOGIC SURGERY
in 2000.15 Of these patients, 153 patients had follow-up data including 96 with primary treatment and 57 with secondary treatment. They reported no hearing pregamma knife in 105 patients, including 53 of 96 primary treatment and 52 of 57 secondary treatment patients. The Gardner-Robertson grading system and subjective assessment of hearing was used; however, no pure-tone average or speech discrimination data were reported. Unfortunately, their data set included audiometric data from only 48 patients, and the intervals of audiometric testing were not reported. Despite these limitations, they found that, except for one patient, no change in hearing was observed in the first two years after gamma knife surgery. Their data also showed that the greatest change in Gardner-Robertson grade occurred between years two and four post-gamma knife; however, without understanding the assessment intervals, the precise onset of the hearing loss is unknown. No outcomes regarding balance function were reported. Kim’s group at the Seoul National University reported the hearing outcomes in 25 patients with vestibular schwannomas with serviceable hearing.16 The median tumor volume was 3.0 cm3 (0.16 to 9.1 cm3), and the dose used was 12 ± 0.7 Gy at the 49.8 ± 1.1% isodose line. They reported the hearing outcomes using the Gardner-Robertson grading system, pure-tone averages, and speech discrimination scores. Pre-gamma knife, interim post-gamma knife, and last post-gamma knife data were reported. Similar to our experience, they found that in 16 patients the hearing deteriorated > 20 dB three to six months post-gamma knife and that this hearing loss continued for 24 months. The only prognostic factor for hearing deterioration that they identified was the maximum dose to the cochlear nucleus. In the Medical College of Wisconsin Acoustic Neuroma and Skull Base Surgery Program, we have established a clinical pathway for all of our patients undergoing gamma knife surgery for primary or secondary treatment of their tumors. Pretreatment they undergo a complete videonystagmography test battery, a complete audiologic assessment, and facial nerve electromyography. At sixmonths intervals post-treatment, each patient undergoes a gadolinium enhanced MRI as well as an audiologic test battery and caloric testing to assess peripheral vestibular function. In addition to other standard reporting methods, we have also presented the data in a longitudinal manner for their objective auditory thresholds (Fig. 65-6), speech discrimination ability (Fig. 65-7), and degree of vestibular paresis (Fig. 65-8). We have recently published an expanded cohort of 54 patients with a median follow-up interval of 54.7 months.17 This report focused on the longitudinal outcomes in vestibular function and changes in the Dizziness Handicap Inventory before and after gamma knife surgery. It is clear that most of the change in hearing and balance function occurs during the first six months after gamma knife surgery; however, continued but less rapid
Patient number:
PTA–3 60 50 PTA–3 change (dB) re: pre-op
792
40 30 20 10 0 −10 −20 −30
0
1
2
3
4
5
6
Postoperative time interval (years)
Figure 65-6. Auditory function over time after gamma knife surgery treatment of unilateral vestibular schwannomas. Three-frequency averages of pure-tone thresholds (PTA-3) in dB HL at 0.5, 1, and 2 kHz were determined for all patients with measures at the preoperative time and at least one postoperative interval. The PTA-3 difference was calculated for each time interval relative to the preoperative PTA-3. The differences are plotted as a function of postoperative time interval, with zero representing the preoperative time. A positive difference value indicates a higher or poorer, postoperative PTA-3. In general, over time, the vast majority of patients were found to have PTA-3s that were poorer or similar to preoperative PTA-3s, although a few individuals showed some initial improvement (e.g., subject 5). The greatest changes in PTA-3 were measured at six months post-treatment although continued changes were observed up to five years post-treatment. (Published with permission, copyright © 2008, P. A.Wackym, MD.)
worsening of function can occur up to 12 months. These objective measurements correspond well to the transient facial nerve dysfunction, trigeminal nerve dysfunction, tinnitus, and dysequilibrium occurring in our patients with vestibular schwannomas undergoing gamma knife surgery.3,10,17 A possible mechanism underlying these changes is that there is an initial increased size of the tumor after radiosurgery. Typically this post-treatment edema persists for six months; however, this may remain for up to one year.3,10,17 The labyrinthine artery, a branch of the anterior inferior cerebellar artery, provides essentially all of the blood supply to the cochlea and vestibule and it is likely that the post-radiation edema compromises this blood supply to the inner ear. The resulting inner ear devascularization could certainly explain the rapid change in hearing and balance function seen at the six-month post-treatment assessment in our patients (see Figs. 65-6 and 65-8). Several of our patients have had tumor control or regression and improvement of hearing and vestibular function. This is clearly divergent from the natural history of vestibular schwannomas. In contrast, worsening of auditory and vestibular function and the development of disequilibrium has occurred in a number of our patients. Continued systematic studies of these patients and expansion of the cohort of patients studied are
793
Chapter 65 • Stereotactic Radiosurgery of Skull Base Tumors
40 20 0 −20 −40 −60 −80
0
1
2
3
4
Vestibular Paresis
100 Vestibular paresis change (%) re: pre-op
Speech recognition change (%) re: pre-op
Patient number: Speech Recognition
5
6
Postoperative time interval (years)
Figure 65-7. Speech recognition testing was performed using the Northwestern University Auditory Test No. 6 (NU-6) monosyllabic words. The stimuli were presented at 40 dB sensation level, i.e., above speech recognition threshold, or if this was too loud, at the patient’s most comfortable listening level. Speech recognition was scored in percent correct. As with PTA, the differences between pre- and postoperative speech recognition were calculated and plotted as a function of postoperative time interval. Positive values are consistent with an improvement in speech recognition. Approximately half of the patients showed improvement in speech recognition at six months post-treatment, while the other half showed a decrease in performance. Of those patients who experienced a reduction in speech discrimination ability, there was a greater range of change than that observed in the patients who enjoyed an improvement in speech discrimination ability. It should be noted that the greatest changes in speech discrimination ability occurred at six months post-gamma knife treatment. However, over time, the patients generally demonstrate speech recognition performance similar to or poorer than pre-treatment performance. (Published with permission, copyright © 2008, P. A.Wackym, MD.)
important to determine the efficacy of gamma knife surgery and to compare to other forms of radiotherapy, as well as microsurgery and expectant management. Recognition of symptoms such as disequilibrium and knowledge regarding the expected time course of vestibular paresis progression are important not only for patient counseling but provide the opportunity to intervene with vestibular rehabilitation or nonspecific vestibular suppression until compensation has been completed, should this be needed clinically.17 One final issue to consider is tumor growth after radiosurgery (Fig. 65-9). It is important to appreciate that there is an increased size of the tumor after radiosurgery. In fact, we observed a statistically significant increase in tumor size for patients whose tumors extended outside of the internal auditory canal six months after gamma knife surgery and a statistically significant decrease at one year post-treatment.3,10 Typically, post-treatment edema persists for six months; however, this may remain for up to one year. Consequently pretreatment counseling should include this information. There have been anecdotal cases discussed and occasionally reported that describe increased tumor size early after radiosurgery.
Patient number:
80 60 40 20 0 −20 −40 −60
0
1
2
3
4
5
6
Postoperative time interval (years)
Figure 65-8. Vestibular paresis was determined with bithermal caloric testing. A positive difference value indicates greater vestibular paresis post-gamma knife surgery. Both degradation and improvement in vestibular paresis are observed across patients. Within a patient, the postoperative degree of vestibular paresis generally tends to remain stable over time after the relatively large initial change observed at the six month post-treatment assessment. In those patients who had continued reduction in their vestibular function, there was continued difficulty with dysequilibrium until vestibular compensation was complete and the vestibular paresis stabilized. (Published with permission, copyright © 2008, P. A.Wackym, MD.)
Figure 65-9. Gamma knife surgery. Example of serial magnetic resonance imaging studies of a small left vestibular schwannoma. Note at six and 12 months post-gamma knife radiosurgery the tumor is larger than pretreatment. By 18 months the tumor is smaller. (Reproduced with permission, copyright © 2008, P. A.Wackym, MD.)
The challenge is in making a decision about whether to resect these tumors and when.2,5,18-22 Pollock and colleagues emphasized the need to demonstrate sustained tumor growth by serial MRI before making the decision to operate and also to review the case with the surgeon who performed the radiosurgery before a surgical decision is made.5 Another related controversy is whether facial nerve dissection and preservation are more difficult during microsurgical resection after radiosurgery. On one end of the spectrum, descriptions of no increased difficulty have
794
OTOLOGIC SURGERY
been reported;5 and, on the other end of the spectrum,19-22 markedly increased difficulty in separating the tumor from the facial nerve and poorer facial nerve function outcome have also been reported. The report of Watanabe and colleagues included a histopathologic analysis of the resected facial nerve.20 They found microvasculitis of the facial nerve, axonal degeneration, loss of axons, and proliferation of Schwann cells. In light of the mechanism of delayed effects following radiosurgery, these findings are not surprising. Moreover, these findings emphasize the need for the neurotologist to be certain that the treatment plan avoids high radiation doses to the facial nerve. Recall as described earlier that a dose of 12 Gy delivered to the 50% isodose line means that the maximum tumor dose is 24 Gy. If the treatment plan delivers this maximal dose to the area of the facial nerve, it should be expected that greater radiation effects will be observed. For this reason, if the neurotologist and the patient have made a decision to resect a tumor previously treated with radiosurgery, it is important to review the treatment plan to determine the amount of radiation delivered to the facial nerve to counsel the patient appropriately preoperatively.
Figure 65-10. Ceiling-mounted diagnostic-energy x-ray sources emit low-dose x-rays through the patient’s tumor treatment area. Amorphous silicon image detectors capture x-ray images from ceilingmounted diagnostic-energy x-ray sources to produce live radiographs. The operating system (typically located adjacent to the treatment room) correlates patient location detected by image guidance system with reconstructed CT scan and directs the robot to adjust position accordingly. The compact linear accelerator mounted on a computercontrolled robotic arm which adjusts position to maintain alignment with the target, compensating for any patient movement and uses X-band technology for mobility. (Published with permission, copyright © 2008, Accuray [Accuray Incorporated, Sunnyvale, CA].)
ALTERNATIVE TECHNIQUES As noted earlier, tumor size and location may dictate that a method other than gamma knife be considered. Indeed, alternative methods of radiosurgery are available for treating a wide variety of skull base neoplasms. These include the Peacock (NOMOS Inc., Cranberry Township, PA), the SmartBeam IMRT (Varian Medical Systems Inc., Palo Alto, CA), the Precise (Elekta, Inc., Stockholm, Sweden), and the CyberKnife (Accuray, Sunnyvale, CA). Among the more common of these modalities is CyberKnife, which will be briefly reviewed here.
Cyberknife Stereotactic Radiology Overview of Treatment Planning The CyberKnife stereotactic radiosurgery system utilizes a compact 6-MeV linear accelerator, a computercontrolled robotic arm with six degrees of freedom, and an image-guidance technology that does not depend on a rigid stereotactic frame and thereby enables treatment of extracranial sites (Fig. 65-10). Potential benefits of this approach include: 1) increased access to and coverage of any target volume including the ability to treat lesions in and around the cranium that are unreachable with other systems, for example, in the lower posterior fossa and foramen magnum; 2) enhanced ability to avoid critical structures; 3) capability to treat lesions in the neck and spine; 4) ability to treat lesions throughout the body; 5) delivery of highly conformal dose distributions; 6) option of fractionating treatment; and 7) potential to target
multiple tumors at different locations during a single treatment, e.g., skull base and neck. The CyberKnife treatment planning system is designed to support the radiosurgery team in determining the optimal plan, including beam weight, targeting positions, dose distributions, and other factors for each patient’s treatment. The CyberKnife stereotactic radiosurgery system permits the following planning and delivery options: 1) inverse planning; 2) nonisocentric delivery; and 3) hypofractionation. In contrast to most gamma knife procedures, CyberKnife is CT based. MR images can be fused with the CT to provide optimal information on soft tissue as well as skeletal anatomy. CT angiography can be used when vascular skull base lesions such as arteriovenous malformations or extensive glomus jugulare tumors are to be treated with this technique. The flexibility of the robotic arm supporting the linear accelerator allows the CyberKnife to implement a wider range of treatment plans than other systems. Furthermore, because the system does not require the use of a stereotactic head frame temporarily attached to the patient’s head, it allows scanning, treatment planning, and quality assurance to take place at any time prior to treatment itself. The CyberKnife system provides a range of treatment options, including the ability to use either forward or inverse treatment planning. With forward treatment planning, the radiation oncologist determines what dose to deliver from a particular targeting position. The total dose within the lesion is then calculated by the
Chapter 65 • Stereotactic Radiosurgery of Skull Base Tumors
system software. With inverse treatment planning, the radiation oncologist specifies total dose to be delivered to the tumor. The surgeon and radiation oncologist are then able to set boundaries to protect adjacent critical structures. The software subsequently determines targeting positions and the dose to be delivered from each targeting position. While other stereotactic radiosurgery systems offer the inverse planning option, the number of possible plans is limited by the constraints of the delivery system.
Dose Distribution The CyberKnife system offers a choice of a nonisocentric or an isocentric treatment approach. With other stereotactic radiosurgery systems, a fixed calculated isocenter is used. Isocentric treatment, or multi-isocentric treatment, involves filling the lesion with a single or multiple, overlapping spherically shaped dose distributions. Isocentric treatment is effective for spherical lesions. However, with irregularly shaped lesions, isocentric delivery can produce significant dose heterogeneity. In this case the surgeon and radiation oncologist must account for the relationship of the maximum dose to critical structures such as the facial nerve or cochlea. Similarly, they must identify regions which may be under-treated by delivery of inadequate doses. Nonisocentric treatment plans are also possible with the CyberKnife system. The delivery of these treatment plans is possible because of the robotic arm which, because of the six degrees of freedom (discussed later) enables the delivery of radiation to complex treatment volumes. The beams originate from arbitrary points in the workspace and are delivered into the lesion. The result is a nonisocentric concentration of beams within the lesion and asymmetric irradiation. Nonisocentric treatment allows the avoidance of critical structures while providing complete coverage of the lesion at the prescribed isodose. With the CyberKnife system, the treatment plan can utilize fractionated or hypofractionated approaches. Fractionated treatment is possible because localization of the lesion is achieved using image guidance technology. Dose delivery over two to five treatment sessions, termed hypofractionation, is another option with the CyberKnife system. Although not directly applicable in managing tumors within the posterior fossa, it has been suggested to be particularly useful in the treatment of large tumors. The argument for fractionation is that lowering the dose for each of a number of treatments, as opposed to a single, larger dose, allows healthy tissue to rejuvenate between treatments. The advantage of fractionated or a single radiation dose remains an active area of investigation and debate. Because of the rigid fixation that occurs with securing the stereotacic headframe in gamma knife surgery, fractionated or hypofractionated delivery of radiation is not possible. Furthermore,
795
it remains to be determined if equal accuracy can be achieved by these two systems or if there is an advantage of fractionation or hypofractionation in the treatment of skull base tumors.
Localization The CyberKnife system’s use of stereotactic principles for tumor localization differs from other stereotactic radiosurgery systems by using an image guidance technology that depends on the skeletal structure of the body as a reference frame. In addition, it continually monitors and tracks patient position during treatment. The CyberKnife’s operating system correlates live radiographic images with preoperative CT scans to determine patient and tumor position repeatedly over the course of treatment. The imaging information is transferred from the computer’s operating system to the robot so that it may compensate for any changes in patient position by repositioning the LINAC.
Treatment Delivery The CyberKnife system’s computer-controlled robotic arm has six degrees of freedom. The robot can position the LINAC to more than 100 specific locations or nodes. Each node has 12 possible approach angles, translating to over 1200 possible beam positions. The treatment planning system determines a set sequence of approach angles, beam weights, and dose distributions. The calculated plan can be incrementally improved by the physicist and physicians. The actual delivery follows a step-and-shoot sequence. The patient is placed in a position approximating that of the CT scan. Image detectors acquire radiographs of the tumor region. The image guidance system software then compares the real time radiographs with the CT information to determine location of the tumor. This information is transmitted to the robot to initialize the pointing of the LINAC beam. The robotic arm then moves the LINAC through the sequence of preset nodes surrounding the patient. At each node, the LINAC stops, and a new pair of images is acquired from which the position is determined again. Corrected position is transmitted to the robot which adapts beam pointing to compensate for any movement. LINAC delivers the preplanned dose of radiation for that position. The entire process is repeated at each node. The total time from imaging to robot compensation is about seven to 10 seconds. The total treatment time depends on the complexity of the plan and delivery paths but is comparable to standard LINAC treatments. Each treatment session ranges from 30 to 90 minutes. Physicians may elect to treat with a single dose, a hypofractionated dose typically of two to five sessions, or a more traditional fractionated regimen. Outcomes following CyberKnife treatment of vestibular schwannomas are emerging at this time.23
796
OTOLOGIC SURGERY
ALTERNATIVE APPLICATIONS Although the majority of stereotactic radiosurgery performed by the neurotologist will be for vestibular schwannoma, other neoplasms and pathologies may be amenable to radiotherapy.24 Several of these were noted earlier in the section on patient selection and this section will focus on a few common pathologies for which the neurotologist may be the primary surgeon. Paragangliomas, more specifically glomus jugulare tumors, are becoming more commonly addressed with primary radiotherapy than with surgical resection. The other chemodectomas such as glomus vagale and carotid body tumors are located too low in the neck for most gamma knife units in use today. Further, these tumors do not typically carry the same morbidity as glomus jugulare tumors and are still commonly addressed surgically. In an effort to reduce the cranial nerve palsies that often accompany glomus jugulare resection, gamma knife surgery has been employed. An evaluation of 42 patients with primary or recurrent/persistent glomus jugulare tumor undergoing gamma knife surgery showed excellent tumor response.25 Approximately 1/3 of tumors shrank and 2/3 showed no size change. A single 3.9 cm tumor was found to have increased 99 months after treatment with 12 Gy at the margin and was re-treated. Progressionfree survival was 100% at seven years and 75% at 10 years. Six patients had complications related to treatment. Five of 26 patients with intact hearing at the time of treatment had subjective decline within the first year. Objective measures of hearing were not performed. One patient had facial parasthesias, one had vocal fold paralysis (the re-treated subject), one had vertigo and imbalance, and one had post-treatment migraine requiring admission. A meta-analysis of glomus jugulare treatment and outcomes compared stereotactic radiosurgery to surgical resection.26 Neurological deficits in those treated with gamma knife, CyberKnife or LINAC showed no change in 58.2%, improved in 39% and permanently worsened in 2.8%. Such deficits included complaints of hearing loss, dizziness, dysphagia, voice change, shoulder dysfunction and headache. Overall, there was an 8.5% incidence of cranial nerve complication with 75% of these being transient. Permanent deficits occurred in three of 141 patients all of which involved facial motor dysfunction; none of which reached House-Brackmann grade VI. Tumor control was achieved in approximately 98% of individuals at 39 months. Conventional surgery for glomus jugulare had a complete resection rate of approximately 92%; some of which represented more than one surgery for resection. The recurrence rate at a mean of 82 months was 3.3%. The mortality rate was 1.3% for conventional surgery compared with 0% for radiotherapy. Cranial nerve deficits varied widely among surgical reports but on
a verage the facial nerve was affected in 4.4% to 11%; the glossopharyngeal in 26% to 42%; the vagus in 13% to 28%; the spinal accessory in 25% to 26% and the hypoglossal in 5% to 21%. Other morbidities included a CSF leakage rate of 8.3%, aspiration in 5.5% and wound infection in 5.5%. Although cranial nerve deficits occur more frequently with conventional surgery, most reports note that the long-term impact of such dysfunction is relatively small. It is important for the surgeon to take into account patient function, age and general health, and tumor size when discussing and weighing treatment options for glomus jugulare tumors. Meningiomas are the second most common benign neoplasm of the cerebellopontine angle and can often present with deficits similar to vestibular schwannomas. Total resection results in excellent tumor control rates and, for all cranial locations, shows a 15-year progression-free survival rate of approximately 68% to 75%.27,28 Experience with partially resected or inoperable meningiomas, however, has shown that radiation therapy can produce excellent tumor control in the majority of cases. The use of stereotactic radiosurgery as a primary treatment to avoid or reduce the incidence of surgical and neurological deficits is increasingly common. Elia and colleagues reviewed stereotactic radiosurgery outcomes for meningioma published since 2001.28 In over 1500 patients the 5-year progression-free control rate was 93.4%. The complication rate ranged from 2.5% to 13% and included neurological and vascular toxicities. Many of these were for tumors around the optic chiasm and carotid arteries and included dosages up to 20 Gy. Kreil and colleagues recently published their series on the treatment of 200 skull base meningiomas with gamma knife surgery.29 There were 21 patients with cerebellopontine angle lesions. Of 20 patients with preoperative hearing loss (not quantified), one improved and 19 remained stable; none showed deterioration. Tinnitus remained stable in seven of seven patients. Vertigo was present in 25 skull base meningiomas and improved in eight and worsened in none. Given the low incidence of complication and the high rate of tumor control, stereotactic radiosurgery should be strongly considered in tumors around sensitive neural structures and in patients medically unsuitable for conventional surgery. In addition to tumors, the neurotologist is often consulted for facial pain syndromes, most notably trigeminal neuralgia. Functional stereotactic radiosurgery using gamma knife has been employed in the treatment of trigeminal neuralgia. In the series of meningiomas reported by Kriel there were 25 patients with preoperative trigeminal neuralgia due to tumor of which 16 improved.29 There were two induced cases of trigeminal neuralgia but these were transient. Such findings indicate that radiation to the trigeminal nerve can induce functional changes. Gorgulho and De Salles reviewed surgical and stereotactic treatments for trigeminal neuralgia.30 Among
Chapter 65 • Stereotactic Radiosurgery of Skull Base Tumors
current treatments, long-term improvement was noted in 70% to 75% of microvascular decompressions, 58% to 77% of radiofrequency rhizotomies, 32% of balloon compressions, 17% to 50% of glycerol rhizotomies, and 45% to 57% of stereotactic radiosurgeries. Immediate improvement was noted in over 90% of patients with stereotactic radiosurgery. Recurrence rates were highest with glycerol rhizotomy and much lower and very similar among the other modalities. Stereotactic radiosurgery was noted to be particularly attractive because it is the least invasive of these methods. Many different treatment protocols for trigeminal neuralgia have been attempted.30 In their review, Gorgulho and De Salles identified several patterns with regards to gamma knife treatment for trigeminal neuralgia that affect outcomes. The root entry zone of the trigeminal nerve, not the nerve proper, should be the preferred target as dosage delivery to this area seems to correlate with pain relief. A minimal dosage of 70 Gy and maximal dosage of 90 Gy should be prescribed. The incidence of post-treatment numbness with this prescription dose ranges from 3% to 55% but bothersome numbness persists in only about 4% to 12%. Treating a longer section of the trigeminal nerve proper does not improve pain control and increases the incidence of post-treatment numbness. Likewise, higher dosage to the nerve does not improve pain control and increases numbness. The overall incidence of complications with stereotactic radiosurgery for trigeminal neuralgia is significantly lower than all other techniques. As with other benign diseases, potential long-term effects of radiation treatment need to be considered in younger individuals.
SUMMARY Stereotactic radiosurgery and radiotherapy are becoming increasingly common in the management of skull base tumors and other disorders. Whether driven by the patient, or the surgeon, the field continues to evolve rapidly. Advances are being made in improving accuracy, effective radiation dose, and parameters necessary to maximize patient outcome. These methods have advantages and disadvantages that must be openly discussed with patients who have vestibular schwannomas or other skull base tumors. It remains the responsibility of the surgeon to provide a balanced view as to the relative risks and benefits of observation, microsurgery, stereotactic radiosurgery or radiotherapy, or a combination of these methods.
FINANCIAL DISCLOSURE None of the authors has a financial interest in any of the companies discussed in this chapter.
797
REFERENCES 1. Kondziolka D, Lunsford L D, McLaughlin M R , Flickinger JC : Long-term outcomes after radiosurgery for acoustic neuroma. N Engl J Med 339:1426-33, 1998. 2. Wackym PA: Stereotactic radiosurgery, microsurgery, and expectant management of acoustic neuroma: basis of informed consent. Otolaryngol Clin North Am 38:653-70, 2005. 3. Wackym PA, Runge-Samuelson C L , Poetker D M, et al: Gamma knife radiosurgery for acoustic neuromas performed by a neurotologist: early experiences and outcomes. Otol Neurol 25:752-61, 2004. 4. Bari M E, Forster D M, Kemeny A A, et al: Malignancy in a vestibular schwannoma. Report of a case with central neurofibromatosis, treated by both stereotactic radiosurgery and surgical excision, with a review of the literature. Br J Neurosurg 16:284-9, 2002. 5. Pollock B E, Lunsford L D, Kondziolka D, et al: Vestibular schwannoma management. Part II. Failed radiosurgery and the role of delayed microsurgery. J Neurosurg 89:949-55, 1998. 6. Rowe J, Grainger A, Walton L , et al: Risk of malignancy after gamma knife stereotactic radiosurgery. Neurosurgery 60:60-5, 2007. 7. Rowe J, Grainger A, Walton L , et al: Safety of radiosurgery applied to conditions with abnormal tumor suppressor genes. Neurosurgery 60:860-4, 2007. 8. Lustig L R , Jackler R K, Lanser M J: Radiation-induced tumors of the temporal bone. Am J Otol 18:230-5, 1997. 9. Hanabusa K, Morikawa A, Murata T, Taki W: Acoustic neuroma with malignant transformation. Case report. J Neurosurg 95:518-21, 2001. 10. Poetker D M, Jursinic PA, Runge-Samuelson C L , Wackym PA : Distortion of magnetic resonance images used in gamma knife radiosurgery treatment planning: implications for acoustic neuroma outcomes. Otol Neurotol 26:1220-8, 2005. 11. Paddick I : A simple scoring ration to index the conformity of radiosurgical treatment plans. Technical note. J Neurosurg 93(Suppl 3):219-22, 2000. 12. Lunsford L D, Niranjan A, Flickinger JC, et al: Radiosurgery of vestibular schwannomas: summary of experience in 829 cases. J Neurosurg 102(Suppl):195-9, 2005. 13. Niranjan A, Lunsford L D, Flickinger JC, et al: Dose reduction improves hearing preservation rates after intracanalicular acoustic tumor radiosurgery. Neurosurgery 45:753-62, 1999. 14. Flickinger JC, Kondziolka D, Niranjan A, Lunsford L D: Results of acoustic neuroma radiosurgery: an analysis of 5 years’ experience using current methods. J Neurosurg 94:1-6, 2001. 15. Prasad D, Steiner M, Steiner L : Gamma surgery for vestibular schwannoma. J Neurosurg 92:745-59, 2000. 16. Paek S H, Chung H -T, Jeong S S, et al: Hearing preservation after gamma knife radiosurgery of vestibular schwannoma. Cancer 104:580-90, 2005. 17. Wackym PA, Hannley MT, Runge-Samuelson C L , et al: Gamma knife surgery of vestibular schwannomas: Longitudinal changes in vestibular function and measurement of the Dizziness Handicap Inventory. J Neurosurg 109(Suppl):137-143.
798
OTOLOGIC SURGERY
18. Pitts L A, Jackler R K : Treatment of acoustic neuromas. N Engl J Med 339:1471-73, 1998. 19. Ho SY, Kveton J F: Rapid growth of acoustic neuromas after stereotactic radiotherapy in type 2 neurofibromatosis. Ear Nose Throat J 81:831-3, 2002. 20. Watanabe T, Saito N, Hirato J, et al: Facial neuropathy due to axonal degeneration and microvasculitis following gamma knife surgery for vestibular schwannoma: a histological analysis. J Neurosurg 99:916-20, 2003. 21. Lee DJ, Westra WH, Staecker H, et al: Clinical and histopathologic features of recurrent vestibular schwannoma (acoustic neuroma) after stereotactic radiosurgery. Otol Neurol 24:650-60, 2003. 22. Friedman R A, Brackmann D E, Hitselberger WE, et al: Surgical salvage after failed irradiation for vestibular schwannoma. Laryngoscope 115:1827-32, 2005. 23. Chang S D, Gibbs IC, Sakamoto GT, et al: Staged stereotactic irradiation for acoustic neuroma. Neurosurgery 56:1254-61, 2005. 24. Knisely J PS, Linskey M E : Less common indications for stereotactic radiosurgery or fractionated radiotherapy for patients with benign brain tumors. Neurosurg Clin N Am 17:149-167, 2006.
25. Pollock B E : Stereotactic radiosurgery in patients with glomus jugulare tumors. Neurosurg Focus 17:63-67, 2004. 26. Gottfried O N, Liu J K, Couldwell WT: Comparison of radiosurgery and conventional surgery for the treatment of glomus jugulare tumors. Neurosurg Focus 17:22-30, 2004. 27. Goldsmith B, McDermott MW: Meningioma. Neurosurg Clin N Am 17:111-120, 2006. 28. Elia A E, Shih H A, Loeffler J S : Stereotactic radiation treatment for benign meningiomas. Neurosurg Focus 23:1-9, 2007. 29. Kreil W, Luggin J, Fuchs I, et al: Long term experience of gamma knife radiosurgery for benign skull base meningiomas. J Neurol Neurosurg Psychiatry 76:1425-1430, 2005. 30. Gorgulho A A, De Salles A A F: Impact of radiosurgery on the surgical treatment of trigeminal neuralgia 66:350356, 2006.
Self-Assessment Questions Q1. Which of the following are true regarding gamma knife surgery for vestibular schwannomas? A. ��������������������������������������������� Tumor control rates are greater than 97% B. Facial nerve motor dysfunction occurs in less than 1% with current dosing C. Trigeminal nerve dysfunction is more common with large tumors D. Malignant transformation or induction is rare E. All of the above Q2. Hearing thresholds as measured by pure-tone averages after gamma knife surgery most commonly: A. Improve immediately B. Behave similar to expectant observation C. Progress rapidly to profound deafness D. Degrade rapidly in the first six months and then slowly worsen E. Do not change Q3. Which of the following tumors would not be amenable to treatment with the more common gamma knife B and C units? A. Intracanalicular acoustic neuroma of 7 mm maximal diameter B: V estibular schwannoma extending into the CPA by 1.5 cm C. 2 cm glomus jugulare tumor extending anterosuperiorly from the jugular bulb D. Glomus vagale tumor extending to the carotid bifurcation E. Petrous apex meningioma of 2.4 cm
Q4. A patient underwent gamma knife surgery of a 2.3 cm CPA vestibular schwannoma six months ago. MRI performed today shows the tumor to be 2.7 cm in maximal diameter. The patient is asymptomatic. The next best course of action is: A. Assure patient this is normal and rescan in six months B. Recommend microsurgical resection for radiation failure C. Counsel patient this may be malignant D. Plan a second round of gamma knife surgery E. Start high-dose steroids with a taper Q5. Which of the following is/are not a component of the gamma knife surgery system? A. Trunnions B. Collimator helmet C. Gamma calipers D. Cobalt 60 (60Co) sources and beam channels E. MRI fiducial box
66
Vascular Considerations in Neurotologic Surgery Robert F. Spetzler, Shaun C. Desai, Vivek R. Deshmukh, and Shervin R. Dashti
The foundation for management of vascular lesions of the petrous bone and cerebellopontine angle (CPA) relies on an understanding of the relevant skull base anatomy and vascular anatomy of this region. Vascular lesions are quite varied and include carotid dissections; carotid and vertebrobasilar aneurysms; arteriovenous fistulas; cavernous malformations; and hypervascular tumors including glomus tumors, hemangiomas, and hemangioblastomas, among others. This chapter presents a concise and clinically relevant review of the anatomy and the diagnostic and therapeutic strategies for these complex lesions.
VASCULAR ANATOMY OF THE PETROUS BONE AND CEREBELLOPONTINE ANGLE Internal Carotid Artery The petrous segment of the internal carotid artery (ICA) begins as the vessel enters the periosteal-lined carotid canal, whereas the cavernous segment originates as it exits the apex of the petrous portion of the temporal bone and traverses the posterior part of the cavernous sinus. The petrous portion of the ICA, which lies immediately posterior to the bony eustachian tube but anterior to the jugular foramen, can be divided into vertical and horizontal segments that connect at the genu. The ICA is lined by cervical sympathetic ganglia throughout its course in the petrous canal, and aneurysms or lesions of the vessel that compress the nerves can cause a classic Horner’s syndrome.1 The ICA is also lined by a venous plexus in the distal part of the canal. The greater superficial petrosal nerve, which originates from the geniculate ganglion of the facial nerve, usually courses above and parallel to the horizontal portion of the petrous ICA. Special caution should be used when exposing the middle fossa floor and the petrous ICA because excessive traction of this nerve can cause postoperative facial nerve palsy.2 Similarly, the trigeminal ganglion lies over the medial portion of the petrous ICA, but it is usually separated from the vessel by dura or a variable layer of bone. The petrous ICA has four main branches of importance to the neurotologic surgeon: (1) caroticotympanic
artery, (2) stapedial artery, (3) artery of the pterygoid canal (vidian), and (4) periosteal artery.1,3 The caroticotympanic artery is often said to be a remnant of the embryonic hyoid artery, and arises from the proximal vertical part of the petrous ICA laterally and supplies the tympanic cavity. It anastomoses most commonly with the inferior tympanic artery. The stapedial artery is the source of the middle meningeal artery during development, but it rarely persists into adulthood. If present, a persistent stapedial artery passes through the floor of the middle ear and runs superiorly in a bony canal, in the obturator foramen of the stapes, and finally through the fal lopian canal into the middle cranial fossa.3 Finally, the artery of the pterygoid canal (vidian) and the periosteal artery are often thought of as small collateral routes most likely branching off from the horizontal segment of the petrous portion or from the internal maxillary artery, and may have relevance in interventional embolization procedures as described later on.1 The intracavernous segment of the ICA begins at the level of the petrolingual ligament near the foramen lacerum, and most consistently gives off two branches known as the dorsal (meningohypophyseal trunk) and lateral (artery of the inferior cavernous sinus) main stem arteries.4 The intracavernous ICA is commonly divided into five segments for anatomic orientation, including a posterior vertical, posterior bend, horizontal, anterior bend, and anterior vertical segments.1 The dorsal main stem artery generally branches from the central one third of the convex outer margin of the posterior bend of the ICA. There is great variability in the origin of the branches of the dorsal main stem, but most investigators agree that three vessels—the tentorial artery (artery of Bernasconi-Cassinari), dorsal meningeal artery, and inferior hypophyseal artery—exist in some form.3 The tentorial arteries travel along the tentorium and may contribute blood supply for tentorial or proximal falcine meningiomas and tentorial dural arteriovenous fistulas, and blood supply to portions of CN III and IV.3,5 The inferior hypophyseal artery supplies the periphery of the anterior pituitary gland and forms a “circulus arteriosus” with the dorsal meningeal arteries around the root of the 799
800
OTOLOGIC SURGERY
dorsum sella. The distal branches of the dorsal meningeal arteries may enter the internal auditory meatus as described in a few patients, and may represent a remnant of the fetal trigeminal artery.3 The second main branch, the artery of the inferior cavernous sinus, generally arises from the central one third of the horizontal segment of the cavernous ICA. Although there is some variability, most studies agree that this vessel supplies various nervous components of the cavernous sinus. Although the dorsal and lateral main stem vessels are the most common branches given off by the intracavernous segment of the ICA, other arteries have been described. McConnell’s capsular artery, when present in about 8% of the population, originates from the horizontal segment of the intracavernous ICA, and supplies any combination of the inferior and peripheral aspect of the anterior lobe of the pituitary gland, the diaphragma sella, and the floor of the sella turcica.6 A persistent fetal trigeminal artery, although rare with an incidence of 0.06% to 0.6%, usually arises from the posterolateral or posteromedial aspect of the intracavernous ICA.3 It is most notable in the literature for producing unique clinical scenarios, including trigeminal neuralgia, oculomotor palsies, hyperprolactinemia, and an increased incidence of intracranial aneurysms.3 Finally, the ophthalmic artery and a superior hypophyseal artery have been shown to originate from the cavernous ICA in 1% to 7.5% and 16% of individuals, respectively.3,7
posterior auricular artery arises above the posterior belly of the digastric and travels between the parotid gland and the styloid process. The artery divides into posterior auricular and occipital branches, and develops a stylomastoid branch, which supplies the structures of the stylomastoid foramen and the facial nerve.2 Although not technically a direct branch of the external carotid artery, but rather a branch of the internal maxillary artery, the middle meningeal artery has clinical relevance in the various neurotologic approaches. As this vessel courses through the foramen spinosum of the sphenoid bone between the two roots of the auriculotemporal nerve, it enters the middle cranial fossa, dividing into two or three main branches. When performing extradural approaches to the floor of the middle fossa, the foramen spinosum is located anteromedial to the geniculate ganglion and anterolateral to the carotid canal. These branches—the anterior branch (frontalis), posterior branch (parietal), and middle branch—supply most of the supratentorial dura and calvaria. On entering the cranium, it also gives off the superficial petrosal artery, which enters the hiatus of the facial canal and supplies the facial nerve. Manipulation of this branch during transpetrosal approaches can result in facial nerve injury.2 Finally, the middle meningeal artery also gives rise to the superior tympanic artery just superior to the foramen spinosum, which runs with the superficial petrosal nerve through the superior tympanic canaliculus and supplies the canal of the tensor tympani muscle.
External Carotid Artery
Vertebrobasilar Arteries
Proximal to its terminal bifurcation, the external carotid artery gives rise to an anterior and posterior group of vessels, the latter of which consists of three arteries that relate to the petrous portion of the temporal bone. The ascending pharyngeal artery usually originates at or near the main bifurcation of the ICA and external carotid artery, and supplies the meninges around the jugular foramen as it passes through the foramen lacerum. As it travels upward with the carotid arteries, it gives off the inferior tympanic artery, which travels through the tympanic cavity with Jacobson’s nerve (inferior tympanic) via the tympanic canaliculus.2 The occipital artery arises from the posterior surface of the external carotid artery and traverses upward between the posterior belly of the digastric muscle and the internal jugular vein, and then medial to the mastoid process. After passing the longissimus capitis muscle, the vessel courses deep to the splenius capitis muscle, finally terminating at the fascia between the attachment of the sternocleidomastoid and the trapezius muscles at the superior nuchal line.2 Its branches supply several muscular and meningeal branches and often anastomose with branches of the external carotid and vertebral arteries.2 Its most notable branch is the mastoid artery, which supplies the posterior portion of the mastoid bone and runs through the mastoid foramen. Finally, the
The vertebrobasilar system has three main branches that pass through the CPA, and any damage to these delicate branches can lead to various brainstem stroke syndromes. The major arterial vessels to consider include the posterior inferior cerebellar artery (PICA), the superior cere bellar artery (SCA), and, most relevant to the CPA, the anterior inferior cerebellar artery (AICA). The AICA typically originates from the lower part of the basilar artery and divides into rostral and caudal trunks as it traverses the central part of the CPA. This main bifurcation often occurs proximal to the facial and vestibulocochlear nerves, and the two trunks formed are usually nerve-related.8 These branches can be divided into a premeatal segment, meatal segment, and subarcuate loop. In one study, a laterally convex curve or “loop” from the meatal segment was found to be medial of the internal acoustic meatus lying in the CPA in 33%, at the entrance of the meatus in 27%, and entering the internal auditory canal (IAC) in 40% of patients.8 A second laterally convex curve, known as a subarcuate loop, can also be seen in the subarcuate fossa and often gives the appearance of an M configuration.8 Although rare, in a few cases the lateral convex loop of the AICA can be embedded in the dura or bone covering the subarcuate fossa, or both.9 In such cases, freeing the artery from the dura and, if
Chapter 66 • Vascular Considerations in Neurotologic Surgery
801
ecessary, drilling the bone around the margin may aid n in freeing this segment in anticipation for exposure of the posterior wall of the internal acoustic meatus.9 In the area of the CPA, the various branches of the AICA give rise to several nerve-related arterial branches from which originate the (1) labyrinthine (IAC), (2) recurrent perforating, (3) subarcuate, and, less commonly, (4) cerebellosubarcuate arteries. The labyrinthine arteries follow the vestibulocochlear nerve into the IAC, supplying the nerves, dura, and bone of the canal, and eventually terminate by giving rise to the anterior vestibular and common cochlear arteries, which supply the inner ear. Although the labyrinthine arteries most commonly arise from the premeatal segment of the AICA, some studies have reported anomalous cases where it originated directly from the basilar artery or from the PICA, or recurrent perforating, subarcuate, and cerebellosubarcuate arteries; however, this difference may be partially explained by the exact anatomic definition of the arteries in the different studies.8 The recurrent perforating arteries often course near the meatus and along the facial and vestibulocochlear nerves before supplying the brainstem. Finally, the subarcuate artery usually travels medial to the meatus, traversing the subarcuate fossa and petromastoid canal, sending branches to the petrous apex, the bone of the semicircular canals, and the posterosuperior portion of the vestibule. The PICA, by definition, originates from the vertebral arteries and usually bifurcates into a medial and lateral trunk at the level of the telovelotonsillar fissure; however, its branches are extremely variable. Because it is located inferiorly, it passes very close to the roots of the lower cranial nerves. The PICA can arise from outside the dura, and at any point from the intradural course of the vertebral artery. The SCA originates most commonly from the basilar artery below, but adjacent to the origin of the posterior cerebral artery. The SCA generally bifurcates into rostral and caudal trunks, and branches from either make up the cortical arteries, which supply the upper two thirds of the petrosal surface, including both lips of the petrosal fissure.8 The first branch of the cortical arteries, when present in about half of the population, is referred to as a marginal branch, and its surface area supplied is inversely related to the supply of the AICA. Several anastomoses exist between the marginal artery and the AICA.
extent of the sigmoid sinus. In addition to draining the petrosal veins that drain the cerebellum and brainstem, it also receives vessels from the inferior surface of the temporal lobe and the cavernous sinus. The transverse sinus begins as a confluence of sinuses at the level of the internal occipital protuberance between the tentorium cerebelli and occipitalis bone and runs laterally to join the sigmoid sinus. The anastomotic vein of Labbé, which drains the temporoparietal region, bridges the inferior surface of the temporal lobe to the transverse sinus. This vein is of special importance in petrosal approaches to the skull base because its injury can lead to speech disturbance and contralateral hemiparesis or hemiplegia.2 This vein is particularly at risk during subtemporal exposures. Excessive retraction of the temporal lobe can result in avulsion of the vein from its insertion into the relatively nonmobile transverse-sigmoid junction. The sigmoid sinus not only drains the superior petrosal sinus, but also receives tributaries from the brainstem, cerebellum, and occipitalis, and vertebral emissary veins. The angle formed between the superior petrosal and sigmoid sinuses and the middle fossa dura represents the sinodural angle, which is an important anatomic landmark. The sinus curves medially and forward, crossing the occipital bone, and finally through the jugular foramen where it enters the superior bulb of the internal jugular vein. Finally, the inferior petrosal sinus connects the cavernous sinus to the medial wall of the bulb of the internal jugular vein as it traverses the petro-occipital fissure and drains the clival area. At its entrance to the jugular foramen, the jugular vein is separated from the inferior petrosal sinus by CN IX, X, and XI. This foramen is located at the lower end of the petro-occipital fissure, and is divided into a larger lateral opening that drains the sigmoid sinus and into a small medial petrosal part that receives the inferior petrosal sinus.2 The jugular bulb is inferior to the posterior floor of the middle ear cavity and generally lies inferior to the ampulla of the posterior semicircular canal; however, its location within the tympanic cavity is highly variable. Often, a high-riding jugular bulb can extend superiorly as high as the lateral semicircular canal and can interfere with visualization in Trautmann’s triangle in transpetrosal approaches as discussed in the next section.2 Finally, the internal jugular vein runs inferiorly from the bulb and is located posterolaterally to the ICA.
Dural Venous Sinuses
VASCULAR CONSIDERATIONS IN NEUROTOLOGIC APPROACHES
One must have an intimate knowledge of venous anatomy in neurotologic surgery to decrease the risk of complications from venous congestion. The petrosal vein (Dandy’s vein) parallels the trigeminal nerve just beneath the tentorium and drains into the superior petrosal sinus. The superior petrosal sinus, which lies at the attachment of the tentorium cerebelli to the superior margin of the petrous ridge, connects the cavernous sinus to the lateral
Transpetrosal Approaches The transpetrosal approach, also known as the transtemporal approach, consists of three craniotomy techniques that open the posterior fossa dura anterior to the sigmoid sinus, through the posterior aspect of the petrous pyramid (Fig. 66-1). This set of approaches requires removal of
802
OTOLOGIC SURGERY
gauze. Intraluminal packing of the sigmoid sinus is not recommended because there is always a risk for embolization of the packing material. Transcochlear Translabyrinthine Retrolabyrinthine
FIGURE 66-1. Surgical corridor afforded by the various transpetrosal approaches. (Courtesy of Barrow Neurological Institute.)
1 to 2 cm of retrosigmoid bone to allow for posterior displacement of the sigmoid sinus. A detailed explanation of the surgical technicalities and indications and exposure to the various approaches is provided elsewhere.
Retrolabyrinthine In this approach, bone is removed up to the bony capsule of the semicircular canals, which provides a limited exposure to the midportion of the CPA. Although this approach was first described as a root entry zone in tic douloureux, it has more recently been used for much less common problems, including vascular decompression of CN VIII for tinnitus and chronic vertigo, and for approaching midbasilar artery aneurysms. More commonly, this approach is used in combination with others, including a subtemporal exposure for transtentorial lesions abutting the brainstem. After the initial incision is made behind the postauricular sulcus, and the mastoid air system is thoroughly exenterated, the posterior limit for drilling is the sigmoid sinus. Using a large diamond burr, the sigmoid sinus must be skeletonized carefully. Some authors have suggested that a thin bone plate be left intact over the anterior aspect of the sinus to protect the vessel from the rotating shaft of the drill as it goes deeper into the temporal bone.2 If this bone plate begins to restrict sigmoid retraction, however, one can always fragment it to facilitate posterior mobilization. Other surgeons believe there is no need for preservation of this bony island, and they remove it and guard the vessel with a retractor. Either way, further exposure must be attained by retracting the sinus superiorly from the jugular bulb. In selected patients, the sigmoid sinus can be ligated to improve exposure; however, this can occur only when angiography shows a patent communication between the two transverse sinuses. Mastoid emissary veins are usually encountered and can be controlled with bipolar cautery; similarly, any lacerations to the sinus can readily be controlled with extraluminal tamponade with hemostatic
Translabyrinthine In the translabyrinthine approach, the semicircular canals and vestibule are completely removed, which allows for excellent access to the IAC and exposure of the CPA. The main indication for this approach is for acoustic neuromas; it also is indicated for meningiomas, epidermoids, and other tumors of the CPA when hearing has been compromised. This approach has also been described for aneurysms of the midportion of the basilar artery. In a translabyrinthine approach, the same precautions must be taken for the sigmoid sinus as discussed earlier for the retrolabyrinthine approach. Sigmoid mobilization must be achieved, however, to have adequate access between the sinus and the external auditory canal. Although there is some anatomic variation to the sigmoid sinus, full exposure can usually be achieved after decompression from the transverse-sigmoid junction superiorly to the jugular bulb inferiorly. A high anterior sigmoid course or a large sinus may inhibit access to the inferior CPA; however, this should not be a problem with acoustic neuroma removal because this neoplasm becomes more accessible in the operative field as it is debulked. Finally, when drilling the mastoid bone toward the semicircular canals, the superior petrosal sinus should be exposed at the sinodural angle. During the labyrinthectomy portion of the procedure, as the lateral and superior semicircular canals are drilled away, bleeding may occur. This is most likely a result of the subarcuate artery because it courses in the bone near the center of the superior semicircular canal. The boundaries of Trautmann’s triangle, the patch of posterior fossa dura in front of the sigmoid sinus, are also important. One limb of the V-shaped incision extends below to the superior petrosal sinus, and the other limb extends above the jugular bulb. The jugular bulb sometimes can lie atypically high behind the posterior semicircular canal and impede the creation of the inferior bony trough, interfering with access to the IAC. This variation can increase the chance of unintentional bleeding and create a potential source for air emboli. If this occurs, better exposure of the superior aspect of the IAC can be achieved by elevating the temporal dura and extending the superior trough to the level of the tentorium. Decompression and inferior displacement of the jugular bulb can always be done, however, if the former option fails to reveal exposure. As the dural flap is reflected posteriorly to expose the meatal structures and the CPA, the subarcuate artery or AICA may be encountered. As noted earlier, the subarcuate artery usually stems from the AICA and passes through the dura on the upper posterior wall of the meatus. This vascular anatomy tends to be variable, however, and in
Chapter 66 • Vascular Considerations in Neurotologic Surgery
some patients the subarcuate artery, along with its origin from the AICA, may be incorporated into the dura on the posterior face of the temporal bone.9
Transcochlear The transcochlear approach is mainly an anteromedial extension of the translabyrinthine approach; the entire inner ear is removed, and the facial nerve is posteriorly rerouted. The surgical extension allows for access to the midclival region, prepontine cistern, and anterior aspect of the CPA. This approach has lost favor in recent years because of substantial morbidity (deafness, facial nerve palsy), although is still indicated in cases of preexisting facial palsy and for life-threatening vascular lesions, such as midbasilar artery aneurysms. The beginning steps of the transcochlear approach represent the same vascular considerations as in the translabyrinthine approach. Similarly, the boundaries in this approach have several important vascular relationships. Medially, the drilling extends to the edge of the clivus, exposing the inferior petrosal sinus from the jugular bulb below to the superior petrosal sinus above. Anteriorly, the ascending portion of the petrous carotid is exposed. After extending dissection medially into the clivus and opening the dura, the initial segment of the AICA and its origin from the basilar artery can be achieved.
Middle Fossa Approaches The middle fossa approaches are a group of procedures that give complete exposure to the IAC and a limited view of the CPA. Several variations exist to this set of approaches, which vary in the exposure of the IAC and CPA. Only Kawase’s approach is discussed here, however.
Kawase’s Approach The middle fossa–transpetrous apex approach, or Kawase’s approach, has mainly been used for lesions that traverse the middle fossa and a small part of the posterior fossa. These include dumbbell-shaped tumors such as petroclival meningiomas and trigeminal schwannomas, and lesions in the petrous apex. This approach can also be used to access aneurysms of the mid-basilar artery and lower basilar artery. Kawase’s approach involves a small posterior fossa craniotomy, after a middle fossa opening, by removing the medial portion of the petrous pyramid and the lateral aspect of the clivus. The ultimate goal is exposure of the anterior CPA and the ventral surface of the pons. As the dura is carefully elevated from the floor of the middle fossa, special attention should be paid to exposing the middle meningeal artery. If necessary, this artery can be sacrificed and divided at the foramen spinosum to avoid excessive bleeding. The boundaries of Kawase’s
803
approach are the IAC posteriorly, petrous carotid artery anteriorly, inferior petrosal sinus inferiorly, and abducens nerve medially. The boundaries of the rhomboid-shaped bony window in the petrous pyramid have several important vascular relationships. The most important is the inferior border of the petrous carotid artery. Often, this vessel can limit posterior fossa exposure. Some authors have advocated that displacement of the intrapetrous carotid artery from its bony canal can be achieved; however, doing so is controversial and probably not justified because of its risk for significant morbidity. More commonly, if more exposure is needed, the horizontal segment of the petrous carotid artery can be skeletonized, leaving a surrounding thin eggshell of bone intact allowing for slightly more mobilization and visualization. A 7 mm length segment of the petrous carotid artery may be exposed without a covering of bone in the floor of the middle fossa deep to the greater petrosal nerve.
Exposure of the Petrous Carotid The petrous ICA may require exposure in particular for revascularization procedures. The petrous ICA is typically located in Glasscock’s triangle, and can be exposed by drilling bone posteromedial to foramen ovale and anterior to the arcuate eminence. The greater superficial petrosal nerve runs superficial to and parallel to the petrous ICA, and can be sacrificed if necessary with resultant disturbance in lacrimation. As stated earlier, excessive traction on the greater superficial petrosal nerve can cause facial nerve palsy.
VASCULAR LESIONS OF THE PETROUS BONE AND CEREBELLOPONTINE ANGLE Cerebral Aneurysms Petrous Internal Carotid Artery Aneurysms Presentation and Pathophysiology Aneurysms of the petrous ICA are rare, although their true incidence is unknown. They are often discovered incidentally on computed tomography (CT) scans for unrelated purposes. They are usually asymptomatic; however, patients can present to an otolaryngologist with a wide variety of signs and symptoms, including cranial nerve palsies, Horner syndrome, epistaxis, vertigo, pulsatile tinnitus, and otorrhagia.1 The triad of otorrhagia, epistaxis, and neurologic deficit is almost pathognomonic for a petrous ICA aneurysm.10 On otoscopic examination, patients with aural symptoms can present with a vascular retrotympanic mass, which is often mistaken for a glomus tympanicum tumor. Biopsy of such a lesion can lead to severe hemorrhage and can be catastrophic. Spontaneous rupture can cause dramatic hemorrhage into the eustachian tube or middle ear or both, and manifest as massive
804
OTOLOGIC SURGERY
epistaxis or otorrhagia or both with the risk of death from exsanguination. These lesions can be classified further as true aneurysms or pseudoaneurysms, which can alter treatment. True aneurysms have walls that are continuous with the unaffected portion of the parent vessel and can develop from a congenitally weakened wall, whereas pseudoaneurysms solely involve the adventitial layer of the vessel and develop when a thrombus and fibrous tissue capsule form in response to a traumatic injury. The pathophysiology of petrous ICA aneurysms is unclear, but traumatic, mycotic (by direct seeding from the middle ear and eustachian tube), and congenital causes have been implicated.11 Trauma is a significant cause of petrous ICA pseudoaneurysms in particular because of the predisposing anatomic arrangement, with a stationary ICA segment connected to a distal mobile cervical segment, predisposing the vessel to stretch forces from forceful trauma. Iatrogenic trauma, especially from temporal bone surgery and myringotomy, has been a well-documented source of petrous ICA aneurysms.1 Management and Treatment After the diagnosis is confirmed by cerebral angiography, the question of management arises. Management of petrous ICA aneurysms remains undefined, with no established treatment paradigm in place. Most authors agree, however, that patients with ruptured petrous ICA aneurysms should undergo urgent treatment to stop active bleeding or to prevent future hemorrhage. Similarly, patients with unruptured petrous ICA aneurysms who experience major chronic symptoms from cranial nerve involvement should undergo intervention. A third group of patients, with incidental findings on CT and who are asymptomatic or experience minimal and minor symptoms, raise the question of whether intervention is necessary. Most surgeons agree that the treatment of asymptomatic patients should be approached on a caseby-case basis. It is reasonable to recommend conservative management, including close observation with serial angiography (magnetic resonance or CT angiography may suffice). Change in lesion size or morphology should prompt further consideration for treatment. Management of petrous ICA dissections should primarily be based on the degree of resultant stenosis of the true lumen. For non–flow-limiting dissections, antiplatelet therapy usually suffices. For dissections with hemodynamically or clinically significant stenoses, stent placement for artery repair should be considered. Various interventional options exist for patients with petrous ICA aneurysms, including endovascular and surgical therapies. Endovascular techniques consisting of coil embolization or stent coiling should be considered the first line of therapy in symptomatic patients or in patients with ruptured aneurysms. For aneurysms that are not amenable to reconstructive procedures, parent vessel sacrifice (deconstructive procedures) should be considered.
A balloon occlusion test with hypotensive challenge is usually performed to evaluate for adequate collateral flow and to assess for any neurologic symptoms or signs of insufficient blood supply. This test produces a high false-negative rate, however, and 2% to 22% of patients with negative findings have been found to have early or delayed ipsilateral cerebral ischemia after permanent occlusion.1 Also, a false-positive result may be obtained if the procedure is complicated by thromboemboli. If the transient occlusion produces no new neuro logic deficits, and if adequate collateral flow exists, occlusion of the ICA can be considered. Detachable balloons were frequently used in the past. More recently, coil occlusion has become the predominant modality for parent vessel sacrifice. Although coils have the advantage of providing a rapid and permanent arterial occlusion, they also have an increased risk of clot formation and distal embolization compared with immediate vessel occlusion with balloon embolization.12 Some reports have shown formation or growth of cerebral aneurysms after balloon or coil embolization.1 Numerous case reports have documented the successful use of stents to treat ruptured petrous ICA aneurysms. Cheng and colleagues12 and Auyeung and associates13 described the successful use of covered stents in three hemodynamically unstable patients who had massive epistaxis as a result of rupture of petrous ICA pseudoaneurysms that were radiation induced. A covered stent may be placed in the petrous ICA because of its lack of major arterial branches, and can be considered for pseudoaneurysms because of their lack of surrounding support. Cheng and colleagues12 concluded that porous stents can alter blood flow pattern, which readily results in stasis and thrombosis of the aneurysm; they cited numerous reports of the successful use of secondary coiling in case of stent failure. Drawbacks to stents include stent-induced intimal hyperplasia, which can cause a hemodynamically significant stenosis and thromboembolic events.14 Longterm studies are still ongoing to determine its overall efficacy and patency rates; however, initial studies show a promising role for this technology. If a patient develops neurologic sequelae after a balloon occlusion test, revascularization should be considered through an extracranial-intracranial bypass. Similarly, a patient with no neurologic deficits on temporary occlusion who exhibits marked asymmetric decrease in hemispheric blood flow (3 cm), with well-defined arterial feeders that are inaccessible surgically. Embolization using polyvinyl alcohol (PVA) particles or N-butyl cyanoacrylate (NBCA) glue should be performed with a microcatheter tip that is placed beyond the normal branches.34 The risk of embolization is particularly high with hemangioblastomas because the feeding arteries are often pial vessels. Experience with preoperative embolization of hemangioblastomas in the petrous bone or CPA has been limited mostly to small studies performed at single institutions. Conway and associates35 had mixed success with the use of preoperative embolization in 4 of 40 patients with hemangioblastomas. Although the embolization alone was sufficient to arrest symptom progression in one patient with a sacral hemangioblastoma, another patient had a lateral medullary infarction after attempted embolization of a medullary hemangioblastoma. The authors concluded that embolization should be reserved for tumors with large, surgically inaccessible arterial feeders.35 Tampieri and colleagues36 treated two patients successfully with large hemangioblastomas, one spinal and one involving the posterior fossa, with only minimal blood loss on surgical resection. Eskridge and coworkers37 reported one complication in nine patients with craniospinal hemangioblastomas: A patient developed malignant posterior fossa edema associated with hydrocephalus after treatment with polyvinyl alcohol. The surgeons concluded that embolization
810
OTOLOGIC SURGERY
B
A
C
D
F
G
E
Chapter 66 • Vascular Considerations in Neurotologic Surgery
811
FIGURE 66-3. A and B, Axial (A) and coronal (B) postcontrast MR images of the brain show right-sided glomus jugulare tumor encroaching on the cerebellopontine angle and underlying right cerebellar edema. C and D, Right internal (C) and external (D) carotid artery angiographic injections show prominent vascular blush at right skull base. The tumor was embolized with NBCA glue through ascending pharyngeal, posterior auricular, and occipital branches of right external carotid artery. E and F, Postembolization angiographic injections of right internal (E) and external (F) carotid arteries show significant flow reduction through glomus jugulare tumor. G, Postcontrast coronal MR image of the brain shows successful resection of tumor. (Courtesy of Barrow Neurological Institute.)
B
A
D
C
E
F
FIGURE 66-4. A and B, Postcontrast axial (A) and coronal (B) short tau inversion recovery MR images show large left-sided carotid body tumor. C and D, Angiogram with left common carotid artery injection shows splaying of internal and external carotid arteries by tumor. There is also a large tumor blush. The tumor was embolized with Onyx-15 through multiple left external carotid artery branches. E and F, Repeat left common carotid artery injection shows near-complete occlusion of discernible tumor vessels. One day after embolization, the patient underwent dissection of the left neck with gross total resection of carotid body tumor. (Courtesy of Barrow Neurological Institute.)
facilitated tumor manipulation and surgical resection.37 Finally, from this author’s experience of 35 patients who underwent preoperative embolization with NBCA liquid adhesive, one of the five patients treated for a hemangioblastoma experienced a complication.34 In this case, distal branches of the PICA were inadvertently occluded during glue embolization of a posterior meningeal artery. The
patient had initial mild dysmetria; however, the deficit had resolved by the 12 month follow-up examination.
Hemangiomas The role of preoperative embolization of hemangiomas is unclear because of their low overall incidence.
812
OTOLOGIC SURGERY
A
B
C
D
FIGURE 66-5. Imaging studies obtained in a 66-year-old woman. A and B, Axial T1-weighted MR images without (A) and with contrast material (B) showing recurrent squamous cell carcinoma on the left side. Lesion infiltrates left internal carotid artery (ICA) and cavernous sinus (arrow in B). Note the absence of flow void in left cavernous ICA. These images were obtained after clip occlusion of left ICA in the neck and supraclinoid process. C, Angiogram showing ICA-to-middle cerebral artery bypass with saphenous vein placed before tumor was resected. D, Axial T1-weighted MR image obtained after tumor resection and left orbital exenteration. Skull base was reconstructed with free tissue transfer by using rectus abdominis muscle flap. (From Feiz-Erfan I, Han PP, Spetzler RF, et al: Salvage of advanced squamous cell carcinomas of the head and neck: Internal carotid artery sacrifice and extracranial-intracranial revascularization. Neurosurg Focus 14(3):e6, 2003. Used with permission from Journal of Neurosurgery.)
Role of Cerebral Revascularization and Bonnet Bypass Untreated head and neck cancer that involves the skull base often is associated with a poor prognosis, especially if it involves the ICA. The risk of ICA rupture because of malignant cervical tumor invasion can be 18%, which highlights the need for ICA resection in certain cases.38 ICA sacrifice comes with the inherent risk of stroke, however, and has led many authors to advocate that resection with revascularization should proceed only in patients with unsuccessful results from a balloon test occlusion (radiographically or clinically). The test has significant drawbacks, however, including a high false-negative rate, often leading to a high rate of perioperative stroke despite a successful balloon occlusion test. It is difficult to predict immediate and delayed postoperative ischemic events.
Consequently, we advocate universal revascularization in all patients with cranial base tumors that involve the carotid artery. Other groups have suggested that in severe metastatic disease, aggressive surgery including carotid sacrifice should be performed only if there is a chance of complete cure of the tumor. More prospective studies are needed to determine a specific set of guidelines on the controversy (Fig. 66-5). The concept of a contralateral extracranial–to– ipsilateral intracranial bypass was originally introduced in 1978 by the senior author (R.F.S.) using a saphenous vein as an interposition graft, the so-called bonnet bypass (Fig. 66-6).39 Since then, based on our experience, we have found that the radial artery as an interposition graft can be used as a bonnet bypass and works with a high technical success rate with minimal complications. Bonnet bypass should be considered if the ipsilateral
Chapter 66 • Vascular Considerations in Neurotologic Surgery
813
CONCLUSION Vascular lesions affecting the petrous bone are among the most challenging in neurotologic surgery. Successful management of these lesions requires an understanding of the vascular anatomy and the various surgical approaches. Diagnosis and treatment of vascular lesions have been revolutionized by the advent of contemporary endovascular techniques. A multimodality and interdisciplinary approach to these complex lesions optimizes patient outcomes.
REFERENCES
FIGURE 66-6. Bonnet bypass using saphenous vein graft in a patient with large carotid body tumor that required sacrifice of the carotid artery. (Courtesy of Barrow Neurological Institute.)
carotid artery is unavailable because of previous surgery or radiation, or if a radical neck dissection is planned. The ipsilateral subclavian artery also can be used. In this case, however, the bypass would pass through the surgical field. Experience from this group has shown that the contralateral superficial temporal artery is an excellent donor vessel to perfuse the ipsilateral hemisphere after carotid sacrifice.38 A radial artery interposition graft is positioned between the contralateral superficial temporal artery and the ipsilateral distal middle cerebral artery. The use of a radial artery and a saphenous vein graft has been described as potential interposition grafts. Revascularization using a radial artery graft is often less taxing because the vessel wall is more resilient than vein grafts, and because the caliber of the artery closely matches that of the superficial temporal artery or middle cerebral artery. The patency rates for radial artery grafts are also higher because the arterial endothelium can support sluggish flow without thrombosis, especially because there is a lack of valves. Although the blood flow in radial artery grafts is high at 40 to 70 mL/min, flow rates with saphenous grafts are even higher at 70 to 140 mL/min.38,40 The lower flow makes radial artery grafts more conducive for anastomosis with smaller distal arteries because less turbulence is created. The grafts can be inadequate, however, for some patients who have very poor or no collateral blood flow. In this situation, the surgeon can divide a radial artery graft into two separate branches of the middle cerebral artery, or use one saphenous vein graft to accommodate the need for higher flow.
1. Liu J K, Gottfried O N, Amini A, Couldwell WT: Aneurysms of the petrous internal carotid artery: Anatomy, origins, and treatment. Neurosurg Focus 17:1-9, 2004. 2. Rhoton A L : The temporal bone and transtemporal approaches. Neurosurgery 47:211-265, 2000. 3. Tubbs R S, Hansasuta A, Loukas M, et al: Branches of the petrous and cavernous segments of the internal carotid artery. Clin Anat 20:596-601, 2007. 4. Rhoton A L : The anterior and middle cranial base. Neurosurgery 51:273-302, 2002. 5. Kakarla U K, Deshmukh VR , Zabramski J M, et al: Surgical treatment of high-risk intracranial dural arteriovenous fistulae: Clinical outcomes and avoidance of complications. Neurosurgery 61:447-459, 2007. 6. McConnell E M : The arterial blood supply of the human hypophysis cerebri. Anat Rec 115:175-203, 1953. 7. Hitotsumatsu T, Natori Y, Matushima T, et al: Microanatomical study of the carotid cave. Acta Neurochir 139:869-874, 1997. 8. Rhoton A L : The cerebellar arteries. Neurosurgery 47:2968, 2000. 9. Tanriover N, Rhoton A L : The anteroinferior cerebellar artery embedded in the subarcuate fossa: A rare anomaly and its clinical significance. Neurosurgery 57:314-319, 2005. 10. Constantino PD, Russel E, Reisch D, et al: Ruptured petrous carotid aneurysm presenting with otorrhagia and epistaxis. Am J Otol 12:378-383, 1991. 11. McGrail K M, Heros RC, Debrun G, Beyerl B D: Aneurysm of the ICA petrous segment treated by balloon entrapment after EC-IC bypass. J Neurosurg 65:249-252, 1986. 12. Cheng K M, Chan C M, Cheung YL , et al: Endovascular treatment of radiation-induced petrous internal carotid artery aneurysm presenting with acute haemorrhage: A report of two cases. Acta Neurochir 143:351-356, 2001. 13. Auyeung K M, Lui WM, Chow LC K, Chan FL : Massive epistaxis related to petrous carotid artery pseudoaneurysm after radiation therapy: Emergency treatment with covered stent in two cases. AJNR Am J Neuroradiol 24:1449-1452, 2003. 14. Wakhloo A K, Lanzino G, Lieber B B, et al: Stents for intracranial aneurysms: The beginning of a new endovascular era? Neurosurgery 43:377-379, 1998.
814
OTOLOGIC SURGERY
15. Sekhar L N, Sen C N, Jho H D: Saphenous vein graft bypass of the cavernous internal carotid artery. J Neurosurg 72:35-41, 1990. 16. Umenzu H, Seki Y, Aiba T, Kumakawa K : Aneurysm arising from the petrous portion of the internal carotid artery: Case report. Radiat Med 11:251-255, 1993. 17. Lawton MT, Hamilton MG, Morcos JJ, et al: Revascularization and aneurysm surgery: Current techniques, indications, and outcome. Neurosurgery 38:83-94, 1996. 18. Vishteh AG, Marciano FF, David C A, et al: Long-term graft patency rates and clinical outcomes after revascularization for symptomatic traumatic internal carotid artery dissection. Neurosurgery 43:761-767, 1998. 19. Candon E, Canovas F, Kabbaj J, et al: Anatomic basis for the treatment of aneurysms of the upper cervical segment of the internal carotid artery by extra-intracranial cervico-petrous bypass with inverted “in situ” saphenous vein graft. Surg Radiol Anat 19:1-6, 1998. 20. Seifert V, Stolke D: Posterior transpetrosal approach to aneurysms of the basilar trunk and vertebrobasilar junction. J Neurosurg 85:373-379, 1996. 21. Kawase T, Bertalanffy H, Otani M, et al: Surgical approaches for vertebro-basilar trunk aneurysms located in the midline. Acta Neurochir 138:402-410, 1996. 22. Kessler L A, Lubic LG, Koskoff YD: Epidural hemorrhage secondary to cavernous hemangioma of the petrous portion of the temporal bone. J Neurosurg 14:329-331, 1957. 23. Bottrill I : Imaging quiz case 2: Intraosseous cavernoustype hemangioma of the petrous temporal bone. Arch Otolaryngol Head Neck Surg 121:348-350, 1995. 24. Sundaresan N, Eller T, Ciric I : Hemangiomas of the internal auditory canal. Surg Neurol 6:119-121, 1976. 25. Madden GJ, Sirimanna K S : Cavernous hemangioma of the internal auditory meatus. J Otolaryngol 19:288-291, 1990. 26. Rocco FD, Paterno V, Safavi-Abbasi S, et al: Cavernous malformation of the internal auditory canal. Acta Neurochir 148:695-697, 2006. 27. Saito N, Sasaki T, Chikui E, et al: Anterior transpetrosal approach for pontine cavernous angiomas. Neurol Med Chir 42:272-274, 2002. 28. MacDonald J D, Antonelli P, Day A L : The anterior subtemporal, medial transpetrosal approach to the upper basilar artery and ponto-mesencephalic junction. Neurosurgery 43:84-89, 1998.
29. Deshmukh VR , Fiorella DJ, McDougall CG, et al: Preoperative embolization of central nervous system tumors. Neurosurg Clin N Am 16:411-432, 2005. 30. Marangos N, Schumacher M : Facial palsy after glomus jugulare tumour embolization. J Laryngol Otol 113:268270, 1999. 31. Herdman RC, Gillespie J E, Ramsden RT: Facial palsy after glomus tumour embolization. J Laryngol Otol 107:963-966, 1993. 32. Pandya S K, Nagpal R D, Desai A P, et al: Death following external carotid artery embolization for a functioning glomus jugulare chemdectoma: Case report. J Neurosurg 48:1030-1034, 1978. 33. LaMuraglia G M, Fabian R L , Brewster DC, et al: The current surgical management of carotid body paragangliomas. J Vasc Surg 15:1038-1044, 1992. 34. Kim L J, Albuquerque FC, Aziz-Sultan A, et al: Low morbidity associated with use of n-butyl cyanoacrylate liquid adhesive for preoperative transarterial embolization of central nervous system tumors. Neurosurgery 59:98-104, 2006. 35. Conway J E, Chou D, Clatterbuck R E, et al: Hemangioblastomas of the central nervous system in von HippelLindau syndrome and sporadic disease. Neurosurgery 48:55-62, 2001. 36. Tampieri D, Leblanc R , TerBrugge K : Preoperative embolization of brain and spinal hemangioblastomas. Neurosurgery 33:502-505, 1993. 37. Eskridge J M, McAuliffe W, Harris B, et al: Preoperative endovascular embolization of craniospinal hemangioblastomas. AJNR Am J Neuroradiol 17:525-531, 1996. 38. Deshmukh VR , Porter RW, Spetzler R F: Use of “bonnet” bypass with radial artery interposition graft in a patient with recurrent cranial base carcinoma: Technical report of two cases and review of the literature. Neurosurgery 56:E202, 2005. 39. Spetzler R F, Roski R A, Rhodes R S, Modic MT: The “bonnet bypass”: Case report. J Neurosurg 53:707-709, 1980. 40. Sekhar L N, Duff J M, Kalavakonda C, Olding M : Cerebral revascularization using radial artery grafts for the treatment of complex intracranial aneurysms: Techniques and outcomes for 17 patients. Neurosurgery 49:646-658, 2001.
Index A Abducens nerve monitoring of, 782 rehabilitation of, after neurotologic skull base surgery, 570–571 Abscesses Bezold’s, 186 of brain with acute infections, 186, 187f organisms causing, 183 stages of, 186 treatment of, 191–193, 191f extradural, with acute infections, 186 treatment of, 189–190, 190f Luc’s, 186 subdural with acute infections, 186 treatment of, 193 Absorbable gelatin sponge for patulous eustachian tube, 97 for tympanoplasty, 152 Acoustic reflex testing, 163 Acoustic tumors neuromas as auditory implants and. See Auditory implants. with chronic otitis media, retrosigmoid approach in, 606 combined therapy of, retrosigmoid approach in, 606 resection of, for retrosigmoid approach, 611–614, 612f translabyrinthine approach for. See Acoustic tumors, translabyrinthine approach for. transotic approach for, 621 neurosurgical techniques for, 600–602 for large tumors, 601–602 for small tumors, 601 translabyrinthine approach for, 591–602, 592f, 594f complication(s) of, 597–600 bleeding as, 599 cerebrospinal fluid leak as, 599 facial nerve and, 598–599 meningitis as, 599–600 hearing sacrifice with, 591 patient selection for, 591 postoperative care for, 597 postoperative follow-up for, 600 results with, 600 surgical procedure for, 593–597, 596f, 598f contracted mastoid management and, 597 vestibular nerve resection for, middle fossa approach for, 581 surgical procedure for, 593–597, 597f, 598f
Acoustic tumors (Continued) contracted mastoid management and, 597 vestibular nerve resection for, middle fossa approach for, 581–590 complications of, 588 indications for, 581 patient counseling for, 582 patient selection for, 588 preoperative evaluation for, 581–582 preoperative preparation for, 582–583 results with, 586–588 course of healing and, 586–587 success rate and, 587–588 surgical anatomy and, 583, 583f surgical technique for, 583–586, 584f–587f Acyclovir for Ramsay Hunt syndrome, 339 Adenocarcinoma of temporal bone, 33 Adenoid(s) hypertrophy of, tympanoplasty and, 150 otitis media and, 75 Adenoid cystic carcinoma of temporal bone, 33 Adenoidectomy complications of, 87 draping for, 80 follow-up for, 84 instrumentation for, 81 for otitis media acute, 77 benefits of, 79 chronic, with effusion, 79 limitations of, 79 pitfalls with, 86 postoperative care for, 84 preoperative preparation for, 80 results with, 86 risks of, 80 surgical site preparation and draping for, 80 surgical technique for, 82f, 83 tympanoplasty and, 150 Adhesions after stapes surgery, 317 Aditus ad antrum obstruction, 183, 185f Adson periosteal elevator, 16, 16f Air-bone gaps, small, stapedectomy for, 301–302 Allergic rhinitis, otitis media and, 75 Allergy treatment for, for otitis media with effusion, 87–88 tympanoplasty and, 150 Alloplastic prostheses for ossicular reconstruction, 166 Amikacin, ototoxicity of, 493–494 Aminoglycosides for acute otitis media, 111 ototoxicity of, 493
Amoxicillin for acute otitis media, 110 for otitis media, acute, 77 Amoxicillin/clavulanic acid for acute otitis media, 110 Anaerobic organisms, chronic suppurative otitis media due to, 108 Analgesia. See under specific procedures. Anesthesia. See also under specific procedures. general, complications of, 128 Anspach drill system, 2, 3f Anterior inferior cerebellar artery, anatomy of, 800–801 Anterior inferior cerebellar artery syndrome with retrosigmoid approach to cerebellopontine angle tumors, 616 Anterior sulcus, blunting in, with outer surface grafting tympanoplasty, 125f, 126 Antibiotics. See Antimicrobial therapy; specific agents. Antihistamines for otitis media, chronic, with effusion, 78 Antileukotrienes for otitis media, chronic, with effusion, 78 Antimicrobial therapy antiviral for Bell’s palsy, 338 for Ramsay Hunt syndrome, 339 following canal wall reconstruction tympanomastoidectomy, 177–178, 180–181 for otitis media acute, 77, 77t, 109–115 failed, causes of, 112–115 granulation tissue and, 111–112 preventing bacterial resistance and, 112 chronic, with effusion, 77–78 Argon laser, 281, 282t, 284 Arriaga-University of Pittsburgh tumor lymph node metastasis staging system, 35–36, 35t Arteriovenous malformations, dural, 805–806 tentorial fistulas and, 806 transverse and sigmoid fistulas and, 806 Atresiaplasty, 55–72 bone-anchored hearing appliances in, 58–59 canal wall down approach for, 65, 67f complications of, 68–69 indications for, 58 patient selection for, 58 pitfalls in, 66–67��� postoperative care and, 65 preoperative evaluation for, 60, 60f–61f results with, 67–68 surgical approach for, 60–65, 62f surgical technique for, 62–65, 62f–67f timing of, 58–59 transmastoid approach for, 65, 67f for unilateral atretic ear, 69
Page numbers followed by f indicate figures; t indicate tables.
815
816
Index
Atticotomy, 209, 215–216, 216f Audiology, cochlear implantation and, 374 Audiometry. See under specific procedures and conditions. Auditory brainstem implant, 703. See also Auditory implants. Auditory brainstem implant for vestibular schwannomas in, 698, 698f Auditory canal. See External auditory canal; Internal auditory canal. Auditory evoked brainstem response, monitoring hearing using, 779–780, 781 Auditory implants, 703–714 anatomic considerations for, 704–705, 705f devices for, 704, 705f implantation technique for, 707–709, 707f–709f patient counseling for, 704 patient selection for, 703, 704t postoperative care for, 709 postoperative complications with, 709–710 preoperative evaluation for, 704 research studies on, 711–712, 712f with auditory midbrain implants, 712 with penetrating electrode ABI, 711–712 results with, 710–711, 710f surgical considerations for, 705–707, 706f–707f Auditory midbrain implant, 703. See also Auditory implants. Aural atresia, 55–72 atresiaplasty for. See Atresiaplasty. Baha for, 399 classification systems for, 56–57, 56t–57t computed tomography in, 60, 60f–61f embryology of, 55–56, 56t initial evaluation of, 57–58 unilateral, atresiaplasty for, 69 Aural fullness, differential diagnosis of, 94, 99f Aural toilet for acute otitis media, 109 Auricle, embryology of, 55–56, 60f Auricular nerve, greater, for facial nerve grafting, 236 Austin “reverse elevator” for undersurface graft tympanoplasty, 142–144, 145f Autografts for ossicular reconstruction, 161, 166, 166f Autophony with patulous eustachian tube, 94 in superior canal dehiscence syndrome, 510
B Bacterial resistance, preventing, 112 Bacteroides spp., otitis media and, 74 Baha. See Bone-anchored cochlea stimulator (Baha). Balloon ballottement, stapes surgery and, 312 Balloon microcompression for trigeminal neuralgia, shortcomings of, 525 Barotrauma after stapes surgery, 316 Basal cell carcinoma of temporal bone, 33 “Beginner’s hump” with mastoidectomy, canal wall down, 211–212, 212f Bell’s palsy, 335–338 audiometry in, 336 facial nerve decompression for, 339–344 herpes simplex virus and, 335 management of, 337, 337f natural history of, 337 testing for, 336, 338f Benign paroxysmal positional vertigo, 467–476 pathophysiology of, 468
Benign paroxysmal positional vertigo (Continued) posterior semicircular canal occlusion for patient counseling and, 468–470, 469f patient selection for, 470 preoperative evaluation for, 471 results with, 471–473 surgical technique for, 471, 472f Bezold’s abscess, 186 BICROS for aural atresia, 59 Biofilms failed antibiotic therapy due to, 113 otitis media with effusion and, 88 treatment of, 113 Biscuit footplate, stapes surgery and, 314, 315f Bleeding. See Hemorrhage. Blood within vestibule, stapes surgery and, 316 Bondy modified radical mastoidectomy, 209, 213–214, 214f Bone cement for dural injury, 238 for encephalocele repair, 248–249 for labyrinthine fistula repair, 231 for ossicular prostheses, 162, 166–167 Bone conduction, hearing through, 397–398. See also Hearing loss, conductive. Bone-anchored cochlea stimulator (Baha), 383–384, 397–410 audiometric criteria for, 400–401, 400f–401f for aural atresia, 58 Baha Cordelle II as, 400, 401f Baha Divino as, 400, 400f Baha Intenso as, 400, 400f in children, 408–409 in chronic ear disease, 398 with chronic external otitis, 399 complications of, 407–408 handling of, 408 for conductive loss in only hearing ear, 399 for conductive or mixed hearing loss, 398 for congenital malformations, 399 contraindications to, 401 in Down syndrome, 399 hearing through bone conduction and, 397–398 historical background of, 397 osseointegration and, 398 patient counseling and, 399, 400f patient preparation for, 402, 403f patient selection for, 398–399 pitfalls with, 407–408 postoperative management for, 406–407, 406f for single-sided deafness, 399 stability of, 408, 408f surgical instruments for, 402 surgical technique for, 402–406, 406f Baha coupling placement and, 405–406 for countersink drilling, 405 for guide hole drilling, 405 subcutaneous soft tissue reduction and, 405 for suturing flap, 406 for thin hairless flap preparation, 402–405 Bonnet bypass, internal carotid artery cancer and, 812–813, 812f–813f Boric acid for patulous eustachian tube, 97 Botulinum toxin for hemifacial spasm, 529 for hyperkinesis, with facial reanimation, 770–771 Brain abscesses with acute infections, 186, 187f organisms causing, 183 stages of, 186 treatment of, 191–193, 191f
Brain herniations, 245–252, 246f–247f computed tomography of, 247, 247f diagnosis of, 246–248, 247f–248f magnetic resonance imaging of, 247, 247f pathogenesis of, 245–246, 247f treatment of, 248–250, 248f–249f Brain injury cerebrospinal fluid leaks due to, 246 treatment of, 248, 248f–249f translabyrinthine approach for. See Acoustic tumors, translabyrinthine approach for. Brain prolapse, 245–252, 246f–247f computed tomography of, 247, 247f diagnosis of, 246–248, 247f–248f magnetic resonance imaging of, 247, 247f pathogenesis of, 245–246, 247f treatment of, 248–250, 248f–249f Brow elevation in facial paralysis, 748f, 749–750
C Canal wall defect repair, 206 for defects resulting from disease, 205f, 206 for defects resulting from surgery, 206 Canalithiasis, 468 Canalplasty, 21–32 for collapsing external auditory canal, 28–29 draping for, 22, 23f for external auditory canal exostoses, 21–28 analgesia for, 22, 23f complications of, 28 indications for, 21 pharmacologic preparation for, 22 postoperative care for, 25–28 problems in, 28 psychological preparation for, 22 site preparation for, 22, 23f surgical technique for, 22–25, 23f–24f for anterior exostoses, 25, 26f–27f for posterior exostoses, 24f, 25, 26f for keratosis obturans, 29 for medial third stenosis of external auditory canal, 28, 29f for post-traumatic suture dehiscence, 31 psychological preparation for, 22 for scutum defects, 29–31, 30f for tympanic bone osteonecrosis and osteoradionecrosis, 29 Candida, acute otitis media due to, 109 Canthoplasty for lower lid reapposition in facial paralysis, 742 lateral, 745–747, 746f medial, 744f, 745 Carotid artery external, anatomy of, 800 glomus tumors of, 551 injury of with chronic otitis media surgery, 241 with limited temporal bone resection, 40 with radical temporal bone resection, 50 internal anatomy of, 799–800 aneurysms of high cervical, 805 petrous, 803–805 Cartilage, preparation for ossicular reconstruction, 169 Cartilage tympanoplasty. See Tympanoplasty, cartilage. Cauterization for tympanic membrane closure, 114f, 115–116 Cavernous hemangiomas, 806–807 Cavernous malformations, 806–807
Index Ceftriaxone for acute otitis media, 110 Cefuroxime for acute otitis media, 110 Central nervous system auditory implants. See Auditory implants. Cerebellar artery, superior, anatomy of, 800–801 Cerebellar dysfunction following retrosigmoid approach to cerebellopontine angle tumors, 617 Cerebellopontine angle extended middle cranial fossa approach to, 641–648, 642t advantages of, 647 limitations of, 645–647 surgical anatomy and, 641–642, 642f surgical technique for, 642–645, 643f–646f technique for, 641 vascular anatomy of, 799–801 dural venous sinuses and, 801 external carotid artery and, 800 internal carotid artery and, 799–800 vertebrobasilar arteries and, 800–801 Cerebellopontine angle lesions recurrent, following retrosigmoid approach, 618 retrosigmoid approach to, 603–620 for acoustic neuroma with chronic otitis media, 606 in combined therapy of acoustic neuromas, 606 complications of, 616–618 from patient positioning, 616–617 vascular, 616 contraindications to, 606–607 dressings for, 614 hearing preservation and, 605–606 indications for, 605–606 instruments for, 607–608 patient counseling for, 605 patient positioning for, 607 complications from, 616–617 patient preparation for, 607 patient selection for, 605–607 postoperative care for, 614–615 preoperative evaluation for, 605 results with, 615–616 for revision surgery, 606 surgical anatomy and, 603–605, 604f surgical site preparation for, 607 surgical technique for, 608–614 for acoustic neuroma resection, 611–614, 612f for cerebellopontine angle exposure, 608–609 craniectomy and, 608 for craniotomy closure, 614 hemostasis and, 614 for internal auditory canal closure, 612f, 614 for internal auditory canal exposure, 609–611, 610f for tumors extending into inferior portion of cerebellopontine angle, 606 for tumors with limited extension into Meckel’s cave, 606 transcochlear approach to, 631–640 advantages of, 631 complications of, 639–640 computed tomography and, 632 disadvantages of, 632 magnetic resonance imaging and, 632 patient evaluation for, 632 postoperative care for, 637 preoperative counseling for, 632 results with, 637–639
Cerebellopontine angle lesions (Continued) surgical anatomy and, 632 surgical technique for, 632–637, 633f closure and, 636–637 external auditory canal closure and, 633–634 facial nerve rerouting and, 634–635, 635f incision and, 633 labyrinthectomy and skeletonization of internal auditory canal and, 634, 634f mastoidectomy and, 633, 634f setup and, 633 transcochlear drill-out and, 635, 635f tumor removal and, 635–636, 636f–639f transotic approach for, 621–630 alternatives to, 629–630 draping for, 622 dressings for, 628 indications for, 621 intraoperative monitoring for, 622 patient positioning for, 622 postoperative care for, 628 preoperative evaluation for, 621–622 results with, 629, 629t surgical site preparation for, 622 surgical technique for, 622–628 blind sac closure of external auditory canal and, 622, 623f eustachian tube obliteration and, 622 otic capsule exenteration and, 622–624, 623f skin incision and, 622, 623f subtotal petrosectomy and, 622, 623f tumor removal and, 624–625, 625f–626f unroofing of labyrinthine portion of facial nerve and, 624, 625f wound closure and, 625–628, 627f tips and pitfalls in, 627f, 628–629 vascular, 803–807 cavernous malformations and cavernous hemangiomas as, 806–807, 808f dural arteriovenous malformations as, 805–806 internal carotid aneurysms as, 803–805 high cervical, 805 petrous, 803–805 vertebrobasilar aneurysms as, 805 vascular tumors as, 807–813 cerebral revascularization and bonnet bypass and, 812–813, 812f–813f preoperative embolization and, 807–811 glomus tumors and, 807–809, 810f–811f hemangioblastomas and, 809–811 hemangiomas and, 811 Cerebral blood flow, evaluation of, in temporal bone malignancies, 44 Cerebral revascularization, internal carotid artery cancer and, 812–813, 812f–813f Cerebrospinal fluid leaks with cochlear implantation, 378 diagnosis of, 246–248, 247f–248f with endoscopic endonasal approaches to skull base and paranasal sinuses, 679 following facial nerve tumor surgery, 371 head trauma and, 246 treatment of, 248, 248f–249f management of, 727–732, 731f ear canal closure with eustachian tube and middle ear obliteration for, 731–732, 731f lumbar drain for, 727–729, 729f
817
Cerebrospinal fluid leaks (Continued) middle fossa obliteration of the eustachian tube for, 731f, 732 pressure dressings for, 727, 728f for refractory leaks, 731–732, 731f wound exploration and reclosure for, 731 with retrolabyrinthine vestibular neurectomy, 451 with retrosigmoid approach to cerebellopontine angle tumors, 617 spontaneous encephaloceles and, 246 with surgery for facial nerve trauma, 360 following transcanal labyrinthectomy, 488 with translabyrinthine approach for acoustic tumors, 599 following translabyrinthine vestibular neurectomy, 464 Cerebrospinal fluid otorrhea, 107–118 from fistulas, 107 management of, 109–115 antimicrobial therapies for, 109–115 aural toilet for, 109 failed therapy and, 112–115 granulation tissue and, 111–112 tympanic membrane closure for, office techniques for, 115–116 from otitis media acute, with open tympanic membrane, 109 chronic suppurative otitis, 107–109 spontaneous, 107 from temporal bone fractures, 107 traumatic, 107 Ceruminous carcinoma of temporal bone, 33 Cervical sympathetic chain deficits, rehabilitation after neurotologic skull base surgery for, 578 Chair, hydraulic, for undersurface graft tympanoplasty, 142 Chemotherapy with limited temporal bone resection, 41 Children Baha in, 408–409 ossicular reconstruction in, 163 otosclerosis in, partial stapedectomy for, 279 stapedectomy in, 302 stapedotomy in, 312 trigeminal neuralgia in, microvascular decompression for, 527 Chissone’s classification system for congenital aural atresia, 57 Cholesteatomas canal wall reconstruction tympanomastoidectomy for. See Tympanomastoidectomy, canal wall reconstruction. cartilage tympanoplasty for. See Tympanoplasty, cartilage. diagnosis of, 197 drainage procedures for. See Petrous apex lesions, drainage procedures for. management of, labyrinthine fistulas and, 229–231, 229f–230f mastoidectomy for. See Mastoidectomy. matrix removal and, 229–231, 229f–230f occult, failed antibiotic therapy due to, 113 of pars flaccida, canalplasty for, 29–31, 30f recurrent, 196 residual, tympanoplasty and, 222 Cholesterol granulomas, 76 treatment of, drainage procedures for. See Petrous apex lesions, drainage procedures for. Chorda tympani nerve identification of, 308 injury of, during laser stapedectomy, 271
818
Index
Chorda tympani nerve (Continued) preservation of, in stapes surgery, 308 stapedectomy and, 257, 260f Chronic ear surgery. See Tympanoplasty. Ciprofloxacin for acute otitis media, 110 CO2 laser, 281, 282t, 284 Cochlea, drilling out of, in transcochlear approach, 635, 635f Cochlear blood flow, measurement using laser Doppler flowmetry, 782 Cochlear hypoplasia, 374–375 Cochlear implantation, 373–382 bilateral, 379–380 combined electroacoustic stimulation and, 378f, 380 complications of, 378–379, 379t criteria for, 373, 374t instrumentation for, 15, 15f medical evaluation for, 373–375 audiologic, 374 imaging and, 374–375 physical examination and, 374 promontory evaluation and, 375 patient selection for, 373 revision, 379 selection of ear for, 375 surgical technique for, 375–378, 375f–377f following transcanal labyrinthectomy, 490 Cochlear microphonics in Meniere’s disease, 415 Cochlear nerve, anatomy of, 442–443, 444f Cochleosacculotomy, 477–482 patient selection for, 478 rationale for, 477–478 results with, 481–482 surgical technique for, 478–481, 479f–481f Combined electroacoustic stimulation, 378f, 380 Compound muscle action potential in Bell’s palsy, 336–337 Computed tomography. See under specific conditions and procedures. Conductive hearing loss. See Hearing loss, conductive. Congenital aural atresia. See Aural atresia. Corticosteroids for Bell’s palsy, 337–338 intratympanic, for inner ear conditions, 502–503 clinical studies of, 502–503 experimental studies of, 502 for otitis media, chronic, with effusion, 78 Corynebacterium, otitis media and, 74 Cosmetic deformities, following infratemporal fossa surgery, 664 Cranial base. See Skull base entries. Cranial nerve(s) abducens (VI) monitoring of, 782 rehabilitation after neurotologic skull base surgery for, 570–571 deficits of with limited temporal bone resection, rehabilitation of, 41 after radical temporal bone resection, rehabilitation for, 50 facial (VII). See Facial nerve entries. glossopharyngeal (IX). See Glossopharyngeal �entries. hypoglossal (XII) loss of function of, with limited temporal bone resection, 40 monitoring of, 782 rehabilitation after neurotologic skull base surgery and, 578
Cranial nerve(s) (Continued) hypoglossal/facial anastomosis for facial nerve paralysis and, 755–764 end-to-side, via facial nerve translocation from fallopian canal, 759, 760f nerve growth factors and conduits for, 757–758 newer modifications of, 758–759, 758f–759f patient selection for, 755–756 results with, 759–761, 760t–761t surgical technique for, 756–757, 756f–757f oculomotor (III), monitoring of, 782 spinal accessory (XI) anatomy of, 443, 444f monitoring of, 782 in glomus tumor surgery, 555–556 palsy of, with jugular foramen schwannomas, 552 preservation of, in glomus tumor surgery, 556 rehabilitation after neurotologic skull base surgery and, 577–578 trigeminal (V). See Trigeminal nerve; Trigeminal neuralgia. trochlear (IV), monitoring of, 782 vagus (X). See Vagus nerve. vestibulocochlear (VIII) anatomy of, 445f, 446 compression of, microvascular decompression for, 530 direct recording from, monitoring hearing using, 780–781 monitoring of, during neurologic procedures, 11–12 Cranial rhizopathies, microvascular decompression for. See Microvascular decompression. Craniectomy for retrosigmoid approach, 608 Craniotomy closure of, for retrosigmoid approach, 614 for middle cranial fossa vestibular neurectomy, 431–433, 432f, 434f middle fossa, internal auditory canal decompression without tumor removal with, for vestibular schwannomas, 697 retrosigmoid, with partial removal of vestibular schwannoma, 697, 697f Cross-facial grafting, 755 hypoglossal/facial anastomosis as, 755–764 end-to-side, via facial nerve translocation from fallopian canal, 759, 760f nerve growth factors and conduits for, 757–758 newer modifications of, 758–759, 758f–759f patient selection for, 755–756 results with, 759–761, 760t–761t surgical technique for, 756–757, 756f–757f Crus, division of, 309 Cupulolithiasis, 468 Curettes, stapes, 6, 7f CyberKnife stereotactic radiosurgery, 794–795 dose distribution and, 795 localization and, 795 treatment delivery and, 795 treatment planning and, 794–795, 794f
D De la Cruz classification of congenital aural atresia, 56–57, 56t “Dead ear,” definition of, 457
Deafferentation procedures for vestibular disorders, 457 Deafness. See Hearing loss. Decongestant therapy for otitis media, chronic, with effusion, 78 Delivery technique, failed antibiotic therapy due to, 112–113 Diplacusis, binaural, after stapes surgery, 317 Discharge, chronic, preoperative treatment of, before mastoidectomy, 197–198 Distention theory of Meniere’s disease, 414 Dix-Hallpike maneuver, 468–469, 469f in benign paroxysmal positional vertigo, 467 Dizziness. See also Vertigo. complicating operations for chronic ear infections, 128 following laser stapedectomy, 272 migraine-related, 456, 510 risk of, with mastoidectomy, 210 Dizziness Handicap Inventory in superior canal dehiscence syndrome, 511 Down syndrome, Baha in, 399 Drainage, chronic, preoperative treatment of, before mastoidectomy, 197–198 Drainage theory of Meniere’s disease, 414 Draping. See under specific procedures. Drills, 1–2, 2f–3f “Dry mopping” for acute otitis media, 109 Dry mouth complicating operations for chronic ear infections, 128 Dural defects with limited temporal bone resection, 40 Dural elevation for middle cranial fossa vestibular neurectomy, 433, 434f Dural herniations, 245–252, 246f–247f computed tomography of, 247, 247f diagnosis of, 246–248, 247f–248f magnetic resonance imaging of, 247, 247f pathogenesis of, 245–246, 247f treatment of, 248–250, 248f–249f Dural injury with chronic otitis media surgery, 237–238, 238f–239f Dural venous sinuses anatomy of, 801 mastoidectomy and, 219 Dysgeusia after stapes surgery, 316
E Ear packing after stapes procedures, 7 Ear specula, 5–6, 6f Ectopic tissue, introduced, after stapes surgery, 320 Elderly patients, stapedectomy in, 302 Electrocautery equipment, 1 Electrocochleography in Meniere’s disease, 415 monitoring hearing using, 780–781 perilymphatic fistulas and, 326 Electrodiagnostic testing. See also specific techniques. in Bell’s palsy, 336 Electromyography in Bell’s palsy, 337 for facial nerve monitoring, 773–774, 774f–775f during facial nerve decompression, 339, 342–343 with facial nerve trauma, 352–353 Electroneuronography in Bell’s palsy, 336 with facial nerve trauma, 351 Electronystagmography in Meniere’s disease, 415
Index Electronystagmography (Continued) for translabyrinthine vestibular neurectomy, 458–459 ELITE approach, 715–726, 717f complications of, 724 results with, 724–726 surgical procedure for, 716–724, 718f–722f, 724f Embolization, preoperative, 807–811 glomus tumors and, 807–809, 810f–811f hemangioblastomas and, 809–811 hemangiomas and, 811 Embryology of ear, 55–56, 56t Encephaloceles, temporal bone, 245–252, 246f–247f computed tomography of, 247, 247f diagnosis of, 246–248, 247f–248f pathogenesis of, 245–246, 247f treatment of, 248–250, 248f–249f Endolymphatic hydrops, pathophysiology of, 413–414 Endolymphatic sac surgery, 411–428 complications of, 418 draping for, 10 endolymphatic shunt procedure as, 417–418, 418f–421f instrumentation for, 10–11, 11f, 19 outcomes with, 418–425, 421t hearing and, 424–425 from 1995 to present, 419–423, 422t patients’ perspective on, 423–424 physicians’ perspective on, 424 patient preparation for, 10 patient selection for, 417 physiologic monitoring during, 10 procedure for, 10 sac anatomy and embryology and, 411–412 sac physiology and, 413 Endoscopy, perilymphatic fistulas and, 326 Enterobacteriaceae, chronic suppurative otitis media due to, 108 Entropion of upper lid, correction of, in facial paralysis, 748f, 750 Envoy Esteem, 386t, 391–394, 393f Epithelial cysts following outer surface grafting tympanoplasty, 125f, 126 Estrogen, conjugated, for patulous eustachian tube, 97 Eustachian tube dysfunction of allergic inflammation in, otitis media and, 76 otitis media and, 75 function of, tests of, tympanoplasty and, 151 obliteration of, transotic approach for cerebellopontine angle lesions and, 622 patulous, 93–106 anatomy and physiology of, 93–94, 94f clinical presentation of, 94 diagnosis of, 95–97, 95f, 99f etiology of, 94 transnasal and transoral occlusion of, 98–104, 100f–104f treatment of, 97–98, 99f retracted, aural fullness and, 99f Exostoses of external auditory canal, canalplasty for. See Canalplasty, for external auditory canal exostoses. Extended middle cranial fossa approach, 641–648, 642t advantages of, 647 limitations of, 645–647 surgical anatomy and, 641–642, 642f surgical technique for, 642–645, 643f–646f technique for, 641
External auditory canal aplasia of. See Aural atresia. atresia of, unilateral, Baha for, 399 closure of blind sac, in transotic approach for cerebellopontine angle lesions, 622, 623f in transcochlear approach, 633–634 collapsing, canalplasty for, 28–29 enlargement of, in undersurface graft tympanoplasty, 142, 143f exostoses of, canalplasty for. See Canalplasty, for external auditory canal exostoses. hypoplasia of. See Aural atresia. keratosis obturans of, canalplasty for, 29 medial third stenosis of, canalplasty for, 28, 29f post-traumatic suture dehiscence and, canalplasty for, 31 scutum defects and, canalplasty for, 29–31, 30f stenosis of, atresiaplasty and, 68 trauma to, during tympanostomy tube insertion, 84 tympanic bone osteonecrosis and osteoradionecrosis and, canalplasty for, 29 External otitis, chronic Baha for, 399 Extradural abscesses with acute infections, 186 treatment of, 189–190, 190f Extreme lateral infrajugular transcondylar approach, 715–726, 717f complications of, 724 results with, 724–726 surgical procedure for, 716–724, 718f–722f, 724f Eye(s) in facial paralysis. See Facial nerve paralysis, eye care in. in neurofibromatosis 2, 696, 696t Eye movements in superior canal dehiscence syndrome, 507–508, 508f Eyes-closed-turning test for perilymphatic fistulas, 326
F Facial nerve anatomy of, 443 bone overlying, lowering of, for mastoidectomy, canal wall down, 211–212, 212f congenitally ectopic, stapes surgery and, 314 deficits of with infratemporal fossa surgery, 663 translabyrinthine approach for. See Acoustic tumors, translabyrinthine approach for. with translabyrinthine approach for acoustic tumors, 598–599 dehiscent, stapedectomy and, 257–259, 297 laser, 269–271 embryology of, 56 excessive manipulation of, with limited temporal bone resection, 40 grafting of, 236–237, 236f hypoglossal/facial anastomosis for facial nerve paralysis and, 755–764 end-to-side, via facial nerve translocation from fallopian canal, 759, 760f nerve growth factors and conduits for, 757–758 newer modifications of, 758–759, 758f–759f
819
Facial nerve (Continued) patient selection for, 755–756 results with, 759–761, 760t–761t surgical technique for, 756–757, 756f–757f injury of in atresiaplasty, 60, 61f with cochlear implantation, 378 mastoid segment of, 203, 205f during mastoidectomy, 86, 203 in transcanal labyrinthectomy, 488 tympanic segment of, 203, 205f injury to, audiometry with, 350 labyrinthine portion of, unroofing of, transotic approach for cerebellopontine angle lesions and, 624, 625f mastoid segment of, transection with cutting burr, 235, 235f mastoidectomy and, 219–220, 219f microvascular decompression for hemifacial spasm and, 529–530 overhanging, stapes surgery and, 314, 315f in radical temporal bone resection, 44 rerouting of, in transcochlear approach, 634–635, 635f sacrifice of, with limited temporal bone resection, 40 tumors of, 363–372 computed tomography of, 363, 364f hemangiomas as, 363, 371, 371t magnetic resonance imaging of, 363, 364f neuromas as, 363–365, 369–370, 371t schwannomas as, in neurofibromatosis 2, 694–695 surgery for complications of, 371 graft material for, 367, 368f middle fossa approach for, 365–367, 366f nerve grafting and, 367–369, 368f, 370f patient counseling and, 365, 365f patient selection for, 363–365 pitfalls of, 369 radiation therapy and radiosurgery and, 365 rerouting and, 369, 370f results with, 369–370, 371t surgical techniques for, 365–369 translabyrinthine approach for, 367, 368f transmastoid approach for, 366f, 367 tumor removal and, 367 Facial nerve decompression for Bell’s palsy, 339–344 anesthesia for, 339 draping for, 339 intraoperative electromyography in, 339, 342–343 limitations of, 344 postoperative care for, 343–344 preoperative preparation for, 339, 340f–341f special considerations for, 344 surgical technique for, 340–343, 341f–344f Facial nerve monitoring, 773–778 anesthetic issues and, 775–776 antidromic, 778 during atresiaplasty, 67 electromyography for, 773–774, 774f–775f during endolymphatic sac surgery, 10 historical background of, 773 indications for, 773 during mastoidectomy, 220 during neurologic procedures, 11–12 physiology and, 773
820
Index
Facial nerve monitoring (Continued) practical application of, 776–778 antidromic monitoring and, 778 artifactual responses and, 777 avoiding trauma and, 776 locating facial nerve and, 776 prognosis and, 776–777, 777t troubleshooting facial nerve monitors and, 777–778 stimulation for, 774–775 burst activity and, 775 trains and, 775 during surgery for traumatic facial paralysis, 356–357 during transcanal labyrinthectomy, 485 for tympanoplasty, 152 Facial nerve paralysis with acute infections, 185 treatment of, 189 in Bell’s palsy. See Bell’s palsy. with chronic otitis media surgery, 234–236, 235f complicating operations for chronic ear infections, 128 eye care in, 733–754, 752t anesthesia for, 733–734 brow elevation for, 748f, 749–750 criteria for, 733 draping for, 734–735, 734f lid suture taped to cheek for, 750–752, 751f lower lid reapposition procedures for, 742–749 canthoplasty for, 742 fascia lata suspension of lower lid for, 747, 748f lateral canthoplasty for, 745–747, 746f lid position assessment and, 742–745 lid stents for, 747–749 medial canthoplasty for, 744f, 745 midface support for, 749 patient preparation for, 733 surgical preparation for, 733–734 temporary tarsorrhaphy suture for, 751f, 752 upper lid entropion correction for, 748f, 750 upper lid reanimation procedures for, 735–737 enhanced palpebral spring implantation for, 738–740, 739f, 741f gold weight implantation for, 740–742, 741f, 742t palpebral spring implantation for, 736f, 737–738 silicone rod prosthesis implantation for, 742, 743f hypoglossal/facial anastomosis for, 755–764 end-to-side, via facial nerve translocation from fallopian canal, 759, 760f nerve growth factors and conduits for, 757–758 newer modifications of, 758–759, 758f–759f patient selection for, 755–756 results with, 759–761, 760t–761t surgical technique for, 756–757, 756f–757f mastoidectomy and, 87 after radical temporal bone resection, rehabilitation for, 50 in Ramsay Hunt syndrome, 338–339 reanimation for. See Facial reanimation. risk of, with mastoidectomy, 210 after stapes surgery, 316–318 following transcochlear approach, 639
Facial nerve paralysis (Continued) following translabyrinthine vestibular neurectomy, 464 traumatic, 347–362, 348f extratemporal facial nerve injury and, 349 intracranial facial nerve injury and, 347 intratemporal facial nerve injury and, 347–349, 349t, 350f–353f ossicular damage with, 354 otorrhea with, 354–355 patient evaluation and, 349–354 prognosis based on electric studies in, 351–353 radiologic evaluation of, 353 surgery for complications of, 360 extratemporal nerve segment and, 359 intratemporal nerve segment and, 356–359, 357f–358f patient positioning for, 356 patient preparation for, 356 pitfalls of, 359 postoperative care for, 359 preoperative preparation for, 355–359 results with, 360 surgical technique for, 356–359 timing of, 353–354 Facial reanimation, 765–772 dynamic procedure(s) for, 765–770 free muscle flaps as, 767–770 nerve grafts as, 765–766 nerve substitution as, 766 temporalis muscle transposition as, 766–767, 768f–769f static procedure(s) for, 770–771 for lower lip rehabilitation, 770 static slings as, 770 surgical management of hyperkinesis as, 770–771 Facial recess, opening of, in mastoidectomy, 199–201, 201f, 205f Facial weakness with labyrinthine fistulas, 228 Fascia grafts in tympanoplasty. See Specific techniques under Tympanoplasty. Fascia lata suspension for lower lid reapposition in facial paralysis, 747, 748f Fat graft tympanoplasty, 115f, 116 Fatigability in benign paroxysmal positional vertigo, 467 Fenestra, small, stapedectomy and, 300 Fentanyl citrate for revision stapedectomy, 288 Fisch classification, 715 Fisch operating table, 430, 430f Fistula(s) perilymphatic. See Perilymphatic fistulas. stapedectomy and, 300 Fistula test for perilymphatic fistulas, 326 Flap necrosis with Baha implant surgery, 407, 407f Food allergy, treatment of, for otitis media, 78 Footplate biscuit, stapes surgery and, 314, 315f extraction of, total, instrumentation for, 6–7, 7f fixed foreshortened incus with, ossicular reconstruction for, 165, 165f with mobile malleus and incus, ossicular reconstruction for, 164 floating laser stapedectomy and, 271 in stapedectomy, 259 stapedectomy and, 296f, 297–298, 298t fractured, stapes surgery and, 316 fragments of
Footplate (Continued) left in oval window, revision stapedectomy for, 287 in vestibule, stapes surgery and, 316 mobile, with mobile incus and malleus and absent stapes superstructure, ossicular reconstruction for, 164 obliterated, in stapedectomy, 259, 261f relationship to vestibule, stapes surgery and, 311–312 removal of, 309 Footplate surgery, classification of, 276 Free muscle flaps for facial reanimation, 767–770 Fungi, chronic suppurative otitis media due to, 108
G Gamma knife surgery for skull base tumors. See Skull base tumors, stereotactic radiosurgery of. Gas lasers, 281 Gelfoam for patulous eustachian tube, 97 for tympanoplasty, 152 Gentamicin intratympanic, for inner ear conditions, 497–501 clinical studies of, 497–501, 498t–499t, 499f–500f experimental studies of, 497 results with, 501–502, 501t ototoxicity of, 493–494 Glasscock-Jackson classification, 715 Glomus jugulare tumors, 715–716 extreme lateral infrajugular transcondylar approach for, 715–726, 717f complications of, 724 results with, 724–726 surgical procedure for, 716–724, 718f–722f, 724f radiation therapy for, 566 stereotactic radiosurgery of, 796 surgery for complications of, 565–566 results of, 564–565 Glomus tumors arteriography in, 553 audiometry and, 553 biopsy in, 553 brain perfusion and flow studies in, 553 carotid artery, 551 computed tomography in, 552–553, 555 embolization in, 553 glomus vagale, 552 jugular bulb, 551 jugular foramen schwannomas as, 552 magnetic resonance imaging in, 552–553 surgery for, 551–568 complications of, 564f–567f, 565–566 with glomus jugulare tumors, 565–566 with glomus tympanicum tumors, 565 patient counseling for, 554 patient selection for, 551–552, 552t preoperative evaluation for, 553 results of, 564–565 with glomus jugulare tumors, 564–565 with glomus tympanicum tumors, 564 surgical approaches for, 554–564 complete carotid mobilization as, 562–564 fallopian bridge, 561–562, 563f infratemporal fossa, 557–561, 557f–562f
Index Glomus tumors (Continued) mastoid-extended facial recess, 555, 555f mastoid-neck, 555–557, 556f–557f transcanal, 554–555, 554f transcondylar, 562, 563f transdural, 552 tympanic, 551 tympanomastoid, 551 Glomus tympanicum tumors, surgery for complications of, 565 results of, 564 Glomus vagale tumors, 552 Glossopharyngeal nerve anatomy of, 443, 444f compression of, microvascular decompression for, 531–532 deficits of, rehabilitation after neurotologic skull base surgery for, 571 monitoring of, 782 paresis/palsy of with jugular foramen schwannomas, 552 with limited temporal bone resection, 40 preservation of, in glomus tumor surgery, 556 schwannomas, in neurofibromatosis 2, 695 Glossopharyngeal neuralgia, microvascular decompression for, 531–532 complications of, 532 operative technique for, 532 patient selection for, 531–532 results with, 532 Glycerol rhizotomy, shortcomings of, 525 Gold weight implantation for upper lid reanimation in facial paralysis, 740–742, 741f, 742t Gram-negative organisms, otitis media due to, 74 acute, 109 chronic suppurative, 108 Granulation tissue in chronic suppurative otitis media, 108, 111–112 extradural with acute infections, 186 treatment of, 189–190, 190f Granulomas cholesterol, 76 treatment of, drainage procedures for. See Petrous apex lesions, drainage procedures for. reparative following laser stapedectomy, 272 after stapes surgery, 320 Greater auricular nerve grafts for facial nerve tumors, 367, 368f
H Haemophilus influenzae intracranial complications due to, 183 meningitis due to, with cochlear implantation, 374t, 379 otitis media due to, 74, 77 acute, 109 otogenic complications due to, 183 Head trauma cerebrospinal fluid leaks due to, 246 treatment of, 248, 248f–249f translabyrinthine approach for. See Acoustic tumors, translabyrinthine approach for. Headache migraine-related dizziness and, 456, 510 following retrosigmoid approach to cerebellopontine angle tumors, 605–607 with retrosigmoid vestibular neurectomy, 451
Hearing through bone conduction, 397–398 following canal wall reconstruction tympanomastoidectomy, 181 after endolymphatic sac surgery, 424–425 good, definition of, 457 measurable, definition of, 457 monitoring of, 778–782 auditory evoked brainstem response for, 779–780, 779f direct eighth cranial nerve recording for, 780–781 electrocochleography for, 780–781 indications for, 778–779 laser-Doppler cochlear blood flow for, 782 otoacoustic emissions for, 781 pathophysiology and, 778 preoperative testing and, 778 following ossicular reconstruction, 170 partial stapedectomy in children and, 279 following perilymphatic fistula surgery, 330 preservation of with labyrinthine fistula repair, 231 in neurofibromatosis 2, 696–697 auditory implants for. See Auditory implants. retrosigmoid approach to cerebellopontine angle tumors and, 605–606 serviceable, definition of, 457 stapes prostheses and, stainless steel vs. polytef, 279 following tympanoplasty, 159 tympanostomy tubes and, in otitis media, 79 vestibular schwannomas and, 695 Hearing aids air conduction, implantable hearing aids vs., 384 bone-anchored, for aural atresia, 58–59 implantable, 383–396 Baha. See Bone-anchored cochlea stimulator (Baha). conventional hearing aids versus, 384–385 electromagnetic, 385 Envoy Esteem, 386t, 391–394, 393f historical background of, 385–386 MET, 386t, 394, 395f piezoelectric, 385 requirements for, 384 RION, 386, 386t–387t, 387f Soundtec, 386t, 391, 392f TICA, 386–389, 386t, 388f Vibrant Soundbridge, 386t, 389–391, 390f–391f, 391t reasons patients do not wear, 383 Hearing loss in aural atresia, 57 following cartilage tympanoplasty, 137 with chronic otitis media surgery, 233–234, 234f complicating operations for chronic ear infections, 128 conductive audiometry in, 163 Baha for, 398 differential diagnosis of, 509 inner ear, 272 in only hearing ear, Baha for, 399 stapedectomy and, 298–299, 300t after stapes surgery, 318 following surgery for facial nerve trauma, 360 after endolymphatic sac surgery, 418 with labyrinthine fistula repair, 231
821
Hearing loss (Continued) mastoidectomy and, 87 mixed, Baha for, 398 noise-induced, atresiaplasty and, 68 with perilymphatic fistulas, pathophysiology of, 325 risk of, with mastoidectomy, 210 sensorineural with chronic otitis media surgery, 233–234 with labyrinthine fistulas, 228 following laser stapedectomy, 272 mastoidectomy and, 86 progression of, after stapes surgery, 317 stapedectomy and, 298–299, 298t stapes surgery and, 316 single-sided, Baha for, 399 with translabyrinthine approach for acoustic tumors, 591 Helium-neon laser, 281 Hemangiomas, cavernous, 806–807 Hematomas, complicating operations for chronic ear infections, 128 Hemifacial spasm botulinum toxin for, 529 microvascular decompression for, 529–530 complications of, 530, 531t operative technique for, 528f, 529–530 patient selection for, 529 results with, 530 Hemorrhage adenoidectomy and, 87 from carotid artery, with chronic otitis media surgery, 241 from jugular bulb, with chronic otitis media surgery, 240–241 following limited temporal bone resection, 40 with retrosigmoid approach to cerebellopontine angle tumors, 616 from sigmoid sinus, with chronic otitis media surgery, 238–240, 239f–240f from superior petrosal sinus, with chronic otitis media surgery, 240 following transcochlear approach, 639 with translabyrinthine approach for acoustic tumors, 599 Hemostasis canal injection for, in stapes surgery, 307 for perilymphatic fistula surgery, 330 for retrosigmoid approach, 614 for tympanoplasty, 152 Hemotympanum, idiopathic, 74 diagnosis of, 76 recurrent, mastoidectomy and, 87 tympanostomy tubes for, 80 Herniation of brain, 245–252, 246f–247f computed tomography of, 247, 247f diagnosis of, 246–248, 247f–248f magnetic resonance imaging of, 247, 247f pathogenesis of, 245–246, 247f treatment of, 248–250, 248f–249f Herpes simplex virus, Bell’s palsy and, 335 Herpes zoster cephalicus, 338 Herpes zoster oticus, 338 Histologic evaluation in temporal bone malignancies, 44 Honeycomb bone, 363 Hook for vestibular neurectomy, 13, 14f House-Urban middle fossa retractor, 16, 16f Hydraulic chair for undersurface graft tympanoplasty, 142 Hydrocephalus otitic with acute infections, 186 treatment of, 193
822
Index
Hydrocephalus (Continued) following retrosigmoid approach to cerebellopontine angle tumors, 617 Hydrocortisone for acute otitis media, 110–111 Hydrops endolymphatic, pathophysiology of, 413–414 inner ear, aural fullness and, 99f Hydroset for ossicular prostheses, 162 Hydroxyapatite. See also Bone cement. for ossicular reconstruction, 166 for ossicular replacement, 162 with polyethylene (HAPEX), for ossicular replacement, 162, 166, 167f Hyperacusis after stapes surgery, 317 in superior canal dehiscence syndrome, 507–508 Hyperkinesis with facial reanimation, surgical management of, 770–771 Hypertension, neurogenic, microvascular decompression for, 532–533 complications of, 533 operative technique for, 532 patient selection for, 532 results with, 528f, 532–533 Hypoglossal nerve rehabilitation after neurotologic skull base surgery and, 578 schwannomas, in neurofibromatosis 2, 695 Hypoglossal/facial anastomosis for facial nerve paralysis, 755–764 end-to-side, via facial nerve translocation from fallopian canal, 759, 760f nerve growth factors and conduits for, 757–758 newer modifications of, 758–759, 758f–759f patient selection for, 755–756 results with, 759–761, 760t–761t surgical technique for, 756–757, 756f–757f Hypokinesis with facial reanimation, 770–771
I Immune function, depressed, failed antibiotic therapy due to, 113 Immunotherapy for otitis media, chronic, with effusion, 78 Implantable hearing aids. See Hearing aids, implantable. Incudostapedial joint division of, in stapes surgery, 309 fused, stapedectomy and, 295 identification of, 309 Incudostapedial joint prostheses, 167 Incus absent with mobile stapes and fixed malleus, ossicular reconstruction for, 164 with mobile stapes and mobile malleus, ossicular reconstruction for, 164, 164f dislocation of with facial nerve trauma, 354 during laser stapedectomy, 271 stapes surgery and, 312–313 erosion of revision stapedectomy and, 287 stapedectomy and, 299 fixation of, stapes surgery and, 310f, 313 foreshortened, with fixed footplate or prior stapedectomy, ossicular reconstruction for, 165, 165f
Incus (Continued) mobile with mobile footplate and malleus and absent stapes superstructure, ossicular reconstruction for, 164 with mobile malleus and fixed footplate, ossicular reconstruction for, 164 necrosis of with mobile stapes and malleus, ossicular reconstruction for, 164, 164f after stapes surgery, 318, 319f partial absence of, stapedectomy and, 295–297, 296f Incus/bridge prosthesis, 167 Infection(s). See also specific infections. acute, 183–194 clinical presentation of, 185–186 clinical significance of, 183 complications of aural, 183 intracranial, 183, 184f less obvious, 186, 187f obvious, 185 pathogenesis of, 183, 185f definition of, 183 diagnosis of, 186–188, 188f etiology of, 183 imaging of, 189 treatment of, 188–193 following canal wall reconstruction tympanomastoidectomy, 180–181 chronic, complications of operations for, 128 complicating operations for chronic ear infections, 128 following infratemporal fossa surgery, 663 respiratory upper, after stapes surgery, 317 viral, otitis media and, 75 sequestered nidus of, failed antibiotic therapy due to, 113 wound with cochlear implantation, 378 with retrolabyrinthine vestibular neurectomy, 451 after stapes surgery, 317 with surgery for facial nerve trauma, 360 Infection control for tympanoplasty, 151–152 Infratemporal fossa anterior transfacial (facial translocation) approach for, 660–661, 661f–662f anesthesia for, 652 diagnostic and staging work-up for, 649–650, 650f, 650t patient selection for, 649 pitfalls and complications of, 663–664 postoperative care for, 662–663 reconstructive considerations for, 651 rehabilitation considerations for, 650–651 lateral Fisch infratemporal approaches for anesthesia for, 652 diagnostic and staging work-up for, 649–650, 650f, 650t patient selection for, 649 pitfalls and complications of, 663–664 postoperative care for, 662–663 reconstructive considerations for, 651 rehabilitation considerations for, 650–651 lateral Fisch infratemporal fossa approaches for, 658–660, 660f type B, 658–659 type C, 659–660 type D, 660 postauricular (transtemporal) approach to, 657–658, 658f–659f
Infratemporal fossa (Continued) anesthesia for, 652 diagnostic and staging work-up for, 649–650, 650f, 650t patient selection for, 649 pitfalls and complications of, 663–664 postoperative care for, 662–663 reconstructive considerations for, 651 rehabilitation considerations for, 650–651 preauricular (subtemporal) approach to, 652–657, 653f, 655f–657f anesthesia for, 652 diagnostic and staging work-up for, 649–650, 650f, 650t patient selection for, 649 pitfalls and complications of, 663–664 postoperative care for, 662–663 reconstructive considerations for, 651 rehabilitation considerations for, 650–651 transorbital approach to, 661–662 anesthesia for, 652 diagnostic and staging work-up for, 649–650, 650f, 650t patient selection for, 649 pitfalls and complications of, 663–664 postoperative care for, 662–663 reconstructive considerations for, 651 rehabilitation considerations for, 650–651 Inner ear conductive hearing loss, 272 Instrumentation, 1–20 for adenoidectomy, 81 for Baha implant surgery, 402 for cochlear implant surgery, 15, 15f for endolymphatic sac surgery, 10–11, 11f, 19 for mastoidectomy, 8–9, 81 for middle cranial fossa surgery, 16, 16f–17f, 19–20 for middle cranial fossa vestibular neurectomy, 430, 431f for neurologic procedures, 11–12, 12f–15f for neurologic surgery, 19 for retrosigmoid approach, 607–608 for stapes surgery, 5, 6f–7f, 18 for tympanoplasty, 7–8, 9f–11f, 18 for tympanostomy tubes, 80–81, 82f for vestibular neurectomy, 13, 14f Internal auditory canal anatomy of, 442, 443f closure of, for retrosigmoid approach, 612f, 614 decompression of, without tumor removal, middle fossa craniotomy with, for vestibular schwannomas, 697 exposure of, 433–435, 436f for retrosigmoid approach, 609–611, 610f extended middle cranial fossa approach to, 641–648, 642t advantages of, 647 limitations of, 645–647 surgical anatomy and, 641–642, 642f surgical technique for, 642–645, 643f–646f technique for, 641 skeletonization of, in transcochlear approach, 634, 634f tumors with extension into, as contraindication to retrosigmoid approach, 606 Internal carotid artery anatomy of, 799–800 aneurysms of high cervical, 805 petrous, 803–805 management and treatment of, 804–805 presentation and pathophysiology of, 803–804 Irrigation for acute otitis media, 109
Index
J
Kanamycin, ototoxicity of, 494 Keratosis obturans, canalplasty for, 29 Klebsiella pneumoniae, otitis media and, 74
Labyrinthine fistulas (Continued) secondary to chronic otitis media, 227–228, 228t Labyrinthitis with acute infections, 185 serous, after stapes surgery, 317 treatment of, 189 Laser(s) comparison of, 284–285, 285f limitations of, 287–288 physics of, 281–282, 282f–284f safety and use of, for stapes surgery, 307 types of, 281, 282t use of, during 1990s to 2007, 285, 286t Laser myringotomy, 81–83 Laser stapedectomy. See Stapedectomy, laser. Lateralization, atresiaplasty and, 68 Lermoyez’s syndrome, 414 Liberatory maneuver, 468 Lid position, assessment of, for lower lid reapposition, in facial paralysis, 742–745 Lid stents for lower lid reapposition in facial paralysis, 747–749 Lid suture taped to cheek in facial paralysis, 750–752, 751f Lower lid reapposition procedures in facial paralysis, 742–749 brow elevation for, 749–750 canthoplasty for, 742 fascia lata suspension of lower lid for, 747, 748f lateral canthoplasty for, 745–747, 746f lid position assessment and, 742–745 lid stents for, 747–749 medial canthoplasty for, 744f, 745 midface support for, 749 Lower lip rehabilitation for facial reanimation, 770 Luc’s abscess, 186
L
M
Labyrinth, chemical treatment of, 493–506 intramuscular streptomycin for, 494–496 clinical studies of, 494 results with, 496 subtotal vestibulectomy using, 494–496 intratympanic corticosteroids for, 502–503 clinical studies of, 502–503 experimental studies of, 502 intratympanic gentamicin for, 497–501 clinical studies of, 497–501, 498t–499t, 499f–500f experimental studies of, 497 results with, 501–502, 501t streptomycin application to lateral semicircular canal for, 496–497 clinical studies of, 496 experimental studies of, 496 results with, 496–497 surgical method for, 496 Labyrinthectomy incomplete, with transcanal labyrinthectomy, 488–489 partial, 473–474 transcanal. See Transcanal labyrinthectomy. in transcochlear approach, 634, 634f transmastoid, translabyrinthine vestibular nerve section vs., 458 Labyrinthine fistulas iatrogenic, 231–233, 232f–233f intraoperative management of, 229–231, 229f–230f mastoidectomy for, 197, 205–206 patient counseling about, 198
Magnetic resonance imaging. See under specific conditions and procedures. Malleus fixation of, stapes surgery and, 310f, 313 fixed with fixed stapes, but absent incus, ossicular reconstruction for, 165, 165f with mobile stapes and absent incus, ossicular reconstruction for, 164 stapedectomy and, 257, 260f, 295, 297t, 300 fracture of, with facial nerve trauma, 354 mobile with fixed stapes, but absent incus, ossicular reconstruction for, 165, 165f with mobile footplate and incus and absent stapes superstructure, ossicular reconstruction for, 164 with mobile incus and fixed footplate, ossicular reconstruction for, 164 with mobile stapes and absent incus, ossicular reconstruction for, 164, 164f with mobile stapes and incus necrosis, ossicular reconstruction for, 164, 164f Malleus/oval window prosthesis, 167 Mastoid exenteration of, in mastoidectomy, 199, 201f obliteration of. See Tympanomastoidectomy, canal wall reconstruction, with mastoid obliteration. Mastoid cavity draining, management of, 113 exteriorized, indications for, 197
Jahrsdoerfer’s grading system for congenital aural atresia, 57, 57t “Joker,”, 16, 16f Jugular bulb injury with chronic otitis media surgery, 240–241 Jugular bulb tumors, 551 Jugular foramen tumors arteriography in, 553 audiometry and, 553 brain perfusion and flow studies in, 553 computed tomography in, 552–553, 555 embolization in, 553 magnetic resonance imaging in, 552–553 schwannomas as, 552 surgery for, 551–568 complications of, 564f–567f, 565–566 patient counseling for, 554 patient selection for, 551–552, 552t preoperative evaluation for, 553 results of, 564–565 surgical approaches for, 554–564 complete carotid mobilization as, 562–564 fallopian bridge, 561–562, 563f infratemporal fossa, 557–561, 557f–562f mastoid-extended facial recess, 555, 555f mastoid-neck, 555–557, 556f–557f transcanal, 554–555, 554f transcondylar, 562, 563f
K
823
Mastoid surgery. See also Mastoidectomy. complications of, 128 obliteration procedure as, 209, 216–217 reconstructive, 209, 217–218, 218f risks of, 128 Mastoid tip, mastoidectomy and, canal wall down, 212–213, 213f Mastoidectomy. See also Tympanomastoi dectomy. canal wall down procedure for, 209–220 atticotomy and, 209, 215–216, 216f decision making for, 210 definitions for, 209 dural venous sinuses and, 219 facial nerve and, 219–220, 219f indications for, 209–210 mastoid obliteration procedure and, 209, 216–217 mastoid reconstruction procedure and, 209 modified radical (Bondy), 209, 213–214, 214f patient counseling for, 210–211 patient positioning for, 211 patient preparation for, 211 pitfalls in, 219 postoperative care for, 218–219 preoperative evaluation for, 210 radical, 209, 216, 217f radical cavity reconstruction and, 217–218, 218f revision, 218 surgical techniques for, 211–213 facial ridge lowering adequacy and, 211–212, 212f mastoid tip management and, 212–213, 213f meatoplasty adequacy and, 213, 214f saucerization adequacy and, 211, 211f with tympanoplasty, 214–215, 215f tympanoplasty with, 209 canal wall up technique for. See Mastoidectomy, intact canal wall procedure for. complications of, 87 draping for, 80 for dural venous thrombophlebitis, 190–191, 191f follow-up for, 84 instrumentation for, 8–9, 81 intact canal wall procedure for, 195–208 canal wall defect repair and, 206 for defects resulting from disease, 205f, 206 for defects resulting from surgery, 206 complete canal wall up, for mastoiditis, 188 controversy over, 196 definitions for, 195 evolution of technique and, 195–196 exteriorized mastoid cavity and, indications for, 197 facial nerve and, 203 mastoid segment of, 203, 204f–205f tympanic segment of, 203, 204f–205f indications for, 196–197 labyrinthine fistula management and, 205–206 modified radical, 195 patient preparation for, 198 postoperative care for, 203 preoperative counseling for, 198 preoperative evaluation for, 197–198 preoperative preparation for, 197–198 radical, 195 surgical technique for, 198–203 completion of operation and, 201–202 elimination of disease and, 201, 204f–205f
824
Index
Mastoidectomy (Continued) mastoid exenteration and, 199, 200f–201f opening facial recess and, 199–201, 200f–201f������������������ , 204f–205f plastic sheeting and, 201, 204f–205f for removal of middle ear disease, 198 tympanoplasty with, 195 for labyrinthine fistulas, 197 laser, 81–83 modified radical, 214–215, 215f for otitis media, chronic, with effusion, 79 for petrositis, 189, 189f–190f pitfalls with, 86 postoperative care for, 84 preoperative audiometry for, 210 preoperative preparation for, 80 results with, 86 surgical site preparation and draping for, 80 surgical technique for, 174f–177f, 175–176 laser, 83–84, 85f in transcochlear approach, 633, 634f tympanoplasty with, 152, 195 Mastoiditis with acute infections, 185–186 masked, with acute infections, 186 treatment of, 188–189 Meatoplasty, mastoidectomy and, canal wall down, 213, 214f Meckel’s cave, tumors with extension into, retrosigmoid approach for, 606 Meniere’s disease, 414–417 audiometry in, 415 aural fullness and, 96–97 cochleosacculotomy for, 477–482 patient selection for, 478 rationale for, 477–478 results with, 481–482 surgical technique for, 478–481, 479f–481f definition of, 411 diagnosis of, 415 endolymphatic sac surgery for. See Endolymphatic sac surgery. epidemiology of, 414 superior canal dehiscence syndrome vs., 510 theories explaining symptoms of, 414 treatment of, 415–416, 457 chemical. See Labyrinth, chemical treatment of. intratympanic, 416–417 retrolabyrinthine vestibular neurectomy for. See Vestibular neurectomy, retrolabyrinthine. surgical. See Endolymphatic sac surgery; Transcanal labyrinthectomy; Vestibular neurectomy, middle cranial fossa; Vestibular neurectomy, retrosigmoid (suboccipital). vertigo in, 456 Meningiomas in neurofibromatosis 2, 695, 696f, 696t removal of, in transcochlear approach, 635–636, 636f–639f stereotactic radiosurgery of, 796 Meningitis with acute infections, 185 aseptic, following retrosigmoid approach to cerebellopontine angle tumors, 617 bacterial organisms causing, 183 following retrosigmoid approach to cerebellopontine angle tumors, 617 with cochlear implantation, 379 with facial nerve trauma, 355
Meningitis (Continued) with retrolabyrinthine vestibular neurectomy, 451 following transcochlear approach, �������� 640 following translabyrinthine approach for acoustic tumors, 599–600 treatment of, 192f, 193 Meningoencephaloceles, 245–252, 246f–247f computed tomography of, 247, 247f diagnosis of, 246–248, 247f–248f magnetic resonance imaging of, 247, 248f pathogenesis of, 245–246, 247f treatment of, 248–250, 248f–249f Meperidine (Demerol), setup of, 264 MET, 386t, 394, 395f Microraspatory, angled, 430, 431f Microscope, operating, 3 Microtia, 56 repair of, before atresiaplasty, 60 Microvascular decompression, 523–536 audiometric tests for, 523 brainstem auditory evoked potentials for, 523, 524f–526f for glossopharyngeal neuralgia, 531–532 complications of, 532 operative technique for, 532 patient selection for, 531–532 results with, 532 for hemifacial spasm, 529–530 complications of, 530, 531t operative technique for, 528f, 529–530 patient selection for, 529 results with, 530 for neurogenic hypertension, 532–533 complications of, 533 operative technique for, 532 patient selection for, 532 results with, 528f, 532–533 patient positioning for, 526–527, 528f for positional vertigo and tinnitus, 530–531 complications of, 531 operative technique for, 531 patient selection for, 528f, 530–531 results with, 531 preoperative evaluation for, 523, 524f–526f for trigeminal neuralgia, 523–529 in children, 527 complications of, 529 operative technique for, 526–527, 528f patient selection for, 523–526 results with, 527–529, 528f Midazolam (Versed) for revision stapedectomy, 288 setup of, 264 Middle cranial fossa approach, extended, 641–648, 642t advantages of, 647 limitations of, 645–647 surgical anatomy and, 641–642, 642f surgical technique for, 642–645, 643f–646f technique for, 641 Middle cranial fossa surgery draping for, 15 instrumentation for, 16, 16f–17f, 19–20 operating room setup for, 3, 5f patient preparation for, 15, 16f vestibular neurectomy as. See Vestibular neurectomy, middle cranial fossa. Middle cranial fossa vestibular neurectomy. See Vestibular neurectomy, middle cranial fossa. Middle ear allergic inflammation in, otitis media and, 76 exposure of, for tympanoplasty, 154, 155f–156f
Middle ear (Continued) preparation of, for undersurface graft tympanoplasty, 144, 147f Middle ear effusion, asymptomatic, 74 Middle fossa craniotomy, internal auditory canal decompression without tumor removal with, for vestibular schwannomas, 697 Middle fossa retractor articulated, 430, 431f introduction of, 433, 436f Midface support for lower lid reapposition, in facial paralysis, 749 Migraine-related dizziness, 456, 510 Mimmix for ossicular prostheses, 162 Minor’s syndrome. See Superior semicircular canal dehiscence syndrome. Mixed hearing loss, Baha for, 398 Mondini malformation,192f Moraxella catarrhalis otitis media and, 77 otitis media due to, 74 acute, 109 Mucosal disease. See Otitis media. Muscle plication for facial reanimation, 770 Musculoskeletal disorders, aural fullness and, 96–97 Myectomy, regional, for hyperkinesis, with facial reanimation, 770–771 Myringoplasty complications of, 128 risks of, 128 Myringotomy for facial paralysis, 189 for labyrinthitis, 189 for mastoiditis, 188 for meningitis, 192f for patulous eustachian tube, 97 for tube placement. See Tympanostomy tubes.
N Nasal obstruction, otitis media and, ������� 75 Nasal septum, tympanoplasty and, 150 Nasopharynx, posterior wall of, deep removal of, during adenoidectomy, 86 Neodymium:yttrium-aluminum-garnet (Nd:YAG) laser, 281 Neomycin for acute otitis media, 110–111 Nerve grafts for facial reanimation, 765–766 Nerve growth factors for hypoglossal/facial anastomosis, 758 Nerve substitution for facial reanimation, 766 Neurectomy peripheral, for tic relief, 525 vestibular. See Vestibular neurectomy. Neurofibromatosis 2, 691–702 definition of, 691–692, 692t eye findings in, 696 family history of, 693 Gardner form of, 692 genetic testing for, 699 imaging studies in, 692 magnetic resonance imaging in, 692 management of, 698–699 meningiomas in, 695, 696f, 696t molecular genetics of, 692–693 neurofibromatosis 1 differentiated from, 691 prevalence and incidence of, 692, 692f screening for, 693 spinal tumors in, 695–696
Index Neurofibromatosis (Continued) symptoms of, 696t tumor types in, 693–694, 694f vestibular schwannomas in, 694–695 hearing loss and, 695 treatment option(s) for, 696–698 auditory brainstem implant as, 698, 698f hearing preservation and, 696–697. See also Auditory implants. middle fossa craniotomy and internal auditory canal decompression without tumor removal as, 697 non-hearing preservation, translabyrinthine/suboccipital approach and total tumor removal as, 697–698 observation without surgery as, 697 retrosigmoid craniotomy with partial removal as, 697, 697f stereotactic irradiation as, 698 Wishart form of, 691–692 Neurofibromatosis 1, neurofibromatosis 2 differentiated from, 691 Neurogenic hypertension, microvascular decompression for, 532–533 complications of, 533 operative technique for, 532 patient selection for, 532 results with, 528f, 532–533 Neurolysis for hyperkinesis with facial reanimation, 770–771 Neuromas, acoustic auditory implants and. See Auditory implants. with chronic otitis media, retrosigmoid approach in, 606 combined therapy of, retrosigmoid approach in, 606 resection of, for retrosigmoid approach, 611–614, 612f translabyrinthine approach for. See Acoustic tumors, translabyrinthine approach for. transotic approach for, 621 Neurosurgery for brain abscess, 191–193, 191f draping for, 11, 11f–12f instrumentation for, 11–12, 12f–15f, 19 for subdural abscesses, 193 Neurotologic surgery skull base cranial nerve deficit rehabilitation after, 569–580, 570f for abducens nerve, 570–571 for cervical sympathetic chain, 578 for glossopharyngeal nerve, 571 for hypoglossal nerve, 578 patient evaluation for, 569 for spinal accessory nerve, 577–578 for trigeminal nerve, 569–570, 570f, 573f for vagus nerve, 571–577, 573f–574f, 576f transtemporal, 519–522 collaboration in, 520–521 nomenclature for, 520, 520f organizational framework for, 519, 520f vascular considerations in, 801–803 with middle fossa approaches, 803 of Kawase, 803 with petrous carotid exposure, 803 with transpetrosal approaches, 801–803, 802f retrolabyrinthine, 802 transcochlear, 803 translabyrinthine, 802–803
Neurovascular complications following infratemporal fossa surgery, 663–664 Nitinol-polytef piston for laser stapedectomy, 269 Noncompliance, failed antibiotic therapy due to, 112 Nose, allergic inflammation in, otitis media and, 76 Nystagmus in benign paroxysmal positional vertigo, 467 perilymphatic fistulas and, 325–326
O Oculomotor nerve monitoring, 782 Operating microscope, 3 setup of, for stapes surgery, 307, 308f Operating room, 1–3 drills in, 1–2, 2f–3f electrocautery equipment in, 1 general equipment for, 18 for middle cranial fossa procedures, 3, 5f observers in, 1, 2f operating microscope in, 3 setup of for laser stapedectomy, 263–264, 264f for neurologic surgery, 3 for routine otologic surgery, 3, 4f for tympanoplasty, outer surface grafting techniques, 120, 121f suction in, 1, 2f Operating table, 1, 2f positioning at, 3, 4f Osseointegration, 398 Ossicular chain. See also Incus; Malleus; Stapes enties. damage to, with facial nerve trauma, 354 dislocation of, during stapedectomy, 295 total fixation of, ossicular reconstruction for, 165, 165f tympanoplasty and, 222 Ossicular prostheses extrusion of, 237 materials for, 161 for partial ossicular replacement, 161–162 selection of, 165–167, 166f–167f for total ossicular replacement, 161–162 Ossicular reconstruction, 161–172 factors influencing outcome with, 170 historical background of, 161–162 patient evaluation for, 162–163 patient selection for, 162–163 postoperative care for, 170 prosthesis selection for, 165–167, 166f–167f surgical considerations in, 163–165, 164f–165f surgical technique for, 167–170 cartilage preparation and, 169 exposure and assessment and, ��������� 167, 168f prothesis placement and, 169–170, 169f tympanoplasty and, 152 Ossiculoplasty, complications of, 237 Osteogenesis imperfecta, stapes surgery and, 313 Osteonecrosis of tympanic bone, canalplasty for, 29 Osteoradionecrosis of tympanic bone, canalplasty for, 29 Otic capsule exenteration, transotic approach for cerebellopontine angle lesions and, 622–624, 623f Otitis externa, chronic Baha for, 399
825
Otitis media acute chronic suppurative otitis media arising from, 108 with effusion, 73 epidemiology of, 74 guidelines for management of, 88 with intact tympanic membrane, 109 with open tympanic membrane, 109 pathophysiology of, 75 recurrent, 73 treatment of, 77, 77t after stapes surgery, 316 treatment of, 77, 77t without effusion, 73 chronic, 227–244 acoustic neuroma with, retrosigmoid approach in, 606 with effusion pathophysiology of, 75–76 treatment of, 77–79 labyrinthine fistulas and iatrogenic, 231–233, 232f–233f secondary to otitis media, 227–231, 228t, 229f–230f ossicular reconstruction for, 162 preoperative counseling in, 227 second-stage tympanoplasty for, 222–223 stapedectomy in, 302 suppurative, 73–74, 107–109 bacteriology of, 108 definition of, 107–108 etiology of, 108 granulation tissue in, 108 pathophysiology of, 108–109 surgery for, complications of dural injury as, 237–238, 238f–239f facial nerve grafting and, 236–237, 236f facial nerve injury as, ��������� 234, 235f labyrinthine fistulas as, 231–233, 232f–233f sensorineural hearing loss as, 233–234, 234f vascular injury as, 238–241, 239f–240f definition of, 73–74 diagnosis of, 76 with effusion, 73 allergy treatment for, 87–88 biofilms and, 88 guidelines for management of, 88 risk factors for, 74 microbiology of, 74 tympanoplasty and, 221–223, 224f tympanostomy tubes for. See Tympanostomy tubes. Otoacoustic emissions, monitoring hearing using, 781 Otologic drills, 1–2, 2f–3f Otorrhea cerebrospinal fluid. See Cerebrospinal fluid otorrhea. in chronic suppurative otitis media, 73–74 with facial nerve trauma, 354–355 tympanoplasty and, 150–151 tympanostomy tubes and, 86–87 biofilm formation and, 88 Otosclerosis far advanced, stapedectomy for, 301 juvenile, partial stapedectomy for, 279 obliterative laser stapedectomy and, 271 stapedectomy and, 297 stapes surgery and, 314, 315f
826
Index
Otosclerosis (Continued) stapedectomy for, total. See Stapedectomy, total. surgery for. See Stapedectomy; Stapes surgery. Otosclerotic inner ear syndrome after stapes surgery, 317 Otosclerotic regrowth after stapes surgery, 318, 319f Oval window deep, stapedectomy and, 261 narrow niche and, stapes surgery and, 314 narrow oval window niche and promontory drilling and, stapedectomy and, 297 protection of, with labyrinthine fistula repair, 232, 232f–233f tissue seal at, 312 Oval window drill-out, atresiaplasty and, 69 Oval window seal, stapes surgery and, 312
P Pain following transcochlear approach, 639–640 Palisade technique for cartilage tympanoplasty, 133–135, 134f–136f Palpebral spring implantation enhanced, for upper lid reanimation, in facial paralysis, 738–740, 739f, 741f for upper lid reanimation, in facial paralysis, 736f, 737–738 Paper patching for tympanic membrane closure, 114f, 115–116 Paragangliomas stereotactic radiosurgery of, 796 of temporal bone, radiation therapy for, 715 Paranasal sinuses, endoscopic endonasal approaches to, 667–680 complications of, 679 coronal plane, 668, 670f anterior fossa, 676–677 infratemporal fossa, 677–679, 678f lower infratemporal fossa/parapharyngeal space, 679 posterior fossa, 679 supraorbital, 676 transcondylar, 679 transorbital, 676 transpterygoid, 676, 677f–678f historical background of, 667 learning curve for, 679 reconstruction and, 679–680 sagittal plane, 668, 668f foramen magnum, 675 middle clivus, 674–675, 674f superior clivus, 673–674, 674f transclival, 673–675 transcribriform, 669–671, 671f transfrontal, 668–669 transodontoid, 675, 675f transplanum/transtuberculum, 671–672, 672f transsellar, 672–673, 673f Particle repositioning technique, 468–469, 469f Patient positioning. See under specific procedures. Patulous eustachian tube reconstruction procedure, 98–101 Pediatric patients Baha in, 408–409 ossicular reconstruction in, 163 otosclerosis in, partial stapedectomy for, 279 stapedectomy in, 302 stapedotomy in, 312 trigeminal neuralgia in, microvascular decompression for, 527
Penetrating auditory brainstem implant, 703. See also Auditory implants. Peptococcus spp., otitis media and, 74 Peptostreptococcus spp., otitis media and, 74 Perichondrial grafts harvesting of, 158f, 159 for undersurface graft tympanoplasty, 144, 146f Perichondrium/cartilage island flap for tympanoplasty, 132–133, 132f–133f placement problems with, 136–137 poor fit of, 135–136 Perilymph, excessive flow of, stapedectomy and, 259 Perilymphatic fistulas, 323–334, 324f acquired, 323 congenital, 323 hearing loss and, pathophysiology of, 325 idiopathic, 323 implosive and explosive, 323 recurrent, 331 after stapes surgery, 320 superior canal dehiscence syndrome vs., 510 surgery for anesthesia for, 327, 330 dressings and, 330 instruments for, 329 patient counseling and, 326–327 patient selection for, 325 pitfalls with, 330 postoperative care for, 330 preoperative evaluation for, 325–326, 327f preoperative preparation for, 327–329 results with, 330–331 surgical technique for, 328f, 329–330 Perilymphatic gusher, stapes surgery and, 314–316 Petroclival area, petrosal approach to, 681–690 closure and, 687–688 combined petrosal variations of, 681–682, 682f preoperative evaluation for, 682–683 preoperative preparation for, 683 results with, 688–689, 688f surgical techniques for, 683–687 middle fossa exposure and, 685–687 posterior exposure and, 683–685, 684f–687f Petrosectomy, subtotal, 544t, 546, 546f transotic approach for cerebellopontine angle lesions and, 622, 623f Petrositis with acute infections, 186 treatment of, 189, 189f–190f Petrous apex lesions drainage procedures for, 537–550, 538t complications of, 544–545, 544t computed tomography and, 537, 538t infralabyrinthine, 539f, 540–541, 544t pitfalls of, 541 middle fossa approach for, 544t, 546, 549f patient counseling for, 540 patient selection for, 538, 539f preoperative evaluation for, 538–540 results with, 544 subtotal petrosectomy for, 544t, 546, 546f transcanal infracochlear approach for, 541–544, 542f, 544t translabyrinthine, 544t transsphenoidal approach for, 544t, 545–546 magnetic resonance imaging and, 537, 538t
Petrous bone vascular anatomy of, 799–801 dural venous sinuses and, 801 external carotid artery and, 800 internal carotid artery and, 799–800 vertebrobasilar arteries and, 800–801 vascular lesions of, 803–807 cavernous malformations and cavernous hemangiomas as, 806–807, 808f dural arteriovenous malformations as, 805–806 internal carotid aneurysms as, 803–805 high cervical, 805 petrous, 803–805 tentorial fistulas as, 806 transverse and sigmoid fistulas as, 806 vertebrobasilar aneurysms as, 805 vascular tumors of, 807–813 cerebral revascularization and bonnet bypass and, 812–813, 812f–813f preoperative embolization and, 807–811 glomus tumors and, 807–809, 810f–811f hemangioblastomas and, 809–811 hemangiomas and, 811 Physiologic monitoring during endolymphatic sac surgery, 10 Pilots, stapedectomy in, 302 Plastic sheeting for tympanoplasty, 223 Platform posturography for translabyrinthine vestibular neurectomy, 458–459 Plester ossicular prosthesis, 167 Pneumatization, lack of, atresiaplasty and, 60 Pneumocephalus with facial nerve trauma, 355 Polyethylene, thermal fused (Polycel), for ossicular reconstruction, 161–162 Polyethylene sponge (Plastipore) for ossicular reconstruction, 161–162, 166 Polymyxin B for acute otitis media, 110–111 Positional vertigo microvascular decompression for, 530–531 complications of, 531 operative technique for, 531 patient selection for, 528f, 530–531 results with, 531 paroxysmal, benign, 467–476 pathophysiology of, 468 posterior semicircular canal occlusion for patient counseling and, 468–470, 469f patient selection for, 470 preoperative evaluation for, 471 results with, 471–473 surgical technique for, 471, 472f Posterior canal dehiscence, posterior semicircular canal occlusion for, 474 Posterior inferior cerebellar artery, anatomy of, 800–801 Posterior semicircular canal occlusion for benign paroxysmal positional vertigo patient counseling and, 468–470, 469f patient selection for, 470 preoperative evaluation for, 471 results with, 471–473 surgical technique for, 471, 472f for posterior canal dehiscence, 474 for superior semicircular dehiscence, 473 Posturography, platform, for translabyrinthine vestibular neurectomy, 458–459 Potassium iodide for patulous eustachian tube, 97 Potassium titanyl phosphate crystal laser, 281, 282t, 284 Prednisone for Bell’s palsy, 337–338 for Ramsay Hunt syndrome, 339
Index Premarin for patulous eustachian tube, 97 Prepontine cistern, extended middle cranial fossa approach to, 641–648, 642t advantages of, 647 limitations of, 645–647 surgical anatomy and, 641–642, 642f surgical technique for, 642–645, 643f–646f technique for, 641 Promontory, evaluation of, for cochlear implantation, 375 Propionibacterium acnes, otitis media and, 74 Proteus mirabilis, intracranial complications due to, 183 Pseudomonas, acute otitis media due to, 109 Pseudomonas aeruginosa chronic suppurative otitis media due to, 108 intracranial complications due to, 183 otitis media and, 74 otogenic complications due to, 183
Q Quix test for perilymphatic fistulas, 326
R Radiation therapy for facial nerve tumors, 365 for glomus jugulare tumors, 566 following limited temporal bone resection, 41 with radical temporal bone resection, 50 stereotactic, for vestibular schwannomas, 698 Radiofrequency rhizotomy, shortcomings of, 525 Radiosurgery for facial nerve tumors, 365 stereotactic, for skull base tumors. See Skull base tumors, stereotactic radiosurgery of. Ramsay Hunt syndrome, 338–339 Reanimation for facial paralysis. See Facial reanimation. of upper lid in facial paralysis, 735–737 enhanced palpebral spring implantation for, 738–740, 739f, 741f gold weight implantation for, 740–742, 741f, 742t palpebral spring implantation for, 736f, 737–738 silicone rod prosthesis implantation for, 742, 743f Respiratory infections upper, after stapes surgery, 317 viral, otitis media and, 75 Retrolabyrinthine vestibular neurectomy. See Vestibular neurectomy, retrolabyrinthine. Retrosigmoid craniotomy with partial removal of vestibular schwannoma, 697, 697f Retrosigmoid vestibular neurectomy. See Vestibular neurectomy, retrosigmoid (suboccipital). Revision stapedectomy, laser. See Stapedectomy, laser, revision. Rhinitis, allergic, otitis media and, 75 Rhinorrhea following infratemporal fossa surgery, 664 Rhizotomy glycerol, shortcomings of, 525 radiofrequency, shortcomings of, 525 RION, 386, 386t–387t, 387f Round window closure of, stapedectomy and, 259 obliteration of, stapes surgery and, 314 Rupture theory of Meniere’s disease, 414
S Sacculotomy, perilymphatic fistulas and, 325 Salicylic acid for patulous eustachian tube, 97 Salivary gland tumors, involving temporal bone, 33 limited temporal bone resection for. See Temporal bone resection, limited. Saucerization for mastoidectomy, canal wall down, 211, 211f Scalp flap, necrosis of, following infratemporal fossa surgery, 663 Schuknecht’s classification system for congenital aural atresia, 57 Schwannomas facial nerve, in neurofibromatosis 2, 694–695 glossopharyngeal, in neurofibromatosis 2, 695 hypoglossal, in neurofibromatosis 2, 695 jugular foramen, 552 trigeminal nerve, in neurofibromatosis 2, 694 vagal nerve, in neurofibromatosis 2, 695 vestibular in neurofibromatosis 2. See Neurofibromatosis 2, vestibular schwannomas in. stereotactic irradiation for, 698 Scutum defects, canalplasty for, 29–31, 30f Sedation See under specific procedures. Selector ultrasonic aspirator for vestibular neurectomy, 13, 14f Semicircular canal, lateral, streptomycin application to, 496–497 clinical studies of, 496 experimental studies of, 496 results with, 496–497 surgical method for, 496 Sensorineural hearing loss. See Hearing loss, sensorineural. Sigmoid sinus injury with chronic otitis media surgery, 238–240, 239f–240f Silicone rod prosthesis implantation for upper lid reanimation in facial paralysis, 742, 743f Silicone sheeting for tympanoplasty, 223 extrusion of, 225 Sinusitis, tympanoplasty and, 150 Skin grafts following limited temporal bone resection, 38f, 39–40 Skin reactions with Baha implant surgery, 407 Skull base, tumors with extension into, as contraindication to retrosigmoid approach, 606 Skull base tumors endoscopic endonasal approaches to surgery for, 667–680 complications of, 679 coronal plane, 668, 670f anterior fossa, 676–677 infratemporal fossa, 677–679, 678f lower infratemporal fossa/parapharyngeal space, 679 posterior fossa, 679 supraorbital, 676 transcondylar, 679 transorbital, 676 transpterygoid, 676, 677f–678f historical background of, 667 learning curve for, 679 reconstruction and, 679–680 sagittal plane, 668, 668f foramen magnum, 675 middle clivus, 674–675, 674f superior clivus, 673–674, 674f
827
Skull base tumors (Continued) transclival, 673–675 transcribriform, 669–671, 671f transfrontal, 668–669 transodontoid, 675, 675f transplanum/transtuberculum, 671–672, 672f transsellar, 672–673, 673f extreme lateral infrajugular transcondylar approach for resection of, 715–726 complications of, 724 results with, 724–726 surgical procedure for, 716–724, 718f–722f, 724f neurotological surgery for. See Neurotologic skull base, surgery���� �����������. stereotactic radiosurgery of, 785–798 CyberKnife, 794–795 dose distribution and, 795 localization and, 795 treatment delivery and, 795 treatment planning and, 794–795, 794f outcomes with, 791–794, 792f–793f patient selection for, 785–786 preoperative counseling for, 786 surgical technique for, 786–787 anesthesia and, 788 frame attachment and, 787–791, 788f gamma knife unit and, 786–787, 787f imaging and, 789 treatment and, 791 treatment planning and, 789–791, 789f–790f Solid state lasers, 281 Sonotubometry in patulous tympanic membrane, 95 Soundtec Direct Drive Hearing System, 386t, 391, 392f Spinal accessory nerve anatomy of, 443, 444f monitoring of, 782 in glomus tumor surgery, 555–556 palsy of, with jugular foramen schwannomas, 552 preservation of, in glomus tumor surgery, 556 rehabilitation after neurotologic skull base surgery and, 577–578 Spinal tumors in neurofibromatosis 2, 695–696 Squamous cell carcinoma of external auditory canal, staging of, 35–36, 35t of temporal bone, 33 Stapedectomy, 293–304 atrophic tympanic membrane and, 295 bilateral, 302 in children, 302 in chronic otitis media, 302 contraindications to, 254 dehiscent facial nerve and, 297 drill, laser stapedectomy vs., 311 in elderly patients, 302 in far advanced otosclerosis, 301 fixed malleus and, 295, 297t, 300 floating footplate and, 296f, 297–298, 298t fused incudostapedial joint and, 295 indications for, 254 instrumentation for, 6 intraoperative audiometry and, 293–294, 294f intraoperative audiometry for, 293–294, 294f laser, 263–274 complications of intraoperative, 271 postoperative, 272
828
Index
Stapedectomy (Continued) draping for, 264, 264f drill stapedectomy vs., 311 with floating footplate, 271 nitinol-polytef piston for, 269 in obliterative otosclerosis, 271 operating room setup for, 263–264 with overhanging facial nerve, 269–271 patient counseling about, 263 patient selection for, pitfalls in, 272–273 postoperative care for, 271–272 preoperative evaluation for, 263 preparation for, 263–264 results with, 272–273 revision, 281–292 advantages of, 287 anesthesia for, 288 comparison of lasers for, 284–285, 285f failed stapes surgery analysis and, 285–287, 287t historical background of, 282–288 laser development and, 283–284 laser physics and principles for, 281–282, 282f–284f, 282t laser use during 1990s to 2007 and, 285, 286t limitations of lasers and, 287–288 technique for, 288–290, 289f sedation for, 264 surgical technique for, 264–269, 264f–270f narrow oval window niche and promontory drilling and, 297 obliterative otosclerosis and, 297 ossicular dislocation during, 295 partial, 275–280 cup/piston stapes prostheses for indications for, 278 modification of, 278 historical background of, 275–276 in juvenile otosclerosis, 279 piston concept and, 275 results with, 278 stainless steel versus polytef prostheses for, 279 surgical technique for, 276–278, 277f wire/stapes prostheses for, 278–279 partial absence of incus and, 295–297, 296f patient counseling for, 253 patient selection for, 253–254 perilymphatic fistulas following, 323 in pilots, 302 pitfall(s) in, 257–261 chorda tympani nerve and, 257, 260f deep oval window as, 261 eardrum perforation as, 257 facial nerve abnormalities as, 257–259 floating footplate as, 259 intraoperative vertigo as, 259–261 malleus fixation as, 257, 260f obliterated footplate as, 259, 261f perilymph flow as, 259 round window closure as, 259 postoperative care for, 257 prior, foreshortened incus with, ossicular reconstruction for, 165, 165f revision, 298–301 for conductive hearing loss, 298–299, 300t laser. See Stapedectomy, laser, revision. recommendations for, 300–301 for sensorineural hearing loss, 298–299, 298t surgical findings in, 299–300, 300t surgical technique for, ������������������� 299, 296f
Stapedectomy (Continued) routine, 294–295, 296f for small air-bone gaps, 301–302 small fenestra technique for, 300 stapedotomy vs., 311 surgical technique for, 254–257, 256f, 258f total, 253–262 anesthesia for, 254 tympanic membrane perforation during, 295 tympanomeatal flap tears during, 295 Stapedial artery, persistent, stapes surgery and, 313 Stapedial footplate. See Footplate. Stapedotomy anesthesia for, 312 in children, 312 laser, laser type for, 312 prosthesis displacement at, 312 site for, 311 stapedectomy vs., 311 Stapes absent superstructure of, with mobile footplate, incus, and malleus, ossicular reconstruction for, 164, 164f displacement of, 315f, 318 fixation of, congenital, stapes surgery and, 313 fixed, with fixed or mobile malleus, but absent incus, ossicular reconstruction for, 165, 165f fracture of crura of, with facial nerve trauma, 354 mobile with fixed malleus and absent incus, ossicular reconstruction for, 164 with mobile malleus and absent incus, ossicular reconstruction for, 164, 164f mobile with mobile malleus and incus necrosis, ossicular reconstruction for, 164, 164f Stapes curettes, 6, 7f Stapes prosthesis(es) availability of, 311 ballottement of, 312 cup/piston indications for, 278 modifications of, 278 wire prostheses compared with, 278–279 displacement of, at stapedotomy, 312 extrusion of, 318 incus erosion and, 295 long, overinsertion of, 318, 319f loose coupling of, 318–320 nitinol, 288 placement of, 309 polytef, 299 preparation and loading of, 309 removal of, 309 Robinson fistulas with, 300 malfunction of, 299 sizes of, 311, 311t sizing of, 309, 310f stainless steel, Robinson, 294 stainless steel vs. polytef, 279 types of, 311 wire, incus erosion with, 295–297, 296f, 299 wire loop, 318, 319f Stapes superstructure, removal of, 309 Stapes surgery, 305–322. See also Stapedectomy; Stapedotomy. balloon ballottement and, 312 biscuit footplate and, 310f, 314
Stapes surgery (Continued) blood within vestibule and, 316 congenital stapes fixation and, 313 congenitally ectopic facial nerve and, 314 draping for, 4, 5f failed, analysis of, 285–287, 287t footplate fragments in vestibule and, 316 footplate/vestibular relationships and, 311–312 fractured footplate and, 316 incus dislocation and, 312–313 incus fixation and, 310f, 313 instrumentation for, 5, 6f–7f, 18 malleus fixation and, 310f, 313 narrow oval window niche and, 314 obliterative otosclerosis and, 314, 315f operative setup for, 3–7, 7f–8f osteogenesis imperfecta and, 313 oval window seal and, 312 overhanging facial nerve and, 314, 315f patient positioning for, 4–5 patient preparation for, 3, 5f perilymphatic gusher and, 314–316 persistent stapedial artery and, 313 preoperative evaluation for, 305–306 informed consent and, 306 medical conditions and, 305–306 physical examination in, 306 primary, complications after, 316–320 medical management of, 316–317 surgical management of, 317–320 prostheses for. See Stapes prosthesis(es). revision, surgical risk reduction in, 320 round window obliteration and, 314 sensorineural hearing loss and, 316 stapedectomy as. See Stapedectomy. stapedotomy as. See Stapedotomy. surgical technique prerequisites for residents and, 306–311 tympanic membrane perforation and, 312 tympanomeatal flap tears and, 312 tympanosclerosis and, 313 vascular anomalies and, 313 Staphylococcus, acute otitis media due to, 109 Staphylococcus aureus intracranial complications due to, 183 otitis media and, 74 otogenic complications due to, 183 Static slings for facial reanimation, 770 Stereotactic irradiation for vestibular schwannomas, 698 Stereotactic surgery for glomus jugulare tumors, 566–567 radiosurgery as of skull base tumors. See Skull base tumors, stereotactic radiosurgery of. for trigeminal neuralgia, shortcomings of, 525–526 Steroids for Bell’s palsy, 337–338 intratympanic, for inner ear conditions, 502–503 clinical studies of, 502–503 experimental studies of, 502 for otitis media, chronic, with effusion, 78 Streptococcus pneumoniae intracranial complications due to, 183 meningitis due to, with cochlear implantation, 374t, 379 otitis media due to, 74, 77 acute, 109 otogenic complications due to, 183 Streptococcus pyogenes intracranial complications due to, 183 otogenic complications due to, 183
Index Streptomycin application to lateral semicircular canal, for inner ear conditions, 496–497 clinical studies of, 496 experimental studies of, 496 results with, 496–497 surgical method for, 496 intramuscular, for inner ear conditions, 494–496 clinical studies of, 494 results with, 496 subtotal vestibulectomy using, 494–496 ototoxicity of, 493–494 Subdural abscesses with acute infections, 186 treatment of, 193 Suboccipital vestibular neurectomy. See Vestibular neurectomy, retrosigmoid (suboccipital). Suction in operating room, 1, 2f Suction tubes for stapes procedures, 7, 7f Superior petrosal sinus injury with chronic otitis media surgery, 240 Superior semicircular canal dehiscence syndrome, 456, 507–518 audiography in, 508, 508f aural fullness and, 96, 99f bilateral, 512 computed tomography in, 509, 510f diagnosis of, 96 diagnostic evaluation for, 507–509, 508f–510f differential diagnosis of, 509–510 long-term results with, 516 operative technique for, 512–515, 513f–516f posterior semicircular canal occlusion for, 473 postoperative care for, 515 preoperative decision making for, 510–512 symptoms of, 507 Suppurative otitis media, chronic. See Otitis media, Chronic suppurative. Supramid Extra for tympanoplasty, 223 Surgeons, tremor in, 307 Surgical drills, 1–2, 2f–3f Suture dehiscence, post-traumatic, canalplasty for, 31 Synkinesis with facial reanimation, 770–771
T Tarsorrhaphy suture, temporary, in facial paralysis, 751f, 752 Tast disturbance, complicating operations for chronic ear infections, 128 Teflon for patulous eustachian tube, 97 Temporal bone drainage procedures for tumors of. See Petrous apex lesions, drainage procedures for. encephaloceles of, 245–252, 246f–247f computed tomography of, 247, 247f diagnosis of, 246–248, 247f–248f pathogenesis of, 245–246, 247f treatment of, 248–250, 248f–249f fractures of cerebrospinal fluid otorrhea due to, 107 facial nerve injury due to, 347–349, 349t longitudinal, 347, 349–354, 350f–351f transverse, 348, 352f–353f glomus tumors of, surgery for. See Glomus tumors, surgery for. gunshot wounds to, 349 malignancies of, 43–54. See also specific tumors.
Temporal bone (Continued) angiography in, 44 biopsy of, 34 diagnostic evaluation of, 43–44 diagnostic tests for, 34–35 history and physical examination in, 34 limited temporal bone resection for. See Temporal bone resection, limited. recurrence of, 51 staging of, 35–36, 35t temporal bone resection for. See Temporal bone resection, limited; Temporal bone resection, radical. ossification centers forming, 245 paragangliomas of, radiation therapy for, 715 post-traumatic suture dehiscence and, canalplasty for, 31 Temporal bone resection lateral, 43. See also Temporal bone resection, radical. limited, 33–42 chemotherapy and, 41 complications of, 40 postoperative reconstruction and, 38f, 39–40 radiation following, 41 rehabilitation of lower cranial nerve defects and, 41 surgical technique for, 36–39, 37f–38f radical, 43–54 adjuvant treatment with, 50 complications of, 50 diagnostic evaluation for, 43–44 follow-up for, 51 postoperative care and, 50 rehabilitation following, 50 results with, 51–52, 51f surgical procedure for, 44–50, 45f, 47f–49f subtotal, 43. See also Temporal bone resection, radical. total, 43. See also Temporal bone resection, radical. Temporalis fascia in undersurface graft tympanoplasty, 142 Temporalis muscle flaps following limited temporal bone resection, 40 for middle cranial fossa vestibular neurectomy, 430–431, 432f Temporalis muscle retractor, 430, 431f Temporalis muscle transposition for facial reanimation, 766–767 complications of, 767 patient evaluation for, 766 patient selection for, 766 results with, 767 surgical technique for, 766–767, 768f–769f Temporomandibular joint dysfunction of, aural fullness and, 96–97, 99f with infratemporal fossa surgery, 663 Tensor veli palatini, 93 release of, for patulous eustachian tube, 98 Thrombophlebitis, dural venous sinus with acute infections, 186 treatment of, 190–191, 191f Tinnitus complicating operations for chronic ear infections, 128 microvascular decompression for, 530–531 complications of, 531 operative technique for, 531 patient selection for, 528f, 530–531 results with, 531 perilymphatic fistulas and, 325 Titanium for ossicular prostheses, 162, 166 Topognostic testing in Bell’s palsy, 337
829
Torus tubarius, trauma to, during adenoidectomy, 86 Totally Implantable Communication Assistance device, 386–389, 386t, 388f Transcanal labyrinthectomy, 483–492 anesthesia for, 484–485 cochlear implantation and, �������� 490 complications of intraoperative, 488 postoperative, 488–489 histopathology of, 489, 489f informed consent for, 484 patient counseling for, 484 patient selection for, 483–484 postoperative care for, 485–488 preoperative evaluation for, 484 results with, 490 surgical technique for, 484–485 incision and exposure and, 485, 486f preparation for vestibule opening and, 485, 486f vestibular end organ removal and, 485, 486f–488f Transdural glomus tumors, 552 Translabyrinthine vestibular nerve section, transmastoid labyrinthectomy vs., 458 Translabyrinthine vestibular neurectomy. See Vestibular neurectomy, translabyrinthine. Transmastoid labyrinthectomy, translabyrinthine vestibular nerve section vs., 458 Transtemporal skull base surgery, 519–522 collaboration in, 520–521 nomenclature for, 520, 520f organizational framework for, 519, 520f Transtemporal-supralabyrinthine approach. See Vestibular neurectomy, middle cranial fossa. Tremor of surgeon’s hand, 307 Triad of Gradenigo, 186 Trichloroacetic acid for tympanic membrane closure, 116 Trigeminal nerve anatomy of, 443, 444f deficits of with infratemporal fossa surgery, 663 rehabilitation after neurotologic skull base surgery for, 569–570, 570f, 573f schwannomas of, in neurofibromatosis 2, 694 Trigeminal neuralgia microvascular decompression for, 523–529 in children, 527 complications of, 529 operative technique for, 526–527, 528f patient selection for, 523–526 results with, 527–529, 528f stereotactic radiosurgery of, 796 Trismus following infratemporal fossa surgery, 663 Trochlear nerve monitoring, 782 T-tubes with cartilage tympanoplasty, 137, 137f Tullio phenomenon, 507 Tumarkin’s otolithic catastrophe, 478 Tympanic bone osteonecrosis of, canalplasty for, 29 osteoradionecrosis of, canalplasty for, 29 Tympanic glomus tumors, 551 Tympanic membrane atrophic management of, in cartilage tympanoplasty, 137 stapedectomy and, 295 collapse of, following tympanoplasty, 221 in idiopathic hemotympanum, 74 lateralization of, with outer surface grafting tympanoplasty, 125f, 126
830
Index
Tympanic membrane (Continued) perforation of during laser stapedectomy, 271 in stapedectomy, 257 stapedectomy and, 295 stapes surgery and, 312 perforations of closure of cauterization and paper patching for, 114f, 115–116 fat graft tympanoplasty for, 115f, 116 persistent, tympanostomy tubes and, 87 reconstruction of. See Tympanoplasty. remnant of, preparation for tympanoplasty, 154 re-retraction of, following canal wall reconstruction tympanomastoidectomy, 181 Tympanomastoid glomus tumors, 551 Tympanomastoidectomy, canal wall reconstruction completeness of cholesteatoma removal and, 181 with mastoid obliteration, 173–182 advantages and disadvantages of canal wall up and canal wall down techniques and, 173–174 draping for, 174 hearing results with, 181 mastoidectomy considerations and, 174f–177f, 175–176 patient positioning for, 174 patient preparation for, 174 patient selection for, 174 pitfalls in, 180–181 postoperative care for, 177–178 rationale for, 174 reconstruction and, 176–177, 178f–180f surgical technique for, 175–177 Tympanomeatal flaps design and elevation of, 307–308, 308f placement of, for undersurface graft tympanoplasty, 144, 147f tears of during stapedectomy, 295 stapes surgery and, 312 Tympanometry, impedance, in patulous eustachian tube, 95, 95f Tympanoplasty, 221–226 cartilage, 131–140, 157–159 complications of intraoperative, 135–137 postoperative, 137–138, 137f patient selection for, 131–132 postoperative care for, 135 results with, 138 revision surgery and, 138 surgical technique for, 132–135, 158f, 159 palisade technique and, 133–135, 134f–136f perichondrium/cartilage island flap and, 132–133, 132f–133f postauricular approach and, 132 complications of, 128 draping for, 7, 8f fat graft, 115f, 116 instrumentation for, 7–8, 9f–11f, 18 with mastoidectomy, 149 canal wall down, 209, 214–215, 215f intact canal wall, 195 mastoidectomy with, 152 outer surface grafting technique for, 119–130 advantages and disadvantages of, 126 anesthesia for, 120 blunting in anterior sulcus and, 125f, 126 epithelial cysts and, 125f, 126
Tympanoplasty (Continued) healing problems with, 126 historical background of, 119 operating room arrangement for, 120, 121f patient counseling for, 120 patient evaluation of, 119–120 patient positioning for, 120 patient selection for, 119–120 postoperative care for, 124–126 preoperative preparation for, 120 preparation in operating room for, 120 surgical technique for, 122–124 canal skin replacement and, 124, 125f closure and, 124 dead skin removal and, 122, 123f ear canal enlargement and, 122–124, 123f fascia placement and, 123f, 124 fascia removal and, 122 packing preparation for, 124 postauricular exposure and, 122 transmeatal incisions and, 121f, 122 tympanic membrane remnant deepithelialization and, 124 vascular strip replacement and, 124, 125f tympanic membrane lateralization and, 125f, 126 packing after, 8–9, 10f patient positioning for, 7 for petrositis, 189, 189f plastic extrusion and, 225 plastic sheeting for, 223 preoperative counseling for, 223 preoperative evaluation for, 223 risks of, 128 second look, following canal wall reconstruction tympanomastoidectomy, 178 staging of, 221–223 of canal wall down procedures, 223–225 controversy regarding, 225 mucosal disease factors and, 221–222 mucous membrane indications for, 223, 224f ossicular chain factors and, 222 residual cholesteatoma and, 222 timing of second stage and, 222–223 tympanic membrane collapse following, 221 undersurface graft technique for, post auricular approach for, 149–160 adenoidal hypertrophy and adenoidectomy and, 150 allergy and, 150 anesthesia for, 153, 153f cartilage graft tympanoplasty and, 157–159, 158f complications of, 151 disease eradication and, 154 eustachian tubal tests before, 151 facial nerve monitoring and, 152 fascia harvest for, 154, 155f fundamental preoperative principles of, 150 graft placement and, 154–156, 156f–157f grafting techniques and exposure for, 152 hemostasis and, 152 historical background of, 149–150 imaging before, 151 incisions for, 154, 155f infection control and, 151–152 informed consent for, 151 mastoidectomy and, 152 middle ear exposure for, 154, 155f–156f nasal or sinus condition and, 150 objectives of, 150 ossicular reconstruction and, 152 otorrhea and, 150–151 postoperative care for, 156–157
Tympanoplasty (Continued) predisposing conditions and, 150 preoperative preparation for, 150–151 rare disorders and, 150 results with, 157 surgical preparation for, 153, 153f surgical technique for, 153–156 tympanic membrane remnant preparation and, 154 undersurface graft technique for, transcanal approach for, 141–148 graft selection for, 141–142 patient selection for, 141 postoperative care for, 145–147 preoperative evaluation for, 141 preparation for, 142, 143f rationale for, 141 requirements for, 141 surgical evaluation for, 142 surgical technique for, 142–144 canal enlargement and speculum placement and, 142–144, 143f, 145f canal incisions and middle ear exposure and, 144, 145f external canal packing and, 144, 147f graft placement stabilization and, 144, 147f injection and, 142 middle ear preparation and, 144, 147f perforation placement and, 143f, 144, 145f perichondrial graft and, 144, 146f wound closure following, 9–10 Tympanosclerosis, stapes surgery and, 313 Tympanostomy tubes, 73–92 biofilms and, 88 complications of, 79–80, 86–87 dislodgement of, 86 draping for, 80 follow-up for, 84 for idiopathic hemotympanum, 80 insertion of, 81–84 instrumentation for, 80–81 for otitis media acute, 77 chronic, with effusion, 78–79 limitations of, 79 for patulous eustachian tube, 97 pitfalls with, 84–86 postoperative care for, 84 preoperative patient counseling for, 79–80 preoperative preparation for, 80 results with, 86 risks of, 79–80 surgical site preparation and draping for, 80 Tympanotomy, posterior, for mastoiditis, 188
U Upper lid entropion correction in facial paralysis, 748f, 750 Upper lid reanimation procedures in facial paralysis, 735–737 enhanced palpebral spring implantation for, 738–740, 739f, 741f gold weight implantation for, 740–742, 741f, 742t palpebral spring implantation for, 736f, 737–738 silicone rod prosthesis implantation for, 742, 743f Urban rotary suction-dissector for vestibular neurectomy, 13, 14f Utricle, failure to find, in transcanal labyrinthectomy, 488
Index
V Vagus nerve anatomy of, 443, 444f compression of, microvascular decompression for, 531–532 deficits of, rehabilitation after neurotologic skull base surgery for, 571–577, 573f–574f, 576f jugular foramen schwannomas and, 552 monitoring of, 782 in glomus tumor surgery, 555–556 paresis/palsy of with jugular foramen schwannomas, 552 with limited temporal bone resection, 40 preservation of, in glomus tumor surgery, 556 schwannomas of, in neurofibromatosis 2, 695 Valacyclovir for Ramsay Hunt syndrome, 339 Vascular anomalies, stapes surgery and, 313 Vascular clips for vestibular neurectomy, 13–14, 14f Vascular compressive syndromes, microvascular decompression for. See Microvascular decompression. Vascular injury with chronic otitis media surgery, 238–241 Vascular lesions, neurotologic surgery for, 801–803 middle fossa approaches for, 803 of Kawase, 803 with petrous carotid exposure, 803 transpetrosal approaches for, 801–803, 802f retrolabyrinthine, 802 transcochlear, 803 translabyrinthine, 802–803 Vascular tumors of petrous bone and cerebellopontine angle, 807–813 cerebral revascularization and bonnet bypass and, 812–813, 812f–813f preoperative embolization and, 807–811 glomus tumors and, 807–809, 810f–811f hemangioblastomas and, 809–811 hemangiomas and, 811 Velopharyngeal insufficiency, adenoidectomy and, 87 Ventilation tubes. See Tympanostomy tubes. Vertebrobasilar arteries anatomy of, 800–801 aneurysms of, 805 Vertigo. See also Dizziness. differential diagnosis of, 456, 467–468, 510 after endolymphatic sac surgery, 418 intraoperative, stapedectomy and, 259–261 positional, microvascular decompression for, 530–531 complications of, 531 operative technique for, 531 patient selection for, 528f, 530–531 results with, 531 positional, benign paroxysmal, 467–476 pathophysiology of, 468 posterior semicircular canal occlusion for patient counseling and, 468–470, 469f patient selection for, 470 preoperative evaluation for, 471 results with, 471–473 surgical technique for, 471, 472f after stapes surgery, 317 transcanal labyrinthectomy to control, 483 in vestibular system disorders, 455–456 Vestibular disorders, treatment of, 457–458. See also Vestibular neurectomy, translabyrinthine. decision making and, 457–458
Vestibular disorders, treatment of (Continued) transmastoid labyrinthectomy vs. translabyrinthine vestibular nerve section for, 458 Vestibular evoked myogenic potential(s) in Meniere’s disease, 415 for translabyrinthine vestibular neurectomy, 458–459 Vestibular evoked myogenic potential thresholds in superior canal dehiscence syndrome, 508, 509f Vestibular nerve resection, middle fossa approach for, for acoustic tumors. See Acoustic tumors, vestibular nerve resection for. Vestibular nerve resection, translabyrinthine, transmastoid labyrinthectomy vs., 458 Vestibular neurectomy endoscopic assisted, 448 instrumentation for, 13, 14f middle cranial fossa, 429–440 alternatives to, 439–440 anesthesia for, 430 complications of, 439 draping for, 430 dressing for, 435 instrumentation for, 430, 431f intraoperative monitoring for, 430 patient positioning for, 430, 430f patient selection for, 429 pitfalls in, 437–439 postoperative care for, 435 preoperative counseling and, 430 preoperative evaluation for, 429–430 preoperative medication for, 430 results with, 439 surgical site preparation for, 430 surgical technique for, 430–435, 431f bony exenteration and blue lining of superior semicircular canal and, 433, 436f craniotomy and, 431–433, 432f, 434f dural elevation and, 433, 434f exposure of meatal plane and arcuate eminence and, 433, 434f internal auditory canal exposure and, 433–435, 436f medial cranial fossa retractor introduction and, 433, 436f middle cranial fossa floor repair and, 435, 438f skin incision and, 430, 432f temporal muscle flap and, 430–431, 432f vestibular neurectomy and, 435, 437f–438f wound closure and, 435, 438f tips for, 437–439 retrolabyrinthine, 439, 441–454, 445f combined with retrosigmoid approach, 448, 448f complications of of, 451–452, 451t efficacy of, 449–451, 449t–450t historical background of, 441–442 neuroanatomy and, 442–443, 443f–444f postoperative course and, 448–449 preoperative evaluation for, 442 retrosigmoid (suboccipital), 439, 441–454, 447f combined with retrolabyrinthine approach, 448, 448f complications of, 451–452, 451t efficacy of, 449–451, 449t–450t historical background of, 441–442 neuroanatomy and, 442–443, 443f–444f postoperative course and, 448–449 preoperative evaluation for, 442
831
Vestibular neurectomy (Continued) translabyrinthine, 455–466 anesthesia for, 459 complications of, 464 diagnostic considerations for, 455–456 draping for, 459 electronystagmography and, 458 follow-up for, 461–464 functional auditory system assessment and, 459 high-resolution computed tomography and, 459 historical background of, 455 literature overview for, 464 magnetic resonance imaging and, 459 patient positioning for, 459 platform posturography and, 458–459 postoperative care for, 461–464 results with, 464 surgical procedure for, 459–461, 459f–460f, 462f–463f transmastoid labyrinthectomy vs., 458 vestibular evoked magnetic potentials and, 458 Vestibular neuronitis, 456 Vestibular schwannomas in neurofibromatosis 2. See Neurofibromatosis 2, vestibular schwannomas in. stereotactic irradiation for, 698 Vestibular symptoms. See also Dizziness; Vertigo. following perilymphatic fistula surgery, 330 Vestibular-evoked myogenic potential, 456 Vestibule, footplate relationship with, 311–312 Vestibulectomy, subtotal, with intramuscular streptomycin, 494–496 indications for, 494–495 pretreatment evaluation and patient counseling for, 495 treatment technique for, 495–496 Vestibulocochlear nerve anatomy of, 445f, 446 compression of, microvascular decompression for, 530 direct recording from, monitoring hearing using, 780–781 monitoring of, during neurologic procedures, 11–12 Vibrant Soundbridge, 167, 386t, 389–391, 390f–391f, 391t Vibratory transducer in ossicular reconstruction, 167 Vibroplasty, 170 Visible-spectrum lasers, 281, 282t, 284, 285f
W Wound infection with cochlear implantation, 378 with retrolabyrinthine vestibular neurectomy, 451 after stapes surgery, 317 with surgery for facial nerve trauma, 360
Y Yeasts, acute otitis media due to, 109
Z Zygomas, 186
Self-Assessment Questions Chapter 1 NONE
Chapter 2 1. E xostoses of the external auditory canal would be expected most frequently in the following: a. Patient with chronic otitis externa b. College swimmer from a warm climate c. 23-year-old thrice-weekly surfer from Florida d. 43-year-old thrice-weekly surfer from Oregon Answer: d.
2. R apid and complete healing of the skin of the external auditory canal is facilitated by a. Performance of surgery prior to severe skin attenuation from chronic otitis externa b. Preservation of skin of the external auditory canal c. Local anesthesia d. a and b e. b and c Answer: d.
3. C onductive hearing impairment occurs with large exostoses of the external auditory canal when a. Debris prevents normal tympanic membrane vibration b. Narrowing of the external auditory canal reaches 5 mm c. Narrowing of the external auditory canal reaches 2 mm d. All of the above e. a and c Answer: e.
4.
edial third stenosis may result in M a. Weeping and chronic otitis externa b. Conductive hearing impairment c. Formation of dense scar lateral and medial to the tympanic membrane d. a and b e. All of the above
Answer: d.
5. O steonecrosis of the tympanic bone presents with bone exposure in the external auditory canal and may be associated with a. Radiation history to the temporal bone b. Lupus vasculitis c. Diabetes d. a and b e. All of the above Answer: e.
Chapter 3 1.
he most common malignancy of the temporal bone is T a. Adenoid cystic carcinoma b. Pleomorphic adenoma c. Warthin’s tumor d. Squamous carcinoma e. Ceruminoma
Answer: d.� Most temporal bone carcinoma originates in the
skin of the external auditory canal. 2. T he mechanism for salivary gland tumor involvement of the temporal bone is a. Direct extension b. Hematogenous metastases c. Ectopic salivary rests within the temporal bone d. Squamous metaplasia e. Absence of the fissures of Santorini e
e
SELF-ASSESSMENT QUESTIONS
Answer: a.� The proximity of the partoid and fissures of San-
torini make direct extension the most common mode for salivary gland tumors to involve the temporal bone. 3. A fungating mass of the external auditory canal is biopsied in the office. The pathologist reports acute and chronic inflammation. The next step in management is a. Consult infectious disease b. Image and rebiopsy c. Lateral temporal bone resection d. Subtotal temporal bone resection e. Radiation therapy Answer: b.� Necrotic carcinomas frequently are confused with
inflammatory changes. The otologist should be suspicious and plan deeper biopsies once imaging proves that vital structures are not jeopardized by the planned biopsy. 4. T he best study to distinguish postoperative scar from tumor recurrence is a. PET-CT imaging b. PET imaging c. CT d. MRI with contrast e. MRI-CT fusion imaging Answer: a.� PET-CT has proven increasingly useful at dem-
onstrating the expected increased metabolic activity of carcinoma in relation to the osseous anatomy. 5. T he most significant prediction of postoperative survival in temporal bone carcinoma is a. Preoperative chemotherapy and radiation b. Clear surgical margins c. Postoperative chemotherapy and radiation d. Patient age e. Extensiveness of resection Answer: b.� Clear surgical margins reflect complete oncologi-
cally sound removal of the lesion. While adjuvant treatments definitely have a role, a clear surgical margin is the best predictor of survival.
Chapter 4 1. T he overall prevalence of primary temporal bone malignancy is about a. Six cases per thousand b. Six cases per hundred thousand c. Six cases per million d. Six cases per hundred million Answer: c.
dvanced malignancies of the temporal bone are best 2. A treated with a. Surgery only b. Radiation only c. Surgery and radiation d. Chemotherapy Answer: c.
3. R adiologic assessment of temporal bone malignancies may include a. CAT scan b. MRI c. Cerebral angiography d. All of the above Answer: d.
ong (>8 cm) facial nerve defects should be grafted 4. L with a. The greater auricular nerve b. The cervical cutaneous nerves c. The cranial nerve XI d. The sural nerve Answer: d.
5. T he most common site for temporal bone treatment failure is a. Local b. Locoregional c. Regional d. Distant site Answer: a.
Chapter 5 1. What are the goals of atresia surgery? a. Perform ossicular reconstruction with the patient’s own ossicular chain instead of using a prosthesis b. Create a patent, skin-lined external auditory canal and achieve a postoperative air-bone gap within 20 to 30 dB c. Perform surgery before age 6 in both unilateral and bilateral cases d. Perform hearing restoration surgery and microtia repair in a one-stage procedure Answer: b.
2. W hat are the four anatomic parameters, as seen radiographically, crucial for atresia surgical operability? a. The status of the inner ear, the degree of pneumatization of the mastoid, the course of the facial
SELF-ASSESSMENT QUESTIONS
nerve, and the presence of the oval window and stapes footplate b. Thickness and form of the atretic bone, soft tissue contribution to the atresia, size and status of the middle ear cavity, and the presence or absence of congenital cholesteatoma c. The status of the inner ear, the degree of pneumatization of the mastoid, thickness and form of the atretic bone, and the presence or absence of congenital cholesteatoma d. Soft tissue contribution to the atresia, size and status of the middle ear cavity, the course of the facial nerve, and the presence of the oval window and stapes footplate
e
c. Noise-induced sensorineural hearing loss. Minimize drilling on the ossicular chain when dissecting it away from the atretic bone. d. Ossicular reconstruction prosthesis extrusion. Cover the prosthesis with cartilage prior to grafting the new drum.
Answer: b.
Chapter 6 None
Answer: a.
Chapter 7
3. W hat is the most common cause of inoperability in congenital aural atresia? a. Absence of the oval window b. A facial nerve overlying the oval window c. Poor mastoid pneumatization d. Unilateral atresia
1. W hich of the following does not contribute to the closing of the lumen of the Eustachian tube? a. Coiling properties of the cartilaginous skeleton b. Active contraction of the associated muscles c. Pressure of tissues neighboring the lumen d. All of the above
Answer: c.
Answer: b.
4. W hat are the main postoperative complications of atresiaplasty? a. Lateralization of the tympanic membrane, large meatus, sensorineural hearing loss, and facial nerve palsy b. Ossicular reconstruction prosthesis extrusion, otorrhea, stenosis of the meatus, and facial nerve palsy c. Sensorineural hearing loss, tympanic membrane perforation, and ingrowth of grafted skin into external auditory canal Merocel wicks d. Lateralization of the tympanic membrane, stenosis of the meatus, sensorineural hearing loss, and facial nerve palsy
2. W hich are the most common symptoms of the patulous Eustachian tube? a. Autophony made worse in recumbent position b. Autophony and pressure-induced dizziness c. Aural fullness and autophony d. Symptoms associated with retraction and atelectasis of the tympanic membrane
Answer: d.
5. W hat is the most common delayed cause of a poor hearing outcome in atresia surgery and what can be done to prevent it? a. Meatal stenosis. Use one-piece split-thickness skin grafts to cover all exposed bone, and Silastic sheets and Merocel wicks in the external auditory canal for 2 weeks. b. Tympanic membrane lateralization. Use Silastic sheets to line the external auditory canal and Merocel wicks for 2 weeks, Gelfim disks, and tabs in the fascia graft to maintain the tympanic membrane graft in position.
Answer: c.
3. W hich of the following is not considered to participate in the pathogenesis of the patulous Eustachian tube? a. Cigarette smoking b. Weight loss c. Changes in estrogen levels d. Scarring from previous nasopharyngeal procedures Answer: d.
4.
iagnosis of patulous Eustachian tube is certain with D a. Appropriate findings on axial CT b. Appropriate findings on sonotubometry c. Presence of typical symptoms d. Observation of lateral and medical excursions of the drum
Answer: d.
e 5.
SELF-ASSESSMENT QUESTIONS
reatment options do not currently include: T a. Discontinuing topical antidecongestants b. Instillation of local irritants c. Augmentation of the nasopharyngeal Eustachian tube orifice d. Injection of Botox to the tensor veli palatine
Answer: d.
Chapter 8 1. W hich of the following is not normally found as part of the pathophysiology of chronic suppurative otitis media? a. Capillary proliferation b. Rarefying osteitis c. Multinucleated giant cells d. New bone formation e. Granulation tissue Answer: c.� All of the other four items are specifically men-
tioned as routinely present in individuals with chronic suppurative otitis media. Large multinucleated giant cells are classic for tuberculosis and/or the presence of foreign bodies but do not appear regularly in patients with chronic suppurative otitis media. 2. W hich of the following statements is true of acute otitis media through a tympanostomy tube or perforation, but not true of acute otitis media with an intact tympanic membrane? a. It occurs frequently without fever. b. Drainage from the area is commonly the only sign. c. Significant pain is uncommon. d. Pseudomonas is the causative organism in a significant number of infections. e. S treptococcus pneumoniae is the causative organism in a number of infections. Answer: d.� Pseudomonas is only very rarely recovered from
children with otitis media and an intact tympanic membrane. However, it makes up a significant percentage of primary isolates in children with acute otitis media through a tympanostomy tube, especially in older children. Answers a–c and e are true of acute otitis media through a tympanostomy tube but are not true of acute otitis media with an intact tympanic membrane. 3. W hich of the following statements is true about commercially available antibiotic ear drops? a. The antibiotic concentration in commercially available ear drops is hundreds of times higher than the concentration that can be achieved in middle ear fluid following systemic administration.
b. Antibiotic–steroid suspensions are too viscous to pass through a tympanostomy tube. c. The use of topical quinolone drops is directly responsible for the increase in quinolone-resistant isolates in children with ear disease. d. The currently available evidence suggests that the use of a steroid compromises healing of the tympanic membrane. e. There is no evidence that resolution of acute otitis media through a tympanostomy tube or tympanic membrane perforation is enhanced when a steroid is added to a topical antibiotic drop.
Answer: a.� The others are all false. Although concerns have
been raised that a steroid might delay wound healing, there is no evidence to support the assertion that outcomes are worse when steroids are used. There is now good evidence that a steroid improves both clinical cure and facilitates the eradication of bacteria when it is included in an antibiotic drop. 4. W hich of the following statements is true of fat graft tympanoplasty? a. Donor fat is usually harvested from the abdomen. b. Fat graft tympanoplasties do not require rimming of the perforation. c. The literature justifies performing this procedure bilaterally and simultaneously. d. It is an important technique for perforations involving 75% of the tympanic membrane or less. e. It is appropriate for any patient with a tympanic membrane perforation and a hearing loss of 50 dB or less. Answer: c.� Mitchell, et al assert that the risks and complica-
tions from fat graft tympanic myringoplasty are sufficiently low so that they can be performed bilaterally and simultaneously. The procedure is not likely to be successful in tympanic membrane perforations that involve more than 25% of the drum. The procedure should probably not be performed in individuals with hearing losses over 25–30 dB because the middle ear ossicles cannot be explored or repaired. Fat grafts are usually taken from earlobes. The margin of the perforation needs to be de-epithilialized as usual. 5. W ith respect to paper patching of tympanic membrane perforations, which of the following statements is true? a. The procedure usually needs to be performed in an outpatient surgery setting. b. It is an excellent technique for large perforations. c. Closure may require repeated applications of the paper patch. d. If successful, the perforation closes in a few days. e. “Rimming” of the perforation is not necessary. Answer: c.� Multiple repetitions of the procedure are almost
always required to get complete closure. Paper patching is often
SELF-ASSESSMENT QUESTIONS
done in the office. The perforation must be de-epithelialized. A solution, usually trichloroacetic acid, is used to de-epitheliaze the perforation. Several months are usually required for successful closure.
e
Chapter 10
1. O ne of these complications is not seen with underlay tympanoplasty grafting: a. Reperforation b. Blunting c. Infection d. Myringitis of the graft
1. A ll of the following are true concerning cartilage tympanoplasty except a. Hearing results are worse than with conventional materials. b. Toothed forceps should be avoided during manipulation. c. Although softening occurs, long-term graft survival is the norm. d. Cartilage becomes more brittle with age and more subject to fracture. e. The postoperative tympanogram is often type B despite no effusion present.
Answer: b.
Answer: a.� Hearing results are worse with cartilage. Several
2. O ne of these complications is not seen with overlay grafting: a. Reperforation b. Blunting c. Lateralization d. Short healing time
studies comparing cartilage, fascia, and perichondrium have been published that show no significant difference in hearing results between the materials. Cartilage can be somewhat brittle, especially in the older patient, so toothed forceps should not be used to manipulate the graft. Softening can occur, and the postoperative tympanogram is frequently a small volume B, due to the noncompliant nature of the graft, even with normal hearing and no effusion.
Chapter 9
Answer: d.
3. Which one of the following statements is correct? a. Blunting is a complication of underlay tympanoplasty. b. The rate of take for overlay tympanoplasty is greater than for underlay tympanoplasty. c. Lateralization is a complication of underlay tympanoplasty. d. Myringitis is not seen after underlay tympanoplasty. Answer: b.
4. A dvantages of the outer surface technique include all the following except a. It provides excellent exposure. b. It allows for removal of as much remnant as necessary. c. The take rate is lower than overlay tympanoplasty. d. This technique can be used in all cases
2. C artilage should be considered as a graft material in the following situations: a. The atelectatic ear b. Cholesteatoma c. A perforation anterior to the annulus d. A draining perforation e. All of the above Answer: e.� Cartilage can be used as a graft material in any
ear considered to be at high risk for failure with traditional techniques using temporalis fascia or perichondrium. Included in this would be the high-risk perforation, the atelectatic ear, and cholesteatoma.The high-risk perforation comprises a revision surgery, a perforation anterior to the annulus, a perforation draining at the time of surgery, a perforation larger than 50%, or a bilateral perforation.
5. D isadvantages of the outer surface technique include all of the following except a. Healing problems b. Lateralization of the graft c. Blunting in the anterior sulcus d. Absence of epithelial cysts
3. T he following are true statements concerning the perichondrium/cartilage island flap except a. A strip of cartilage 1–2 mm in width is removed to facilitate the malleus. b. The graft is placed as an underlay graft, medial to the malleus. c. Tragal cartilage is most suitable for total tympanic membrane reconstruction. d. One should expect a cosmetic defect after harvest in the tragal area. e. The graft is placed so that the perichondrium side is out, toward the canal.
Answer: d.
Answer: d.� One should expect a cosmetic defect after harvest
Answer: c.
in the tragal area. If the cartilage is harvested correctly, leaving
e
SELF-ASSESSMENT QUESTIONS
a strip of cartilage in the dome of the tragus, no cosmetic deformity is seen. The rest of the statements are true concerning the island flap technique described here.
Chapter 11
4. W hen using the palisade technique for cartilage tympanoplasty, the following statements are considered true, except for a. Cartilage can be harvested from the tragus or cymba. b. The cartilage is cut into rectangular strips. c. The technique is favored when ossiculoplasty is performed in a malleus-present situation. d. The technique does not require one large piece of cartilage. e. The technique is particularly suited for cholesteatoma surgery.
1. T he authors identified four advantages associated with transcanal medial graft placement tympanoplasty. Which one of the following was not considered an advantage? a. It is quicker and more direct. b. It results in less surgical trauma and reduces the likelihood of healing problems such as adhesions, narrowing, and stenosis of the ear canal. c. It is technically easier to perform. d. It results in less postoperative discomfort for the patient. e. There is reduced risk of tympanic membrane blunting and burying of squamous epithelium beneath the graft.
Answer: b.� The cartilage is cut into rectangular strips. One
Answer: c.� Although transcanal medial graft placement may
major difference between the palisade technique described here and that described by Heermann, et al, is that, instead of placing rectangular strips of cartilage side to side, an attempt is made to cut one major piece of cartilage in a semilunar fashion. A second semilunar piece is placed between this first piece and the canal wall to reconstruct the scutum precisely, and any spaces that result between this cartilage and the canal wall or scutum are filled in with small slivers of cartilage.The remaining statements are all true with regard to the palisade technique.
be technically difficult in some circumstances, the advantages gained with this approach are felt by the authors to outweigh the difficulties associated with the procedure.
5. W hich of the following statements apply to the postoperative period following cartilage tympanoplasty? a. Eardrum intubation, if necessary, will likely require a trip to the operating room. b. A tympanogram may be unreliable. c. A second-look surgery may be required in cases of cholesteatoma removal. d. Persistent effusion is the most significant complication. e. All of the above Answer: e.� While the tympanic membrane remains relatively
insensate after cartilage reconstruction, it is often necessary to take the patient to the operating room for eardrum intubation as tube placement can be difficult. Impedance tympanometry is unreliable after cartilage tympanoplasty and will generally yield a low-volume, type B tympanogram, despite normal hearing, due to the noncompliant nature of the cartilage. One serious disadvantage of using cartilage for reconstruction in cholesteatoma surgery is that it creates an opaque tympanic membrane posteriorly, which could potentially hide residual disease. If disruption of the cholesteatoma sac occurs, consider the advisability of performing a second-look surgery. The most significant pitfall seen in the postoperative period is persistent effusion with conductive hearing loss, requiring intubation of the reconstructed eardrum. This is seen in about 7% to 10% of cases and can be problematic in cases in which the entire tympanic membrane is reconstructed with the cartilage/perichondrium island flap.
2. I dentify which one of the following is not a contraindication for transcanal tympanoplasty surgery. a. A narrow ear canal that will only accommodate up to a 4.5-mm speculum b. An elderly and debilitated patient c. Involvement of only hearing ear and patient not a satisfactory surgical risk d. A perforation with persistent and active drainage e. Very questionable Eustachian tubal function Answer: d.� Drainage at the time of surgery is not believed
by the authors to be an absolute contraindication to surgery. If drainage cannot be controlled preoperatively, this constitutes an indication for surgery. 3. W hich one of the following techniques is not advocated by the authors? a. The head is secured with tape. b. Preoperative antibiotics are usually administered. c. The ear, auricle, and surrounding skin are cleansed with povidone-iodine solution. d. The hair is secured with liquid spray adhesive and usually is not shaved. e. The head is extended with the chin up. Answer: b.� Studies have shown that no significant benefits
result from the use of preoperative antibiotics. 4. W hich one of the following tympanoplasty techniques is advocated by the authors? a. The entire bony ear canal skin is removed to allow satisfactory exposure. b. If there is a prominent anterior bony canal wall “hump,” a postauricular incision should be used.
SELF-ASSESSMENT QUESTIONS
c. The graft is placed lateral to the malleus handle. d. The ear speculum should be supported and manipulated by the surgeon’s fingers during the procedure. e. Following graft placement, the ear canal is filled with moist Gelfoam.
Answer: c.� The authors believe that graft placement lateral
to the malleus handle avoids interference with ossicular chain reconstruction and lessens the likelihood of postoperative middle ear adhesions. 5. W hich one of the following postoperative methods is advocated by the authors? a. Cotton is placed in the ear canal and usually left in place for 3 weeks. b. A mastoid dressing is usually placed at the end of the procedure. c. If drainage occurs postoperatively, the patient is advised to come into the office immediately and the ear canal is cleaned using the microscope. d. The patient is advised to keep the operated ear dry for 3 weeks, at which time the ear is examined in the office with a microscope. e. Active autoinflation of the ear is begun 3 days postoperatively. Answer: d.� It is important to keep the ear dry and to avoid
inflating the ear for 3 weeks. Cotton should be placed in the meatus, changed as necessary for drainage, and discontinued after drainage has ceased. If drainage worsens, the patient is started on antibiotic ear drops.
Chapter 12 1. W hich of the following conditions is not associated with a predisposition for chronic ear disease? a. Chronic tonsillitis b. Adenoid hypertrophy c. Allergic rhinitis d. Chronic sinusitis Answer: a.
2. F or which of the following are temporal bone CT scans without contrast used for preoperative assessment in chronic ear surgery? a. Sinonasal disease b. Epidural abscess c. Brain hernia d. Inner ear fistula Answer: d.
3. A ntimicrobial prophylaxis is not indicated in chronic ear surgery in which of the following situations?
e
a. Violation of the dura with or without cerebrospinal fluid leak b. Simple perforation c. Labyrinthine fistula d. Presence of indwelling devices such as a cochlear implant
Answer: b.
4. T he undersurface grafting technique greatly reduces which of the following postoperative complications? a. Infection b. Anterior sulcus blunting c. Graft failure d. Recurrent cholesteatoma Answer: b.
5. T he vascular strip is defined as the canal skin between a. The annulus and bony cartilaginous junction b. The 12 o’clock and 6 o’clock positions in the ear canal c. The malleus and the anulus d. The tympanomastoid and tympanosquamous sutures Answer: d.
Chapter 13 1. W hich of the following factors may increase the risk of extrusion following ossicular reconstruction with a titanium prosthesis? a. Age of patient b. Inadequate length of prosthesis c. Absence of cartilage between prosthesis and tympanic membrane d. Hydroxyapatite platform on prosthesis e. Absence of malleus handle Answer: c.� Many allograft prostheses, including titanium
and Plastipore, require a disk of cartilage to be interposed between the platform and tympanic membrane to prevent extrusion. Hydroxyapatite has demonstrated excellent biocompatibility and the use of cartilage is not always required with this allograft material; however, it is probably prudent to use cartilage even in these cases. 2. W hich of the following reconstruction techniques can be used in the setting of a mobile malleus and stapes superstructure and a foreshortened incus? a. Total ossicular reconstruction prosthesis b. Malleus-to-oval-window prosthesis c. Stapes piston
e
SELF-ASSESSMENT QUESTIONS
d. Bone cement e. Removal of incus and placement of total ossicular replacement prosthesis
Answer: d.� When the gap between a foreshortened incus and
a mobile stapes superstructure is small, bone cement can be used to reconstruct the ossicular chain and restore continuity between the incus and stapes. In situations where the gap is large, an incus bridge prosthesis can be used in conjunction with bone cement.
3. S uccessful ossicular reconstruction is most likely to be achieved when a. The Eustachian tube function is normal. b. The canal wall remains intact. c. Adequate tension is placed on the prosthesis. d. A mobile stapes superstructure exists. e. All of the above are correct. Answer: e.� Each of the above factors contributes to improved
a. Graft the tympanic membrane and use a partial ossicular replacement prosthesis to reconstruct the ossicular chain. b. Pack the ear with antibiotic-soaked packing and stage the operation. c. Place a ventilation tube in the newly reconstructed tympanic membrane and then use a partial ossicular replacement prosthesis. d. Place Gelfilm or Sialastic sheeting over the promontory and reconstruct at a later time. e. Stage the operation and once the tympanic membrane has healed reconstruct the ossicular chain with a malleus-to-footplate prosthesis.
Answer: d.� A totally denuded promontory may result in
severe scarring between it and the tympanic membrane. In cases where the mucosa is missing or severely diseased, it is best to cover the promontory with Gelfilm or Sialastic sheeting, and reconstruct the ossicular chain during a second-look procedure.
success in ossicular reconstruction. 4. I n your preoperative discussions with your patients undergoing ossicular chain reconstruction, what are the anticipated hearing results for partial ossicular replacement prostheses (PORPs) and total ossicular replacement prostheses (TORPs)? a. Complete closure of air-bone gap with PORPS and 5 dB air-bone gap with TORPs b. PORPs with a closure of air-bone gap to within 5 dB in 90% of cases and to within 10 dB in 90% of TORPs c. Closure of air-bone gap to within 15 dB with PORPs and to within 25 dB with TORPs in majority of patients d. PORPs with closure of air-bone gap to within 5 dB and TORPs to within 10 dB in the majority of patients e. An equivalent air-bone gap if the stapes footplate is mobile
Chapter 14
Answer: c.� Although we all would like to completely close the air-bone gap in our patients, it is important to have realistic expectations for yourself and your patients. The results with PORPs are generally better than with TORPs. In patients undergoing ossicular chain reconstruction with PORPs, two thirds should close the air-bone gap to within 15 dB, and two thirds of patients with TORPs should be within 25 dB.
2. I n the canal-wall-reconstruction tympanomastoidectomy technique, bone pate collection should be performed a. After tympanic membrane grafting and replacement of the posterior canal wall b. After a complete mastoidectomy has been performed but prior to making bony canal cuts with a microsagittal saw c. Prior to any entry into mastoid cells d. Following removal of the bony posterior canal wall
5. I n a patient undergoing cholesteatoma removal using an intact canal wall approach, the surgeon had to remove the mucosa off the promontory and also remove the involved incus. The malleus was mobile and the stapes was intact and mobile. What are the surgeon’s best options for the management of this ear?
1. A ll of the following are thought to contribute to the higher rate of recidivism seen with canal-wall-up tympanomastoidectomy (vs. canal-wall-down technique) except a. Tympanic membrane re-retraction with recurrent antral or epitympanic cholesteatoma formation b. Suboptimal exposure to the attic, antrum, and middle ear due to the presence of the posterior canal wall leading to persistent disease c. The presence of nitrogen-resorbing mucosa lining the mastoid cavity postoperatively d. Compromised exposure of the sinus tympani due to the presence of the posterior canal wall Answer: d.
Answer: c.
3. I n the canal-wall-reconstruction tympanomastoidectomy technique, slices of calvarial bone are placed
SELF-ASSESSMENT QUESTIONS
a. In the attic and sinus tympani b. In the sinus tympani and facial recess c. In the facial recess and attic d. Only in the attic
Answer: c.
4. T he bony posterior canal wall is cut in the following fashion during the canal-wall-reconstruction tympanomastoidectomy technique: a. One beveled cut superiorly and one beveled cut inferiorly b. Two cuts superiorly making a right angle and one beveled cut inferiorly c. Two cuts inferiorly making a right angle and one beveled cut superiorly d. One beveled cut superiorly and one straight cut inferiorly
e
with a right-sided unilateral profound sensorineural hearing loss. You suspect she has what? a. A right-sided meningoencephalocele through the mastoid tegmen b. A right-sided Mondini malformation c. A right-sided enlarged vestibular aqueduct syndrome d. A patent Hyrtl’s fissure
Answer: b.� She had a severe Mondini malformation with
dehiscence of the medial wall of the vestibule with a large cerebrospinal fluid (CSF) connection between the internal auditory canal and the vestibule. She also had an incomplete stapes footplate allowing the CSF to bulge a CSF containing thin mucosal cyst filling the oval window niche.
5. C ontraindications to canal wall reconstruction tympanomastoidectomy include a. Mastoid cholesteatosis b. Sinus tympani involvement c. Facial paralysis d. Tegmen defect with meningoencephalic herniation
3. A 40-year-old woman presents with a seizure, from which she fully recovered. She had never had seizures before. Her complete neurological examination was normal. The only abnormality found on full examination was an infected cholesteatoma in her right ear, which had drained intermittently for several years. You are concerned about what? a. Brain abscess b. Subdural abscess c. Meningitis d. Otitic hydrocephalus
Answer: a.
Answer: a.� She had a large right temporal lobe brain
Answer: b.
abscess.
Chapter 15 1. A 35-year-old man with no previous ear disease presents with persistent left ear conductive hearing loss for 1 month following an upper respiratory infection concomitant with left ear pain, all of which resolved quickly with oral antibiotics. He did ultimately mention that the left ear was still very mildly painful. On examination, the left ear had seromucinous middle ear effusion; otherwise, the history and physical examination was normal. What would you do? a. Follow him for another month expecting full recovery. b. Give him another round of oral antibiotic treatment. c. Have him self-inflate the ear to speed aeration of the middle ear. d. Order a temporal bone CT because you are concerned he might have masked mastoiditis. Answer: d.� He had mastoiditis with fairly extensive bone
destruction. 2. A 13-year-old girl presents with a history of three episodes of meningitis associated with episodes of rightsided acute suppurative otitis media. She was born
4. A 60-year-old woman presents with a spontaneous conductive hearing loss in the left ear that had been present for 3 months. She had been treated with antibiotics with no resolution. She had no previous ear problems. On examination, the left middle ear is filled with amber serous-appearing fluid. After a myringotomy and tube in the office, her ear developed a copious thin discharge continuously so that she had to, at times, place a cotton ball in the meatus and change it several times a day. You investigate for what? a. Chronic suppurative otitis media b. Cerebrospinal fluid leak c. Chronic serous otitis media d. Fungal external otitis Answer: b.� She had a cerebrospinal fluid leak from a menin-
goencephalocele through the tegmen of the mastoid. 5. A 20-year-old man presents with a draining right ear and pain. When questioned about the pain, he tells you that the pain is in his right ear and behind his right eye. On examination, he has a perforation in the right drum and mucopurulent discharge. The rest of the history and examination is normal. You suspect what?
e10
SELF-ASSESSMENT QUESTIONS
a. Mastoiditis b. Otitis external associated with the drainage c. Brain abscess d. Petrositis
a. 3–6 months b. 6–9 months c. 9–18 months d. 18–24 months e. 24–36 months
Answer: d.� He had petrositis that was discovered by CT. Answer: c.� These are guidelines established by Sheehy.
Chapter 16
Chapter 17
5.
NONE
Answer: c.� This thickness is commonly used for staging. The
NONE
Chapter 18 1. T he indications for staging tympanoplasty include which of the following? a. Mucosal disease b. Stapes fixation c. Concern of residual cholesteatoma d. All of the above Answer: d.� All are well-recognized indications.
2. F actors that have been blamed for postoperative collapse of the tympanic membrane include which of the following? a. Multiple surgeries b. Eustachian tube dysfunction c. Recurrent infections d. Middle ear mucosal disease e. b and d
Thick” Silastic sheeting has which dimension? “ a. .005 inch b. .010 inch c. .040 inch d. .1 inch e. .2 inch
.005-inch thickness is too easily displaced by scar tissue for use with severe mucosal disease.
Chapter 19 1. H ow is a cholesteatoma from a pars flaccida retraction pocket most likely to spread? a. Anteriorly, in the lateral mallear space, into the anterior epitympanum b. Posteriorly, lateral to the body of the incus, to the aditus ad antrum c. Inferiorly via the posterior pouch of von Tröltsch, lateral to the incus, and into the mesotympanum. d. Posteriorly, medial to the body of the incus, to the aditus ad antrum. e. Inferiorly via the posterior pouch of von Tröltsch, medial to the incus, and into the mesotympanum. Answer: b.� Anterior spread is relatively rare. Inferior spread
is lateral to the incus but is less common than the posterior route.
Answer: e.� Classically, Eustachian tube dysfunction has
3. F or patients that have middle ear cholesteatoma found at the first stage, what percentage will have residual disease in the middle ear at the second stage? a. 5% b. 15% c. 33% d. 66% e. 90%
2. F or which situation is it appropriate to attempt repair of a cholesteatoma fistula? a. A 3-mm lateral semicircular canal fistula during a second-stage surgery, with normal hearing in the contralateral ear b. A 1-mm lateral semicircular canal fistula in an only hearing ear. c. A 1.5-mm lateral semicircular canal fistula, with normal hearing in the contralateral ear. d. A 0.5-mm cochlear fistula with normal hearing in the contralateral ear. e. A 1.5-mm fistula involving the lateral semicircular canal and vestibule.
Answer: c.� This is an empiric observation.
Answer: c.� Repair should not be attempted with fistulas
4. W hat is the time interval between stages when the indication is possible residual cholesteatoma?
greater than 2 mm, in an only hearing ear, or when it involves any labyrinthine structure other than the lateral semicircular canal.
been blamed for retraction and collapse of the grafted tympanic membrane. With the increased use of staging, many surgeons have recognized the role of middle ear mucosal disease in the pathogenesis of tympanic membrane collapse.
SELF-ASSESSMENT QUESTIONS
e11
3. W hich would be the best approach for an acquired cholesteatoma that involves the geniculate ganglion, but without a sensorineural hearing loss? a. Mastoidectomy combined with a middle fossa craniotomy b. Radical mastoidectomy c. Bondy modified radical mastoidectomy d. Canal-wall-up mastoidectomy with malleus head removal to expose the anterior epitympanum e. Translabyrinthine
Chapter 20
Answer: a.� A middle fossa craniotomy will give the expo-
bone and encephalocele, and 59% occurred as a complication of previous mastoid surgery.
sure needed for cholesteatoma removal. Mastoidectomy alone is unlikely to give adequate exposure of the geniculate ganglion. A radical mastoidectomy would not improve the exposure or facilitate postoperative care compared with other canal-wall-down mastoidectomy techniques. A translabyrinthine approach should be considered if there is no salvageable hearing. 4. A pproximately 50% of the mastoid segment of the facial nerve is disrupted during removal of a large cholesteatoma. What should be done? a. Remove all cholesteatoma and plan nerve repair at a second stage. b. Remove all cholesteatoma and evaluate facial function following surgery. c. Decompress the nerve, remove the injured section, and place an interposition graft. d. Reroute the facial nerve and anastomose it. e. Decompress the nerve and put a small nerve graft into the defect. Answer: c.� Repair is best done at the time of injury. The
injured segment should be removed if it is greater than 30% of the total diameter of the nerve. Rerouting requires much more extensive drilling, as well as transection of the greater superficial petrosal nerve. 5. T here is profuse bleeding from a 2-mm tear in the jugular bulb. How should you proceed? a. Firm intraluminal packing with Surgicel b. A muscle plug reinforced with fibrin glue c. Ligation of the sigmoid sinus and internal jugular vein d. Cover the injury with gelatin foam e. Surgicel packing between the bulb and the overlaying bone Answer: e.� Packing between the bulb and the overlaying
bone usually works with a defect this size. Firm intraluminal packing may injure the nerves in the jugular foramen. The muscle plug and gelatin foam are inadequate. Ligation is rarely necessary.
1. E ncephaloceles occur most commonly with what other condition? a. Chronic otitis media b. Head trauma c. Idiopathic d. Previous mastoid surgery e. Arachnoid granulations Answer: d.� In 1989, Iurato reviewed 139 cases of temporal
2. F or an encephalocele to develop which event or events must occur? a. Dural injury b. Bone dehiscence c. Dural injury and bone dehiscence d. Chronic otitis media e. Head trauma Answer: c.� For an encephalocele to develop, two preexisting
conditions are necessary and must occur: bone dehiscence and dural injury. 3. T he most common presentation of cerebrospinal fluid (CSF) leak in the temporal bone is a. Middle ear effusion b. Otorrhea c. Meningitis d. Seizure e. Mass in the ear canal Answer: a.� Middle ear effusion and hearing loss are the most
common presentation of CSF leak and encephalocele in the temporal bone. Subsequent myringotomy may result in otorrhea. Meningitis, seizures, and mass in the ear canal are all uncommon presenting conditions. 4. C onfirmation of spinal fluid in the ear is best made with which test? a. Glucose content b. Protein content c. β-2 transferrin d. CT e. MRI Answer: c.� Identification of β-2 transferrin in a specimen is
over 90% sensitive for identifying spinal fluid. CT and MRI are more valuable for locating possible sites of spinal fluid leakage. 5. The most common site of encephalocele formation is: a. Middle fossa b. Posterior fossa
e12
SELF-ASSESSMENT QUESTIONS
c. Ear canal d. Middle ear e. Oval window
Answer: a.
Chapter 21 1. P atients with far advanced otosclerosis are not candidates for a stapedectomy. a. True b. False Answer: b.
2. T he advantages of stapes surgery using local anesthesia includes all of the following except a. More bleeding during surgery b. Immediate feedback in terms of vertigo when manipulating the footplate c. Typically shorter recovery time Answer: a.
3. I f too much posterosuperior canal wall bone is removed, adhesions to the incus or a retraction pocket can form. a. True b. False Answer: a.
a. Tympanic membrane perforation b. Acute otitis externa c. Active Ménière’s disease d. Conductive hearing loss of at least 15 dB confirmed by tuning fork e. Only hearing ear
Answer: d.� Perforation, infection, or active Ménière’s disease
present at stapedectomy can result in a postoperative hearing loss. 2. P ossible risks and complications of stapedectomy surgery include a. Worsened hearing b. Tinnitus c. Dizziness d. Taste disturbance e. All of the above Answer: e.� All are well-recognized postoperative complica-
tions. 3. T he tympanomeatal flap should be elevated so that which two landmarks can be visualized? a. Neck of the malleus and round window b. Half of the diameter of the fallopian canal and stapedial tendon c. Cochlear form process and Jacobson’s nerve d. Eustachian tube orifice and a long process of incus e. Subiculum and ponticulum Answer: a.� Exposure of these two landmarks will allow the
4. T he limits of exposure in stapes surgery include all of the following except a. Round window niche b. Pyramidal eminence c. Upper edge of tympanic fallopian canal d. Malleus handle Answer: c.
5. A surgeon might consider stopping the procedure if he or she encounters a: a. Fixed malleus b. Tear in the tympanic membrane c. Solid or obliterated footplate d. Prolapsed facial nerve covering the footplate Answer: d.
tympanomeatal flap to be folded forward, providing adequate access to the middle ear and facilitating crimping of the prosthesis. 4. F or a right-handed surgeon operating on a right ear, what two landmarks should be visualized at the conclusion of curetting? a. Neck of the malleus and round window b. Half of the diameter of the tympanic fallopian canal and stapedial tendon c. Cochlear forum process and Jacobson’s nerve d. Eustachian tube orifice and a long process of incus e. Subiculum and ponticulum Answer: b.� This exposure will allow visualization of the foot-
Chapter 22
plate and introduction of instruments. On a left ear, a righthanded surgeon will require more exposure for instrument access and the entire diameter of the fallopian canal should be exposed.
1. R elative contraindications for stapedectomy include all of the following except
5. W hich of the following statements are true about reparative granuloma?
SELF-ASSESSMENT QUESTIONS
a. A reparative granuloma is a controversial entity whose modern existence is in doubt. b. It may be due to retained ethylene oxide in gelfoam wire prostheses. c. The attributed symptomatology may be explained by serous labyrinthitis. d. The presentation can be explained by expected postoperative findings. e. All of the above
Answer: e.
Chapter 23 1. F or the partial stapedectomy technique, how much of the footplate should be removed? a. All of it b. 25% c. Only that part which comes out easily d. 75% Answer: c.
2. T o properly size a piston-cup prosthesis, how is the correct length determined? a. The measurement from the surface of the footplate to the lateral surface of the incus is used. b. The measurement from the surface of the footplate to the medial surface of the incus is used. c. A prosthesis 4 mm in length is used universally, as it protrudes 0.2 to .03 mm into the vestibule. d. One half of a mm is added to the measurement from the surface of the footplate to the lateral surface of the incus. Answer: c.
e13
5. F or the treatment of juvenile otosclerosis, the prosthesis length should be sized a. The same as adult b. One half mm shorter than adult c. One half mm longer than adult d. None of the above Answer: a.
Chapter 24 1. L aser energy power (watts) per area of spot size (cm2) × exposure time (sec) is called a. Power density b. Fluence c. Intensity d. Photon Answer: b.
2. T he most common indication of failure of primary stapedectomy requiring revision surgery is a. Dizziness and vertigo b. Sensorineural hearing loss c. Recurrence of conductive hearing loss d. Facial nerve palsy Answer: c.
3. T he most common intraoperative finding associated with revision stapedectomy is a. Perilymph fistula b. Prolapsed facial nerve c. Displaced prosthesis d. Fibrosis of oval window tissue Answer: c.
3. W hat is the minimum diameter of the opening into the vestibule required for the piston-cup prosthesis? a. Larger than 0.8 mm b. Larger than 2.0 mm c. Half of the footplate d. The entire footplate Answer: a.
4. The vein graft size and oval window position should be a. 15 × 15 mm and adventitial side toward the vestibule b. 5 × 5 mm and endothelial side toward the vestibule c. 4 × 8 mm and adventitial side toward the vestibule d. 15 × 15 mm and endothelial side toward the vestibule Answer: c.
4. T he major advantage of lasers in revision stapes surgery is a. The ability to define the margins of the oval window b. Less bleeding and better visualization of oval window c. Thinning of existing membrane to more thoroughly identify the status of the oval window (footplate fragments, bone regrowth, etc.) d. All of the above Answer: d.
5. T he cardinal rule of laser surgery of the oval window, regardless of the type of laser, is a. To never fire the laser directly into an open vestibule b. Only perform revision surgery once
e14
SELF-ASSESSMENT QUESTIONS
c. That the inner ear is immune to damage from laser energy d. That there is nothing more to learn about laser revision stapes surgery
c. There is no difference in audiometric results in stapedotomy as compared to partial stapedectomy, except in overclosure. d. The vein graft should be placed with adventitia up. e. Place the prosthesis with two-handed technique.
Answer: a. Answer: d.� The vein graft should be placed adventitia down,
Chapter 25 1. W hich of the following statements regarding intraoperative audiometry is correct? a. It is not useful in cases done under local anesthesia. b. It requires testing by an audiologist. c. The frequency with the greatest air-bone gap is usually tested. d. It does not assist in placement of the prosthesis in revision stapedectomy. e. It will not prevent opening of the oval window membrane in revisions. Answer: c.� Intraoperative audiometry is done under local
anesthesia, does not require an audiologist, uses the greatest air-bone gap frequency, and assists in decisions regarding the footplate and prosthesis placement. 2. W hich of the following statements is correct regarding promontory drilling? a. It cannot be done with patient under local anesthesia. b. It will cause sensorineural hearing loss. c. It will yield poorer hearing results in stapedectomy. d. It should be done in a superior to inferior direction. e. It should be done in a medial to lateral direction. Answer: e.� Promontory drilling begins at the footplate prom-
ontory junction and sweeps laterally. All other statements are false. 3. A vein tissue graft is not useful in which of the following intraoperative situations? a. Sealing of the oval window after footplate removal b. Centering of the stapes prosthesis over the central area of footplate removal c. Preventing perilymph leak d. Recreating a mobile oval window membrane e. Repair of a necrosed distal long process Answer: e.� Vein tissue graft will not repair a necrosed incus.
All other statements are correct. 4. In routine stapedectomy, which is not correct? a. Always use a speculum holder. b. Check both the malleus and incus visually and by palpation.
intima up. All other statements are correct. 5. T he most appropriate candidate for revision stapedectomy is a patient with a history of: a. Sensorineural hearing loss without an oval window tissue graft b. Sensorineural hearing loss with an oval window tissue graft c. Malleus/incus fixation that is total d. Stapedectomy in which the hearing initially improved then declined e. Significant erosion of the incus Answer: d.� The most likely candidate is one whose hearing
initially improved, then declined and who presents with a conductive hearing loss.
Chapter 26 1. P resence of acoustic reflexes on a preoperative audiogram should prompt the operating surgeon to a. Cancel the surgery. b. Order a CT scan. c. Consider superior semicircular canal dehiscence as a cause of the conductive hearing impairment. d. Do all of the above. Answer: d.
2. W ith gentle pressure on a stapedotomy prosthesis after placement, patient dizziness should prompt the surgeon to consider that a. The prosthesis may be too long. b. The otosclerotic inner ear syndrome may be operative and may require treatment with calcium fluoride. c. A fistula exists in the round window. d. Placing a vein seal would be helpful. Answer: a.
3. The law of additive inadequacy describes a. Reduction in surgeon ability with age b. Unavoidable deterioration in surgical equipment over time c. Surgical inadequacies that add up to poor results d. Additive reduction in outcome with multiple surgeons Answer: c.
SELF-ASSESSMENT QUESTIONS
4. A dequate surgical results can be achieved in all of the following except a. Superior semicircular canal dehiscence b. X-linked progressive mixed deafness c. Revision stapedotomy with obliterative otosclerosis d. a and b e. a, b, and c Answer: d.
5. S tudies have shown the following may produce sensorineural hearing loss following stapedotomy: a. Viral infection b. Blood in the inner ear c. Gusher d. Otosclerosis e. All of the above Answer: e.
Chapter 27 1. E xamination findings suggestive of perilymphatic fistula (PLF) include a. Rotatory nystagmus with fatigue and direction reversal b. Nystagmus induced by loud noise or pressure to the eardrum c. Abnormal electrocochleography (ECoG) with elevated summating potential/action potential (SP/AP) ratio (>0.5) d. Positional nystagmus and a positive turning test Answer: d.� Nystagmus induced by noise or positive or nega-
tive pressure with intact tympanic membrane is indicative of a superior semicircular canal fistula. Rotary nystagmus with fatigue and direction reversal is classic for benign positional paroxysmal vertigo. Abnormally elevated SP/AP ratios have been noted in patients with PLF; however, they cannot be distinguished from Ménière’s patients who also have abnormal ECoG findings. 2. M iddle ear exploration for PLF is recommended in which of the below cases: a. Mild hearing loss and/or disequilibrium following recent trauma b. Sudden onset of hearing loss with no disequilibrium c. Fluctuating hearing loss, episodic vertigo d. Disequilibrium, worsening with coughing or straining, conductive hearing loss Answer: a.� Conservative, nonsurgical treatment is advo-
cated in patients seen in the first 7 days after development of symptoms. However, if the patient’s hearing fails to recover
e15
in 10 days of bed rest, surgical intervention should be carried out in the next 4 days. Sudden sensorineural hearing loss without history of prior antecedent traumatic event should be treated medically. Fluctuating hearing loss and episodic vertigo is likely Ménière’s disease. Conductive hearing loss with disequilibrium worsened by Valsalva-type maneuvers is likely superior semicircular canal dehiscence. 3. C lear fluid is encountered in the middle ear space during exploration of PLF. The most appropriate next step would be to a. Send for β-2 transferrin assay b. Suction the fluid c. Have the patient perform a Valsalva maneuver d. Patch both the oval and round windows Answer: b.� It is important to remove any fluid present on
initial exposure of the middle air space as this is likely diffusion of local anesthetic into the middle ear space. Once the fluid is suctioned, having the patient perform a Valsalva maneuver and putting pressure over the incudostapedial joint can help determine the presence of a leak. β-2 transferrin assay has not been shown to be helpful for intraoperative determination of perilymph. There is lack of general consensus as to whether both the oval and round windows should be patched without visual confirmation of PLF. 4. A natomical features of the round window that increase the likelihood of PLF are a. Deep round window niche b. Large overhanging promontory c. The inability to visualize round window transtympanically d. A 45-degree angle of the round window membrane to the promontory Answer: d.� Tears of the round window membrane occur most
frequently when its position is 45 degrees to the promontory and there is little or no overhanging promontory, allowing direct visualization of the round window membrane transtympanically. 5.
ecurrent PLF should be suspected if R a. Disequilibrium persists 2 weeks after surgery. b. Whirling episodic vertigo persists or develops. c. Hearing levels fail to improve. d. A fat graft was used to patch the fistula and 6 weeks of healing have taken place.
Answer: d.� Multiple authors have recommended avoiding use of fat as a graft material as it has been noted to have an increased failure rate. Six weeks should be allowed for the wound to completely heal before contemplating reexploration.Whirling episodic vertigo is typical for endolymphatic hydrops and not PLF. Hearing improvement has been reported in less than 50% of patients who have undergone PLF repair.
e16
SELF-ASSESSMENT QUESTIONS
Chapter 28 1. B ell’s palsy is characterized by all of the following pathophysiologic findings except a. HSV-1 DNA in facial perineural fluid at the time of nerve decompression in some patients b. Lymphocytic infiltration, neural edema, and myelin degeneration most prominent at the labyrinthine segment c. Constriction of the facial nerve at the meatal foramen with distal ischemia d. Degeneration of the cisternal segment of the facial nerve e. A spectrum of injury to the facial nerve from neuropraxia to neurontomesis Answer: d.
2. W hich of the following clinical findings is most consistent with the diagnosis of Bell’s palsy? a. Progression to complete paralysis over 1–7 days b. A palpable parotid mass c. Recent hearing loss associated with the facial paralysis d. Other cranial neuropathies associated with the onset of the facial paralysis e. Continued complete facial paralysis 3–6 months after the initial onset Answer: a.
3. W hen is electrodiagnostic testing of the facial nerve indicated in the setting of Bell’s palsy? a. Immediately after the onset of complete paralysis b. Two weeks after the onset of complete paralysis c. Worsening paralysis after appropriate medical therapy d. Seventy-two hours after the onset of complete facial paralysis e. Immediately once eye closure becomes incomplete Answer: d.
4. A ppropriate surgical management of Bell’s palsy involves all of the following except a. Complete paralysis of less than 14 days’ duration b. A transmastoid approach c. Greater than 90% duration on electroneurography performed at least 72 hours after the onset of complete paralysis d. Lack of voluntary motor unit potentials on facial electromyogram e. Bony decompression of the labyrinthine, geniculate, and proximal tympanic segments Answer: b.
5. A ll of the following findings are consistent with the diagnosis of Ramsay-Hunt syndrome in a patient with acute facial paralysis except a. Periauricular vesicular eruptions b. Otalgia c. Predominant involvement of the frontal and orbital branches of the facial nerve d. Other cranial neuropathies e. Skip regions of facial nerve involvement Answer: c.
Chapter 29 1. W hat segment of the facial nerve is most commonly damaged as a result of temporal bone trauma? a. Intracranial b. Meatal c. Labyrinthine/perigeniculate d. Tympanic e. Mastoid Answer: c.� The labyrinthine/perigeniculate portion of the
facial nerve is involved in up to 90% of cases. 2. A ll of the following are most commonly associated with longitudinal temporal bone fractures except a. Frontal/occipital impact b. Conductive hearing loss c. Ossicular damage d. Bloody otorrhea e. Fracture through the foramen ovale Answer: a.� Frontal/occipital impact is most commonly asso-
ciated with transverse temporal bone fractures. 3. W hich of the following findings would favor surgical exploration rather than observation in a patient with traumatic facial nerve paralysis? a. Immediate onset of complete facial nerve paralysis after penetrating injury b. Delayed onset of facial nerve paralysis with greater than 95% degeneration on electroneuronography (ENoG) c. Bony fragment impingement of the facial nerve on CT scan d. None of the above e. All of the above Answer: e.� All of the findings would favor surgical explora-
tion. In a and c, the chance of a complete transection of the facial nerve is high. Progressive deterioration of facial nerve function with a greater than 95% degeneration on ENoG are also associated with poor outcomes.
SELF-ASSESSMENT QUESTIONS
4. W hat is the best approach for surgical decompression of the facial nerve in a patient who has complete hearing loss on the paralyzed side? a. Transmastoid b. Translabyrinthine c. Middle cranial fossa d. Suboccipital e. None of the above
e17
a. Transcanal b. Transmastoid/extended facial recess c. Translabyrinthine d. Suboccipital e. Transcondylar
Answer: c.� The translabyrinthine approach provides expo-
sure of the facial nerve from the brain stem to the stylomastoid foramen.
Answer: b.� In a patient without residual hearing, the pre-
ferred approach is translabyrinthine because it provides adequate exposure of the perigeniculate area without many of the risks of serious complications associated with the middle cranial fossa approach. 5. W hat is the best result (House-Brackmann grade) in terms of facial nerve function after exploration and primary anastamosis? a. I b. II c. III d. IV e. V Answer: c.� The best surgical result that can be attained after
primary anastamosis is House-Brackmann III.
Chapter 30 1. T he most common presenting symptom for a facial nerve tumor patient is a. Facial paralysis b. Facial twitching c. Hearing loss d. Pulsatile tinnitus e. Otorrhea Answer: c.� Facial nerve tumors can cause a sensorineural
hearing loss if they develop in the internal auditory canal or a conductive hearing loss if they are present in the middle ear or mastoid. 2. P roven approaches to the management of facial nerve neuromas include a. Observation b. Surgical resection and nerve repair c. Decompression d. Radiation therapy e. a, b, and c Answer: e.� There are few reports of radiation treatment for
facial nerve tumors and no assessments of long-term efficacy. 3. T he entire length of the intracranial/intratemporal nerve can be accessed through which approach?
4. T he expected facial nerve result after facial nerve repair is a. I–II b. II–III c. III–IV d. IV–V e. V–VI Answer: c.� The intracranial and intratemporal facial nerve
is monofasicular. After facial nerve anasatomosis, synkenesis is an expected result. 5.
acial nerve repair is best accomplished F a. With as many sutures as possible b. Without a nerve graft c. In a tensionless fashion d. In a second-stage procedure e. With a laser welding technique
Answer: c.� It has been shown that a tensionless anastomosis
is important for success.
Chapter 31 1. W hich of the following is considered to be an absolute contraindication to cochlear implantation? a. Duration of deafness greater than 30 years b. Auditory neuropathy c. Enlarged vestibular aqueduct d. Michel aplasia e. All of the above Answer: d.
2. P atients with cochlear implants are at higher risk for developing meningitis. The CDC has recommended which of the following vaccinations to reduce the incidence of meningitis in cochlear implant recipients? a. Pneumovax for all patients greater than 2 years of age b. Prevnar vaccine for all patients less than 5 years of age c. Hib vaccine for all patients less than 5 years of age
e18
SELF-ASSESSMENT QUESTIONS
d. Meningococcal vaccination for all patients greater than 5 years of age e. a, b, and c f. All of the above
a. Patient age b. Handling of soft tissue c. Cooling during drilling d. Gender e. Patient hygiene
Answer: e. Answer: c.� Excessive heat during drilling will cause trauma
3. B ilateral cochlear implantation is experimental and should only be performed as part of a clinical trial. a. True b. False Answer: b.
4. A ssuming audiometric criteria for cochlear implantation are met, which of the following patients would be the poorest candidate for cochlear implantation? a. A 14-year-old male with congenital, profound sensorineural hearing loss who uses sign language as his primary mode of communication b. An 8-year-old female with progressive hearing loss following meningitis c. An 82-year-old female with gradually progressive hearing loss d. A 14-month-old male with congenital sensorineural hearing loss e. A 15-year-old female with congenital, progressive sensorineural hearing loss with normal speech and language Answer: a.
to the osteocytes and could result in soft tissue healing instead of osseointegration. 2. T o establish a reaction-free skin penetration, it is important to a. Remove all periosteum b. Be careful not to remove any of the periosteum c. Use an extra long abutment d. Make the skin at the penetration site thin and hairless e. Use firm packing under healing cap Answer: d.� Thin skin at the implant site will reduce the rela-
tive mobility between implant and skin, which is the key to a lasting, reaction-free skin penetration. The important daily cleaning is facilitated if no hair follicles are present. 3. I mportant for success with an ear-level, bone-anchored hearing aid is when the a. Air-bone gap is less than 20 dB b. Air-bone gap is larger than 20 dB c. Cochlea reserve is better than 35 dB d. Cochlea reserve is worse than 60 dB e. Bilateral cochlea deafness
5. W hich of the following techniques should be used during surgery for cochlear implantation? a. Skin and periosteal incisions should overlap by at least 1 cm b. The internal device should be placed as close as possible to the postauricular crease c. The cochleostomy should not be packed with soft tissue, as this increases the risk of developing meningitis d. The cochleostomy should be placed between the round window and oval window to ensure insertion occurs in the scala vestibuli
Answer: c.� Air-bone gap is of no importance at any level.
Answer: a.
Answer: a.� Even in large defects, healing will take place with
At 60 dB cochlea reserve, a body-worn aid is probably needed. 4. T he recommended way to handle postoperative skin necroses is a. Conservative handling with mild ointment b. Revision surgery as soon as possible with a free graft c. Removal of skin-penetrating abutment d. Removal of coupling and bone implant e. Long-term intravenous antibiotics a conservative attitude, even if it could take some time.
Chapter 32 NONE
Chapter 33 1. W hat is the most important factor to establish osseointegration?
5. W hich of the following is the best method to use to handle damage to the sigmoid sinus during boneanchored hearing aid surgery? a. Plug with bone wax, close wound, and wait 6 months for next trial b. Plug with periosteum and find a new implant site c. Perform a mastoidectomy to identify the damaged area
SELF-ASSESSMENT QUESTIONS
d. Use a p-PTFE membrane to stop bleeding e. Enlarge the defect, fill with muscle tissue, and use fibrin glue
Answer: b.� Sigmoid sinus is a low-pressure system and dam-
age of the wall is of minor importance. It is easily stopped with some soft tissue, and a new implant site close by can often be found.
Chapter 34 1. W hat is the pathophysiologic correlate of Ménière’s disease? a. Scarring and fibrosis of the periductal and saccular tissues within the vestibular aqueduct and opercular regions b. Contraction of the mastoid cavity with narrowing of Trautman’s triangle by anteromedial displacement of the sigmoid sinus c. Dilation of the membranous endolymphatic spaces including the scala media, saccule, and endolymphatic system d. Shortening and narrowing of the vestibular aqueduct e. Episodic vertigo lasting more than 30 minutes associated with nausea and vomiting, fluctuating or deteriorating hearing often in the low frequencies initially, and tinnitus and/or aural pressure. Answer: c.� Endolymphatic hydrops, as documented by tem-
poral bone histopathologic analysis of Ménière’s patients, shows dilatation of the membranous endolymphatic spaces, particularly the scala media and saccule. Answers a, b, and d are common findings within temporal bones of Ménière’s patients that may contribute to the symptomatology and development of the pathophysiologic state of hydrops. Answer e is the 1995 Committee on Hearing and Equilibrium symptom diagnosis for possible Ménière’s disease. 2. The functional role of the endolymphatic sac is to a. Maintain homeostasis of the endolymph with a graded concentration of sodium and potassium b. Provide immunologic support for the inner ear c. Remove debris and infectious waste by phagocytes d. Provide hormonal maintenance of inner ear fluid balance by secretion of Saccin and other cell-signaling effectors e. All of the above Answer: e.� Physiologic evidence that the endolymphatic
sac participates in inner ear immunity and control of fluid dynamics by humoral and local cellular activity are helping to identify the direct role that the endolymphatic system plays on hearing and balance function.
e19
3. A ccording to the 1995 Committee on Hearing and Equilibrium guidelines, the appropriate minimal follow-up time prior to reporting data in Ménière’s disease is a. 6 months b. 12 months c. 18 months d. 24 months e. 36 months Answer: d.� 1995 guidelines of the American Academy of
Otolaryngology—Head and Neck Surgery standardize the diagnosis and reporting of Ménière’s disease to minimize confounding effects of the natural history of the disease. They also promote standard benchmarks with which to compare data between competing treatment arms and studies. According to these guidelines, observations over the 6 months preceding intervention should be compared with observations within the 18 to 24 months following treatment. For comparison of audiometric data, the worst audiogram in the 6 months prior to treatment should be compared to the poorest audiogram following treatment. 4. E ndolymphatic sac procedures accumulatively offer class A and B results in what percentage of patients? a. Less than 50% b. 51%–65% c. 66%–80% d. 81%–90% e. Greater than 90% Answer: d.� According to the 1995 Committee on Hearing
and Equilibrium guidelines, class A responses to treatment of Ménière’s disease completely eliminate vertigo when comparing the 6 months prior to treatment with the 6-month period 18 to 24 months following intervention. Class B results occur when an intervention reduces the frequency of definite vertigo spells to less than 40% of the pretreatment levels. Looking at endolymphatic sac interventions between 1985 and 1995 with adequate follow-up, Grant andWelling showed a weighted effect of 86% class A/B results. A similar analysis between 1995 and 1997 shows endolymphatic surgery to be 84% effective. 5. W hat is the most common intervention for medically refractory Ménière’s disease in patients with serviceable hearing? a. Intratympanic gentamicin b. Tube insertion with application of micropressure, Meniette system c. Labyrinthectomy d. Vestibular nerve section e. Endolymphatic sac procedure Answer: e.� Endolymphatic sac surgery is the preferred
primary surgical treatment of Ménière’s disease by American Otologic and Neurotologic Society members in patients refractory to medical management with serviceable hearing,
e20
SELF-ASSESSMENT QUESTIONS
b ilateral disease, and in an only hearing ear. The total numbers of endolymphatic procedures is greater than numbers for other interventions. Intratympanic gentamicin therapy evolved throughout the 1990s and is used as first-line surgical treatment in patients with hearing loss and unilateral disease. Surgical labyrinthectomy numbers are falling, and numbers of vestibular nerve sections remain at a steady level.
Chapter 35
Chapter 36
1. The meatal plane is defined by a. The blue line of the posterior canal and a 60-degree angle b. The blue line of the superior canal and a 60-degree angle c. The blue line of the lateral canal and a 60-degree angle d. Bisecting the angle defined by the superior canal and the greater superficial petrosal nerve e. The flat bone 1 cm medial to the geniculate ganglion
1. W hich of the following is the most important component of the preoperative evaluation when considering vestibular neurectomy? a. Clinical history and physical examination b. CT scan of the temporal bone c. Caloric testing results d. MRI e. Audiometric findings
Answer: b.
2. A n absolute contraindication to middle fossa vestibular nerve section is a. An only hearing ear b. Bilateral Ménière’s disease c. Incapacitating vertigo d. Diuretic allergy e. Age less than 60 years Answer: a.
3. E xposure of the tegmen tympani facilitates orientation by a. Identification of the malleus and incus b. Identification of the lateral canal c. Identification of the superior canal d. Identification of the posterior canal e. Identification of the stapedius muscle
a. Less than 5% b. Greater than 20% c. Greater than 30% d. Greater than 40% e. Greater than 50%
Answer: a.
Answer: a.� All of the other components of the preopera-
tive workup may be helpful, but clinical history and physical examination are the most important components when identifying the cause of dizziness prior to considering any surgical treatment for relief of vertigo. 2. W hich patient is best suited for treatment of vertigo with a vestibular neurectomy? a. A patient with vestibular neuritis b. A patient with benign paroxysmal positional vertigo c. A patient with bilateral Ménière’s disease d. A patient with migrainous vertigo e. A patient with unilateral Ménière’s disease Answer: e.� Patients with unilateral Ménière’s disease have
the best response to selective vestibular neurectomy. There is no surgical treatment of migrainous vertigo and, if surgical treatment is undertaken for benign paroxysmal positional vertigo, it is most likely to be canal occlusion or singular neurectomy. Vestibular neuritis is much more unlikely to be treated successfully with vestibular neurectomy. Bilateral Ménière’s disease is a relative contraindication to vestibular neurectomy.
Answer: a.
4. C utting rather than avulsing the vestibular nerve is preferred because of possible a. Injury of the cochlear nerve b. Cerebrospinal fluid leak c. Increased dural injury d. Increased facial nerve injury e. Less effective vertigo control Answer: a.
5. Middle fossa vestibular nerve section hearing loss risk is
3. W hich of the following statements best describes the course of the facial nerve from the brain stem to the internal auditory canal? a. The facial nerve leaves the brain stem inferior and slightly anterior to the eighth cranial nerve and rotates in an anterior and superior direction so that it resides anterior to the superior vestibular nerve and superior to the cochlear nerve in the internal auditory canal. b. The facial nerve leaves the brain stem anterosuperior to the eighth cranial nerve and maintains that position into the internal auditory canal where
SELF-ASSESSMENT QUESTIONS
it is situated anterior to the cochlear nerve and inferior to the superior vestibular nerve. c. The facial nerve leaves the brain stem anterosuperior to the eighth cranial nerve and rotates until it is located anterior to the cochlear nerve and inferior to the superior vestibular nerve. d. The facial nerve leaves the brain stem posteroinferior to the eighth cranial nerve and rotates until it is anterior to the cochlear nerve and inferior to the superior vestibular nerve in the internal auditory canal. e. The facial nerve leaves the brain stem directly inferior to the eighth cranial nerve and rotates posteriorly such that it is located posterior to the cochlear nerve and inferior to the superior vestibular nerve in the internal auditory canal.
e21
Chapter 37 NONE
Chapter 38 1. A ll of the following are characteristic features of the nystagmus of a positive Dix-Hallpike maneuver except a. Predominantly torsional, side specific b. Latency of 3–5 seconds c. Fatigability with repeat testing d. Limited duration of 60–90 seconds e. Reversal when resuming the sitting position Answer: d.
Answer: a.
4. W hich of the following is the most commonly cited complication associated with the retrosigmoid vestibular neurectomy? a. Cerebrospinal fluid leak b. Wound infection c. Aseptic meningitis d. Subdural hematoma e. Chronic headache
2. B enign paroxysmal positional vertigo (BPPV) can result from, or has been correlated to, all of the following except a. Ménière’s disease b. Autoimmune inner ear disease c. Migraine d. Stapedectomy e. Labyrinthitis Answer: b.
Answer: e.� The retrosigmoid approach is associated with an
approximately 10% chance of headache that can be long-lasting and disabling. 5. W hich of the following statements best describes the success rate of posterior fossa vestibular neurectomy at eliminating vertigo and maintaining hearing? a. Posterior fossa vestibular neurectomy is greatly inferior to labyrinthectomy in both controlling vertigo and preserving hearing. b. Posterior fossa vestibular neurectomy is slightly inferior to labyrinthectomy in controlling vertigo and preserves hearing in approximately two thirds of patients. c. Posterior fossa vestibular neurectomy is slightly inferior to labyrinthectomy in preserving hearing while controlling vertigo in approximately two thirds of patients. d. Posterior fossa vestibular neurectomy is superior to labyrinthectomy at controlling vertigo, but not at preserving hearing. e. Posterior fossa vestibular neurectomy is superior to labyrinthectomy in both controlling vertigo and preserving hearing. Answer: b.� Labyrinthectomy is the gold standard for control of
vertigo but results in complete loss of hearing. In contrast, posterior fossa vestibular neurectomy controls vertigo in greater than 90% of patients and preserves hearing in 60% to 70% of patients.
3.
he most common form of BPPV results from T a. Anterior canal canalithiasis b. Lateral canal cupulolithiasis c. Lateral canal canalithiasis d. Posterior canal cupulolithiasis e. Posterior canal canalithiasis
Answer: e.
4. T he following statements about posterior canal occlusion surgery for BPPV are all correct except a. Free-floating particles can be seen in about 30% of cases. b. The risk of hearing loss is about 20%. c. Transient disequilibrium is seen in virtually all patients. d. Bilateral occlusions can be safely done in a sequential manner. e. Intraoperative auditory monitoring is not required. Answer: b.
5. T ransmastoid canal occlusion surgery has been used for all of the following except a. Otosclerosis b. Horizontal canal BPPV c. Acoustic neuroma excision
e22
SELF-ASSESSMENT QUESTIONS
d. Superior canal dehiscence e. Trigeminal schwannoma
Answer: a.
Chapter 39 1. Cochleosacculotomy creates a permanent defect in a. Osseous spiral lamina and cochlear duct and saccule b. Vestibular aqueduct and endolymphatic sac and saccule c. Cochlear aqueduct and endolymphatic sac and saccule d. Singular nerve e. Round window membrane Answer: a.
2. T he appropriate pick length for cochleosacculotomy without drilling the round window niche is a. 1 mm b. 2 mm c. 3 mm d. 4 mm e. Any pick length which can pierce the round window Answer: c.
3.
earing loss after cochleosacculotomy is H a. Almost zero b. Progressive for weeks c. Immediate d. Reversible with mannitol e. Only in the high frequencies
Answer: b.
4.
ertigo control with cochleosacculotomy is V a. 99% b. 50% c. 65% d. Improved with time e. Predictable by the presence of immediate nystagmus
Chapter 40 1. T he most important element of a transcanal labyrinthectomy to achieve mechanical destruction of the five vestibular end organs is a. Identification and removal of the saccule b. Identification and removal of the utricle c. Mechanical probing of the three semicircular canals d. Surgical widening of the oval window Answer: b.
2. T he most common cause of cerebrospinal fluid leakage during the course of a transcanal labyrinthectomy is a. Fracture of the crisbose bone at the medical aspect of the vestibule b. Widely patent cochlear aqueduct c. Anomalous spinal fluid communication along course of the facial nerve d. Widely patent communication of the spinal fluid with the perilymphatic space of the posterior semicircular canal Answer: a.
3. F acial nerve injury may complicate a transcanal labryinthectomy. The most common site of the injury is at the a. Descending segment b. Horizontal segment c. Geniculate ganglion d. Internal auditory canal Answer: b.
4. T he most difficult vestibular neuroephithelium to destroy by a transcanal labyrinthectomy is a. Maculae utriculi b. Macular sacculi c. Crista ampullaris of lateral semicircular canal d. Crista ampullaris of posterior semicircular canal Answer: d.
5. T he vertigo control rate in Ménière’s disease is affected by a strong placebo effect. a. True b. False
5. F ailure to obliterate the vestibule with a tissue seal following transcanal labyrinthectomy may a. Result in incomplete destruction of the vestibular neuroepithelium b. Result in delayed facial paresis c. Increase the chance of postoperative meningitis d. Result in tramautic neuroma of the superior or inferior vestibular nerve
Answer: a.
Answer: c.
Answer: c.
SELF-ASSESSMENT QUESTIONS
Chapter 41
NONE
Chapter 42 1. W hat is the most common serious complication of superior canal dehiscence (SCD) plugging surgery in the initial 24 hours after surgery? a. Aphasia b. Hematoma c. Seizure d. Meningitis
e23
c. Reassure the patient that the symptoms will improve with time d. Hold high-dose steroids and reassess in 4 hours
Answer: a.
Chapter 43 NONE
Chapter 44 NONE
Answer: b.
2. W hich factor puts patients at considerably greater risk of hearing loss after SCD plugging surgery? a. SCD greater than or equal to 4 mm in size b. Preoperative air-bone gap greater than 40 dB c. Prior stapes surgery d. Pulsatile tinnitus Answer: c.
3. W hich of the following is not a relative contraindication to SCD plugging? a. Prior middle ear surgery b. Prior successful SCD plugging on the contralateral side c. Age greater than 70 years d. Preoperative oscillopsia symptoms Answer: d.
4. W hich of the following symptoms are not likely to be improved after SCD plugging? a. Disorientation when rapidly rotating the head to look at something on the floor b. Disorientation and oscillopsia in response to loud noise c. Disturbing sound of one’s own voice d. Pulsatile tinnitus Answer: a.
5. T wo hours after SCD plugging surgery the patient has severe head pain on the side of the surgery and difficultly naming some common items. What is the appropriate next step in management? a. CT scan of the head without intravenous contrast b. Order a patient-controlled anesthesia machine so he or she does not have to ask the nurse for narcotics
Chapter 45 1. G radenigo’s syndrome usually includes all of the following except a. Retro-orbital pain b. Fourth cranial nerve palsy c. Otorrhea d. Hearing loss e. Sixth nerve palsy Answer: b.
2. T he most serious complication of the infracochlear approach to the petrous apex is a. Sensorineural hearing loss b. Facial nerve injury c. Tearing of the tympanic membrane d. Carotid artery injury e. Rupture of the jugular bulb Answer: d.
3. C haracteristic imaging of cholesterol granuloma includes a. Lack of pneumatization of contralateral petrous apex on CCT b. Hyperintense T1W and T2W images on MRI c. Hypointense T1W and hyperintense T2W images on MRI d. Hyperintense T1W and hypointense T2W images on MRI e. None of the above Answer: b.
4. I n the infralabyrinthine approach to the petrous apex, the structure at greatest risk is a. The carotid artery b. The jugular bulb
e24
SELF-ASSESSMENT QUESTIONS
c. The facial nerve d. The posterior semicircular canal e. The endolymphatic sac
Answer: b.
5. T he infracochlear approach to the petrous apex includes all of the following except a. Transection of the external auditory b. Elevation of the tympanomeatal flap c. Disarticulation of the incudostapedial joint d. Removal of tympanic bone e. Skeletonization of the carotid artery
c. Ascending pharyngeal artery d. Occipital artery e. Infratemporal fossa artery
Answer: c.� The ascending pharyngeal branch of the external
carotid artery is the most common arterial supply of glomus tumors. 5. P reoperative biopsy of glomus tumor is routinely used for surgical planning and patient counseling. a. True b. False Answer: b.� Biopsy risks uncontrollable hemorrhage or major
Answer: c.
arterial/venous injury.
Chapter 46
Chapter 47
1. T he distinction between a tympanomastoid glomus tympanicum versus a tympanomastoid glomus jugulare is a. Visible mass in a tympanicum b. Erosion of jugular bulb c. Erosion of carotid canal d. Middle ear and mastoid involvement e. Secretion of vasoactive chemicals
1. D uring resection of an anterior skull base malignancy, extensive removal of bone at the lateral aspect of the sphenoid is required. Abrupt bleeding is encountered that obliterates the entire surgical field. Vigorous packing with absorbable hemostatic agents successfully controls the bleeding. Postoperatively the patient complains of ipsilateral double vision on lateral gaze. No other focal neurologic deficit can be detected. The most likely cause of this deficit is a. Acute cerebral hemorrhage b. Edema in Dorello’s canal c. Overzealous packing of the cavernous sinus d. Carotid artery aneurysm e. Injury to cranial nerve V
Answer: b.� Jugulare tumors arise from the jugular bulb.
2.
ocal cord paralysis occurs soonest with V a. Glomus tympanicum b. Glomus jugulare c. Glomus vagale d. Jugular foramen schwannoma e. Jugular foramen meningioma
Answer: c.� Early onset of vocal cord paralysis is characteris-
tic of glomus vagale. 3.
reoperative embolization in glomus jugulare surgery P a. Reduces intraoperative blood loss b. Eliminates the need for jugular bulb resection c. Improves cranial nerve outcomes d. Alters periauricular incisions due to vascular compromise e. Has no proven intraoperative or postoperative effect
Answer: a.� Preoperative embolization significantly reduces
intraoperative blood loss. 4. T he most common arterial supply of glomus jugulare tumors is the a. Internal carotid artery b. External carotid artery
Answer: c.
2. A ppropriate studies for detection of aspiration in patients with lower cranial nerve deficits after skull base surgery include (choose all that apply) a. Bedside flexible endoscopic evaluation of swallowing (FEES) b. Subjective report from the patient c. Chest x-ray d. Modified barium swallow (MBS) e. Flexible fiberoptic laryngoscopy Answer: a, c, d, e.
3. V elopalatal insufficiency associated with lateral skull base surgery may be caused by all of the following except a. Tensor veli palatini paralysis b. Loss of cranial nerve X nodose ganglion fibers c. Nerve of Hering injury d. Palatopharyngeus muscle dysfunction Answer: c.
SELF-ASSESSMENT QUESTIONS
4. A patient undergoes lateral skull base tumor resection that includes sacrifice of cranial nerve VII and removal of tumor at foramen ovale. Acceptable rehabilitation of the cranial deficits would include (choose all that apply) a. Facial nerve cable graft b. Gore-tex or alloderm orbicularis oris sling c. Temporalis muscle sling d. Masseter muscle sling e. Temporal fossa implant Answer: a, b, e.
5. A 26-year-old sales professional undergoes routine resection of a glomus vagale tumor. She has previously undergone Silastic medialization at an outside institution but complains of a breathy voice and nasal speech. Swallowing evaluation and airway evaluations show decreased pharyngeal squeeze without aspiration, poor glottal closure with vocal cord atrophy, and paralysis of the left hemipalate. The best surgical option(s) for rehabilitation include (choose all that apply) a. Vocal cord augmentation with collagen injection b. Arytenoid adduction only c. Revision silastic medialization with arytenoid adduction d. Palatal adhesion e. Cricopharyngeal myotomy Answer: c, d.
Chapter 48 1. T he indications for a middle fossa approach for removing an acoustic tumor include a. Small tumor b. Good hearing c. Intracanalicular location d. Location mainly in the cerebellopontine angle e. a, b, and c Answer: e.� The exposure of the cerebellopontine angle is lim-
ited through the middle fossa approach. 2. A coustic tumors arising from which nerve have a higher incidence of hearing preservation? a. Superior vestibular nerve b. Inferior vestibular nerve c. Cochlear nerve d. Facial nerve Answer: a.� Tumors arising in the superior compartment of
the internal auditory canal tend to have less involvement of the cochlear nerve and cochlear blood supply.
e25
3. S earch of the landmarks that aid in the identification of the internal auditory canal include a. Arcuate eminence b. Greater superficial petrosal nerve c. Geniculate ganglion d. Superior semicircular canal e. All of the above Answer: e.� The arcuate eminence overlies the superior semi-
circular canal. Bisecting the angle between the superior canal and the greater superficial petrosal nerve locates the course of the internal auditory canal. The geniculate ganglion is at the lateral end of the internal auditory canal. 4.
he direction of tumor dissection should be T a. Lateral to medial b. Medial to lateral c. Anterior to posterior d. Inferior to superior e. All of the above
Answer: b.� Medial to lateral dissection prevents traction
injury to the facial and cochlear nerves at their exit points in the lateral internal auditory canal. 5.
he first middle fossa approach was reported by T a. William House b. Howard House c. John House d. Walter Dandy e. R. H. Parry
Answer: e.� Parry reported the use of the middle fossa
approach in the early part of the 20th century. It was refined and popularized by William House in the 1960s.
Chapter 49 1. S ome advantages the translabyrinthine approach has over the retrosigmoid approach for removal of acoustic neuromas include a. Extradural drilling that decreases the seeding of bone dust into the subarachnoid space b. Exposure of the entire facial nerve c. Less cerebellar retraction during tumor removal d. a and c e. All of the above Answer: e.� All drilling is complete prior to opening the dura.
In a retrosigmoid craniotomy, any internal auditory canal drilling must be performed after the cistern has been opened. By completing a mastoidectomy and opening the entire length of the internal auditory canal, the facial nerve can be exposed from brain stem to stylomastoid foramen (and possibly beyond if parotidectomy is performed), which allows for easier grafting if necessary.The trajectory of the translabyrinthine craniotomy
e26
SELF-ASSESSMENT QUESTIONS
enters the cerebellopontine angle from a more anterior origin than the retrosigmoid craniotomy; this allows for less cerebellar retraction.
2. A 45-year-old male with a 3.6-cm enhancing right cerebellopontine angle (CPA) lesion, which fills the internal auditory canal and does not have a detectable “dural tail” on MRI, is a good candidate for a translabyrinthine craniotomy because a. No other surgical approach will allow removal of a tumor this large. b. There is an 80% chance that the facial nerve function will be House-Brackmann grade I or II 1 year after surgery. c. There is a poor chance that hearing will be spared if total tumor removal is attempted, and the entire internal auditory canal and CPA component of the tumor can be exposed. d. This is an urgent case given the tumor size, and the translabyrinthine approach is a faster method of exposure. e. None of the above.
Answer: c.� Because of the tumor’s size, this patient has a
poor chance of hearing preservation with tumor removal, regardless of approach. Its size does not prohibit its removal through other approaches, and the translabyrinthine craniotomy is not necessarily faster than other craniotomies. The reported percentage of patients with House-Brackmann grade I or II 1 year following surgery for removal of unilateral, sporadic vestibular schwannomas greater than 3.5 cm in a single stage is about 50%. 3. W hich of the following is not routinely employed during a translabyrinthine craniotomy? a. Facial nerve monitoring b. Facial electromyography c. Short-acting paralytic agents for anesthesia d. Auditory brain stem response e. Monopolar electrocautery Answer: d.� Auditory brain stem response is not routinely
used during any case in which hearing is expected to be lost. Short-term paralytics, facial nerve monitoring, and facial electromyography are routinely employed as they allow the surgeon feedback regarding facial nerve irritation/trauma during tumor dissection. There are no contraindications to monopolar electrocautery during the approach. 4. Y ou are planning a translabyrinthine craniotomy for removal of an acoustic neuroma in a 59-year-old male in whom you had previously performed a tympanoplasty with mastoidectomy 10 years ago for chronic otitis media. His tympanic membrane appears healed, but your old operative report indicates that he had a small mastoid. For this patient you should
a. Perform the case by taking down the bony canal wall, closing off the ear canal laterally, and converting to a transcochlear or transotic approach to improve exposure. b. Perform the case as a routine translabyrinthine craniotomy, decompressing a large amount of middle fossa dura and dura posterior to the sigmoid sinus. c. Stage the case because of risk of infection by first closing the ear canal skin and taking down the bony canal wall followed by a translabyrinthine craniotomy 6 months later if no evidence of infection is noted. d. Abort the procedure as the risk of infection is high and the exposure will be poor. e. None of the above.
Answer: b.� A contracted mastoid is not a contraindication
to the translabyrinthine craniotomy.Wide dural decompression will be necessary to improve exposure. 5. T he potential benefits of performing a cranioplasty after a translabyrinthine craniotomy include a. Decreasing the rate of cerebrospinal fluid leak b. Prevention of a noticeable postauricular defect c. Protects the bony ear canal d. a and b e. a and c Answer: d.� Performing a cranioplasty helps bolster the fat
graft into the dural defect and potentially drops the cerebrospinal fluid leak rate.The nature of the cranioplasty is prevention of a noticeable defect in the skull; it does nothing to protect the bony ear canal.
Chapter 50 NONE
Chapter 51 1. T he transotic exposure adds which dimension of exposure of the internal auditory canal beyond what the translabyrinthine exposure offers? a. Anterior b. Posterior c. Inferior d. Superior e. Lateral Answer: a.
SELF-ASSESSMENT QUESTIONS
2.
ar canal management in transotic exposure requires E a. Sterile preparation with antibacterial solutions b. Two-layer closure c. Temporary transection and stenting to prevent stenosis d. Skin graft lining e. Wide meatoplasty
e27
c. Sinonasal tumors invading the infratemporal fossa, the masticator space, or the pterygomaxillary fossa, and tumors of the nasopharynx extending into the infratemporal fossa d. Trigeminal nerve sections, vestibular nerve sections, and CPA biopsies
Answer: a.
Answer: b.
3. T he facial nerve near the root-entry zone in relation to the anterior inferior cerebellar artery (AICA) is always: a. Posterior b. Superior c. Anterior d. Inferior e. Lateral Answer: c.
4. T he distinction between transcochlear and transotic exposure is that a. The facial nerve is mobilized posteriorly in transotic exposure. b. The facial nerve is mobilized anteriorly in transotic exposure. c. The facial nerve is mobilized anteriorly in transcochlear exposure. d. The facial nerve is mobilized posteriorly in transcochlear exposure. e. The facial nerve is better visualized in transotic exposure. Answer: d.
5. T he meatal foramen of the fallopian canal is situated at the a. Internal auditory canal porous b. Internal auditory canal fundus c. Geniculate ganglion d. Stylomastoid foramen e. Second genu Answer: b.
Chapter 52 1. W hat are the indications for the transcochlear approach? a. Lesions arising anterior to the internal auditory canal (IAC), petrous apex lesions, skull base and midline intradural lesions from the clivus b. Small acoustic tumors, with moderate extension into the cerebellopontine angle (CPA), and good preoperative hearing
2. W hat are the advantages of the transcochlear approach? a. The added exposure of removing the external auditory canal (EAC) and the cochlea, which increases access medially and anteriorly to the facial nerve with the safety of not requiring transposition of this nerve b. Hearing preservation and provision of an anterior plane of dissection on IAC in cases of acoustic neuroma c. Excellent exposure of the midline, allowing complete removal of the tumor, its base of implantation, and its blood supply with no cerebellar or temporal lobe retraction d. Extensive exposure inferiorly in the area of the jugular foramen and foramen magnum Answer: c.
3. W hat are the disadvantages of the transcochlear approach? a. Does not allow exposure of the lateral aspect of the pons and upper medulla, cranial nerves V through XI, and the midbasilar artery b. Sacrifice of residual hearing in the operated ear and risk of temporary facial palsy c. Limited by the cerebellum and the brain stem d. Requires the use of brain retractors Answer: b.
4. W hat are the intracranial structures that can be exposed by the transcochlear approach? a. Arcuate eminence, middle meningeal artery, greater and lesser superficial petrosal nerves, V3, V2, and VI entering into the superior orbital fissure b. The jugular foramen and foramen magnum c. Cerebellum and middle cranial fossa d. Entire lateral aspect of the pons and upper medulla, cranial nerves V through XI, as well as the midbasilar artery Answer: d.
5. W hat is the most common complication after transcochlear surgery and what steps should be taken to treat it?
e28
SELF-ASSESSMENT QUESTIONS
a. Temporary facial nerve paresis. Prompt eye care including lubrication with drops, nighttime ointments, and a moisture shield to prevent corneal complications should be given. Soft lenses, spring and gold weights, canthoplasty should be administered, and “watchful waiting” in cases of complete paralysis, if the facial nerve is anatomically intact, should be done. b. Temporary facial nerve paresis. Prompt eye care including lubrication with drops, nighttime ointments, and a moisture shield to prevent corneal complications should be given. Surgical intervention for facial reanimation in cases of complete paralysis, with a facial nerve that is anatomically intact, should be done. c. Delayed postoperative intracranial hemorrhage. Immediate reopening of the surgical wound and removal of the fat in the intensive care unit, and operative evacuation of the hematoma and control of the bleeding site or sites, should be done. d. Meningitis. Aggressive antibiotic therapy, if it is infectious, and treatment with dexamethasone if it is chemical aseptic meningitis, should be administered.
Answer: a.
Chapter 53 1. T he electromagnetic field (EMF) approach is a useful one for a. Acoustic neuroma greater than 2.5 cm b. Anterior petrous meningioma c. Large pituitary adenoma d. Meningioma extending into the jugular foramen Answer: b.
2. T he superior vestibular nerve can be safely followed laterally for a distance equaling a. 3 mm b. Three quarters the distance of the labyrinthine facial nerve c. One half the distance of the labyrinthine facial nerve d. 2.5 mm Answer: c.
4. T he EMF approach can be used for all of the following lesions except a. Petroclival meningioma b. Acoustic neuroma extending into the posterior fossa c. Lower clival lesions d. Infraclinoidal basilar tip aneurysms Answer: c.
5. A s a rule, the internal auditory canal (IAC) can be identified topographically by a. Bisecting the angle between the greater superficial petrosal nerve (GSPN) and the arcuate eminence b. Bisecting the angle between the arcuate eminence and the middle meningeal artery c. Following the course of the GSPN d. Following the course of the IAC Answer: a.
Chapter 54 1. H igh risk of stroke with carotid sacrifice on xenon blood flow studies with test occlusion of the carotid are predicted by flows a. Greater than 35 mL/min/100 gm of tissue b. 21–35 mL/min/100 gm of tissue c. Less than 20 mL/min/100 gm of tissue d. Greater than 50 mL/min/100 gm of tissue e. Greater than 100 mL/min/100 gm of tissue Answer: c.
2. T he frontalis portion of the facial nerve is best protected in the lateral subtemporal skull base approach by dissecting a. Deep to the deep temporal fascia off the zygomatic arch b. Superficial to the deep temporal fascia off the zygomatic arch c. Superficial to the fat pad over the temporal muscle d. Along each branch of the frontalis portion of the facial nerve e. Along at least two branches of the frontalis portion of the facial nerve Answer: a.
3.
he inferior extent of the EMF approach includes T a. The foramen magnum b. The inferior petrosal sinus c. The midclivus d. The superior petrosal sinus
Answer: b.
3. T he most common morbidity associated with surgery of the infratemporal fossa is a. Trigeminal nerve dysfunction b. Facial nerve dysfunction c. Dysphagia
SELF-ASSESSMENT QUESTIONS
d. Dysphonia e. Ataxia
Answer: a.
4.
he best treatment of postoperative trismus is T a. Temporomandibular joint (TMJ) resection b. TMJ prosthesis c. Stretching therapy d. Laser scar ablation e. Steroid injection into the pterygoid muscles
Answer: c.
5. U nilateral rhinorrhea not related to postoperative cerebrospinal fluid leak following skull base surgery occurs from a. Loss of sympathetic fibers along the internal carotid artery b. Loss of parasympathetic fibers along the internal carotid artery c. Pterygopalatine nerve ablation d. Sinusitis e. Hematoma liquefaction Answer: a.
e29
3. W hat are the four anatomical structures that form the boundaries of the “quadrangular space”? a. Cavernous internal carotid artery (ICA), optic nerve, first branch of trigeminal nerve, and vidian canal b. Vertical/paraclival ICA, petrous/horizontal ICA, cranial nerve (CN) VI, and maxillary branch of trigeminal nerve c. Vertical/paraclival ICA, petrous/horizontal ICA, CN IV, and cavernous sinus d. Cavernous ICA, optic nerve, vidian canal, and gasserian ganglion e. Vertical/paraclival ICA, petrous/horizontal ICA, CN VI, and gasserian ganglion Answer: b.� These four structures are the key anatomic struc-
tures that limit the endonasal approach to Meckel’s cave. 4. W hich of the following are options for vascularized tissue repair of endonasal skull base defects? a. Temporoparietal fascial flap b. Nasal septal mucosal flap c. Turbinate flap d. All of the above e. None of the above Answer: d.� All are potential candidates for a vascularized
Chapter 55 1. T he lateral extent of access via endoscopic endonasal approach at the level of the superior orbit is a. Lamina papyracea b. Periorbita c. Midorbital line d. Medial rectus muscle e. Anterior ethmoidal artery Answer: c.� Following removal of the lamina papyracea, the
periorbita can be gently displaced laterally to allow access to lesions above the orbit as far lateral as midorbit. 2. I nferior extent of access via endonasal approach can be determined preoperatively by drawing a line (Kassam line) between which two structures extended into the depth on sagittal CT? a. Nasal tip and odontoid process b. Tip of bony nasal bridge and hard palate c. Tip of bony nasal bridge and odontoid process d. Middle turbinate and hard palate e. Middle turbinate and C1 Answer: b.� These two bony structures create a fulcrum that
limits the inferior extent of exposure. By drawing a line between them on preoperative sagittal imaging, one can get a rough idea of the extent of lowest access.
flap. The nasal septal flap is the most versatile given its size and location. However, it usually must be harvested during the approach. 5. W hich of the following is the most common complication of endoscopic endonasal surgery prior to the introduction of the nasal septal flap? a. Cerebrospinal fluid leak b. Arterial injury c. Infection d. Stroke e. Cranial nerve palsy Answer: a.� Cerebrospinal fluid leak was the most common
complication of endoscopic endonasal surgery for skull base lesions prior to the introduction of the septal flap. Despite this, infection rates were very low.
Chapter 56 1. A combined petrosal approach is appropriate for petroclival tumors that a. Are superior to the internal auditory meatus b. Are inferior to the internal auditory meatus c. Extend into the infratemporal fossa d. a, b, and c e. a and b
e30
SELF-ASSESSMENT QUESTIONS
Answer: e.� The combined petrosal approach is best suited for
petroclival tumors that are both superior and inferior to the internal auditory canal. Otherwise, the middle fossa approach alone or a posterior approach alone can be used. The infratemporal fossa is not accessible with the combined petrosal approach.
a. Cerebrospinal fluid leak b. Cranial nerve injury c. Cerebrovascular accident d. Pituitary hypofunction e. Subdural hematoma
Answer: b.� Cranial nerve injury is the most common com-
2. M edial access to petroclival tumors with the combined petrosal approach is most commonly limited by a. The jugular bulb b. The angle between the clivus and brain stem c. The distance between the sigmoid sinus and facial nerve d. The tentorium and superior petrosal vein e. The angle between the cerebellum and brain stem
plication. Cranial nerves V and VII are the most commonly compromised, but III–XI are at risk with this approach. Some authors advocate subtotal excision with stereotactic irradiation as a result, but the long-term results are unknown.
Chapter 57 NONE
Answer: b.� The two structures that cannot be retracted,
removed, or moved are the brain stem and clivus. It is therefore the angle between them that limits access to the medial aspect of these tumors. More anterior exposure helps to open this angle. 3. P reoperative evaluation should include examination of the venous drainage system to exclude a. A dominant sigmoid sinus system b. A high jugular bulb c. Failure of the transverse sinus to communicate with the confluence (torcular herophili) d. Insertion of the vein of Labbé into the superior petrosal sinus e. a, c, and e Answer: e.� All of these vascular variations place the patient
at risk for venous infarction if not recognized preoperatively. 4. S ignificant posterior compression of the brain stem by the tumor a. Often requires more aggressive removal of the otic capsule for adequate brain stem exposure and tumor excision b. Requires no change in the amount of otic capsule removal because brain stem position does not affect tumor exposure c. Often requires less aggressive removal of the otic capsule for adequate brain stem exposure and tumor excision d. Requires more aggressive cerebellar retraction to adequately expose the brain stem e. Often makes sigmoid sinus transection necessary Answer: c.� The angle between the brain stem and clivus can
often be opened by posterior brain stem compression, allowing better access to the clivus than with a tumor that does not compress the brain stem. 5. T he most common complication from the combined petrosal approach is
Chapter 58 1. T he auditory brain stem implant (ABI) and the other CNS auditory implants described in this chapter a. Are direct replacements for cochlear implants b. Provide equivalent speech perception performance to cochlear implants c. Were developed to provide hearing sensations to deaf individuals with nonviable peripheral auditory neural systems d. All of the above Answer: c.
2. T he primary benefit of first-tumor side ABI implantation includes a. Better outcomes because implantation can occur when tumors are smaller b. Better outcomes because implantation can occur when patients are younger c. The opportunity to gain experience with the device before becoming completely deaf on the second-tumor side d. An opportunity to implant a more advanced device when the second-side tumor is removed Answer: c.
3. A primary contributor to a satisfactory outcome with an ABI is a. Implantation when vestibular schwannomas are as small as possible b. Implantation as soon as possible after onset of complete deafness c. A thorough and frank appraisal preoperatively of the potential benefits and limitations of the device d. Learning sign language and lipreading preoperatively Answer: c.
SELF-ASSESSMENT QUESTIONS
4. T he primary benefit of the penetrating auditory brain stem implant (PABI) was found to be a. Improved speech recognition performance b. Fewer nonauditory sensations c. Lower electrical auditory thresholds and a wide range of pitch sensations d. Greater ease of placement in the target neurons than the surface ABI Answer: c.
5. W hich of the following statements is true about auditory midbrain implantation? a. Speech perception is much better than the PABI. b. Speech perception is much worse than the regular surface ABI. c. Initial patients were not able to understand speech without lipreading cues. d. It clearly avoids any issues related to performance limitations with regular ABIs because of possible neural damage due to vestibular schwannomas or their removal. Answer: c.
Chapter 59 1. T he major advantage of the ELITE procedure for resection of large glomus jugulare tumors is a. Easier reconstruction of the skull base b. Better results in hearing preservation c. Improved exposure of intradural tumor d. Exposure of the tympanic portion of the facial nerve Answer: c.
2. I n the classic ELITE approach, management of the facial nerve typically includes a. Classic transposition b. Skeletonization of intratympanic portions of nerve c. Resection for improved access, with later grafting d. Skeletonization of vertical portion with limited anterior translocation
e31
Answer: Craniovertebral instability can occur secondary
to over-resection of the condyle. At least 50% of the condyle should be preserved. 5. D iscuss options for cranial base reconstruction and for prevention of cerebrospinal fluid leak. Answer: Options for reconstruction include fascia for
smaller defects, a pericranial flap, or a microvascular free flap for reconstruction of larger defects. Use of a vascularized myofascial flap for reconstruction is thought to reduce the incidence of postoperative cerebrospinal fluid leak. Abdominal fat is used to fill the mastoid defect. Lumbar drainage is used in most cases.
Chapter 60 1. A ll of the following are advantages of using a pumpregulated drainage system for lumbar drainage of a cerebrospinal fluid leak EXCEPT: a. Patient can move about more freely b. Decreased risk of tension pneumocephalus c. Strictly regulated flow of CSF d. Lower risk of meningitis Answer: d. There is no study showing that pump-regulated
drainage is associated with a lower risk of meningitis, but pump-regulated drainage does allow the patient to move about more freely, decreases the risk of accidental tension pneumocephalus due to unregulated CSF drainage, and does provide for a strict regulation of CSF flow. 2. T RUE or FALSE: Titanium mesh cranioplasty and hydroxyapatite cranioplasty reduce the risk of cerebrospinal fluid leak after translabyrinthine acoustic neuroma surgery. a. True b. False Answer: a. This is true (cf. references by Fayad and
Arriaga).
Answer: Dissection of the suboccipital triangle, identification
3. W hich of the following techniques would NOT be appropriate for closure of persistent CSF rhinorrhea after translabyrinthine acoustic neuroma surgery? a. Ear canal closure with Eustachian tube and middle ear obliteration (blind sac closure) b. Wound exploration and reclosure with new abdominal fat graft c. Middle fossa obliteration of the Eustachian tube d. Lumbar drainage
of the vertebral sulcus (“J” groove of the C1 lamina), or use of a Doppler probe.
Answer: c. Since the patient no longer has hearing, middle
Answer: d.
3. N ame three methods of identifying the V3 segment of the vertebral artery
4. N ame one uncommon complication of the ELITE surgical procedure that is unique to this approach.
fossa surgery to obliterate the Eustachian tube is not indicated after translabyrinthine surgery.
e32
SELF-ASSESSMENT QUESTIONS
4. M ethods of sealing the CSF space after posterior fossa surgery include: a. Abdominal fat b. Hydroxyapatite cement c. Fibrin glue d. Collagen matrix e. All of the above Answer: e. All of the answers represent methods used to seal
the CSF space after skull base surgery.
4. T he best method of assessing lower lid position on the paralyzed side of the face is a. Ask the patient how much the lid position has changed. b. Assess the position with straight ahead gaze. c. Assess the position with up gaze. d. Assess the position with down gaze. e. The gaze of the patient should be directed such that the limbus of the contralateral lid is placed just tangential to the inferior limbus. Answer: e.
Chapter 61 1. P atients who are to be considered for eyelid surgery include a. Those who are either symptomatic or who show signs of conjunctival or corneal injury, or both, despite maximum tolerated medical therapy b. Those who require rapid ocular rehabilitation to resume their usual occupation and responsibilities c. Those whose ocular status is currently stable but who are at high risk of corneal complications d. All of the above e. None of the above
5. A lid suture taped to the cheek and temporary tarsorrhaphy suture are a. Old methods that are only of historical interest b. Ways to close the eye on a windy day c. Complex procedures requiring special skills d. Only useful for a day or two e. Useful temporizing measures until a definitive solution to the eyelid closure problem can be implemented Answer: e.
Chapter 62
2. U nipolar cautery should not be used in eyelid surgery if a. The patient has an auditory brain stem implant b. The patient has a cochlear implant c. The patient has a prior spring d. All of the above e. Only a and b
1. W hat is the best method, when possible, for repairing a facial nerve injury where the nerve ends are visible during surgery for removal of a cerebellopontine angle neoplasm? a. Cable grafting b. Primary tension-free anastomosis c. Hypoglossal-facial anastomosis d. Cross-facial grafting e. Temporalis muscle dynamic reanimation
Answer: e.
Answer: b. Primary anastomosis of nerve ends, without
Answer: d.
3. A dvantages of the palpebral spring compared to the gold weight include the following: a. The spring closes the eye during sleep, when the patient is supine, whereas the weight does not. b. Lids that are hard to close may require large bulky weights, compared to a fine wire spring. c. Patients who are exposed to extremes of hot or cold may experience discomfort in the lid due to the heating or cooling of the weight, which is much less of a concern with the spring due to less mass to heat or cool. d. To adjust the closing tension with a gold weight requires replacement, whereas the spring can be adjusted without removal. e. All of the above. Answer: e.
tension, is the preferred method for repairing discontinuities of the facial nerve. Cable grafting requires more than one suture line, but should be used in cases where a tensionless anastomosis is impossible. The other options are appropriate when direct facial nerve repair has failed to reanimate the facial palsy. 2. W hich of the following procedures generally results in the most tongue dysfunction? a. Jump graft interposition between the hypoglossal nerve and facial nerve b. Facial nerve transposition from the fallopian canal, with end-to-side anastomosis to the hypoglossal nerve c. Direct hypoglossal-facial anastomosis using the distal end of the hypoglossal nerve
SELF-ASSESSMENT QUESTIONS
e33
Answer: c. The conventional direct anastomosis of the distal
Answer: d. A single epineurial suture is the standard method
hypoglossal nerve to the main trunk of the facial nerve provides the most amount of viable neurons to the facial nerve, but results in hemiglossal atrophy. Partial anastomoses such jump grafting and end-to-side anastomosis may largely spare ipsilateral hypoglossal function.
of securing a nerve to the end of a collagen tubule. Multiple sutures are used when nerve ends are directly anastomosed.
3. W hat clinical situation is the most appropriate for use of the hypoglossal-facial anastomosis? a. Extirpation of malignant parotid neoplasm with sacrifice of the pes anserinus and upper and lower divisions of the intraparotid facial nerve b. Extirpation of cerebellopontine angle tumor with loss of the facial nerve at the brainstem with no visible proximal stump c. Bell’s Palsy with no visible facial function but with active motor unit potentials on voluntary EMG d. Extirpation of cerebellopontine angle tumor with laceration of the facial nerve at the porus acousticus, with present proximal and distal nerve ends Answer: b. The most appropriate use of hypoglossal-facial
nerve anastomosis is when there is no potential for recovery of nerve function from the facial nerve itself, either through tensionless anastomosis or grafting. In the case of the parotid neoplasm, direct facial nerve anastomosis or muscular facial reanimation may be attempted, but hypoglossal-facial nerve anastomosis is impossible due to loss of the main trunk and upper and lower divisions of the facial nerve. 4. W hich electrophysiological test results are most compatible with the use of the hypoglossal-facial anastomosis? a. Fibrillation potentials on voluntary EMG, with 100% degeneration on electroneuronography (ENoG) b. Motor unit potentials on voluntary EMG, but with >90% degeneration on ENoG c. No degeneration on ENoG and active motor unit potentials on EMG Answer: a. Option 1 is the only test result that signifies com-
plete loss of facial nerve function, and is hence compatible with hypoglossal-facial nerve anastomosis. 5. W hat is the most appropriate way to secure a collagen tubule to the nerve end to be anastomosed? a. Six 9-0 nylon interrupted epineurial sutures b. Fibrin glue c. Cyanoacrylate d. One 8-0 interrupted polypropylene epineurial suture
6. T /F Recent studies suggest that partial hypoglossal nerve anastomoses may provide similar facial nerve outcomes while sparing patients morbidity from hemiglossal atrophy. a. True b. False Answer: a. This is true. References may be found in the
chapter text.
Chapter 63 1. T he ideal procedure for any patient with a disruption of the facial nerve is a. Primary repair b. Grafting to reestablish continuity between the facial nerve nucleus and facial musculature c. Temporalis muscle transposition d. None of the above e. a and b Answer: e.
2. B ased on the author’s experience, what percentage of patients have fair to superb nerve graft results? a. 5% b. 20% c. 60% d. 80% e. 95% Answer: d.
3. T he goal of the temporalis muscle transposition procedure is to restore spontaneous mimetic expression to a patient with facial paralysis. a. True b. False Answer: b.
4. T ime post-onset of the facial paralysis is an important consideration in which of the following procedures? a. Temporalis muscle transposition b. Greater auricular nerve graft c. Free muscle grafting d. All of the above e. None of the above Answer: b.
e34 5.
SELF-ASSESSMENT QUESTIONS
he preferred nerve to use in a grafting procedure is T a. Sural cutaneous b. Medial branchial cutaneous c. Greater auricular d. None of the above e. a and b
Answer: c.
6. W hich of the following are important factors in considering a temporalis muscle transposition procedure? a. Airway structure b. Appearance of nasolabial structures c. Type of smile on unaffected side d. b and c e. All of the above Answer: e.
Chapter 64 1. T he rationale for using intraoperative monitoring is to reduce the risk of permanent postoperative neurologic deficits. The way in which monitors do this is to a. Identify facial nerve when not visible in field b. Preserve useful hearing in very small acoustic tumors c. Assist in microvascular decompression for hemifacial spasm, trigeminal neuralgia, and vestibular nerve section d. Direct intracranial dissection when anatomy is unknown e. All of the above Answer: e.� There are a variety of scenarios in which intra-
operative monitoring can be a useful adjunct to reduce the risk of permanent postoperative neurological deficits. 2. A pitfall of intraoperative monitoring includes a. Operating room is noisy and electrical interference is abundant. b. Result of intraoperative monitoring is easily obtainable. c. The computer equipment can identify clinically significant changes in waveform without neurophysiologic personnel. d. Facial nerve monitoring is required in all otologic procedures. e. Maintenance of equipment is rarely necessary. Answer: a.� Obtaining a reproducible signal in the operat-
ing room is difficult due to electrical noise and interference. Neurophysiologic personnel should be available for setup and troubleshooting equipment that is regularly maintained. Neurophysiologic personnel help to identify changes in the amplitude and latencies and can help distinguish true changes from
rtifact. Facial nerve monitoring is required in otologic proa cedures when the facial nerve is particularly at risk, that is, unusual anatomy, previously operated ear, or extensive disease. 3. M ethods to monitor neural conductivity of the eighth cranial nerve include recording a. Brain stem–evoked auditory potentials b. Evoked potentials from exposed intracranial portion of cranial nerve VIII c. Surface electrode on cochlear nucleus d. Laser Doppler blood flow analysis e. Visual inspection Answer: a, b, c.� These are three methods to check neural
conductivity of cranial nerve VIII. 4. T o reduce the time to obtain interpretable evoked potentials a. Minimize electrical interference reaching recording electrodes b. Optimize filtering of recorded potential to accentuate background noise c. Decrease stimulus repetition rate and strength d. Increase electrode impedance e. Ignore methods for quality control that do not require record replication Answer: a.� Evoked potentials are most readily obtained in
environments with minimal electrical interference, background noise, low electrode impedance, and optimal stimulus repetition rate and strength. Low electrode impedance and increased stimulation rate and strength improve the quality of electrical signal information to be interpreted. 5. C hanges involving the following reflect clinically important changes in hearing a. Increase peak V latency b. Decreased peak V amplitude c. Change in peak V without change in peak III d. Change in interpeak latency e. Loss of reproducible signal Answer: e.� Whereas all of these changes could indicate
potential hearing compromise, the loss of reproducible signal is the strongest indication of a clinically important change in hearing.
Chapter 65 1. W hich of the following is true regarding gamma knife surgery for vestibular schwannomas? a. Tumor control rates are greater than 97%. b. Facial nerve motor dysfunction occurs in less than 1% with current dosing. c. Trigeminal nerve dysfunction is more common with large tumors.
SELF-ASSESSMENT QUESTIONS
d. Malignant transformation or induction is rare. e. All of the above.
Answer: e.
2. H earing thresholds as measured by pure-tone averages after gamma knife surgery most commonly a. Improve immediately b. Behave similar to expectant observation c. Progress rapidly to profound deafness d. Degrade rapidly in the first 6 months and then slowly worsen e. Do not change
e35
4. A patient underwent gamma knife surgery of a 2.3-cm CPA vestibular schwannoma 6 months ago. MRI performed today shows the tumor to be 2.7 cm in maximal diameter. The patient is asymptomatic. The next best course of action is a. Assure patient this is normal and rescan in 6 months b. Recommend microsurgical resection for radiation failure c. Counsel patient this may be malignant d. Plan a second round of gamma knife surgery e. Start high-dose steroids with a taper Answer: a.
Answer: d.
3. W hich of the following tumors would not be amenable to treatment with the more common gamma knife B and C units? a. Intracanalicular acoustic neuroma of 7 mm maximal diameter b. Vestibular schwannoma extending into the cerebellopontine angle (CPA) by 1.5 cm c. 2-cm glomus jugulare tumor extending anterosuperiorly from the jugular bulb d. Glomus vagale tumor extending to the carotid bifurcation e. Petrous apex meningioma of 2.4 cm maximal diameter Answer: d.
5. W hich of the following is not a component of the gamma knife surgery system? a. Trunnions b. Collimator helmet c. Gamma calipers d. Cobalt 60 (60Co) sources and beam channels e. MRI fiducial box Answer: c.
Chapter 66 NONE